Sei sulla pagina 1di 291

In a solid, the particles are tightly packed due to strong forces of interaction between them;

therefore, they are rigid and have definite volume.

The constituent particles are not free to move but only oscillate about their mean positions so they
posses fixed shape.

Solids are rigid and incompressible, they cannot flow.

Solids have many free surfaces.

Types of Solid

Property Crystalline Amorphous

Geometry The internal arrangement of particles The internal arrangement of particles is


is regular so they possess definite irregular. Thus they do not have any
and regular geometry. definite geometry.

It has definite edges, faces and It has no definite edges, faces or


interfacial angles. interfacial angles.

Order Long range order in arrangement of Short- range order limited only to a few
constituent particles atomic distances.

Melting point Definite Not definite and soften on heating ,then


flow

Heat of fusion Definite Not definite

Fluidity Rigid Fluid

Cleavage They break into two pieces with plane They break into two pieces with irregular
surfaces surfaces

Nature True solid as these are rigid Pseudo solids / super cooled liquid as
these have the property of fluiditys.

Isotropy Crystalline solids are anisotropic. This Amorphous solids are isotropic in nature.
implies that physical properties such This implies that various physical
as refractive index, conductivity, properties are same in all the directions.
thermal expansion etc are different in This is because of random arrangement
different directions. This is due to of particles.
orderly arrangement of particles

Effect of heat There is a sudden change in volume There is no sudden change in volume on
when it melts. melting.

jksahu74@gmail.com 1|Page
Silica (SiO2)

Quartz Amorphous Silica (Glass)


It is crystalline in nature It is light (fluffy) white power

All four corners of SiO44- tetrahedron are The SiO44- tetrahedral are randomly joined,
share by others to give a network solid giving rise to polymeric chains, sheets or three
dimensional units.

It has high and sharp melting point (1710oC) It does not have sharp melting point

Liquid Crystal

Solid Transition point Melting point


Turbid liquid Clear liquid

Allotropes of Carbon

Diamond Graphite

It occurs naturally in free state It occurs naturally, as well as manufactured


artificially.

It is the hardest natural substance known It is soft and greasy to touch

It has high relative density (about 3.5) Its relative density is 2.3

It is transparent and has high refractive index (2.45) It is black in colour and opaque

It is non-conductor of heat and electricity It is a good conductor of heat and


electricity

It burns in air at 9000C to give CO2 It burns in air at 7000C to give CO2

It occurs as octahedral crystals It occurs as hexagonal crystals.

Quartz can be converted to glass by melting and then rapid cooling.

Quartz is used for converting sunlight into electricity ( Photovoltaic cell)

Two or more solids having similar crystalline form are called isomorphous solids and this
property is called isomorphism.

A substance which exists in more than one crystalline form is called polymorphic and this
property is called polymorphism.

jksahu74@gmail.com 2|Page
Different Types of solid

Type of solid Constituent particles Attractive forces Examples Physical Electrical conductivity Melting point
Nature
[1]Molecular solids

(a) Non-polar
Dispersion or Ar, CCI4, H2, I2, Soft Insulator Very low
London force CO2

(b) Polar
Molecules Dipole-dipole HCI, SO2 Soft Insulator Low
interactions

(c) Hydrogen
bonded Hydrogen H2O (ice) Hard Insulator Low
bonding
Insulator in solid
[2] Ionic solids lons Coulombic or NaCI , MgO, Hard but brittle state but conductors in High
electrostatic Zns , CaF2 molten state and
in aqueous solutions

[3] Metallic solids Positive ions in a sea or Metallic bonding Fe, Cu, Ag, Hard but Conductors in solid Fairly high
delocalized electrons Mg malleable and state as well as in
ductile molten state.

SiO2 (quartz),
[4] Covalent or SiC,
network solid Atoms Covalent C (Diamond) Hard Insulators Very high
bonding AIN

Graphite Soft Conductor

jksahu74@gmail.com 3|Page
Space lattice or Crystal lattice
A regular three dimensional arrangement of points in space is called space lattice or crystal lattice.
The points represent the constituent particles of the crystal, know is lattice points.

Unit Cell
An unit cell is the smallest portion of the crystal lattice. When it is moved repeatedly a distance
equal to its own dimension along each direction, a three dimensional crystal lattice is generated.

Types of unit cell


1. Primitive Unit cell. It is also called simple unit cell, which has particles as points only at its
corners.
2. Non-Primitive unit cell - In this type of unit cells, particles as points are present not only at the
corners but also at some other positions.
Parameters of a unit cell
Depending upon the symmetry of the axial distance (a,b,c) and also the axial angle between the
edge ( α ,β and γ ) the various crystal systems on the basis of unit cells present, classified into
fourteen different types of lattices, called Bravis lattices.

α
β

γ
a

jksahu74@gmail.com 4|Page
Fraction of an atom that occupies a unit cell for various positions in the unit cell

Position of atom in unit cell Contribution of unit cell Shared by

Body centred atom ( nb ) 1 1 unit cell

1f
f
ff
Face centred atom ( nf ) 2 2 unit cell

1f
f
ff
Edge centred atom ( ne) 4 4 unit cell

1f
f
ff
Corner atom ( nc ) 8 8 unit cell

nf
f
f
bf
ff nf
f
ff
f
f nf
f
f
ef
ff nf
f
cf
f
ff
Number of atoms per unit cell or Rank ( z ) = + + +
1 2 4 8

Unit Cell nb nf ne nc Rank of unit cell ( z )

Simple cubic 0 0 0 8 0f
f
ff 0f
f
ff 0f
f
ff 8f
f
ff
+ + + =1
1 2 4 8

Body centered 1 0 0 8 1f
f 0f
f
ff 0f
f
ff 8f
f
ff
+ + + =1+ 1=2
cubic 1 2 4 8

Face centered 0 6 0 8 0f
f
ff 6f
f
ff 0f
f
ff 8f
f
ff
+ + + =3+ 1=4
cubic 1 2 4 8

Co-ordination number

The number of nearest neighbours of any constituent particle in a crystal lattice is called Co-
ordination number. In fact, the co-ordination number is the number of oppositely charged ions
surrounding each ion in ionic solids.

C.No of triangular arrangement = 3

C.No. of tetrahedral arrangement = 4

C.No. of body centered cubic arrangement =8

Any close (tight) packing having C.N. = 12 i.e. hcp and ccp i.e. fcc having C.N. = 12

For simple cubic, C.N. =6

jksahu74@gmail.com 5|Page
Different crystal systems

Crystal system Space lattice Examples


Simple :Lattice points at Body centered Points at the Face centered : Points at
the eight corners of the unit eight corners and at the body the eight corners and at the Pb, Hg, Ag, Au, Cu,
Cubic cells. centered. six face centres
a=b=c ZnS, diamond, KCl,
0
α = β = γ = 90
CsCl, NaCl, Cu2O,
a
a CaF2 and alums, etc.
a
Simple :Lattice points at the eight corners of Body centered: Points at the eight corners
the unit cells. and at the body centered. SnO2, TiO2, ZnO2,

Tetragonal NiSO4, ZrSiO4,


a=b≠c
α = β = γ = 90
0 c PbWO4, White Sn
a
a
etc.

Simple :Lattice End centered : Also Body centered: Points Face centered :
points at the eight called side centered or at the eight corners and Points at the eight
corners of the unit base centered. Points at at the body centered. corners and at the KNO3, K2SO4,
cells. the eight corners and at six face centres
two face centres opposite PbCO3, BaSO4,
to each other.
Orthorhombic rhombic sulphur,
a≠b≠c
α = β = γ = 90
0 c MgSO4.7H2O

b
a

jksahu74@gmail.com 6|Page
Simple :Lattice points at the eight corners of the unit cells.

α NaNO3, CaSO4,
Rhombohedral
a=b=c calcite, quartz,
0
α = β = γ ≠ 90
As, Sb, Bi, etc.

a
a a

Simple : Points at the twelve corners of the hexagonalprism and at the


centres of the two hexagonal faces.
Hexagonal ZnO, PbS, CdS, HgS,
a=b≠c
α = β = 90
0 graphite, ice,
γ =120
0 c
Mg, Zn, Cd, etc.

120o a
a
Simple :Lattice points at the eight End centered : Points at the eight
corners of the unit cells corners and at two face centres Na2SO4.10H2O
Monoclinic opposite to each other.
a≠b≠c Na2B4O7.10 H2O
0
α = γ = 90
0 c CaSO4.2 H2O
β ≠ 90
Monoclinic sulphur
β b
a

jksahu74@gmail.com 7|Page
Simple :Lattice points at the eight corners of
the unit cells CaSO4. 5 H2O

Triclinic K2Cr2O7
a≠b≠c
0
α c
α ≠ β ≠ γ ≠ 90
β γ H3BO3 , etc.
b
a

Packing efficiency

Packing efficiency = f
f
ff
f
ff
f
ff
f
ff
f
ff
f
ff
f
ff
f
ff
f
ff
f
ff
f
ff
f
ff
f
ff
f
ff
f
ff
f
ff
f
ff
f
ff
f
ff
f
ff
ff
f
ff
f
ff
f
ff
f
ff
fff
ff
f
ff
f
ff
f
ff
f
ff
f
ff
f
ff
f
ff
f
ff
f
ff
f
ff
f
ff
f
ff
f
ff
f
ff
f
ff
ff
f
ff
f
ff
f
ff
f
ff
f
ff
f
ff
ff
f
ff
f
ff
f
ff
f
ff
Volume occupied by all the spheres in a unit cell
B 100
Volume of unit cell

Property Simple cubic Body centered cubic Face centered cubic


w
w
w
w
ww
w w
w
w
w
ww
w
Edge length and radius a = 2r a p3 = 4r a p2 = 4r

Volume occupied by spheres 4f


f
ff 3 4f
f
ff 4f
f
ff 3
1 B πr 2 B πr 3 4 B πr
3 3 3
w
w
w
w
ww
w c3
a = 2 p2 r
3 ` a3 h i3 b
Volume of unit cell 3
f
f
ff
f
ff
f
ff
f
f
a = 2r = 8r 4r
3
3
=j w
w
ww
w
wwk
p3
a

1 B fffffπr 2 B fffffπr 3 4 B fffffπr 3


Packing efficiency 4 3 4 4
f
f
ff
f
ff
f
ff
f
f
3f
f
ff
f
ff
f
ff
f
ff
f
ff f
f
ff
f
ff
f
fff
f
f
3ff
f
ff
ff
ff
f
ff
f
ff
f f
f
ff
f
ff
f
ff
f
ff
f
f
3f
ff
f
ff
f
ff
f
ff
f
ff
f
f
B100 = 52.4% B100 = 68 % b w w
ww
w
w
w c3 B 100 = 74%
2 p2 r
3 f g3
f
ff
f
ff
f
ff
ff
f
ff
8r 4r
w
ww
w
w
w
p3

Free space per unit cell 100 – 52.4 = 47.6 % 100 – 68 = 32 % 100 – 74 = 26 %

jksahu74@gmail.com 8|Page
Characteristics of Cubic lattices

Property Simple Cubic BCC FCC

Number of lattice points involved 8 9 14

Unit Cell Volume a3 a3 a3

Lattice points per cell 1 2 4

Nearest Neighbor Distance a √3


2 √2

Number of Nearest Neighbors 6 8 12


Or Co-ordination number
Second Nearest Neighbor Distance a√2 a a

Number of Second Nearest Neighbors 12 6 6

Density of Unit cell

zB fffffffff
M

Density of unit cell d = ff


f
ff
f
ff
f
ff
f
ff
f
ff
f
ff
f
ff
ff
f
ff
ff
f
ff
f
ff
f
ff
f
ff
ff
f
ff
f
ff
f
ff
f
f f
f
ff f
= ff
f
ff
f
ff
f
ff
f
ff
f
ff
f
ff
f
ff
f
ff
f
ff
f
ff
f
ff
f
fff
f
ff
f
ff
f
ff
f
ff
f
ff
f
ff
f
ff
f
ff
f
ff
f
ff
f
ff
= ff
f
ff
f
ff
ff
fff
fff ff
f f
f
ff
f
ff
f
ff
f
ff
f
ff
f
` a Mass of unit cell zBMass of one atom N A
zM
3 3
= 3
volume of unit cell a a a NA

Z: Number of atoms per unit cell , M: Molar mass of the element ,

a : Edge length of unit cell in centimeter , NA : Avogadro number

Three dimensional Close packing of spheres

Hexagonal close packing (hcp) Cubic close packing (ccp) or fcc

jksahu74@gmail.com 9|Page
Octahedral voids Tetrahedral voids

It refers to an empty space surrounded by six It refers to an empty space surrounded by four
spheres which lie at the vertices of a regular spheres which lie at the vertices of a regular
octahedron. C.N = 6 tetrahedron. C.N =4

There are six octahedral voids around each There are eight tetrahedral voids around each
sphere. sphere.

Radius ratio = 0.414 Radius ratio = 0.225.

The number of octahedral voids belonging to a . The number of tetrahedral voids belonging to a
sphere in close packing arrangement. sphere in close packing arrangement.

ff
f
ff
ff
f
ff
ff
f
f
Numberf
ff
f
ff
ff
f
ff
ff
ff
f
f
offf
f
ff
ff
f
ff
ff
f
ff
ff
f
ff
ff
f
ff
f
octahedralf
f
ff
ff
f
ff
ff
f
ff
ff
f
ff
ff
f
f
voidsf
ff
f
ff
ff
ff
f
ff
ff
f
ff
ff
f
f
aroundf
ff
f
ff
ff
f
f
af
ff
f
ff
ff
f
ff
ff
f
ff
f
spheref
f
ff
ff
f
ff
ff 6f
f
f ff
f
ff
ff
f
ff
ff
f
f
Numberf
ff
f
ff
ff
f
ff
ff
f
f
off
ff
f
ff
ff
f
ff
ff
f
ff
ff
f
ff
ff
f
ff
ff
f
f
tetrahedralf
ff
f
ff
ff
f
ff
ff
f
ff
ff
f
ff
f
voidsf
f
ff
ff
f
ff
ff
f
ff
ff
f
ff
f
aroundf
f
ff
ff
f
ff
ff
af
f
ff
ff
f
ff
ff
f
ff
ff
f
f
spheref
ff
f
ff
ff
f
f 8f
f
ff
= = =1 = = =2
Number of sphere around a void 6 Number of sphere around a void 4

O : Octahedral voids T : Tetrahedral voids

jksahu74@gmail.com 10 | P a g e
In a ccp structure, there is In ccp structure, there are
1 octahedral void in the 8 tetrahedral voids.
centre of the body and 12
octahedral void on the In close packed structure,
edges. Each one of which there are eight
is common to four other
unit cells. Thus, in cubic spheres in the corners of
close packed structure. the unit cell and each

Octahedral voids in the centre of the cube =1 sphere is in contact with three groups giving rise
Effective number of octahedral voids located at
1f
f
ff to eight tetrahedral voids.
the 12 edge of = 12 × =3
4
∴ Total number of octahedral voids = 4

(i) Derivation of the relationship between the radius (r) of the octahedral void and the
radius (R) of the atoms in close packing

( √2 1.414 & √3 1.732

A sphere into the octahedral void is shown in the diagram. A sphere


above and a sphere below this small sphere have not been shown in
the figure. ABC is a right angled triangle. The centre of void is A.

Applying Pythagoras theorem.

BC2 = AB2 + AC2 2R2 = (R + r)2

(2R)2 = (R + r)2 + (R + r)2 √2R = R + r

4R2 = 2(R + r)2 r = √2R – R = (1.414 –1)R

ff
f
ff
ff
f
ff
ff
2
4R r = 0.414 R
= (R + r)2
2

(ii) Derivation of the relationship between radius (r) of the tetrahedral void and the radius
(R) of the atoms in close packing:

To simplify calculations, a tetrahedral void may be represented


in a cube as shown in the figure. In which there spheres form
the triangular base, the fourth lies at the top and the sphere
occupies the tetrahedral void.

Let the length of the side of the cube = a

From right angled triangle ABC, face diagonal


w
w
w
w
ww
w
w
w
ww
w
w
w
ww
w
w
w
ww
w
w
w
ww
w
w
w
ww
w
w
w
ww
w
w
w
ww
w
w
w
ww
w
w
w
w w
w
w
w
ww
w
w
w
ww
w
w
w
ww
w
w
w
ww
w
w
w
ww
w
w
w
ww w
w
w
w
ww
w
AB = qAC + BC = qa + a = a p2
2 2 2 2

As spheres A and B are actually touching each other, face diagonal AB = 2R

jksahu74@gmail.com 11 | P a g e
∴ 2R = a√2 => a = R√2
ww
w
w
w
ww
w
w
w
ww
w
w
w
ww
w
w
w
ww
ww
ww
w
w
w
ww
w
w
w
ww
w
w
w
ww
w
w
w
ww w
w
w
w ww
w
w
ww
w
w
w
ww
w
w
w
w
b w
w
w
w
w
ww
w
w
w
w
w
w
w
w
w
w
w
w
w
w
w
w
w
ww
w
w
w
w
wc2
w
w
ww
w
w
w
ww
w
w
w
ww
w
w
w
w
w
w
w
w
ww
w
Again from the right angled triangle ABD AD q
= AB + BD = a 2 + a = a p3
2 2 s p 2

But as small sphere (void) touches other spheres, evidently body diagonal AD = 2(R + r).

∴ 2(R + r) = a√3 = R√2 √3

ww
w
w
w
ww
p3
f
ff
f
ff
f
ff
f
ff ff
f
ff
f
ff
f
ff
f
f
1.732f
f
ff
f
f
R + r = ww
w
w
w
ww R =
p2
R = 1.225 R
1.414
1+ f
rf
f
ff
= 1.225
R
rf
f
ff
f
= 0.225
R

=> r = 0.225 R

(iii) Derivation of the relationship between radius (r) of the cubic body centered void and
the radius (R) of the atoms in close packing:

In Simple cubic , the corner atoms along a side touch each


other. So a = 2R

The cubic body centre void touch each sphere.


f
f
ff
f
ff
f
ff
f w
w
w
w
ww
w w
w
w
w
ww
w
rf
f
ff
f p
= a p3
` a
So BC = 2 R + r 1+ = 3 = 1.732
w
w
w
w
ww
w
= 2 R p3
R
# f
w
w
w
w
ww
w rf
f
ff
R + r = R p3
` a = 0.732
=> R

Radius ratio in ionic crystal

The ratio of radius of the smaller sphere (Cation) with that of the larger (anion) is called the radius
b c
+
ff
f
ff
f
ff
f
ff
f
ff
f
ff
f
ff
f
ff
f
ff
f
ff
f
ff
f
ff
f
ff
f
ff
f
ff
f
ff
f
ff
f
ff
f
ff
f
f
Radius of cation rf
f
ff
f
ff
f
ff
ratio. Radius ratio in ionic crystal = ` @a
Radius of anion r

S.No. Types of packing Co-ordination number Radius ratio Example

1 Tetrahedral Void 4 0⋅225 − 0⋅414 ZnS

2 Octahedral Voild 6 0⋅414 − 0⋅732 NaCl

3 Cubic body centred void 8 0⋅732 − 1⋅00 CsCl

High pressure increases the co-ordination number and high temperature decreases the co-
ordination number. Thus high pressure converts NaCl structure into CsCl structure and high
temperature converts CsCl into NaCl.
jksahu74@gmail.com 12 | P a g e
Why is co-ordination number of 12 not found in ionic crystals?

Maximum radius ratio in ionic crystals lies in the range 0.732 – 1 which corresponds to a
coordination number of 8. Hence coordination number greater than 8 is not possible in ionic
crystals.

Structure of Ionic Crystals

Types Anion Cation Coordination

NaCl fcc All octahedral voids 6:6

CsCl SC Body centre 8:8

ZnS fcc Alternate tetrahedral voids 4:4

CaF2 SC Alternate body centre 8:4

Na2O fcc All tetrahedral voids 4:8

DIMENSIONAL PROPERTIES OF HEXAGONAL UNIT CELL

In hexagonal unit cell, each corner atom shared by 6 hexagonal


unit cell, each face shared by 2 unit cell and inside the body
there are three atoms.

∴ The rank of hexagonal unit cell (z)

1f
f
ff
` a 1f
f
ff` a b c
= 12 corners + 2 faces + 3 inside the body = 6
6 2

1
a a
6 2

5 3
a 4

Sphere m and y are black colour but sphere Z is white ( Layer A is Black & Layer B is white )

hf
f
ff
ZX = sphere m, y and z touch each other; So length of yn = r ;
2

Angle ∠ n y x = 300

jksahu74@gmail.com 13 | P a g e
w
w
w
w
ww
w
w
w
ww
w
w
w
ww
w
w
w
ww
w
w
w
ww
w
w
w
ww
w
w
w
ww
w
w
w
ww
w
w
Intriangle ∆xyz, zx = qyz @ yx
2 2

v
ww
w
w
w
ww
w
w
w
ww
w
w
w
ww
w
w
w
ww
w
w
w
ww
w
w
w
ww
w
w
w
ww
w
w
ww
w
w
w
ww
w
w
w
ww
w
ww
w
w
ww
w
w
ww
w
So in triangle ∆ ynx, Cos 30 = ff
f
ff
f
f u
yn v
w
ww
w
w
w
ww
w
u u
0 h i2
u` a2 u2f
=u jff
ff
f
ff
f
ff
f
f
= 2r t f
ff
2r
w
ww
w
w
wk
yx
w
ww
w
w
ww t 2r @ p
p3
f
ff
f
ff
f
ff
f
ff f
rf
f
ff
f
f
3 3
v
w
ww
w
w
w
ww
w
=> = u
hf
f
ff u2f
f
ff
2r t
2 yx => =

= > yx = f
ff
f
ff
f
ff
f
ff
2r 2 3
w
ww
w
w
ww v
w
ww
w
w
w
ww
w
u
p3 u2f
= > h = 4r tfff
3

The base of hexagonal unit cell consists of six equilateral triangles having side length “a”
w
ww
w
w
ww
p3
f
ff
f
ff
f
ff
f
ff2
The area of triangle = a
4
ww
w
w
w
ww ww
w
w
w
ww
p3 p3 ` w
w
w
w
ww
w
f
ff
f
ff
f
ff
f
ff2 f
ff
f
ff
f
ff
f
ff
B 2r = 6 p3 r
a2 2
Therefore Area of hexagon = 6 B a = 6B
4 4

Volume of hexagonal unit cell = (Area of the hexagonal base) × (Height of the hexagonal unit cell)
v
w
ww
w
w
w
www
ww
w
w
w
ww2 u w
ww
w
w
w
w 3
u 2f
f
ff
= 6 p3 r B4r t = 24 p2 r
3
f
ff
f
ff
f
ff
f
ff
f
ff
f
f
Volumef
f
ff
f
ff
f
ff
f
ff
f
f
off
ff
ff
f
ff
f
ff
f
ff
f
f
totalf
ff
f
ff
f
ff
f
ff
f
ff
f
ff
f
f
numberf
f
ff
f
ff
f
ff
f
ff
f
f
off
ff
f
ff
f
ff
f
ff
f
ff
f
ff
f
ff
f
f
spheresf
f
ff
f
ff
f
ff
ff
f
f
inf
ff
ff
f
ff
f
f
thef
f
ff
f
ff
f
ff
f
ff
f
f
unitf
f
ff
f
ff
f
ff
f
ff
f
f
cell f
Packing efficiency = B 100
Volume of unit cell
6 B fffffπr
4 3

f
f
ff
f
ff
f
ff
f
ff
3f
f
ff
f
ff
f
ff
f
ff
f
ff
f
f
ww
ww
w
w
w 3 B100
24 p2 r
=

4f
f
ff f
f
ff
f
f1f
f
ff
f
ff
f
ff
f
f
πB w w
ww
w
w
wB 100
p
=
3 4 2
ff
f
ff
f
πf
f
ff
f
ff
f
ff
f
f ff
ff
f
ff
f
f
3.14f
f
ff
f
ff
f
f
Bf
f
ff
f
ff
f
ff
f
f
100f
f
ff
f
f ff
f
ff
f
ff
f
f
314f
f
ff
f
ff
f
ff
= w w
ww
w
w
wB100 =
p
= = 74 %
3 2 3 B1.414 4.242

IMPERFECTIONS IN SOLIDS: DEFECTS IN CRYSTALS

When deviations exist from the regular (or periodic) arrangements around an atom or a group of
atoms in a crystalline substance, the defects are called point defects.

Type of point defects – point defects in a crystal may be classified into three types

(a) Stoichiometric defects (b) Non – stoichiometry defects (c) Impurity defects

Stoichiometry defect

The defects that do not disturb the ratio of cations and anions as represented by the molecular
formula, are called stoichiometric defect.
jksahu74@gmail.com 14 | P a g e
Schottky defect Frenkel defect

It is due to missing of equal number of cations and anions from the It is due to missing of cations from the lattice sites, and these
lattice sites. cations occupy the interstitial sites.

A+ B− A+ B− A+ B− A+ B−

B− A+ A+ B− A+ B− A+
+
A
B − A+ B−
B −
A+ B−

B− A+ B− A+ B− A+ B− A+

This results in the decrease in the density of the crystal. It has no effect on the density of the crystal

This type of defect is shown by highly ionic compounds which have This type of defect is present in ionic compounds which have low
high Co – ordination number and small difference in the sizes of co ordinations number and larger difference in size of cation and
cations and anions anions
NaCl, KCl, KBr and CsCl.etc. AgCl, AgBr, AgI, ZnS etc.

It is vacancy defect It is interstitial defect


AgBr shows both Schottky and Frenkel defect.

Non - Stoichiometry defect

Metal excess defects


Metal excess defects due to anion vacancies When there is an excess of metal ions in non-stoichiometric compounds, the crystal lattice
( F- Centre) has vacant anion site. The anion sites occupied by electrons are called F-centre(Farbe :
+ -
x + -
x
means colour)
Anion vacancy occupied

M M
The F-centers are associated with the cololur of the compounds.
by electron ( F-centre)

- + + Excess of K in KCl makes the crystal violet. Excess of Li in LiCl makes the crystal pink.
x M - M

Solid containing F-centre are paramagnetic, because the electrons occupying the F-centers
+
- + - are unpaired.
M x M x

- +
- + When the crystal having F-centers are exposed to light, they become photo conductor.
x M x M

jksahu74@gmail.com 15 | P a g e
Metal excess defects due to interstitial cations The extra cation occupying the interstitial site and in order to maintain electrical neutrality an
Zn2+ O2− Zn2+ O 2−
electron present in another interstitial site.

Zinc oxide is white in colour at room temperature on heating; it looses oxygen and turns
O2− Zn2+ O2− Zn2+ yellow.
Zn2+ Heat 1
2+ O2−
ZnO Zn2+ + O2
+ 2 e-
Zn O 2−
O 2− 2
2e−
Now there is excess of zinc in the crystal and its formula becomes Zn 1 + x O
O2− Zn2+ O2− Zn2+ The excess zinc ions move to interstitial site and the electrons to neighbouring interstitial site.

Metal deficiency defects


Metal deficiency due to cation vacancies The non-stoichiometric compounds may have metal deficiency due to the absence of a metal
Fe2+ O 2− Fe3+
from its lattice site. The charge is balanced by an adjacent ion having higher positive charge.
O 2−
This type of defects are generally shown by compounds of transition elements.

O2− O 2− Fe2+ FeO, mostly found with a composition of Fe0.95 O. In crystals of FeO, some Fe2+ ions are
missing and the loss of positive charge is made up by the presence of required number of Fe3+
ions. Holes are created.
Fe2+ O 2− Fe3+ O2−

O2− Fe2+ O 2− Fe2+

Point defects due to the presence of foreign atoms


These defects arise when foreign atoms are present at the lattice site (in place of host atoms) or at the vacant interstitial sites.. The introduction
of defects in a particular crystalline solid by the addition of other elements is known as doping.
Doping increases the conductivity of crystal. For example, if we mix strontium chloride (SrCl2) with sodium chloride, some strontium (Sr2+) ions
occupy the lattice sites of sodium ions (Na+) and equal number of sodium (Na+) sites remain vacant. Such vacancies in the crystal increase the
electrical conductivity because certain ions from the neighbouring sites can move into these vacant holes. In this defect the number of positive
ions is less as compared to negative ions. Crystals with such defects also act as semiconductor. Since the conductivity is due to holes, these are
known as P-type semiconductors.

jksahu74@gmail.com 16 | P a g e
ELECTRICAL PROPERTIES

Electrical conductivity of solids may arise through the motion of electrons and positive holes
(electronic conductivity) or through the motions of ions (ionic conductivity). The conduction through
electrons is called n-type conduction and through positive holes is called p – types conduction.
Electrical conductivity of metal is due to motion of electrons and it increases with the number of
electrons available to participate in the conduction process. Pure ionic solids where conduction
can take place only through motion of ions are insulators. However, the presence of defects in the
crystal structure increases their conductivity.

Electrons sea model of metallic bonding

A metal consists of a lattice of positive ions (Kernel) immersed in a sea of valence electrons
(mobile electrons). The force of attraction between the mobile electrons and the kernels is known
as metallic bond. The electrical and thermal conductivity of metals can be explained by the
presence of mobile electrons in metal.

The Band Model of Metallic Bonding

The band model of metal is based on molecular orbital theory.


When a large no. of orbital overlap in metal, it results a continuous energy level produced by a
large number of molecular orbital is called energy band.

Coduction band
2p0 2p band ( conduction band )
111-63
511 Kj/mol

Kj/mol

2s2 2s- band ( valence band )

Valence band [ conductor ] 1s2 1s- band


[ Semi conductor ]
[ Insulator ]

The lowest unoccupied energy band is known as conduction band.


The highest occupied energy band is known as valence band.
The energy difference between the top of valence band and the bottom of the conduction band is
known as energy gap.
On the basis of electrical conductivity the solids can be classified into three types
(a) Metal (conductors): They allow the maximum portion of the applied electric field to flow through
them and have conductivities in order of 106 – 108 ohm-1.
(b) Insulators: They have low conductivities i.e. they do not practically allow the electric circuit to
flow through them. The electrical conductivity is in order 10-10 – 10-20 ohm-1 m-1
(c) Semi conductors: The solids with intermediate conductivities at the room temperature. Semi
conductors allow a portion of electric current to flow through them.
Actually semi conductors are those solids which are perfect insulators at absolute zero, but
conduct electric current at room temperature.

jksahu74@gmail.com 17 | P a g e
(1) Intrinsic semi conductors (semi-conductors due to thermal defects)
At zero Kelvin pure substance silicon and germanium act as insulators
because electrons fixed in covalent bonds are not available for
conduction. However at higher temperature some of the covalent bonds
are broken and the electrons so released become free to move in the
crystal and thus conduct electric current. This type of conduction is known
as intrinsic conduction as it can be introduced in the crystal without
adding an external substance.

(2) Extrinsic semi conductors: (semi conductors due to impurity defects)

The conductivity of pure silicon and germanium is very low at room temperature. The conductivity
of silicon and germanium can be increased by doping with impurities producing n-type
semiconductors or p – type semi conductors .

N-type semiconductors

It is obtained by doping group – 14 elements with group – 15


elements. Suppose Si is doped with Sb with 5 valence electrons,
out of 5 valence electrons, only 4 valence electrons are involved
in bond formation. The fifth electron is not bound anywhere and
can be easily promoted to the conduction band. The conduction
is thus mainly caused by the movement of electrons.

P-type semiconductors

It is obtained by doping group – 14 elements with group – 13


elements. Suppose Si is doped with B which has 3 valence
electrons, 3 valence electrons are involved in bond formation
with neighbouring Si atom. A vacancy is left which can be filled
by the transfer of a valence electron from a neighboring Si atom.
The movement of electron into the vacancy leaves behind a hole
which carries positive charge. Another electron from a
neighbouring Si atom can move into the hole leaving behind
another hole. It appears as if the hole has moved through the
lattice. The movement of positively charged hole is responsible for the conduction of charge.

Periodic Table of Semiconductors

Elements Group 15
Elements Group 13 Elements Group 14
5-Electrons in Outer Shell
3-Electrons in Outer Shell 4-Electrons in Outer Shell
(Negatively Charged)
(Positively Charged) (Neutrally Charged)
(6) Carbon ( C )
(5) Boron ( B )
(15) Phosphorus ( P )
(13) Aluminium ( Al ) (14) Silicon ( Si )
(33) Arsenic ( As )
(31) Gallium ( Ga ) (32) Germanium ( Ge )
(51) Antimony ( Sb )

jksahu74@gmail.com 18 | P a g e
MAGNETIC PROPERTIES

The magnetic properties of different materials are studies in terms of their magnetic moments which arise due to the orbital motion and spinning
motion of the electron. Due to the magnetic moment of the electrons different substances behave differently towards the external applied
magnetic field. Based on the behavior in the external magnetic field, the substances are divided into different categories.

A region within a magnetic material which has uniform magnetization (the individual magnetic moments of the atoms are aligned with one
another and they point in the same direction) is called magnetic domain.

Properties Description Alignment of magnetic dipoles Examples Applications


Diamagnetic Feebly repelled by the magnetic fields. Non- All paired electrons TiO2, V2O5, NaCl, Insulator
metallic elements (excepts O2, S) inert gases and
species with paired electrons are diamagnetic C6H6 (benzene)

Paramagnetic Substances which are weakly attracted by At least one unapired electron O2, Cu2+, Fe3+, Electronic
magnetic field are called paramagnetic appliances
substances. These substance have permanent TiO, TiO3, VO,
magnetic dipoles due to the presence of some
species (atoms, ions or molecules) with unpaired VO2, CuO
electron. The paramagnetic substances lose their
magnetism in the absence of magnetic field

Ferromagnetic Substances which show permanent magnetism Fe, Ni, Co, CrO2 CrO2 is
even in the absence of the magnetic field are used in
called Ferromagnetic substances. Such audio and
substances remain permanently magnetised, video tapes
once they have been magnetised. This type of
magnetism arises due to spontaneous alignment
of magnetic moment due to unpaired electrons in
the same direction.

jksahu74@gmail.com 19 | P a g e
Antiferromagnetic This arises when the dipole alignment is zero MnO, MnO2,
due to equal and opposite alignment.
( Substances which are expected to possess Mn2O, FeO,
paramagnetism or Ferromagnetism on the
basis of unpaired electron but actually they Fe2O3, NiO,
posses zero net magnetic moment)
Cr2O3, CoO,

Co3O4.
Ferrimagnetic Substance which are expected to posses
large magnetism on the basis of the unpaired
electrons but actually have small net magnetic
moments are called Ferrimagnetic substances
( Unequal number of opposite dipole Fe3O4, ferrites
alignment)

12 – 16 and 13 – 15 compounds

Combination of elements of Gr – 13 and Gr – 15 or Gr – 12 and Gr – 16 produce compounds which stimulate average valence of four as in

Ge or Si.

12 – 16 compounds →ZnS, CdS, CdSe, HgTe

13 – 15 compounds → InSn, AlP, GaAs

jksahu74@gmail.com 20 | P a g e
NUMERICALS ON SOLID STATE
=========================================================================
ww
w
w
w
w w
w
w
ww
w
p 2 = 1.414 and p3 = 1.732

==========================================================================
(1) What type of crystal system is with cell dimensions a = 0.328 nm, b = 0.328nm,
0
c = 0.527nm and α = β = γ = 90

0
Since a = b ≠ c and α = β = γ = 90 so the crystal is tetragonal crystal system.

(2) What is the total volume of atoms in a face-centred cubic unit cell of a metal
( r in atomic radius)?

Number of atoms per unit cell in fcc = 4


4f
f
ff 3 16
ff
ff
ff
f
f 3
# Total volume of atoms present in fcc unit cell = 4 B πr = πr
3 3

(3) Potassium crystallizes in body centred cubic lattice. Calculate the approximate number
of unit cells in 1 g of potassium. Atomic mass of potassium is 39 u.

1 mole of potassium = 39 g = 6.022 x 1023 atoms

# No A of atoms present in 1 g of potassium = ff


f
ff
f
ff
f
ff
f
ff
f
ff
f
ff
f
ff
f
ff
f
ff
f
ff
f
ff
f
ff
f
ff
23
6.022 B10
39
A body centred cubic unit cell contains 2 atoms

# No A of unit cells present = f


f
ff
f
ff
f
ff
ff
f
ff
f
ff
f
ff
f
ff
f
ff
f
ff
f
ff
f
ff
f
ff
f
f
23
6.022 B10 21
= 7.72 B10
39 B2

(4) In a crystalline solid, anions B- are arrange in a cubic close packing. Cations A+ are
equally distributed between octahedral and tetrahedral voids. If all the octahedral voids
are occupied , what is the formula of the Solid?

Suppose the number of anions B- = N

Then total number of octahedral voids = N

Number of tetrahedral voids = 2N

As octahedral and tetrahedral voids are equally occupied by cations A+ and all the octahedral
voids are occupied ( given), Therefore N cations are present in octahedral voids and N cations
are present in tetrahedral voids.

In other words , corresponding to N anions B-, there are N + N = 2N cations A+.

Thus , cations A+ and anions B- are in ratio 2N : N = 2:1

Hence the formula of the solid will be A2B.

jksahu74@gmail.com 21 | P a g e
(5) The pyknometric density of sodium chloride crystal is 2.165 x 103 kg m-3 while its X-ray
density is 2.178 x 103 kg m-3. What is the fraction of the unoccupied sites in sodium
chloride crystal?

Molar volume from pyknometric density = ff


ff
f
ff
f
ff
f
ff
f
ff
f
ff
f
ff
f
ff
f
ff
f
ff
f
fff 3
M
3
m
2.165 B10

Molar volume from X- ray density = ff


ff
f
ff
f
ff
f
ff
f
ff
f
ff
f
ff
f
ff
f
ff
f
ff
f
fff 3
M
3
m
2.178 B10
f g
Therefore volume unoccupied = ff
f
ff
f
ff
ff f
f
ff
f
1f
ff
ff
f
ff
f
ff
f ff
f
ff
1f
ff
ff
f
ff
f
ff
f 3 0.013
f
f
ff
f
ff
f
ff
f
ff
f
ff
f
ff
f
ff
f
ff
f
ff
f
ff
f
ff
f
ff
ff
f
ff
f
ff
f
ff
f
ff
f
@3
M M B 10
3
@ m =
10 2.165 2.178 2.165 B 2.178

ff
f
ff
f
ff
f
ff
f
ff
f
ff
f
ff
f
ff
f
ff
f
ff
f
ff
f
ff
f
ff
f
ff
f
ff
f
ff
f
ff
f
@3
0.013 M B10
ff
ff
ff
f
ff
ff
ff
f
ff
ff
ff
ff
f
ff
f
2.165B2.178f
ff
ff
f
ff
ff
ff
f
Therefore fraction unoccupied = = 5.96 x 10-3
f
f
ff
f
ff
f
ff
f
ff
f
ff
f
ff
f
ff
f
ff
f
ff
f
ff
@3
M B10
2.165

(6) The edge length of unit cell of a metal having molecular weight 75 g/mol is 5 A0 which
crystallizes in cubic lattice. If the density is 2 g/cc, then find the radius of metal atom. (
NA= 6 x 1023 ) Give the answer in pm.
b cb c3 b c
@8 23 @1
f
f
ff
ff
f
ff
f
ff
f
ff
f
ff
f
ff
f
ff df
f
ff
f
ff
f
ff
f
ff
f
fff
f
ff
f
ff
f
ff
f
ff
f
ff
f
ff ff
zB M Ba 3
B N 2f
f
ff
f
ff
f
ff
f
f
gcmf
f
ff
f
ff
f
ff
f
f
@3
f
ff
f
ff
f
f
5f
f
ff
f
f
Bf
f
ff
f
ff
ff
f
f
10f
f
ff
f
ff
f
ff
f
ff
f
f
cmf
f
ff
f
ff
f
ff
f
ff
f
ff
6f
f
ff
f
ff
f
ff
f
f
B10f
f
ff
f
ff
f
ff
f
ff
f
ff
f
ff
f
f
molf
f
ff
f
ff
f
ff
f
ff
f
f
A
d= 3 [ z= = @1
=2
a BNA M 75 g mol

For a metal with z = 2 means that it has body centred cubic structure.
w
ww
w
w
w ww
w
w
w
w
ww
w
w
w
w p
f
ff
f
f
3f
f
ff
f
f p
f
ff
f
f
3f
f
ff
f
f
= a p3
0 0
So 4r [ r= a= B 5 A = 2.165 A = 216.5 pm
4 4

(7) The metal calcium crystalises in a face centred cubic unit cell with a= 0.556 nm.
Calculate the density of the metal if it contains
(i) 0.2 % Frenkel defects (ii) 0.1 % Schottky defect

For face centred cubic unit cell , Z = 4


(i) Density do not change in Frenkel defect, because the numbers of atoms per unit cell remain
unchanged.
b c
@1

d= f
f
ff
ff
f
ff
f
ff
f
ff
f
ff
f
ff
f
ff f
= bf
ff
f
ff
f
ff
f
ff
f
ff
f
ff
f
ff
f
ff
f
ff
f
ff
f
ff
f
ff
f
ff
zBM 4f
f
ff
f
f
Bf
f
ff
f
ff
f
ff
ff
f
f
40f
f
ff
f
gf
f
ff
f
ff
f
ff
f
f
molf
f
ff
f
ff
f
ff
f
ff
f
ff
f
ff
f
ff
f
ff
f
ff
ff
f
fff
f
ff
f
ff
f
ff
f
ff
f
ff
f
ff
ff
f
ff @3
3 c3 b c = 1.5463 gcm
a BN A @7
0.556 B10 cm B 6.022 B10 mol
23 @1

(ii) Due to Schottky defects, vacant spaces will decrease the number of atoms per unit cell.

f g
z= 4@ f
4f
ff
f
ff
f
ff
f
ff
f
f
B0.1f
f
ff
f
f
= 3.996
100
b c
@1

d= f
f
ff
ff
f
ff
f
ff
f
ff
f
ff
f
ff
f
ff
f f ff
f
ff
f
ff
f
ff
f
ff
f
ff
f
ff
f
ff
f
ff
f
ff
f
ff
ff
f
ff
f
ff
ff
f
ff
f
ff
f
ff
f
ff
f
ffff
f
ff
ff
f
ff
f
ff
f
zB M 3.996 B 40 gf
f
ff
f
ff
f
ff
f
f
molf
f
ff
f
ff
f
ff
f
ff
f
ff
f
ff
ff
f
fff
f
ff
f
ff
f
ff
f
ff
f
ff
f
ff
f
ff
f
ff @3
3
= b c 3 b c = 1.5448 gcm
a BN A @7
0.556 B10 cm B 6.022 B10 mol
23 @1

jksahu74@gmail.com 22 | P a g e
(8) In the mineral, “spinel” having the formula MgAl2O4 , oxide ions are arranged in the
cubic close packing, Mg2+ ions occupy the tetrahedral voids while Al3+ ions occupy the
octahedral voids.
(i) What percentage of tetrahedral voids is occupied by Mg2+ ions?
(ii) What percentage of octahedral voids is occupied by Al3+ ios?

According to the formula MgAl2O4 ,

If there are 4 oxide ions, There will be 1 Mg2+ ion and 2 Al3+ ions.
But if the 4 O2- ions are in ccp arrangement,
there will be 4 octahedral voids and 8 tetrahedral voids.
2+
Thus, 1 Mg ion is present in one of the 8 tetrahedral voids.
2+ 1f
f
ff
# % of tetrahedral voids occupied byMg = B 100 = 12.5 %
8
Similarly, 2 Al3+ ions are present in two octahedral voids out of 4 available.
# % of octahedral voids occupied byAl = f
3+ 2f
ff
B 100 = 50 %
4

(9) A metal crystallizes in to two cubic phases, face centred cubic and body centred cubic,
whose unit cell edge length are 3.5 A0 and 3.0 A0 respectively. Claculate the ratio of the
densities of FCC and BCC.

` a f
f
ff
f
ff
f
f
zBMf
f
ff
f
ff
f
ff
f
ff
f
ff f
f
ff
f
ff
f
ff
f
f4f
f
ff
f
f
Bf
f
ff
f
ff
f
f
Mf
f
ff
f
ff
f
ff
f
ff
f
ff
ff
Density of unit cell d = 3
Density of FCC unit cell = b c3
NA Ba NA B 3.5 A
0

f
f
ff
f
ff
f
ff
f
f2f
f
ff
f
ff
f
f
BMf
f
ff
f
ff
f
ff
f
ff
f
ff
f
ff
f
ff
ff
Density of BCC unit cell = b c3
0
NA B 3.0 A
b 3 c3
0
f
ff
f
ff
f
ff
f
ff
f
ff
f
ff
f
ff
f
ff
f
ff
f
ff
f
fff
f
ff
f
ff
f
ff
f
ff
f
ff
f
ff
f ff
f
ff
f
ff
f
ff
f
ff
ff
ff
f
ff
f
ff
f
ff
f
ff
f
ff
f
ff
ff
f
ff
f
ff f
Density of FCC 4 BM Nf
f
f
Af
f
ff
f
f
Bf
f
ff
f
ff
f
ff
f
ff
f
f
3.0f
f
ff
f
f
Af
f
ff
f
ff
f
ff
f
ff
f
ff
f f f
f
ff
f
ff
f
f
108f
f
ff
f
ff
f
f
= b c3 B = = 1.259
Density of BCC N B 3.5 A 0 2 BM 85.75
A

(10) An element ‘X’ with an atomic mass of 60 g mol-1 has density of 6.23 g cm-3. If the edge
length of its cubic unit cell is 400 pm, identify the type of of cubic unit cell. Calculate the
radius of an atom of this element.
` a @1 ` a @3
Atomic mass M = 60 g mol , Density of unit cell d = 6.23 g cm ,
` a @8
Edge length a = 400 pm = 4 B10 cm
b c3
@8 23
f
f
ff
f
ff
f
ff
f
ff
f
ff
f
ff
f
ff
f df
f
ff
f
ff
f
ff
f
ff
f
ff
f
ff
f
ff
f
ff
f
ff
f
ff
f
fff
f f f
f
ff
f
ff
f
ff
f
ff
f
ff
f
ff
f
ff
f
ff
f
ff
f
ff
f
ff
f
ff
f
ff
f
ff
f
ff
f
ff
f
ff
f
ff
f
ff
f
ff
f
ff
f
ff
f
ff
f
ff
f
ff
f
ff
f
ff
f
ff
f
ff
f
ff
f
ff
f
ff
f
ff
f
ff
f
ff
f
ff
f
f
zBM Ba BN
3
A
6.23 B 4 B 10 B6.022 B10
d= 3
[ z= = =4
a BNA M 60

Since z = 4 , it means that the element crystallizes in FCC

For FCC , Radius of atom and edge length of unit cell are related as
ww
w
w
w
ww f
f
ff
f
faf
f
ff
f
ff
f
ff
f
f ff
ff
f
ff
f
ff
f
ff
f
ff
f
f
4 r = a p2 [ r = w
400
w
ww
w
w
w= w
w
ww
w
w
w = 141.4 pm
2 p2 2 p2

jksahu74@gmail.com 23 | P a g e
(11) A unit cell consists of a cube in which there are atoms ‘A’ at the corners and atoms ‘B’
at the face centres. Two ‘A’ atoms are missing from the two corners of unit cell. What is
the formula of the compound?
Contribution made by an atom ‘A’ present in one corner = f
1f
f
8

Contribution made by an atom ‘A’ present at six corners = f


6f
f 3f
f
ff
=
8 4

Contribution made by an atom ‘B’ present at face centre = f


1f
f
2

Contribution made by six atoms ‘B’ present at six face centres = f


1f
f
B6 = 3
2

Formula of the compound = A 3fffffB3 = AB4


4

(12) Calculate the number of atoms in a cubic based unit cell having one atom on each
corner and two atoms on each body diagonal.

There are four body diagonal passing through the centre and connecting opposite corners.
The atoms present on these diagonals are not shared by other unit cells.

So contribution of atoms present on diagonals = 2 x 4=8

Contribution by corner atoms = 8 at corners B f


` a 1f
f
=1
8

Therefore total number of atoms in the unit cell = 1 + 8 = 9

(13) In a cubic close packed structure of mixed oxides, the lattice is made of oxide ions,
one eighth of tetrahedral voids are occupied by divalent ions (A2+) while one half of the
octahedral voids are occupied by trivalent ions (B3+). What is the formula of the oxide.

Suppose number of oxide ion per unit cell = 1

So number of octahedral voids = 1 and Number of tetrahedral voids = 2


b c
2+ 1f
f
ff 1f
f
ff
# Number of divalent ions A = 2B =
8 4
b
3+
c 1f
ff
f 1f
ff
f
Number of trivalent ions B = 1 B =
2 2

So formula of the compound = A 1fffffB1fffffO = AB2 O 4


4 2

(14) Sodium crystallises in a bcc unit cell with edge length 4.29 A0. What is the radius of the
sodium atom? What is the length of the body-diagonal of the unit cell?
w
w
w
w
ww
w
afp3
` a f
ff
f
ff
ff
ff
f
ff
f
ff
f 4.29
f
f
ff
f
ff
f
ff
f
ff
f
ff
f
f
Bf
ff
f
ff
f
ff
f
ff
f
ff
f
f
1.732f
f
ff
f
ff
f 0
For bcc unit cell , radius r = = = 1.86 A
4 4
0
Length of the body @ diagonal = 4r = 4 B 1.86 = 7.44 A

jksahu74@gmail.com 24 | P a g e
(15) In a solid XY, ‘X’ atoms are in ccp arrangement and ‘Y’ atoms occupy all the octahedral
sites. If all the face centred atoms along one of the axis are removed, then what will be
the resultant stoichiometry of the compound?

In CCP type structure, Number of atoms per unit cell = 4


So number of octahedral voids = 4
# Number of Yatoms per unit cell = 4

X atoms are present at the lattice points and all the face atoms along aone axis are removed
[ 2 X atoms are removed
Thus only 4 X atoms will be left on the centre of the face A
1f
f
ff 1f
f
ff
# Number of . X. atoms per unit cell = B8 + B4 = 3
8 2
# Stoichiometry of the compound = X 3 Y4

(16) A compound forms hexagonal close-packed structure. What is the total number of
voids in 0.5 mol of it?
23 23
Number of atoms in close packing = 0.5 mol = 0.5 B 6.022 B10 = 3.011 B10
23
Number of octahedral voids = Number of atoms in the close packing = 3.011 B10
23 23
Number of tetrahedral voids = 2 BNumber of octahedral voids = 2 B3.011 B10 = 6.022 B10
23 23 23
# Total number of voids = 3.011 B10 + 6.022 B10 = 9.033 B 10

(17) A compound is formed by two elements M and N. The element N forms ccp and atoms
of M occupy 1/3 rd of tetrahedral voids. What is the formula of the compound?

Let in the compound there are number of atom ‘N’= 1

So the number of tetrahedral voids = 2

i.e. Number of atom ‘M’ = f


1f
ff
B Number of tetrahedral voids = f
1f
ff
B2 = f
2f
ff
3 3 3

So the ratio of M:N = f


2f
ff
:1 = 2:3 [ Formula of the compound = M2 N3
3

(18) An element with molar mass 2.7 x 10-2 kg mol-1 forms a cubic unit cell with edge length
405 pm. If its density is 2.7x103 kg.m-3, what is the nature of the cubic unit cell?
` a @ 12 ` a @2 @1
Edge length a = 405 pm = 405 B10 m Molar mass M = 2.7 B10 kg mol
b c ` a
23 @1 3 @3
Avogrados number NA = 6.022 B10 mol Density d = 2.7 B10 kg m
` a ff
f
ff
f
f
zBMf
f
ff
f
ff
f
ff
f 3
Now density d = Ba
NA
b c b c b c3
3 @3 23 @ 12
df
f
ff
f
ff
f
ff
f
ff
f
ff
ff
ff
f
ff
f
ff
f
faf
f
ff
f
ff
f ff
f
ff
f
ff
f
f
2.7f
f
ff
f
ff
f
ff
f
ff
f
ff
f
ff
f
ff
f
ff
f
f
kgf
f
ff
f
f
mf
f
ff
f
ff
f
ff
f
ff
f
ff
f
ff
f
ff
f
ff
f
ff
f
ff
f
ff
f
ff
f
f
6.022f
f
ff
f
ff
ff
f
ff
f
ff
f
ff
f
ff
f
ff
f
ff
f
ff
f
ff
f
ff
f
ff
f
ff
f
ff
f
ff
f
f
405f
f
ff
f
ff
f
ff
f
ff
f
ff
f
ff
f
ff
f
ff
f
ff
f
ff
f
f
mf
f
ff
f
ff
f
ff
f
f
3 B10 B B10 B B10
BN A B
#z= = @2 @1
=4
M 2.7 B10 kg mol

Since, No. of atoms per unit cell are 4, so it must be a face centred cubic unit cell.
jksahu74@gmail.com 25 | P a g e
(19) Silver crystallises in fcc lattice. If edge length of the cell is 4.077 x 10-8 cm and density
is 10.5 cm-3, calculate the atomic mass of silver.

f
f
ff
f
ff
f
ff
f
ff
f
ff
f
ff
f
ff
f df
f
ff
f
ff
f
ff
f
ff
f
ff
f
ff
f
ff
f
ff
f
ff
f
ff
fff
f
3
zBM Ba BN A
We know that d = 3
[ M=
a BNA z
According to available data:
` a @8
Edge length a = 4.07 B10 cm
` a b c
Number of atoms per unit cell z = 4 $ of fcc structure
@3
Density of silver = 10.50 g cm
b c
23 @1
Avogadro. s Number NA = 6.022 B10 mol

b c ` a3 b c b c3
@3 23 @1 @8
f
f
ff
f
ff
f
ff
f
ff
f
ff
f
ff
f
ff
f
ff
f
ff
f
ff
f
ff
f
ff
f
ff
f
ff
f
ff
f
ff
f
ff
f
ff
f
ff
f
ff
f
ff
f
ff
f
ff
f
ff
f
ff
f
ff
f
ff
f
ff
f
ff
f
ff
f
ff
ff
f
ff
f
ff
f
ff
f
ff
f
ff
f
ff
f
ff
f
ff
f
ff
f
ff
f
ff
f
ff
f
ff
f
ff
f
ff
f
ff
f
ff
f
ff
f
ff
f
ff
f
ff
f
ff
f
ff
f
ff
f
ff
f
ff
f
ff
f
ff
f
ff
f
ff
f
ff
f
ff
f
ff
f
ff
f
ff
` a 10.50 gcm B 4.077 B 6.022 B10 mol B 10 cm
# Atomic mass of the element M =
4
@1
= 107.12 g mol

(20) A cubic solid is made of two elements P and Q. Atoms of Q are at the corners of the
cube and P at the body centre. What is the formula of the compound? What are the
coordination number of P and Q?

Number of P atoms per unit cell = 1(within the body) x 1 = 1

Number of Q atoms per unit cell = 8(at corners) B f


1f
f
=1
8

So the formula of the compound is PQ

For body centre cubic, coordination numbers of both P and Q are 8

(21) Niobium crystallises in body centred cubic structure. If density is 8.55 g cm-3, Calculate
atomic radius of Niobium using its atomic mass 93 u.
@3 ` a @1
d = 8.55 g cm , z = 2 bcc , M = 93 g mol

d= f
f
ff
f
ff
f
ff
f
ff
f
ff
f
ff
f
ff
f
[ a3 = f
f
ff
f
ff
f
ff
f
ff
f
ff
f
ff
f
f f
= f
f
ff
f
ff
f
ff
f
ff
f
ff
f
ff
f
ff
f
ff
f
ff
f
f2f
f
ff
f
ff
ff
f
ff
f
ff
f
ff
f
ff
f
fgf
f
ff
f
ff
f
ff
f
ff
f
ff
f
ff
f
ff
f
ff
f
ff
f
ff
f
ff
f
fff
f
ff
f
ff
f
ff
f
ff
f
ff
f
ff
f
f
@1
zBM zBM B 93 mol
3 @3 23 @1
= 36.13 B10@ 24 cm 3
a BNA d BNA 8.55 g cm B6.022 B10 mol

b c 1fffff
@ 24 3 3 @8
a = 36.13 B10 cm = 3.31 B10 cm
w
w
ww
w
w
w
w
ww
w
w
w
w afp3
p3 f
ff
f
ff
f
ff
f
ff
f
ff
f
f f
ff
f
ff
f
ff
f
ff
f
ff
f
ff
f
ff
f
ff
f
ff
f
ff
f
ff
f
ff
f
ff
f
ff
f
ff
f
ff
f
ff
f
ff
f
ff
f
ff
f
ff
f
ff
f
ff
f
ff
f
ff
f
ff
f
f
@8
3.31 B10 cm B1.732 @7
In bcc , 4r = a [r= = = 14.29 B10 cm = 14.29 nm
4 4

(22) Copper crystallises into a fcc lattice with edge length 3.61 x 10-8 cm. Show that the
calculated density is in agreement with its measured value of 8.92 g cm-3.
@8 ` a @1
a = 3.61 B10 cm, z = 4 fcc , M = 63.5 g mol

jksahu74@gmail.com 26 | P a g e
f
f
ff
f
ff
f
ff
f
ff
f
ff
f
ff
f
ff
f fff
f
ff
f
ff
f
ff
f
ff
f
ff
f
ff
f
ff
f
ff
f
ff
f
ff
f
ff
f
ff
f
ff
f
ff
f
ff
f
ff
f
ff
f
ff
f
ff
f
ff
f
ff
f
ff
gf
f
ff
f
ff
f
ff
f
ff
f
ff
f
ff
f
ff
f
ff
f
ff
f
ff
f
ff
f
ff
f
ff
f
ff
f
ff
f
ff
f
ff
f
ff
f
ff
f
ff
f
ff
f
f
@1
zBM 4 B63.5 mol @3
d= 3
=b c3 b c = 8.96 g cm
a BNA 3.61 B10
@8
cm B 6.022 B10 mol
23 @1

Thus , the calculated value of density is in agreement with the measured value A

(23) Analysis shows that nickel oxide has formula Ni 0.98 O 1.00. What fractions of nickel
exist as Ni2+ and Ni3+ ions?
2+
Let number ofNi in the oxide = x
3+ ` a
So number of Ni in the oxide = 0.98 @ x
` a ` a
In terms of oxidation number 2x + 3 0.98 @ x + 1 @ 2 = 0
2x + 2.94 @ 3x @ 2 = 0
0.94 @ x = 0 [ x = 0.94
2+ ff
f
ff
f
ff
f
f
0.94f
f
ff
f
# Fraction of Ni = B100 = 96 %
0.98
3+
and Fraction ofNi = 100 @ 96 = 4 %

(24) Ferric oxide crystallises in a hexagonal close packed array of oxide ions with two out
of every three octahedral holes occupied by ferric ions.
Derive the formula of the ferric oxide.

Let number of oxide ions per unit cell = 1

So number of octahedral holes = 1

Then number of ferric ions = f


2f
ff 2f
f
ff
BOctahedral holes =
3 3

Thus the formula of the compound = Fe 2fffffO1 = Fe2 O3


3

(25) Gold (atomic radius = 0.144 nm) crystallises in a face centred unit cell.
What is the length of a side of the cell?
w
w
w
w
ww
w w
w
w
w
ww
w
For FCC unit cell 4r = a p2 [ a = r 2 p2 = 0.144 B 2 B 1.414 = 0.407 nm

(26) Aluminium crystallises in a cubic close-packed structure. Its metallic radius is 125 pm.
(a) what is the length of the side of the unit cell?
(b) How many unit cells are there in 1.00 cm3 of aluminium?

(a) For cubic close packed structure ( fcc)


w
w
w
w
ww
w w
w
w
w
ww
w
4r = a p2 [ a = r 2 p2 = 125 B2 B 1.414 = 354 pm
b c3
(b) Volume of unit cell = a 3 = 354 B10@ 10 cm = 4.4361 B 10@ 26 cm 3

@ 26 3
Now, 4.4361 B10 cm contains 1 unit cell
3 f
f
ff
f
ff
f
ff
f
ff
f
ff
f
ff
f
ff
1f
f
ff
f
ff
f
ff
f
ff
f
ff
f
ff
f
ff
f
ff
f
ff 22
# 1 cm contains @ 26
= 2.26 B10 unit cells
4.4361 B10
jksahu74@gmail.com 27 | P a g e
(27) If NaCl is doped with 10-3 mol% of SrCl2, What is the concentration of cation vacancies?
+ 2+
2 Na ions are replaced by 1 Sr ions
2+
# EachSr ion cause one cation vacancy
@3 @3
Hence, concentration of cation vacancies on being doped with10 mol % SrCl2 =10 mol %

f
f
ff
f
ff
f
ff
f
ff
f
f
@3
10 @5 23 18
= mol =10 B 6.022 B10 = 6.022 B10 Vacancies
100

(28) An element crystallises into a structure which may be described by a cubic type of unit
cell having one atom on each corner of the cube and two atoms on one of the
diagonals. If the volume of this unit cell is 24 x 10-24 cm3 and density of the element is
7.2 g/cm3, calculate the number of atoms present in 200g of the element.

f
f
ff
f
ff
f
ff
f
ff
f
ff
f
f
200gf
f
ff
f
ff
f
ff
f
ff
f
ff
f 3
Volume of 200 g of the element = @3
= 27.778 cm
7.2g cm
@ 24 3
But volume of unit cell = 24 B10 cm
f
f
ff
f
ff
f
ff
f
ff
f
ff
f
ff
f
ff
f
ff
f
ff
f
ff
f
ff
f
ff
f
ff
f
ff
f
ff
f
ff
3
27.778 cm 24
# Number of unit cells contained in 200 g of element = @ 24 3
= 1.1574 B10
24 B10 cm
1f
f
ff
But number of atoms present in each unit cell = B8 at corners + 2 at diagonal = 3
8
24 24
# Total number of atoms present in 200 g of element = 3 B1.1574 B10 = 3.4722 B10

(29) An element ( At. Mass = 60) having face centred cubic unit cell has a density of 6.23 g
cm-3. What is the edge length of the unit cell?

For a cubic unit cell : d = f


f
ff
f
ff
f
ff
f
ff
f
ff
f
[a = f
f
ff
f
ff
f
ff
f
ff
f
f
zM 3 zM
3
a NA NA d
For FCC z = 4

So a = ff
f
ff
f
ff
f
ff
f
ff
f
ff
f
ff
f
3 4f
f
ff
f
ff
f
ff
f
f
B60f
f
ff
f
fff
f
ff
gf
f
ff
f
ff
f
ff
f
f
molf
f
ff
f
ff
ff
f
f
@1
f
ff
f
ff
f
ff
f
ff
f
ff
f
ff
ff
f
ff
ff @ 24 3
23 @3
= 64 B10 cm
6.022 B10 B6.23 g cm
b c 1fffff
@ 24 3
[ a = 64 B10 = 4 B10@ 8 cm = 400 pm

(30) A body centred cubic element of density 10.3 g cm-3 has a cell edge of 314 pm.
Calculate the atomic mass of the element.

f
f
ff
f
f
zMf
f
ff
f
ff
f
ff
f @3
We know that for a cubic unit cell d = 3
g cm
a NA
` a
For BCC Number of atoms per unit cell z = 2
` a @ 10
Edge length a = 314 pm = 314 B10 cm
b c3
@ 10 23

Atomic mass of the element M = f


f
ff
f
ff
f
fff
f
ff
f
ff
ff
ff
f f
= f
f
ff
f
ff
f
ff
f
ff
f
ff
f
ff
f
ff
f
ff
f
ff
f
ff
f
ff
f
ff
f
ff
f
ff
f
ff
f
ff
f
ff
f
ff
f
ff
f
ff
f
ff
f
ff
f
ff
f
ff
f
ff
f
ff
ff
f
ff
f
ff
f
ff
f
ff
f
ff
f
ff
f
ff
f
ff
f
ff
f
ff
f
ff
f
ff
f
ff
da 3
N
` a A
10.3 B 314 B 10 B 6.022 B 10 @1
= 96 g mol
z 2

jksahu74@gmail.com 28 | P a g e
(31) The density of chromium is 7.2 g cm-3.
If the unit cell is cubic with edge length of 289 pm, determine the type of the unit cell.
` a @ 10 ` a @1
Edge length a = 289 pm = 289 B10 cm Atomic mass of chromium M = 52 g mol
` a f
f
ff
f
f
zMf
f
ff
f
ff
f
ff
f
Density of unit cell d = 3
a NA
b c3
@ 10 23
f
ff
f
ff
f
ffff
f
ff
f
ff
ff
ff
f f f
f
ff
f
ff
ff
f
ff
f
ff
f
ff
f
ff
f
ff
f
ff
f
ff
f
ff
f
ff
f
ff
f
ff
f
ff
f
ff
f
ff
f
ff
f
ff
f
ff
f
ff
f
ff
f
ff
f
ff
f
ff
f
ff
f
ff
f
ff
f
ff
f
ff
f
ff
f
ff
f
ff
f
ff
f
ff
f
ff
f
ff
f
ff
da
3
N A
7.2 B 289 B10 B6.022 B10
[z= = =2
M 52
Since the unit cell has 2 atoms so it is body centred unit cellA

(32) An element ‘A’ crystallises in FCC structure. 200 g of this element has 4.12 X 1024
atoms.The density of ‘A’ is 7.2 g cm-3. Calculate the edge length of the unit cell.
24
4.12 B10 atoms having mass = 200 g
f
f
ff
f
ff
f
ff
f
ff
f
ff
f
fff
f
ff
f
ff
f
ff
f
ff
f
ff
f
ff
f
ff
f
fff
f
fff
f
ff
f
ff
f
ff
f
fff
23
200 B6.022 B10 23 @1
# Gm @ atomic mass of A i A e A Mass of 6.022 B10 atoms = 24
= 29.23 g mol
4.12 B10

We know that d = f
f
ff
f
ff
f
ff
f
ff
f
ff
f
zM
3
Since the unit cell having FCC structure , So z = 4
a NA

[a = f
f
ff
f
ff
f
ff
f
ff
f
f f
= f
f
ff
f
ff
f
ff
f
f
3 zM 4f
f
ff
f
ff
f
ff
f
ff
f
ff
f
ff
f
f
B29.23f
ff
f
fff
f
ff
f
ff
f
ff
f
ff
f
ff
f
ff
f @ 24 3
23
= 26.96 B10 cm
NA d 6.022 B10 B7.2
b c 1fffff
[ a = 269.6 B10@ 24 3
= 2.99 B10@ 8 cm = 2.99 A 0

(33) Lithium metal crystal has a body centred cubic structure. Its density is 0.53 g cm-3 and
molar mass is 6.94 g mol-1. Calculate the volume of a unit cell of lithium metal.

Number of atoms per unit cell (z) =2 Since body centred cubic unit cell

f
f
ff
f
f
zMf
f
ff
f
ff
f
ff
f
We know that d= 3
a NA
f
f
ff
f
ff
f
ff
f
ff
f
f f
f
ff
f
ff
f
ff
f
ff
f
ff
f
ff
f
ff
f
ff
f
ff
f
ff
ff
f
ff
f
ff
f
ff
f
ff
f
ff
f
ff
f
ff
f
ff
f
ff
f
ff
f
b c zM 2 B 6.94
3 @ 23 3
[ Volume of unit cell a = = 23
= 4.348 B10 cm
NA d 6.022 B10 B 0.53

(34) Sodium crystallises in the cubic lattice having density 0.9623 g cm-3 and
edge length 430 pm. Calculate the number of atoms in a unit cell.

We know that d = f
f
ff
f
ff
f
ff
f
ff
f
ff
f
zM
3
a NA
b c3
10 23
` a af
f
ff
f
ff
f
ff
f
ff
fff
f
f
df
f
ff
f ff
f
ff
f
ff
f
f
430f
f
ff
f
ff
f
ff
f
ff
f
ff
f
ff
f
ff
f
ff
f
ff
f
ff
f
ff
f
ff
f
ff
f
ff
f
ff
f
ff
f
ff
f
ff
f
ff
f
ff
f
ff
f
ff
f
ff
f
ff
f
ff
f
ff
f
ff
f
ff
f
ff
f
ff
f
ff
f
ff
f
ff
f
ff
f
ff
f
ff
f
ff
f
f
B10 3 B6.022 B10 B0.9623
N A
So Number of atoms in a unit cell z = = =2
M 23

(35) An element crystallises in a bcc structure. The edge length of its unit cell is 288 pm. If
the density of the crystal is 7.3 g cm-3, what is the atomic mass of the element?
@ 10
Edge length of unit cell = 288 pm = 288 B10 cm Since the unit cell is bcc so z = 2

jksahu74@gmail.com 29 | P a g e
We know that d = f
f
ff
f
ff
f
ff
f
ff
f
ff
f
zM
3
a NA
af
f
ff
f
ff
f
ff
f
ff
fff
f
f
df
f
ff
f
3
` a N A
[ Atomic mass of the element M =
z
b c3
@ 10 23
f
f
ff
f
ff
f
ff
f
ff
f
ff
f
ff
f
ff
f
ff
f
ff
f
ff
f
ff
f
ff
f
ff
f
ff
f
ff
f
ff
f
ff
f
ff
f
ff
f
ff
f
ff
f
ff
f
ff
f
ff
f
ff
f
ff
f
ff
f
ff
f
ff
f
ff
f
ff
f
ff
f
ff
f
ff
f
ff
f
ff
f
ff
f
ff
288 B10 B6.022 B10 B7.3 @1
= = 52.51 g mol
2

(36) Chromium metal crystallises with a body centred cubic lattice. The length of the unit
cell is found to be 287 pm. Calculate the atomic radius and the density of chromium.
w
w
w
w
ww
w
w
ww
w
w
w
w afp3
f
ff
f
ff
ff
ff
f
ff
f
ff
f 287
f
f
ff
f
ff
f
ff
f
ff
f
ff
f
ff
ff
ff
f
ff
f
ff
f
ff
f
ff
f
ff
f
ff
f
For bcc 4r = a p3
` aB 1.732
# Atomic radius r = = = 124.27 pm
4 4
` a ` a @ 10
For bcc number of atoms per unit cell z = 2 Edge length a = 287 B10 cm
` a @1
Atomic mass of chromium M = 52 g mol
` a f
f
ff
f
f
zMf
f
ff
f
ff
f
ff
f fff
f
ff
f
ff
f
ff
f
ff
f
ff
f
ff
f
ff
f
ff
f
ff
f
ff
f
ff
f
2f
f
ff
f
f
Bf
f
ff
f
ff
f
ff
f
f
52f
f
ff
f
ff
f
ff
f
ff
f
ff
f
ff
f
ff
f
ff
f
ff
f
ff
f
ff
f
ff
f
ff
f
f @3
# Density of Chromium d = 3
=b c3 = 7.3 g cm
a NA 287 B10
@ 10
B 6.022 B10
23

(37) The nearest neighbour Ag atom in the silver crystal are 287 pm apart.

What is the density of silver? Silver crystallises in fcc form.

For FCC the distance between the neighbour = 2r = 287 pm

w
w
w
w
ww
w w
w
w
w
ww
w
4r = a p2 [ Edge length a = 2r p2 = 287 B1.414 = 406 pm = 406 B10@ 10 cm
` a

` a
Number of atoms per unit cell z = 4

Gm @ atomic mass of silver = 108 g mol@ 1

` a f
f
ff
f
f
zMf
f
ff
f
ff
f
ff
f
f f
f
ff
f
ff
f
ff
f
ff
f
ff
f
ff
f
ff
f
ff
f
ff
f
ff
f
ff
f
f
4f
f
ff
f
f
Bf
f
ff
ff
ff
f
ff
f
f
108f
f
ff
f
ff
f
ff
f
ff
f
ff
f
ff
f
ff
f
ff
f
ff
f
ff
f
ff
f
ff
f
ff
f
f @3
# Density of unit cell d = 3
=b c3 = 10.72 g A cm
a NA 406 B10@ 10 B 6.022 B10 23

(38) An element has a body centred cubic structure with a cell edge of 288 pm. The
density of the element is 7.2 g /cm3. How many atoms are present in 208 g of the
element?

Nuber of atoms per unit cell in bcc = 2


` a @ 10
Edge length of unit cell a = 288 pm = 288 B10 cm
` a @3
Density of unit cell d = 7.2 g Acm

f
f
ff
f
f
zMf
f
ff
f
ff
f
ff
f
We know that d= 3
a NA

jksahu74@gmail.com 30 | P a g e
b c3
@ 10 23

[M= f
aff
f
ff
f
ff
ff
f
ff
ff
f
f
df
f
ff
f f
= f
f
ff
f
ff
f
ff
f
ff
f
ff
f
ff
f
ff
f
ff
f
ff
f
ff
f
ff
f
ff
f
ff
f
ff
f
ff
f
ff
f
ff
f
ff
f
ff
f
ff
f
ff
f
ff
f
ff
f
ff
f
ff
f
ff
f
ff
f
ff
f
ff
f
ff
f
ff
f
ff
f
ff
f
ff
f
ff
f
ff
f
ff
f
f
N
3 288 B10 B 6.022 B10 B7.2
A
= 51.8 g mol@ 1
z 2

Number of atoms present in 208 g of the element = f ff


f
ff
f
ff
f
ff
f
ff
208 23 23
B 6.022 B10 = 24.16 B10 atoms
51.8

(39) Calculate the approximate number of unit cells present in 1g of gold. Given that gold
crystallises in the face centred cubic lattice. (At. Mass of gold=197 u)

Number of atoms prtesent in 1 g of gold = f ff


f
ff
f
ff
f
ff
f
ff
f
ff
f
ff
f
ff
f
ff
f
ff
f
ff
f
ff
f
ff
f
f
23
6.022 B10
197
As face @ centred cubic unit cell contains 4 atoms,
therefore , number of unit cells present in 1 g gold

= f
f
ff
f
ff
f
ff
f
ff
f
ff
f
ff
f
ff
f
ff
f
ff
f
ff
f
ff
f
ff
f
ff
f
23
6.022 B10 20
= 7.64 B10
197 B 4

(40) Atoms of element B forms hcp lattice and those of the element A occupy 2/3rd of
tetrahedral voids. What is the formula of the compound formed by the elements A & B?

Let number of B atoms in unit cell = 1 # Number of tetrahedral voids = 2

Number of atom A per unit cell = 2 B f


2f
ff f
= fff
The ratio of number of atoms A:B = f
4 4f
ff
:1 = 4:3
3 3 3
So the formula of the compound is A4 B3

(41) A solid has structure in which Na, W and O atoms are arranged as follows: Na at the
centre, W at the corner and O at edges of a cube. What is the simple formula of this
compound?

Na at the centre of the cube i A e A Number of Na @ atom per unit cell = 1 B 1 = 1


1f
f
ff
W at the corner i A e A Number of W @ atoms per unit cell = 8 B = 1
8
1f
ff
f
O at the edges i A e A Number of O @ atoms per unit cell = 12 B = 3
4
# The simple formula of the compound is NaWO3

(42) Calculate the packing efficiency of the two dimensional square unit cell.
w
w
w
w
ww
w
In the two dimensional square unit cell 4r = L p2
f
f
ff
f
ff
f
ff
f
f ww
w
w
w
ww
w = 2r p2
4r
[L = w
w
ww
w
w
p2
w
w
w
w
ww
wc2
Area of the unit cell = L = 2r p2
b
2 2
= 8r
Number of atoms per unit cell = 2
2
So area occupied by the atoms in the unit cell = 2πr

jksahu74@gmail.com 31 | P a g e
Packing efficency = f
f
ff
f
ff
f
ff
f
ff
f
ff
f
ff
f
ff
f
ff
f
ff
f
ff
f
ff
f
ff
f
ff
f
ff
f
ff
f
ff
f
ff
f
ff
f
ff
f
ff
ff
f
ff
f
ff
f
ff
f
ff
f
ff
f
ff
f
fff
f
ff
ff
ff
f
ff
f
ff
f
ff
f
ff
f
ff
f
ff
f
ff
f
ff
f
ff
f
ff
f
ff
f
ff
Area occupied by atoms in the unit cell
B100
Area of unit cell
ff
ff
f
ff
fff
ff
f
f
2
2πr
= 2
B100 = π B25 = 3.14 B25 = 78.54%
8r
` a 2+ + 2@ 2@ @ 0 0 0
43 If the radius of Mg , Cs ,O ,S and Cl ions are 0.65 A , 1.69 A , 1.40 A ,
0 0
1.84A and 1.81 A respectively A
Calculate coordination numbers of cations in the crystal of MgS, MgO and CsClA
b c
2+
rf
ff
f
ff
f
ff
f
f
Mgf
f
ff
ff
f
ff
ff
f
ff
f
f ff
f
ff
f
ff
f
f
f 0.65f
f
ff
f ` a
b
2@
c = = 0.353 > 0.225 but less than 0.414 [ C A N ofMg 2 + = 4 Tetrahedral
r S 1.84
b c
2+
rf
ff
f
ff
f
ff
f
f
Mgf
f
ff
ff
f
ff
ff
f
ff
f
ff 065
ff
f
ff
f
ff
f
ff
f
ff 2+ ` a
b c = = 0.464 >0.414 but less than 0.732 [ C A N of Mg = 6 Octahedral
2@
r O 1.40
b c
+
rff
f
ff
f
ff
f
f
Csf
f
ff
f
ff
f
ff
f
ff 1.69
f
f
ff
f
ff
f
ff
f
ff
f + ` a
` @a = = 0.933 > 0.732 [ C A N of Cs = 8 bcc
r Cl 1.81

(44) An element 'A' has BCC structure and another guest atom
'B', of largest possible size are present at each edge centres of
unit cell of 'A' but without distrubing the original unit cell
dimension. Determine the void percentage of the solid.

Atom B is closer to corner than body centre, in close contact A iA eA 2r A + 2rB = a


ww
w
w
w
ww
Atom A present at the lattice points of bcc, So 4r A = a p3
h i
ff
f
f
2rff
ff
ff
f 2r
ff
f
ff
fff
f
ff f
f
ff
f
faf
f
ff
f
ff
f
ff
f
f 1f
f
ff 1ff
f
frf
f
fff
f ff
f
1f
f
ff
f
ff
f
f rf
f
fff
f f
f
f
1f
f
ff
f
ff
f
f 1f
f
ff
A
+ B = w w
ww
w
w
w
B
= w
w
ww
w
w
w [ B = 2j w
w
ww
w
ww @ k= 0.155
p p p
[ +
4r A 4r A a 3 2 2 rA 3 rA 3 2
1f
f
ff
Number of atom . A. per unit cell = 2 and number of atom. B. = B12 = 3
4
f g f g
4f
f
ff
f 3 4f
f
ff
f 3
2B πr A + 3B πrB
f
f
ff
f
ff
f
ff
f
ff
f
ff
3f
f
ff
f
ff
f
ff
f
ff
f
ff
f
ff
f
ff
f
ff
f
ff
f
ff
f
ff
f
ff
f
ff
f
ff
f
ff
f
ff
f
ff
3f
f
ff
f
ff
f
ff
f
ff
f
ff
f
ff
f
f
Packing efficency = 3
B100 = 68.4 %
a
# Void space = 100 @ 68.4 = 31.6 %

(45) An element 'A' has BCC structure and another guest


atom 'B', of largest possible size, are present at the face
centres , but without distrubing the original unit cell
dimension. Determine the packing fraction of this solid.

In the above solid, face centres are closer to the body centre, therefore, the relationship A
ww
ww
w
w
w rf
f
ff
f
f
2 r A + rB = a and 4r A = a p3 [ B = 0.155
b c

rA
Number of atom A per unit cell = 2 and atom B = 3

jksahu74@gmail.com 32 | P a g e
In the above solid, face centres are closer to the body centre, therefore, the relationship A
w
w
ww
w
w
w
2 r A + rB = a and 4r A = a p3 [ f
b c rBf
f
ff
f
= 0.155
rA
Number of atom A per unit cell = 2 and atom B = 3
2 B fffffπr A + 3 B fffffπrB
4 3 4 3

f
f
ff
f
ff
f
ff
f
f
3f
f
ff
f
ff
f
ff
f
ff
f
ff
f
ff
f
ff
f
ff
f
ff
f
ff
f
ff
f
ff
f
ff
f
f
3f
ff
f
ff
f
ff
f
ff
f
ff
# Packing fraction = = 0.684
a3

(46) A compound is formed by two elements X and Y. Atoms of the element Y ( as anion)
make ccp and those of element X ( as cation) occupy all the octahedral voids. What is
the formula of the compound?

Let in ccp number of . Y. atoms per unit cell = 1 # Number of octahedral voids = 1

As all the octahedral voids are occupied by. X. , So number of . X. atoms per unit cell = 1

Ratio of number of atoms of X and Y = 1:1 [ The formula of the compound = XY

(47) X-ray diffraction studies show that copper crystallises in an fcc unit cell with cell edge
of 3.608 x 10-8 cm. In a separate experiment, copper is determined to have a density of
8.92 g/cm3. Calculate the atomic mass of copper.

f
f
ff
f
ff
f
ff
f
ff
f
ff
f af
f
ff
f
ff
f
ff
f
ff
fff
f
f
df
f
ff
f
3
zM N A
We know that d = 3
[ M= For fcc unit cell z = 4
a NA z
b c3 b c
@8 23
` a f f
ff
f
ff
ff
f
ff
f
f
3.608f
f
ff
f
ff
f
ff
f
ff
f
f
B10f
f
ff
f
ff
f
ff
f
ff
f
ff
f
ff
f
ff
f
ff
f
ff
f
ff
f
ff
f
ff
f
f
6.022f
f
ff
f
ff
f
ff
f
ff
f
f
B10f
f
ff
f
ff
f
ff
f
ff
f
ff
f
ff
f
ff
f
ff
f
f
8.92f
f
ff
f
ff f
ff
gf
f
ff
f
ff
f
f
So molar mass of copper M = = 63.1
4 mol
# Atomic mass of copper = 63.1 u

(48) Silver forms ccp lattice and X-ray studies of its crystals show that the edge length of
its unit cell is 408.6 pm. Calculate the density of silver.
` a
Since the lattice is ccp, the number of silver atoms per unit cell z = 4 ;

Molar mass of silver M = 108 f ff


` a gf
f
ff
f
ff
f
f
mol
` a
Edge length of unit cell a = 408.6 pm = 408.6 x10@ 10 cm

So density of silver d = f
` a fff
f
ff
f
ff
f
ff
f
ff
f f
= bf
f
ff
f
ff
f
ff
f
ff
f
ff
f
ff
f
ff
f
ff
f
ff
f
ff
f
ff
f
ff
zM 4f
f
ff
f
ff
fff
f
f
B108f
f
ff
f
ff
ff
f
ff
f
ff
f
ff
f
ff
f
ff
f
ff
f
ff
f
ff
f
ff
f
ff
f
ff
f
fff
f
fff
f
ff @3
3 c3 b c = 10.5 g cm
a NA 408.6 B10 @ 10
6.022 B10 23

(49) In Corundum, oxide ions are arranged in hcp and aluminium ions occupy 2/3rd of
octahedral voids. What is the formula of the corundum?

Let number of oxide ions per unit cell = 1 # Number of octahedral voids = 1

Number of aluminium ions = 1 B f


2f
ff 2f
= f f
f
Ratio of Al @ ions and oxide ions = f
2f
ff
:1 =2:3
3 3 3
So the formula of the compound = Al2 O3
jksahu74@gmail.com 33 | P a g e
A homogenous mixture whose composition can be varies within certain limits is termed a true
solution.

Solution = Solvent (Large quantity ) + Solute { Solvent decide the physical state of the solution }

Depending on the state, solutions can be classified as : Gaseous, Liquid and Solid.

Solution Solvent Solute Examples


Solid Solid Various alloys ( bronze,brass,german silver etc)
Solid Solid Liquid Sodium amalgam
Solid Gas Hydrogen gas adsorbed on palladium
Liquid Solid Sugar dissolve in water
Liquid Liquid Liquid Aqueous ethanol
Liquid Gas Carbonated soft drinks
Gas Solid Camphor vapours in nitrogen gas
Gaseous Gas Liquid Humidity in air
Gas Gas Atmospheric air

Solubility

(A substance dissolves in water if the enthalpy of hydration is more than the lattice enthalpy.)

Solubility represent the maximum amount of a substance that can be dissolved in a specified
amount of solvent at a particular temperature. It depends upon

(1) Nature of solute and solvent :

Like dissolves like

i.e Polar solute dissolves in polar solvent and non- polar solute in non-polar solvent.

(2) Temperature:

For Solute + Solvent @ @ @ Q Solution , ∆ s H =@ ve


Increase of temperature decreases the solubility.

For Solute + Solvent @@@ Q Solution , ∆s H = + ve


Increase of temperature increases the solubility

(3) Pressure:

Pressure does not have any significant effect on solubility of solids in liquids,
because solid and liquids are highly incompressible and remain unaffected by change in
pressure.
jksahu74@gmail.com 34 | P a g e
Henry’s law of gas solubility

The solubility of a gas in a liquid at constant temperature is directly proportional to the pressure of
the gas.

Mole fraction of gas in the solution is proportional to the partial pressure of the gas over the
solution.

The partial pressure (p) of the gas in vapour phase is proportional to the mole fraction (x) of the
gas in the solution.

p ∝ x => p = kH x

p: Partial pressure of the gas in vapour phase

x : Mole fraction of the gas in the solution

kH : Henry’s law constant

The value of KH is a function of nature and temperature of the gas.

Higher the value of KH at a given pressure, lower is the solubility of the gas in the liquid.

With increase of temperature, solubility of gas decreases i.e. KH value increases.

Application of Heny’s law

Fact Explanations

Soft drink and soda water bottles To increase the solubility of carbon dioxide gas.
are sealed under high pressure.
Under water pressure is high. Breathing at increased pressure
The tanks used by scuba divers increases the solubility of atmospheric gases in blood. When
are filled with air diluted with the divers come towards surface the pressure gradually
helium. decreases. Dissolved gases are released which leads to
formation of bubbles of nitrogen in the blood. This blocks
capillaries and creates the disease bends which are painful
and dangerous to life.
High concentration of nitrogen in the blood has toxic effects
also.
At high altitude the partial pressure of oxygen is less than that
Mountaineers( Climbers at high at ground level. At lower pressure solubility of oxygen in blood
altitude ) become patient of decreases. As a result concentration of oxygen in the blood
anoxia ( weak and unable to think and tissues of people living at high altitude or climbers
clearly) become low. Low blood oxygen causes anoxia.

jksahu74@gmail.com 35 | P a g e
Concentration of solutions

Concentration term Definition Advantages(A) and disadvantages (D)

A Most commonly used concentration term because volume


Number of moles of solute per litre of solution. measurement is easy. Results are readily used in
Molarity (M) stoichiometric calculations.
ff
f
ff
f
ff
f
ff
f
ff
f
f
Numberf
f
ff
f
ff
f
ff
f
ff
f
f
off
f
ff
f
ff
f
ff
f
ff
f
ff
f
f
molesf
f
ff
f
ff
f
ff
ff
f
f
off
ff
f
ff
f
ff
f
ff
f
ff
f
ff
f
f
solutef
f
ff
M= D Temperature dependent
Volume of solution in litres
A Independent of temperature
Molality (m) Number of moles of solute per kilogram of solvent
f
ff
f
ff
f
ff
f
ff
f
f
Numberf
f
ff
f
ff
f
ff
f
ff
f
ff
f
f
off
ff
f
ff
f
ff
f
ff
f
ff
f
f
molesf
f
ff
f
ff
f
ff
ff
f
f
off
ff
f
ff
f
ff
f
ff
f
ff
f
ff
f
f
solutef
f
ff D Mass of volatile solvent changes by evaporation
m=
Mass of solvent in kg

A Unitless quantity. Sum of mole fraction of solute and


Ratio of moles of the component to total moles of solvent in a solution is unity.
Mole fraction (X) components in the solution.
f
f
ff
f
ff
f
ff
f
ff
f
ff
f
ff
f
ff
f
ff
f
ff
f
f
Numberf
f
ff
f
ff
f
ff
f
ff
f
ff
f
f
off
ff
f
ff
f
ff
f
ff
f
ff
f
f
molesf
f
ff
f
ff
f
ff
f
f
off
f
ff
ff
f
ff
f
f
thef
f
ff
f
ff
f
ff
f
ff
f
ff
f
ff
f
ff
f
ff
f
ff
f
f
componentf
f
ff
f
ff
f
ff
f
ff
f
ff
f
ff
f
ff
f
ff
f
ff
f
f D In very dilute solutions mole fraction of solute becomes
x=
Total number of moles of all the components negligibly small.

A Independent of temperature
Mass percentage Mass of solute per 100 parts by mass of solution
(W/W %) ff
f
ff
f
ff
f
ff
f
f
Massf
f
ff
f
ff
f
ff
f
ff
ff
f
f
off
ff
f
ff
f
ff
f
f
thef
f
ff
f
ff
f
ff
f
ff
f
ff
f
ff
f
ff
f
ff
f
ff
f
ff
f
f
componentf
f
ff
f
ff
f
ff
f
ff
f
ff
f
ff
f
ff D Density must be known to convert mass % into molarity.
%= B 100
Total mass of the solution

A Independent of temperature. Widely used in environmental


Parts per million(ppm) Number of parts of solute per million parts of solution. applications.
ff
f
ff
f
ff
f
ff
f
ff
f
ff
f
ff
f
f
Numberf
f
ff
f
ff
ff
f
ff
f
ff
f
off
ff
f
ff
f
ff
f
ff
f
ff
f
ff
f
f
partsf
f
ff
f
ff
f
f
off
ff
ff
f
ff
f
f
thef
f
ff
f
ff
f
ff
f
ff
f
ff
f
ff
f
ff
f
ff
f
ff
f
f
componentf
f
ff
f
ff
f
ff
f
ff
f
ff
f
ff
f
ff 6
ppm = B10
Total no A of components in the solution D Measurement demands very exacting analytical
techniques.

jksahu74@gmail.com 36 | P a g e
Raoult’s law

For a solution of volatile liquid, the partial pressure of each component in the solution is directly
proportional to its mole fraction.

Partial pressure ∝ Mole fraction of volatile component

Let a solution contains, A : Volatile solvent ( M.F = X1)

B: Non-volatile solute ( M.F = X2 )

Vapour pressure of solution ∝ Mole fraction of solvent

p ∝ X1

> p = X1p0

Where p: Vapour pressure of solution and p0 : Vapour pressure of solvent.


b c
0
p = p 1 @ x2

p = p 0 @ x2 p 0
x2 p 0 = p 0 @ p Where ∆p = Lowering of vapour pressure
ff
f
f
∆pf
fff
f
x2 = f
pff
f
ff
f
ff
f
fff
ff
f
ff
ff
0
@ p and 0
= Relative lowering of vapour pressure
0 p
p

# x2 = ff
ff
ff
ff
f
∆p
0
p

Raoult’s law also states that relative lowering of vapour pressure is equal to mole fraction of
solute.

When both solute and solvent are volatile in nature

A : Volatile solvent ( M.F = X1 ) having v.p = P10

B : Volatile solute ( M.F = X2 ) having v.p = P20

Let in the solution of A &B , the partial pressure of 'A' is P1 and partial pressure of 'B' is P2
0 0
p1 = x1 p1 and p2 = x2 p2
` a
Vapour pressure of solution p = p1 + p2
0 0
p = x1 p1 + x2 p 2
b c
0 0
= p1 1 @ x2 + x2 p2
0 0 0
= p1 @ p1 x2 + x2 p2
b c
0 0 0
# p = p2 @ p1 x2 + p1

Straight line equation y = mx +c

jksahu74@gmail.com 37 | P a g e
Ideal and non-ideal solution

Ideal solution Non-ideal Solution


The solutiopn which obeys Raoult's law under all Non-ideal solutions do not obey Raoult’s law at any temperature and concentration.
conditions of temperature, pressure and concentration is They form azeotropes.
known as ideal solution. They do not form azeotropes. Positive deviation Negative deviation
Enthalpy of mixing and volume of mixing is zero. Enthalpy of mixing and volume of mixing Enthalpy of mixing and volume of mixing
∆Hmix = 0 & ∆V mix = 0 P = pA + pB is greater than zero. is less than zero.
∆Hmix > 0 & ∆Vmix > 0 P > pA + pB ∆Hmix < 0 & ∆Vmix < 0 P < pA + pB
In this case the intermolecular attractive forces between In this solution, solute-solvent In this solution, solute-solvent
solvent-solute molecules are same with solute-solute and intermolecular attraction forces are lower intermolecular attraction forces are more
solvent-solvent molecules than the solute-solute or solvent-solvent than the solute-solute or solvent-solvent
intermolecular attraction forces. intermolecular attraction forces.
Examples, solutions of :- Examples, solutions of :- Examples, solutions of :-
[1] n-hexane & n-heptane [1] Acetone and CS2 [1] Acetic acid - Pyridine
[2] Chlorobenzene & Bromobenzene [2] Acetone and C2H5OH [2] Chloroform -Acetone
[3] Ethyl bromide & Ethyl iodide [3] Methyl alcohol and water [3] Aniline -Acetone
[4] Carbon tetra chloride & silicon tetra chloride [4]Cyclohexanal and water [4] Water - Sulphuric acid
[5] Benzene & Toluene [5]CCI4 and CHCI3 [5] Water - Nitric acid
Bezene is more volatile than toluene so, p10 >p20
0
P1
P=
P1 +
P2
0
P1 P2
V.P
P2

X1 = 1 X1 = 0
X2 = 0 M.F X2 = 1
Benzene Toluene

Form minimum boiling azeotropes Form maximum boiling azeotropes


jksahu74@gmail.com 38 | P a g e
Composition of mixture in liquid and vapour phase

Azeotropes

Azeotropes are defined as the mixture of liquids which boil at constant temperature like a pure
liquid and possess the same composition of the components in the liquid as well as in vapour
phase.

Azeotropes are mixtures and not compounds,because both the boiling point and composition of
azeotropes is changed whereas for a chemical compound, composition remains constant over a
range of pressure .

There are two types of Azeotropes :-

Minimum boiling point azeotropes Maximum boiling point azeotropes

The solution which shows positive deviation, The solution which shows negative deviation,
form minimum boiling point azeotropes at a form maximum boiling point azeotropes at a
specific composition specific composition.
95% by volume of ethanol with water boil at 68% by mass of nitric acid with water boil at
351.15 K 393.5K

Colligative Properties

The properties of solution that depends upon the number of solute particles irrespective of their
nature is known as colligative properties.

[1] Relative lowering of vapour pressure ff


f
ff
fff
f
f
∆P
0
= x2
p

[2] Elevation in boiling point ( ∆Tb )

[3] Depression in freezing point ( ∆Tf )

[4] Osmotic pressure ( π )


jksahu74@gmail.com 39 | P a g e
Elevation in boiling point ( ∆Tb ) Depression in freezing point ( ∆Tf )
The temperature at which the V.P of liquid is The temperature at which vapour pressure of
equal with the external pressure is known as solid = V.P of liquid of same substance, is
boiling point. known as freezing point.
Solvent
Solution
1 atm Liquid
∆P

Vapour pressure
Vapour pressure

Solid
Solvent
∆P ∆Tf
Solution ∆Tb
0
O Tf Tf
Tb Tb Temperature
Temperature
The boiling point of a solution (Tb) of non- The freezing point of a solution (Tf) of non-
volatile solute is higher than that of pure solvent volatile solute is lower than that of pure solvent
0 0
(Tb0). i.e. Tb > Tb (Tf0) i.e Tf < Tf
0 0
∆Tb = Tb @ Tb ∆Tf = Tf @ Tf
∆Tb ∝ m ∆Tf ∝ m
∆Tb = k b m ∆Tf = k f m
f
f
wf
f
f2f
f
f
Bf
f
ff
f
ff
f
ff
f
ff
f
ff
f
f
1000f
f
ff
f
ff
f f
f
wf
f
f2f
f
f
Bf
f
ff
f
ff
f
ff
f
ff
f
ff
f
f
1000f
f
ff
f
ff
f
∆Tb = k b ∆Tf = k f
M2 B w1 M2 B w1
Where Kb is proportionality constant , known as Where Kf is proportionality constant , known as
Ebullioscopic constant / boiling point elevation cryoscopic constant / freezing point depression
constant / molal elevation constant, depends constant / molal depression constant, depends
upon the nature of solvent. upon the nature of solvent.

For water Kb = 0.52 K kg mol-1 . For water Kf = 1.86 K kg mol-1 .

Molal elevation constant ( kb) is defined as the Molal depression constant (kf) is defined as the
elevation of boiling point of one molal solution. depression in freezing point of one molal
solution.

f
f
ff
f
ff
f
ff
f
ff
f
ff
f
ff
f
ff
f
ff
f
ff
f
ff
f
ff
f
ff
f
bf
f
ff
f
ff
f
f f
f
ff
f
ff
f
ff
f
ff
f
ff
f
ff
f
ff
f
ff
f
ff
f
ff
f
ff
f
ff
f
ff
f
ff
f
ff
f
2 2
RB MB T RB MB T
Kb = Kf =
1000 B∆ vap H 1000 B ∆ fus H

Osmosis
The spontaneous flow of solvent molecules from low
concentrated side to high concentrated side through a
semipermeable membrane is known as osmosis.
[ In diffusion, in order to maintain equal concentration
the solute particles flow from high concentrated side to
low concentrated side.]
V.P of low concentrated side > High concentrated side, So Solvent molecules flow from low
concentrated side to high concentrated side.

jksahu74@gmail.com 40 | P a g e
` a
Osmotic pressure π

The pressure exerted at high concentrated side due to osmosis is known as osmotic pressure.
Osmotic pressure is directly proportional to the concentration of the solution i.e. π ∝ C
Osmotic pressure increases with increase in temperature .i.e. π ∝ T
So π ∝ CT
π = CRT
If two solutions having same concentration i.e. same osmotic pressure is known as
Isotonic solution.

The low concentrated solution is known as Hypotonic solution

The solution having high concentration i.e. higher osmotic pressure is known as
Hypertonic solution.

Osmotic pressure is used for the determination of molar masses of macromolecules. Because,
[1] It is performed in room temperature with help of concentration expressed in molarity.
[2] Compared to other colligative properties, its magnitude is large even for very dilute solution.
[3] Macro molecules are unstable and poor solubility at higher temperature.

Edema
When placed in water containing less than 0.9% (mass/ volume) salt, blood cells collapse due to
loss of water by osmosis. People taking a lot of salt or salty food experience water retention in
tissue cells and intercellular spaces because of osmosis. The resulting puffiness or swelling is
called edema.
Reverse Osmosis
The phenomenon of movement of solvent molecules from high concentrated side to low
concentrated side through semi permeable membrane by applying pressure on the high
concentrated side which is greater than osmotic pressure is known as reverse osmosis.
It is used in desalination of sea water.

Van’t Hoff Factor ( i )


Calculated colligative properties and observed colligative properties sometimes differ due to
abnormal molecular mass. The abnormal molecular masses results due to molecular dissociation
or molecular association. In order to equalise the observed and calculated colligative properties,
we should multiply Van't Hoff factor (i) with the calculated properties.

( Since Number of moles = f


wf
f
ff
f
Colligative properties ∝ Number of moles )
M

1f
f
ff
f
f
So colligative properties ∝
M
jksahu74@gmail.com 41 | P a g e
f
f
ff
f
ff
f
ff
f
ff
f
ff
f
ff
f
ff
f
f
Observedf
f
ff
f
ff
f
ff
f
ff
f
ff
f
ff
f
ff
f
ff
f
ff
f
ff
f
ff
f
ff
f
ff
f
f
colligativef
f
ff
f
ff
f
ff
f
ff
f
ff
f
ff
f
ff
f
ff
f
ff
f
ff
f
f
propertiesf
f
ff
f
ff
f
ff
i=
Calculated colligative properties

= ff
f
ff
f
ff
f
ff
f
ff
f
ff
f
ff
f
ff
f
ff
f
ff
f
ff
ff
f
ff
ff
f
ff
f
ff
f
ff
f
ff
f
ff
f
ff
f
ff
f
ff
f
ff
f
ff
f
ff
f
ff
f
ff
f
ff
f
ff
f
ff
f
ff
f
ff
f
ff
f
ff
f
ff
f
ff
f
ff
f
ff
f
ff
f
ff
f
ff
f
ff
f
ff
f
ff
f
ff
f
ff
ff
f
ff
f
ff
f
ff
f
ff
f
ff
f
ff
f
ff
f
ff
f
ff
f
ff
f
ff
f
ff
f
ff
f
ff
f
ff
f
Number of particles after association or dissociation
Number of particles before association or dissociation
ff
f
ff
f
ff
f
ff
f
ff
f
ff
f
ff
f
ff
f
ff
f
f
Calculatedf
f
ff
ff
f
ff
ff
f
ff
f
ff
f
ff
f
ff
f
ff
f
ff
f
ff
f
f
molecularf
f
ff
f
ff
f
ff
f
ff
f
ff
f
ff
f
f
massf
f
ff
f
ff
f
f
=
Observed molecular mass

Molecules C6H12O6 NaCl CH3COONa 2CH3COOH Na2SO 4 K4[Fe(CN)6]

1 1 5
Van't Hoff Factor (i) 2 2
2 3

Relationship between Van't Hoff factor ( i ) and degree of dissociation ( α )

AxBy x Ay+ + yB x- No. of particles


1 0 0 Before dissociation = 1

1-α xα yα After dissociation = 1- α + xα + yα


= 1- α + α ( x +y )
= 1- α + α n
(n- Total no. of ions)

Van. t Hoff factor i = ff


f
ff
f
ff
f
ff
f
ff
f
ff
f
ff
f
ff
f
ff
f
ff
f
ff
ff
f
ff
ff
f
ff
f
ff
f
ff
f
ff
f
ff
f
ff
f
ff
f
ff
f
ff
f
ff
f
ff
f
ff
f
ff
f
ff
f
ff
f
ff
f
ff
f
ff
f
ff
f
ff
f
ff
f
ff
f
ff
f
ff
f
ff
f
ff
f
ff
f
ff
f
ff
f
ff
f
ff
f 1f
= f
ff
f
ff
f
ff
f
ff
f
ff
ff
f
ff
f
ff
f
ff
f
ff
f
ff
f
ff
f
ff
` a Number of particles after dissociation @ α + αn
Number of particles before dissociation 1

=> i = 1 @ α + αn
` a
=> i =1@α 1@n

α = f
` a 1f
f
ff
f
f
@f
f
ff
f
fif
f
f
=> α 1@n =1@i
1@n
Relationship between Van't Hoff factor ( i ) and degree of association ( β )

nA A n No. of particles
1 0 Before association = 1
β 1 n−1
1 −β
n After association = 1 −β +
β
n =1 − β 1−
n
= 1−β n

F nf
f
ff
f
f
@f
f
ff
f
ff
f
f
1f
f
G
1@β H I
`a f
f
ff
f
ff
f
ff
f
ff
f
ff
f
ff
f
ff
f
ff
f
nf
f
ff
f
ff
f
ff
f
f Jf
nf
f
f
@f
f
ff
f
ff
f
f
1f
f
f
K
Van. t Hoff factor i = = 1@β
1 n
H I
J
nf
f
ff
f
f
@f
f
ff
f
f
1f
f
ff
K
=> β = 1@i
n
` a
nf
f
ff
f
ff
f
1f
f
ff
f
f
@f
f
ff
f
ff
if
f
ff
f
ff
f
β =
n@1
jksahu74@gmail.com 42 | P a g e
NUMERICALS ON SOLUTIONS
(1) Calculate the mole fraction of ethylene glycol in an aqueous solution containing 20% of
C2H6O2 by mass
Assume that we have 100 g solution A

Molar mass of ethylene glycol = 62 f


ff
gf
f
ff
f
ff
f
f
Mass of ethylene glycol = 20 g
mol

So Number of moles nE A G = ff
f
ff
f
ff
b c 20
= 0.322 mol
62
Molar mass of water = 18 f f
f
gf
f
ff
f
ff
ff
Mass of water = 80 g
mol

So number of moles of water nW = f ff


f
ff
ff
b c 80
= 4.444 mol
18

Mole fraction of Ethylene Glycol XE A G = f f


ff
fff
nf
ff
f
fff
Af
f
ff
f
ff
f
ff
f
ff
f
f f
= f
f
ff
f
ff
f
ff
ff
f
ff
f
ff
ff
f
ff
f
ff
f
ff
f
ff
f
ff
f
ff
f
ff
f
ff
f
ff
b c 0.332
E G
= 0.068
nE A G + nW 0.332 + 4.444
b c
Mole fraction of water X W = 1 @ XE A G = 1 @ 0.068 = 0.932

(2) Calculate the molarity of a solution containing 5 g of NaOH in 450 ml solution.

Molar mass of NaOH = 40 f ff


gf
f
ff
f
ff
f
f
; Mass of NaOH = 5 g
mol

So number of moles of NaOH = f f


f
ff
ff
f
f
5
= 0.125 mol
40
Volume of solution = 450 ml = 0.450 L

# Molarity of the solution = f


f
fff
f
ff
ff
f
ff
f
ff
f
ff
Af
f
ff
ff
ff
f
ff
f
ff
f
ff
f
ff
f
ff
f
ff
f
ff
f
ff
ff
f
ff
f
ff
f
ff
f
ff
f
ff
f
ff
f
ff
f
ff
f
ff
f
ff
f 0.125
= f
f
ff
f
ff
f
ff
f
ff
f
ff
f
ff
No moles of NaOH
= 0.278 M
Volume of solution in litre 0.450

(3) Calculate molality of 2.5 g of ethanoic acid in 75 g of benzene.

Molar mass of CH3 COOH = 60 ff


f
gf
f
ff
f
ff
f
f
; Mass of CH3 COOH = 2.5 g
mol
ff
f
ff
f
f
2.5f
ff
So number of moles ofCH3 COOH = = 0.0417 mol
60
` a
Mass of Benzene Solvent = 75 g = 0.075 kg

# Molality of the solution = ff


f
ff
f
ff
f
fAf
f
ff
ff
f
ff
ff
f
ff
f
ff
f
ff
f
ff
f
ff
f
ff
f
ff
f
ff
ff
f
ff
f
ff
f
ff
f
ff
f
ff
f
3f
f
ff
f
ff
f
ff
f
ff
f
ff
f
ff
f
ff
f
ff
f
ff 0.0417
= ff
f
ff
f
ff
f
ff
f
ff
f
ff
f
ff
f
ff
f
No of moles ofCH COOH
= 0.556 m
Mass of solvent in kg 0.075

(4) If N2 gas is bubbled through water at 298K, How many mill moles of N2 gas would
dissolve In 1 liter of water? Assume that N2 exerts a partial pressure of 0.987 bars.
Given that Henry’s law constant for N2 at 293 K is 76.48 kbar.

jksahu74@gmail.com 43 | P a g e
According to Henry. s law : P = KH X

X Nitogen = f
f
ff
f
ff
f 0.987
= f
f
ff
f
ff
f
ff
f
ff
f
ff
f
ff
f
ff
f
ff
f
ff
f
ff
f
ff
f
b c P bar @5
= 1.29 B10
KH 76480 bar
one litre Water contains 55.5 mol of it
Let the solution contains n moles of nitrogenA

X nitrogen = ff
f
ff
f
ff
f
ff
f
ff
ff
f
ff
f
ff
f
ff
f
f
b c n @5
So mole fraction of Nitrogen = 1.29 B10
n + 55.5
b c
n in denominator is neglected as it is << 55.5

Thus n = 1.29 B10@ 5 B55.5 = 7.16 B 10@ 4 mol = 0.716 m mol


(5) Vapour pressure of chloroform and dichloromethane at 298K are 200 mm Hg and 415
mm Hg respectively. Calculate the vapour pressure of the solution prepared by mixing
25.5 g of CHCl3 and 40 g of CH2Cl2 at 298 K. Also calculate mole fraction of each
component in vapour phase.

Moalr mass Number of moles


f
ff
gf
f
ff
f
ff
f
f f
f
ff
f
ff
f
ff
f
ff
f
ff
f
ff
b c 25.5
Chloroform CHCl3 119.5 = 0.213 mol
mol 119.5
ff
f
gff
f
ff
f
ff
f ff
f
ff
f
ff
f
b c 40
Dichloromethane CH2 Cl2 85 = 0.47 mol
mol 85
ff
f
ff
f
ff
f
ff
f
ff
f
ff
f
ff
f
ff
f
f
0.213f
f
ff
f
ff
f
ff
f
ff
f
ff
f
f
Mole fraction of CHCl3 = = 0.312 and M A F ofCH2 Cl2 = 1 @ 0.312 = 0.688
0.213 + 0.47
In liquid phase Raoult. s law will apply:
V A P of the component in the solution = M A F of the componet in the solutionBV A P of pure component
V A P of the solution = V A P ofCHCl3 + V A P of CH2 Cl2 in the solution
= 0.312 B200 + 0.688 B 415 = 347.9 mm Hg
In the vapour phase partial pressure of CHCl3 = 0.312 B200 = 62.4 mm Hg
and Partial pressure of CH2 Cl2 = 0.688 B 415 = 285.5 mm Hg
In vapour phase Dalton. s law of Partial pressure will apply:
i A e A Partial pressure of the component = Mole fractionBTotal pressure
f
f
ff
f
ff
f
ff
f
f
62.4f
f
ff
f
ff ff
f
ff
f
ff
f
ff
f
f
285.5f
f
ff
f
f
[ M A F ofCHCl3 = = 0.18 and M A F of CH2 Cl2 = = 0.82
347.9 347.9
(6) The vapour pressure of pure benzene at a certain temperature is 0.850 bar. A non-
volatile , non-electrolyte solid weighing 0.5 g when added to 39 g of benzene, Vapour
pressure of the solution then, is 0.845 bar. What is the molar mass of the solid
substance?
` a 0
Vapour pressure of pure solvent Benzene P = 0.850 bar
Vapour pressure of solution P = 0.845 bar
` a @1
Mass of the solute w = 0.5 g Let molar mass = M g mol
ff
f
ff
f
ff
ff
b c 0.5
So number of moles of solute n2 =
M

jksahu74@gmail.com 44 | P a g e
` a @1
Mass of solvent = 39 g Molar mass of solvent Benzene = 78 g mol
ff
f
ff
f
ff
b c 39
So number of moles of solvent n1 = = 0.5
78
[ P= f
f
ff
ff
ff
f
1f
f
ff
f
ff
f
ff0
0 n
According to Raoult. s law P = X1 P P
n1 + n2

[ 0.845 = f
f
ff
f
ff
f
ff
f
ff
f
ff
f
ff
fffff ff ff
f
0.5
0.850 [ M = 165.6 g mol@ 1
0.5 + ffffffffffff
0.5
M

OR

ff
f
ff
f
fff
f f
f
ff
fnf
f
ff
2f
f
ff
f
ff
f
ff nf
f
ff
ff Wff
f
ff
f
f
2f
f
ff
f
ff
f
f1f
f
f Pf
f
ff
f
ff
f
ff
f
ff
ff
f
ff
f
ff
f Wff
f
ff
f
2f
ff
ff
f
ff
f
f1f
f
f
0
∆P 2 M @ P M
According to Raoult. s law 0
= X2 = = = [ 0
=
P n1 + n2 n1 M2 W1 P M2 W1
0 @1
P = 0.850 bar , P = 0.845 bar , W1 = 39 g , W 2 = 0.5 g, M1 = 78 g mol M2 = ?

ff
f
ff
f
ff
f
ff
f
f
0.850f
ff
f
ff
f
f
@f
f
ff
f
ff
f
ff
f
ff
f
ff
f
ff
f
f
0.845f
f
ff
f
f 0.5
f
f
ff
f
ff
f
ff
f
ff
f
ff
f
f
B78f
f
ff
f
ff
f
f @1
# = [ M2 = 170 g mol
0.850 M2 B39

(7) 18 g of glucose is dissolved in 1 kg of water in a saucepan. At what temperature will


water boil at 1.013 bar?

f
f
ff
f
f
18f
f
ff
f
f
Number of moles of glucose = = 0.1 Mass of solvent = 1 kg
180
f
f
ff
f
f
0.1f
f
f
Thus molality of the solution = = 0.1 m A
1
b c b c
@1
Elevation of boiling point ∆Tb = Kb m = 0.52 B0.1 = 0.052 K $ Kb for water = 0.52K kg mol
Since water boils at 373.15 K at 1.013 bar pressure, therefore, the boiling point of solution will be
373.15 + 0.052 = 373.202 K
(8) The boiling point of benzene is 353.23 K. When 1.80 g of a non- volatile solute was
dissolved in 90 g of benzene, then boiling point is raised to 354.11 K. Calculate the
molar mass of the solute. [Kb for benzene is 2.53 K kg mol-1
b c
The elevation in boiling point ∆Tb = 354.11 @ 353.23 = 0.88 K
@1
Let molar mass of the non @ volatile solute = M g mol

So numer of moles of solute = ff


ff
f
ff
f
ff
ff
f
f
1.80
M
Mass of sollvent = 90 g = 0.09 kg

# Molality of the solution = ff


f
ff
f
ff
f
ff
f
ff
f
ff
f
ff
f
ff
1.80
0.09 M

[ 0.88 = 2.53 B f
f
ff
f
ff
f
ff
f
ff
f
ff
f
ff
f
f
[M= f
f
ff
f
ff
f
ff
f
ff
f
ff
f
ff
f
ff
f
ff
f
ff
f
ff
f
ff
f
ff
f
1.80 2.53 B1.80
∆Tb = Kb m = 57.5 g mol@ 1
0.09M 0.09 B0.88

jksahu74@gmail.com 45 | P a g e
(9) 45 g of ethylene glycol is mixed with 600 g of water. Calculate the freezing point of the
solution.
@1
Molar mass of ethylene glycol = 62 g mol

# Number of moles of solute = f f


f
ff
f
ff
f
45
= 0.73
62
Mass of solvent = 600 g = 0.6 kg

So molality of the solution = ff


f
ff
f
ff
f
ff
ff
f
f
0.73
= 1.2 m
0.6
b c
Depression of freezing point of the solution ∆Tf = Kf m = 1.86 B1.2 = 2.2 K
b c
@1
Since depression constant for water Kf = 1.86 K kg mol
` a
Freezing point of the aqueous solution = 273.15 K F A P of water @ 2.2 = 270.95 K

(10) 1.00 g of a non-volatile solute dissolved in 50 g of benzene lowered the freezing point
of benzene by 0.40 K. The freezing point depression constant of benzene is
5.12 K kg mol-1. Find the molar mass of the solute.
@1
Let molar mass of the non @ volatile solute = M g mol

# Number of moles of solute = f


1f
f
ff
f
M
Mass of solvent = 50 g = 0.05 kg

So molality of the solution m = f


` a f f
fff
ff
f
1f
f
ff
f
ff
f
ff
f
ff
f
f
0 A 05M
b c
Freezing point constant Kf = 5.12 K kg mol@ 1 for benzene

[ 0.40 = 5.12 B f
f
ff
f
ff
f
ff
f
ff
f
ff
f
ff
f
f f
f
ff
f
ff
f
ff
f
ff
f
ff
f
ff
f
ff
f
ff
f
ff
f
ff
f
ff
f
ff
f
1 5.12
∆Tf = Kf m [ M= = 256 g mol@ 1
0.05M 0.40 B0.05
(11) 200 cm3 of an aqueous solution of a protein contains 1.26 g of the protein. The
osmotic pressure of such a solution at 300K is found to be 2.57 X 10-3 bar.
Calculate the molar mass of the protein.

# Number of moles of protein = f


f
ff
f
ff
f
ff
f
ff
f
@1 1.26
Let the molar mass of protein = M g mol
M
Volume of solution = 200 cc = 0.2 L

So concentration of solution C = f ff
f
ff
f
ff
f
ff
f
ff
f
f
` a 1.26 @1
molL
0.2M
` a @1 @1
Value of gas constant R = 0.083 L bar mol K
f
f
ff
f
ff
f
ff ff
f
ff
f
ff
f
ff
f
ff
f
ff 2.57
f
f
ff
f
ff
f
ff
f
ff
f
ff
f
ff
f
ff
f
ff
f
ff
f
ff
f
ff
f
ff
f
ff
f
@3
` a π 1.26 B10
Osmotic pressure π = CRT [ C= [ =
RT 0.2M 0.083 B300

[M= f
f
ff
f
ff
f
ff
f
ff
f
ff
f
ff
f
ff
f
ff
ff
f
ff
f
ff
f
ff
f
ff
f
ff
f
ff
f
ff
f
ff
f
ff
f
ff
f
ff
f
ff
f
1.26 B0.083 B300 @1
@3
= 61.022 g mol
0.2 B 2.57 B10
(12) 2 g of benzoic acid dissolved in 25 g of benzene shows a depression in freezing point
equal to 1.62 K. Molal depression constant for benzene is 4.9 K kg mol-1.
What is the percentage association of acid if it forms dimer in solution?

jksahu74@gmail.com 46 | P a g e
@1
Molar mass of bezoic acid = 122 g mol

So number of moles of solute benzoic acid = f


` a ff
2f
f
ff
f
ff
f
f
122
` a
Mass of solvent benzene = 25 g = 0.025 kg

# Molality of the solution = f


f
ff
f
ff
f
ff
f
ff
f
ff
f
2f
f
ff
f
ff
f
ff
f
ff
f
ff
f
ff
f
ff
f
= 0.656 m
122 B0.025
b c
Depression in freezing point ∆Tf = Kf m = 4.9 B0.656 = 3.21
b c
∆Tf
Van. t Hoff factor i = bf
`a f ff
f
ff
f
ff
f
ff
f
ff
ff
ff
f
ff
f
fff 1.62
= f
f
ff
f
ff
f
ff
f
ff
f
expt
c = 0.504
∆Tf 3.21
cal

If x represents the degree of association , then:

2 COOH COOH

2
Before association: 1 0
x
After association: 1- x
2

xf
f
ff
f
1@x+
f
f
ff
f
ff
f
ff
f
ff
f
ff
f
f
Numberf
f
ff
f
ff
f
ff
f
ff
f
ff
f
f
off
ff
f
ff
f
ff
f
ff
f
ff
f
ff
f
ff
f
ff
f
ff
f
f
particlesf
f
ff
f
ff
f
ff
f
ff
f
ff
f
f
afterf
f
ff
f
ff
f
ff
f
ff
f
ff
f
ff
f
ff
f
ff
f
ff
f
ff
f
ff
f
f
associationf
f
ff
f
ff
f
ff
f
f f
f
ff
f
ff
f
ff
f
ff
f
ff
f
ff
f
ff
f
f2f
f
ff
f xf
f
ff
Van’t Hoff factor (i) = = = 1@
Number of particles before association 1 2
xf
f
ff ` a
1@ = 0.504 [ x = 2 1 @ 0.504 = 2 B0.496 = 0.992
2
# Degree of association of benzoic acid in benzene = 99.2 %

(13) 0.6 ml of acetic acid having density 1.06 g ml-1, is dissolved in 1 litre of water. The
depression in freezing point observed for this strength of acid was 0.0205 0C.
Calculate the dissociation constant of acid.
Mass of acetic acid = Volume BDensity = 0.6 B1.06 = 0.636 g
@1
Molar mass of acetic acid = 60 g mol

# Number of moles of acetic acid = f


f
fff
ff
f
ff
f
ff
f
ff
f
ff
0.636
= 0.0106
60
Mass of solvent water = Volume Bdensity = 1000 mlB1 f
` a gf
f
ff
f
f
= 1000gm = 1 kg
ml

# Molality of the solution = ff


f
ff
f
ff
f
ff
f
ff
f
ff
f
ff
f
ff
f
0.0106
= 0.0106 m
1
b c
Depression in freezing point ∆Tf = Kf m = 1.86 B0.0106 = 0.0197 K
0
Observed depression in freezing point = 0.0205 C = 0.0205 K
b c

Van. t Hoff factor i = bf


`a f ff
f
ff
f
ff
f
ff
f
ff
f
ff
ff
f
ff
f 0.0205
ff
f
ff
f
ff
f
ff
f
ff
f
ff
f
ff
f
ff
f
∆Tf
cobs = = 1.041
∆Tf 0.0197
cal

jksahu74@gmail.com 47 | P a g e
Since 0.0106 moles of acetic acid present in 1 litre of solution,
so molarity of the solution ( C )= 0.0106 M
Let degree of dissociation of acetic acid = α

- +
CH3COOH CH3COO + H
Number of moles per litre
Before dissociation C 0 0

After dissociation C − Cα Cα Cα

Van. t Hoff factor i = f


f
ff
f
ff
f
ff
f
ff
f
ff
f
ff
f
ff
f
ff
f
ff
f
ff
f
ff
f
ff
ff
f
ff
f
ff
f
ff
f
ff
f
ff
f
ff
f
ff
f
ff
f
ff
f
ff
f
ff
f
ff
f
ff
f
ff
f
ff
f
ff
f
ff
f
ff
f
ff
f
ff
f
ff
f
ff
f
ff
f
ff
f
ff
f
ff
f
ff
f
ff
f
ff
f
ff
f
ff
f C
= f
f
ff
f
ff
f
ff
f
ff
f
ff
f
ff
f
ff
f
ff
f
ff
f
ff
f
ff
f
ff
f
ff
f
ff
f
ff
f
ff
ff
f
ff
f
ff
f
ff
f
ff
f
ff
f C
= f
f
ff
f
ff
`a Number of particles after dissociation @ Cα + Cα + Cα +f
f
ff
f
ff
f
ff
f
f
Cαf
f
ff
f
ff
=1+ α
Number of particles before dissociation C C

α = i @ 1 = 1.041 @ 1 = 0.041

` a2
f
f
ff
f
ff
f
ff
f
ff
f
ff
f
ff
f
ff
f
ff
f
ff
ff
f ff
f
ff
f
ff
f
ff
f
ff
ff
f
f 0.0106
f
f
ff
f
ff
f
ff
f
ff
f
ff
f
ff
f
ff
f
ff
f
ff
f
ff
f
ff
f
ff
f
ff
f
ff
f
ff
f
ff
f
ff
f
ff
f
f
b c Cα BCα Cα 2
B 0.041 @5
Dissociation constant Ka of acetic acid = ` a = = = 1.86 B10
C 1@α 1@α 1 @ 0.041

(14) Calculate the mass percentage of benzene and carbon tetrachloride, if 22 g of benzene
is dissolved in 122 g of carbon tetrachloride.
Mass of solution = Mass of benzene + Mass of carbon tetrachloride = 22 + 122 = 144 g

Mass percentage of benzene = f


f
ff
f
ff
f
ff
f
ff
f
ff
f
ff
f
ff
f
ff
ff
f
ff
f
ff
f
ff
f
ff
f
ff
f
ff
f
ff
f
ff
f
ff
f
ff
B100 = f
f
ff
f
ff
f
ff
f
ff
f
ff
f
Mass of benzene 22g
B 100 = 15.28 %
Mass of solution 144g

Mass percentage of carbontetrachloride = 100 @ 15.28 = 84.72%

(15) Calculate the mole fraction of benzene in solution containing 30% by mass in
carbon tetrachloride.
Mass of solution = 100 g, Mass of benzene = 30g ,
Mass of carbon tetrachloride = 100 @ 30 = 70 g
@1
Molar mass of benzene = 78 g mol and
@1
Molar mass of carbontetrachloride = 154 g mol
f
30f
f
ff
f
ff
Number of moles of benzene = = 0.385 and
78
f
f
ff
f
f
70f
f
ff
ff
Number of moles of Carbontetrachloride = = 0.455
154
ff
f
ff
f
ff
f
ff
f
ff
ff
f
ff
f
ff
f
ff
f
f
0.385f
f
ff
f
ff
f
ff
f
ff
f
ff
f
ff
f
f 0.385
f
ff
f
ff
f
ff
f
ff
ff
f
ff
f
f
Mole fraction of benzene = = = 0.458
0.385 + 0.455 0.84
Mole fraction of carbon tetrachloride = 1 @ 0.458 = 0.542
(16) Calculate the molarity of each of the following solutions:
(a) 30 g of Co(NO3)2.6H2O in 4.3 L of solution (b) 30 ml of 0.5 M H2SO4 diluted to 500 ml.

jksahu74@gmail.com 48 | P a g e
f
ff
f
ff
f
ff
ff
f
` a b c 30
@1
a Molar mass of Co NO3 A 6H2 O = 311 g mol [ Number of moles =
= 0.0966
2 311

# Molarity of solution = f f
f
ff
ff
f
ff
f
ff
ff
f
ff
f
ff
f
ff
0.0966
and Volume of solution = 4.3 L = 0.02 M
4.3

[ M2 = f
Mff
f1f
f
ff
f
ff
f
1f
f
f 0.5
= f
f
ff
f
fff
f
ff
f
ff
f
ff
f
ff
f
ff
f
ff
f
` a V B 30
b Applying molarity equation: M1 V1 = M2 V2 = 0.03 M
V2 500

(17) Calculate the mass of urea required in making 2.5 kg of 0.25 molal aqueous solutions.
@1
Let mass of urea required = w g Molar mass of murea = 60 g mol

f
f
f
wf
f
ff
f
# Number of moles of urea =
60

Mass of solvent = 2.5 kg Molality of the solution = 0.25 m

ff
f
ff
f
ff
f
ff
f
ff
f
f
Numberf
f
ff
f
ff
f
ff
f
ff
f
f
off
f
ff
f
ff
f
ff
f
ff
f
ff
f
f
molesf
f
ff
f
ff
f
ff
ff
f
f
off
ff
f
ff
f
ff
f
ff
f
f
ureaf
f
ff
f
Moality of the solution =
Mass of solvent in kg
f
f
f
wf
f
ff
f
f
f
ff
f
ff
f
ff
f ` a
0.25 = 60 Mass of urea required w [ 0.25 B2.5 B60 = 37.5 g
2.5
(18) Calculate (a) Molality (b) Molarity and (c) Mole fraction of KI
if the density of 20% (mass/mass) aqueous KI is 1.202 g ml-1.
n
20% by mass of solution [ Mass of KI = 20g ; mass of sol = 100 g
# Mass of solvent = 100 @ 20 = 80g = 0.08kg
@1
Molar mass of KI = 166 g mol

# Number of Moles of KI = f
f
ff
f
ff
f
ff
f
f
= 0.12 Number of moles ofH2 O = f
f
ff
f
ff
f
20 80
= 4.44
166 18
@1
Density of solution = 1.202 g ml

# Volume of solution = f f
f
ff
f
fff
f
ff
f
ff
f
ff
f
ff
f
ff
f
ff
f
ff
f
ff
f
ff
f
ff
f
ff
f
ff
f
ff
f
ff
f
ff f
= f f
ff
f
ff
f
ff
f
ff
f
ff
f
f
Mass of solution 100
= 83.2 ml = 0.0832 L
Density 1.202

a Molality of the solution m = f f


f
ff
f
ff
f
ff
f
ff
f
ff
f
ff
f
ff
f
ff
f
ff
f
ff
f
ff
f
ff
ff
f
ff
f
ff
f
ff
f
ff
f
ff
f
ff
f
ff
f
ff
ff
f
ff
f
ff
f
ff
f
ff
f
ff
f
ff
f
ff 0.12
= ff
f
ff
f
ff
f
ff
f
ff
f
` a ` a Number of moles of solute
= = 1.5 m
Mass of solvent in kg 0.08

b Molarity of the solution M = f ff


f
ff
f
ff
f
ff
f
ff
ff
f
ff
f
ff
f
ff
ff
f
ff
f
fff
f
ff
f
ff
f
ff
ff
f
ff
f
ff
f
ff
f
ff
f
ff
f
ff
ff
f
ff
f
ff
f
ff
f
ff
f
ff
f
ff
f
ff f
= f
ff
f
ff
f
ff
f
ff
f
ff
f
ff
f
ff
f
f
` a ` a Number of moles of solute 0.12
= 1.44 M
Volume of solution in litre 0.0832

c Mole fraction of KI = ff
ff
ff
ff
f
nf
f
ff
f
fff
f
ff
f
ff
f
ff
f
f f
= f f
f
ff
f
ff
f
ff
ff
f
ff
f
ff
ff
f
ff
ff
f
ff
ff
f
ff
f
ff
f
` a KI 0.12
= 0.0263
nKI + nH2 O 0.12 + 4.44

(19) H2S , a toxic gas with rotten egg like smell, is used for the qualitative analysis. If the
solubility of H2S in water at STP is 0.195 m. Calculate Henry’s law constant.
b c
Solubility of H2 S gas in water = 0.195 m [ 0.195 moles ofH2 S present in 1 kg 1000 g of water
So Number of moles ofH2 S = 0.195

jksahu74@gmail.com 49 | P a g e
f
f
ff
f
ff
f
ff
f
ff
f
f
1000f
f
ff
f
ff
f
f
gf
f
ff
f
ff
f
ff
Number of moles of water = @1
= 55.55
18 g mol

Mole fraction of H2 S = ff
f
ff
f
ff
f
ff
f
ff
ff
f
ff
f
ff
f
ff
f
ff
f
ff
f
ff
f
ff
f
ff
f
ff
f
ff
f
f
0.195
= 0.0035
0.195 + 55.55
Pressure at STP = 0.987 bar

[ KH = f
Pf
f
ff
f
f2f
f
ff
fff
f f
= f
ff
f
ff
f
ff
f
ff
f
ff
f
ff
f
ff
f
f
H S 0.987
Applying Henry. s law PH2 S = KH XH2 S = 282 bar
XH2 S 0.0035

(20) Henry’s law constant for CO2 in water is 1.67 x 108 Pa at 298 K. Calculate the quantity of
CO2 in 500 ml of soda water when packed under 2.5 atm CO2 pressure at 298 K.
8
KH = 1.67 B10 Pa and PCO2 = 2.5 atm = 2.5 B101325 Pa

[ XCO2 = f
Pf
ff
ff
f
ff
f
2f
f
ff 2.5
= ff
f
ff
ff
f
ff
f
ff
f
ff
f
ff
f
ff
f
ff
f
ff
f
ff
f
ff
f
fff
ff
f
ff
f
f
CO B 101325 @3
Applying Henry. s law PCO2 = KH XCO2 8
= 1.517 B10
KH 1.67 B10

iAeA ff
ff
f
ff
f
f
nf
ff
f
ff
f
f
COf
f
2f
f
ff
f
ff
f
ff
ff
ff @3
= 1.517 B10
nCO2 + nH2 O
h i
nf
f
ff
f
f
COf
ff
ff
ff # CO2 gas is little soluble inwater i A enCO2 <<nH2 O
# 2 = 1.517B10 j k
@3

nH2 O so it is neglected inthe denominator

In500 ml of soda water, water present ≈ 500 ml = 500 g

So number of moles of water nH2 O = f


f
ff
f
ff
f
ff
f
ff
b c 500
= 27.78A
18
f
f
nf
f
ff
f
f
COf
f
ff
2f
f
ff
f
ff @3 @3 @3
# = 1.517 B10 [ nCO2 = 27.78 B1.517 B10 = 42.14 B10 mole
27.78
# Mass ofCO2 in soda water = 42.14 B10@ 3 B44 = 1.854 g

(21) The vapour pressure of pure liquids A and B are 450 and 700 mm Hg respectively at
350 K. Find out the composition of the liquid mixture if total vapour pressure is 600
mm Hg. Also find the composition of the vapour phase.

Inf
f
ff
f
ff
ff
f
ff
f
ff
f
ff
f
ff
f
ff
f
ff
f
ff
f
ff
f
ff
f
ff
f
ff
f
ff
f
f PA = 450 mm PB = 700 mm
0 0
liquid phase PTotal = 600 mm Hg
b c
0 0 0
Applying Raoult. s law PA = X A PA and PB = XB PB = 1 @ X A PB
b c b c
0 0 0 0 0 0 0 0
PTotal = PA + PB = X A PA + 1 @ X A PB = X A PA + PB @ X A PB = PB + X A PA @PB
b c
0 0 0
i A e PTotal = PB + X A PA @PB

[ XA = ff
f
ff
f
ff
f
ff
f
f
` a 100
[ 600 = 700 + X A 450 @ 700 [ 250 X A = 100 = 0.4
250
# XB = 1 @ X A = 1 @ 0.4 = 0.6
0
Partial pressure of. A. i A e PA = X A PA = 0.4 B 450 = 180 mm Hg
Partial pressure of. B. i A e PB0 = XB PB0 = 0.6 B700 = 420 mm Hg

jksahu74@gmail.com 50 | P a g e
Inf
f
ff
f
ff
f
ff
f
ff
f
ff
f
ff
f
f
Vapourf
f
ff
f
ff
f
ff
f
ff
f
ff
f
ff
f
ff
f
f
Phasef
f
ff
f
ff
f
ffDalton. s law of partial pressure is applicable:
b c b c
Partial pressure of. A. PA = MA F of. A. BTotal pressure PTotal

# Mole fraction of. A. = f


f
ff
f
ff
ff
ff
f
ff
f 180
= f
f
ff
f
ff
f
ff
f
ff
P A
= 0.3
PTotal 600
f
f
f
Pf
f
ff
f
f
Bff
f
ff
f 420
ff
f
ff
f
ff
f
ff
f
f
[ Mole fraction of. B. = = = 0.7
PTotal 600

(22) Vapour pressure of pure water at 298 K is 23.8 mm Hg. 50 g of urea is dissolved in
850 g of water. Calculate the vapour pressure of water for this solution and its relative
lowering.
b c
0
Vapour pressure of pure water P = 23.8 mm Hg
` a
Vapour pressure of solution P = ?
` a @1
Mass of solute Urea = 50 g Molar mass of urea = 60 g mol
ff
f
ff
f
ff
b c 50
[ Number of moles of solute n2 = = 0.83
60
` a @1
Mass of solvent Water = 850 g Molar mass of water = 18 g mol
f
f
ff
f
ff
f
ff
f
ff
850 b c
[ Number of moles of solvent n1 = = 47.2
18
According to Raoult. s law , Relative lowering of vapour pressure = Mole fraction of solute
ff
f
ff
f
fff
f ff
f
ff
f
fff
f f f
ff
f
nf
ff
f
2f
f
ff
f
ff
f
ff nf
f
ff
ff
∆P ∆P b c
2
iAe 0
= X2 [ 0
= = since n1 >>n2
P P n1 + n2 n1
h i
ff
f
f
∆Pf
f
ff
f ff
f
ff
f
f
f 0.83f
f
ff
f
ff
f
# Relative lowering of vapour pressure j 0 k= = 0.017
P 47.2

Pf
f
ff
f
ff
ff
ff
fff
f
ff
f
ff
f
f
0
@ P 0 0
iAe 0
= 0.017 [ P @ P = 0.017 P
p
` a 0 0 0
[ Vapour pressure of solution P = P @ 0.017 P = P B 0.983 = 23.8 B 0.983 = 23.3954 mm Hg

(23) Boiling point of water at 750 mm Hg is 99.630C. How much sucrose is to be added to
500 g of water such that it boils at 1000C.
0 0
Boiling point of water = 99.63 C Boiling point of solution =100 C at 750 mm Hg pressure
Let mass of sucrose added = W g Molar mass of sucrose = 342 g mol@ 1

# Number of moles of sucrose = ff


f
ff
f
ff
f
ff
f
f
W
Mass of solvent = 500 g = 0.5 kg
342
f
f
f
Wf
f
ff
f
ff
f
ff
f
f
ff
f
ff
f
ff
f
ff f
` af
ff
f
ff
f
ff
f
f
Wf
f
ff
f
ff
f
ff
f
ff
f
ff
f
ff
f
So molality of the solution m = 342 =
0.5 0.5 B342
b c
0
Elevation of boiling point of solution ∆Tb = 100 @ 99.63 = 0.37 C = 0.37 K

jksahu74@gmail.com 51 | P a g e
b c
@1
We know that ∆Tb = Kb m For water Kb = 0.52 K kg mol

f
f
ff
f
ff
f
ff
f
f
Wf
f
ff
f
ff
f
ff
f
ff
f
ff
f
ff
f ff
f
ff
f
f
0.5f
ff
f
f
Bf
f
ff
f
ff
f
ff
f
f
342f
f
ff
f
ff
f
f
Bf
f
ff
f
ff
ff
f
ff
f
ff
f
f
0.37f
f
ff
f 63.27
f
f
ff
f
ff
f
ff
f
ff
f
ff
f
ff
0.37 = 0.52 [W = = = 121.67 g
0.5 B 342 0.52 0.52
(24) Calculate the mass of ascorbic acid ( Vitamin C, C6H8O6) to be dissolved in 75 g of
acetic acid tolower its melting point by 1.50C. Kf = 3.9 K kg mol-1.
@1
Let mass of ascorbic acid = W g Molar mass of ascorbic acid = 176 g mol

# Number of moles = f f
f
ff
f
ff
f
ff
f
W
176
f
f
f
Wf
f
ff
f
ff
f
ff

# Molality of the solution m = f


f
ff
ff
f
ff
f
ff
f
ff
f
ff
f f
= f
f
ff
f
ff
f
ff
f
ff
f
ff
f
ff
f
ff
f
ff
f
ff
f
ff
f
ff
f
ff
f
ff
` a 176 W
Mass of solvent = 75 g = 0.075 kg
0.075 0.075 B176
∆Tf =1.5 0 C = 1.5 K and Kf = 3.9 K kg mol @1

[ 1.5 = 3.9 ff
f
ff
f
ff
f
ff
f
ff
f
ff
f
ff
f
ff
f
ff
f
ff
f
ff
f
ff
f
ff
f
ff
[W = f
f
ff
f
ff
f
ff
f
ff
f
ff
f
ff
f
ff
f
ff
f
ff
f
ff
f
ff
f
ff
f
ff
f
ff
f
ff
f
ff
f
ff
f
ff
f
ff
f
W 1.5 B0.075 B176
We know that ∆ Tf = Kf m = 5.07 g
0.075 B 176 3.9
(25) Calculate the osmotic pressure in Pascal exerted by a solution prepared by dissolving
1.0 g of polymer of molar mass 185000 in 450 ml of water at 370C.

Mass of polymer = 1 g ; Molar mass of polymer = 185000 g mol ; Number of moles = f


f
ff
f
ff
f
ff
@1 1f
f
ff
f
ff
f
ff
f
ff
f
ff
185000
f
f
ff
f
ff
f
ff
f
f
1f
f
ff
f
ff
f
ff
f
ff
f
f
ff
f
ff
f
ff
f
ff
f
ff
f
ff
f
ff
f f
= f
f
ff
f
ff
f
ff
f
ff
f
ff
f
ff
f
f
` a 1f
f
ff
f
ff
f
ff
f
ff
f
ff
f
ff
f
ff
f
ff
f
ff
Volume of solution = 450 ml = 0.45 L # Concentration of solution C = 185000 M
0.45 0.45 B 185000
0 3 @1 @1
Temperature of solution = 37 C = 310 K R = 8.3 B10 Pa L K mol

We know that osmotic pressure π = CRT = ff


f
ff
f
ff
f
ff
f
ff
f
ff
f
ff
f
f
` a 1f
f
ff
f
ff
f
ff
f
ff
f
ff
f
ff
f
ff
f
ff
f
ff
B8.3 B10 3 B 310 = 30.96 Pa
0.45 B185000
(26) Concentrated nitric acid used in the laboratory work is 68 % by mass in aqueous
solution. What should be the molarity of such a sample of the acid if the density of the
solution is 1.504 g ml-1?
b c
68% by mass of aqueous solution of nitric acid [ 68 g HNO3 present in 100 g of solution

f
f
ff
f
ff
f
ff
f
ff
f
f
Masssf
f
ff
f
ff
f
ff
f f
f
ff
f
ff
f
f
100f
f
ff
f
ff
f
ff
# Volume of 100 g solution = = = 66.489 ml = 0.665 L
Density 1.504

ff
f
f
68f
ff
f
Number of moles of HNO3 = = 1.079
63
ff
ff
f
ff
f
ff
f
f
1.079f
ff
f
ff
f
# Molarity of the solution = = 16.23 M
0.665
(27) A solution of glucose in water is labeled as 10 percent w/w, what would be the molality
and mole fraction of each component in the solution? If the density of the solution is
1.2 g ml-1, then what shall be the molarity of the solution?

jksahu74@gmail.com 52 | P a g e
10 g of glucose is present in 100 g solution [ 10 g of glucose has been dissolved in 90 g water

# Volume of 100 g of solution = f f


f
ff
f
ff
f
ff
f
ff
f
ff
f
ff
f
ff
f
ff 100
= ff
f
ff
f
ff
f
ff
f
Mass
= 83.3 ml = 0.0833 L
Density 1.2

Number of moles of glucose = f f


ff
f
ff
f
ff
f
f
Number of moles of water = f
f
ff
f
ff
f
10 90
= 0.055 =5
180 18

# Molarity of solution = ff
ff
f
ff
f
ff
f
ff
f
ff
f
ff
f
ff
f
f
0.055
= 0.67 M
0.0833
Mass of water = 90 g = 0.09 kg

# Molality of solution = f
f
ff
f
ff
f
ff
f
ff
f
ff
f
ff
0.055
= 0.617 m
0.09
Total number of moles of solute and solvent = 0.055 + 5 = 5.055
f
ff
f
ff
f
ff
f
f
0.055f
f
ff
f
ff
f ff
f
ff
f
f
5f
f
ff
f
ff
f
ff
f
f
# Mole fraction of glucose = = 0.01 and Mole fraction of water = = 0.99
5.055 5.055
(28) How many ml of 0.1M HCl are required to react completely with 1 g mixture of Na2CO3
and NaHCO3 containing equimolar amount of the two.
Mass of mixture = 1 g

` a
Let mass ofNa2 CO3 = x g [ Mass ofNaHCO3 = 1 @ x g

@1 @1
Molar mass ofNa2 CO3 = 106 g mol and Moar mass of NaHCO3 = 84 g mol

According to the question : Number of moles of Na2 CO3 = Number of moles of NaHCO3

f
f
fxf
f
ff
f
ff
f
f 1f
f
ff
f
f
@f
f
ff
f
fxf
f
ff ff
f
ff
f
f
106f
f
ff
f
iAe = [ 84 x = 106 @ 106 x [ x= = 0.557 g
106 84 190
# Mass ofNa2 CO3 in the mixture = 0.557 g and Mass ofNaHCO3 = 1 @ 0.557 = 0.443 g

0.557 g Mass of HCl


Na2CO3
+ 2 HCl CO2 + 2 NaCl + H2O
106 g 73 g
0.557 g Mass of HCl
Mass of HCl required to react with Na2CO3 : =
106 g 73 g
0.443 g Mass of HCl
NaHCO3 + HCl CO2
+ NaCl + H2O
84 g 36.5 g
0.443 g Mass of HCl
Mass of HCl required to react with NaHCO3 : =
84 g 36.5 g

jksahu74@gmail.com 53 | P a g e
ff
f
ff
f
ff
f
ff
f
f
0.557f
f
ff
f
ff
f
ff
f
ff
f
f
B73f
f
ff
f
ff
f ff
f
ff
f
ff
f
ff
f
f
0.443f
f
ff
f
ff
f
f
Bf
f
ff
f
ff
f
ff
f
ff
f
f
36.5f
f
ff
f
f
Total mass of HCl required = + = 0.576 g
106 84
f
f
ff
f
ff
ff
f
f
0.576f
f
ff
f
fff
Number of moles of HCl = = 0.0157
36.5
f
f
ff
f
ff
f
ff
f
ff
f
ff
f
ff
ff
f
ff
f
ff
f
ff
ff
f
ff
ff
f
ff
ff
f
ff
f
ff
f
ff
f
ff
f
ff
f
ff
f
ff
f
fff
f
ff
f
ff
f
ff
f
ff
f
ff
f
[ 0.1 = f
f
ff
ff
f
ff
f
ff
f
ff
f
ff
f
ff
f
ff
f
ff
f
ff
f
ff
[ Volume of solA = ff
f
ff
f
ff
f
ff
f
ff
f
ff
f
ff
f
ff
f
Number of moles of HCl 0.0157 0.0157
Molarity of solution = = 0.157 L
Volume of solution in litre Vol of solA 0.1

(29) A solution is obtained by mixing 300 g of 25% solution and 400g of 40% solution by
mass. Calculate the mass percentage of the resulting solution.

f
f
ff
f
f
25f
f
ff
f
f
Amount of solute present in 300 g of 25% solution = B300 = 75 g
100

Amount of solute present in 400 g of 40% solution = f


ff
f
ff
f
ff
f
ff
40
B400 = 160 g
100
Total mass of the solute = 75 + 160 = 235 g
Total mass of solution = 300 + 400 = 700 g
ff
f
ff
f
f
235f
f
ff
f
f
Mass percentage of resulting solution = B 100 = 33.57 %
700
(30) An antifreeze solution is prepared from 222.6 g of ethylene glycol (C2H6O2) and 200 g of
water. Calculate the molality of the solution. If the density of the solution is
1.072 g mL–1, then what shall be the molarity of the solution?
Mass of solvent = 200 g = 0.2 kg Mass of solution = 200 + 222.6 = 422.6 g
@1
Density of solution = 1.072 g ml
f
f
ff
f
ff
f
ff
f
f
Massf
f
ff
f
ff
f
ff
f
ff
f 422.6
f
f
ff
f
ff
f
ff
f
ff
f
ff
f
ff
f
# Volume of solution = = = 394.2 ml = 0.3942 L
Density 1.072
@1
Molar mass of ethylene glycol = 62 g mol

Number of moles of solute = f


f
ff
f
ff
f
ff
f
ff
f
ff
f
ff
222.6
= 3.59
62

Molality of solution = f
ff
f
ff
f
ff
f
ff
f
ff
f
ff
f
ff
f
ff
f
ff
f
ff
f
ff
ff
f
ff
f
ff
f
ff
f
ff
f
ff
f
ff
f
ff
f
ff
ff
f
ff
ff
f
ff
f
ff
f
ff
f
ff
f
ff
f
ff
f
ff 3.59
= ff
ff
f
ff
f
ff
f
ff
f
f
Number of moles of solute
= 17.95 m
Mass of solvent in kg 0.2

Molarity of solution = ff
f
ff
f
ff
f
ff
f
ff
f
ff
f
ff
f
ff
f
ff
f
ff
f
ff
ff
f
ff
f
ff
f
ff
f
ff
f
ff
f
ff
f
ff
f
ff
ff
f
ff
ff
f
ff
f
ff
f
ff
f
ff
f
ff
f
ff
f
ff
f f
= ff
f
ff
f
ff
f
ff
f
ff
f
ff
f
ff
f
ff
Number of moles of solute 3.59
= 9.10 M
Volume of solution in litre 0.3942
(31) The partial pressure of ethane over a solution containing 6.56 × 10–2 g of ethane is 1
bar. If the solution contains 5.00 × 10–2 g of ethane, then what shall be the partial
pressure of the gas?
.
Applying the Henry. s law m = kH P
@2 . . @2 @1
In the first case 6.56 B10 = kH B 1 bar [ kH = 6.56 B10 g bar
b c
. @2 @2
Putting k h in the second case 5.00 B10 g = 6.56 B10 Bp

ff
f
ff
f
ff
f
ff
f
ff
f
ff
f
ff
f
ff
f
ff
f
ff
f
ff
f
ff
f
ff
@2
5.00 B10
#p= @2
= 0.762 bar
6.56 B10
jksahu74@gmail.com 54 | P a g e
(32) An aqueous solution of 2% non-volatile solute exerts a pressure of 1.004 bar at the normal
boiling point of the solvent. What is the molar mass of the solute?

` a
2 % aqueous solution means ; Mass of solute = 2 g and Mass of sovent water = 100 @ 2 = 98 g

So number of moles of solute n2 = f


ff
f
ff
b c 2
@1
Let molar mass of solute = M g mol
M

Since molar mass of water = 18 g mol@ 1 So number ofmoles of water n1 = f ff


f
ff
f
f
b c 98
= 5.44
18
b c ` a
0
Vapour pressure of water P = 1.013 bar Given V A P of solution P = 1.004 bar
Pf
f
ff
f
ff
f
ff
f
ff
ff
f
ff
f
ff
f
= X2 = f
nff
fff f
= f
ff
f
ff
f
ff
f
ff
f
ff
f
ff
f
f
0
@ P 2 2
According to Raoult. s law 0
P n1 5.44M
ff
f
ff
f
ff
f
ff
f
f
1.013f
ff
f
ff
f
f
@f
f
ff
f
ff
f
ff
f
ff
f
ff
f
ff
f
f
1.004f
f
ff
f
f f f
f
ff
f
ff
f
f
2f
f
ff
f
ff
f
ff
f
ff @1
[ = [ M = 41 g mol
1.013 5.44 M
(33) Heptane and octane form an ideal solution. At 373 K, the vapour pressures of the two
liquid components are 105.2 kPa and 46.8 kPa respectively. What will be the vapour
pressure of a mixture of 25.0 g of heptane and 35 g of octane?

Number of moles of heptane = fff


f
ff
f
ff
f
ff
b c 25
@1
Molar mass of heptane C 7 H16 = 100 g mol = 0.25
100
f
f
ff
f
ff
f
ff
f
f
b c 35
@1
Molar mass of octane C 8 H18 = 114 g mol Number of moles of octane = = 0.307
114
f
f
ff
f
ff
f
ff
f
ff
f
ff
f
ff
f
ff
f
ff
f
ff
f
ff
f
ff
f
ff
f
ff
f
ff
b c 0.25
Mole fraction of heptane XH = = 0.449
0.25 + 0.307
ff
f
ff
f
ff
f
ff
f
ff
f
ff
f
ff
f
ff
f
ff
f
ff
f
ff
f
ff
f
ff
f
ff
f
f
b c 0.307
Mole fraction of octane XO = = 0.551
0.25 + 0.307
0 0
Total presssure of the mixture = PH XH + PO XO = 105.2 B0.449 + 46.8 B0.551 = 73.02 kPa

(34) The vapour pressure of water is 12.3 kPa at 300 K.


Calculate vapour pressure of 1 molal solution of a non-volatile solute in it.
1 Molal solution in water [ 1 mole of solute present in 1000 g of water

Number of moles of solvent = f


f
ff
f
ff
f
ff
f
ff
f
ff
f
1000
# Number of moles of solute = 1 and = 55.55
18
f
f
ff
f
ff
f
ff
f
ff
1f
f
ff
f
ff
f
ff
f
ff
f
ff
f
ff
Mole fraction of solute = = 0.0177
1 + 55.55
Pf
f
ff
f
ff
f
ff
f
ff
ff
f
ff
f
ff
f
0
@ P
According to Raoult. s law 0
= Mole fractio0n of solute
P
ff
f
ff
f
ff
f
f
12.3f
f
ff
f
f
@f
f
ff
f
ff
f
f
Pf
f
ff
[ = 0.0177 [ P = 12.08 kPa
12.3
(35) Calculate the mass of a non-volatile solute (molar mass 40 g mol–1) which should be
dissolved in 114 g octane to reduce its vapour pressure to 80%.

jksahu74@gmail.com 55 | P a g e
[ Number of moles of solute n2 = f f
ff
f
ff
f
f
b c w
Let mass of non @ volatile solute = w g
40
ff
f
ff
f
ff
f
ff
f
b c 114
@1
Molar mass of octane = 114 g mol [ Number of moles of solvent n1 = =1
114
b c ` a
0
If vapour ptressure of pure solvent P = 100 Then Vapour pressure of solution P = 80

Pf
f
ff
f
ff
f
ff
f
ff
ff
f
ff
f
ff
f
= X2 = f
nff
fff
[ ff
f
ff
f
ff
f
ff
f
ff
f
ff
f
ff
f
ff
f
ff
f
ff
f f
= f
f
ff
f
ff
f
0
@ P 2 100 @ 80 w
According to Raoult . s law 0
[w=8g
P n1 100 40

(36) A solution containing 30 g of non-volatile solute exactly in 90 g of water has a vapour


pressure of 2.8 kPa at 298 K. Further, 18 g of water is then added to the solution and
the new vapour pressure becomes 2.9 kPa at 298 K.
Calculate: (i) molar mass of the solute (ii) vapour pressure of water at 298 K.

Number of moles of solute n2 = f f


f
ff
f
ff
b c 30
@1
Let molar mass of solute = M g mol
M
ff
f
ff
f
ff
b c 90
@1
Molar mass of water = 18 g mol So number of moles of solvent n1 = =5
18
0 ` a
Let vapour pressure of solvent = P Vapour pressure of solution P = 2.8 kPa
Pf
f
ff
ff
ff
f
ff
ff
ff
f
ff
ff
f
f nf
f
ff
ff
[ f
Pf
f
fff
ff
f
ff
ff
f
fff
f
ff
ff
f
ff
ff 30
= ff
f
ff
ff
f
ff 6
= f
f
ff
ff
0 0
@ P 2 @ 2.8
According to Raoult. s law 0
= X2 = 0
P n1 P 5M M

[1@ f ff
ff
f
fff
f 6f
ff
f
ff ff
ff
f
ff
ff
f
2.8 2.8 6f
f
ff
ff M
f
ff
ff
f
f
@f
ff
f
ff
f
6f
f
ff
f f
ff
f
ff
f
2.8Mf
f
fff
f
ff
ff ` a
0
= [ 0
= 1 @ = [ 0
=M@6 ………………… 1
P M P M M P
When 18 g water is added to the solution, mass of solvent becomes 90 + 18 = 108 g

Now number of moles of solvent n1 = f


f
ff
ff
f
ff
ff
f
b c 108
=6
18
` a
New Vapour pressure of solution P = 2.9 kPa
Pf
f
ff
ff
f
ff
ff
ff
ff
f
ff
ff
f
f
= X2 = f
nff
fff
[ f
Pf
ff
ff
f
ff
ff
f
ff
fff
f
ff
ff
f
ff
ff 30
= ff
f
ff
ff
f
ff 5
= f
f
ff
ff
0 0
@ P 2 @ 2.9
According to Raoult. s law 0 0
P n1 P 6M M

[1@ f
ff
ff
f
fff
f 5
= f
ff
f
ff
[ f
ff
ff
f
fff
f
=1@ f
5f
f
ff
f M
= f
f
ff
ff
f
ff
ff
f
ff
5f
f
ff
f
[ ff
f
ff
ff
f
ff
ff
f
ff
ff
2.9 2.9 @ 2.9M ` a
0 0 0
=M@5 …………… 2
P M P M M P

Dividing equation 1 by equation 2 : ff


f
ff
f
ff
f
f M
= f
f
ff
f
ff
f
ff
f
ff
f
ff
f
ff
` a ` a 2.8 @ 6
[ 2.8M @ 14 = 2.9M @ 17.4
2.9 M @ 5
[ 2.9M @ 2.8M = 17.4 @ 14
[ 0.1M = 3.4 [ M = 34 g mol@ 1

Putting the value of M in equation 1 we get : f f


ff
f
ff
f
ff
f
ff
f
fff
f
ff
f
ff
f
ff
f
` a 2.8 B34
0
= 34 @ 6 = 28
P

[ P0 = ff
f
fff
f
ff
f
ff
f
ff
f
ff
f
ff
f
ff
f
ff
2.8 B 34
= 3.4 kPa
28
(37) A 5% solution (by mass) of cane sugar in water has freezing point of 271K. Calculate
the freezing point of 5% glucose in water if freezing point of pure water is 273.15 K.

jksahu74@gmail.com 56 | P a g e
Number of moles of cane sugar in 5 g = f f
f
ff
f
ff
f
ff
f
f
5
Mass of solvent = 95g = 0.095kg
342

Molality of the solution m = f


` a f f
ff
f
ff
f
ff
f
ff
f
ff
f
f
5f
f
ff
f
ff
f
ff
f
ff
f
ff
f
ff
f
ff
= 0.153
0.095 B 342
b c
Depression of freezing point ∆Tf = 273.15 @ 271 = 2.15

[ Kf = ff
f
ff
f
ff
f
ff
f
f f
= f
ff
f
ff
f
ff
f
ff
f
ff
f
ff
∆T 2.15
∆Tf = Kf m = 14.05
m 0.153

f
f
f
5f
f
ff
f
ff
f
f
Number of moles of glucose in 5 g = = 0.027 Mass of solvent = 95 g = 0.095 kg
180
` a ff
f
ff
f
ff
f
ff
f
f
0.027f
f
ff
f
f
Molality of solution m = = 0.28
0.095
∆Tf = Kf m = 14.05 B0.28 = 3.9 Freezing point of solution = 273.15 @ 3.9 = 269.25 K

(38) Two elements A and B form compounds having formula AB2 and AB4. When dissolved
in 20 g of benzene (C6H6), 1.0 g of AB2 lowers the freezing point by 2.3 K
whereas 1.0 g of AB4 lowers it by 1.3 K. The molar depression constant for benzene
is 5.1 K kg mol–1. Calculate atomic masses of A and B.
Mass of solvent = 20 g = 0.02 kg

[ Number of moles ofAB2 = f


@1 1f
f
ff
f
f
Let molart mass of AB2 = M1 g mol
M1

Molality of solution m = f
` a fff
f
ff
f
1f
f
ff
f
ff
f
ff
f
ff
f
∆Tf = 2.3 and Kf of benzene = 5.1 K kg mol@ 1
0.02M1

[ 2.3 = 5.1 B f f
f
ff
f
ff
1f
f
ff
f
ff
f
ff
f
ff
f
[ M1 = ff
f
ff
f
ff
f
ff
f
ff
f
ff
f
ff
ff
f
ff
f
ff
f
ff
f
ff
5.1 @1
∆Tf = Kf m = 110.87 g mol
0.02M1 2.3 B0.02
@@@@@@ @@@@@@ @@@@@@ @@@@@@ @@@@@@ @@@@@@ @@@@
[ Number of moles ofAB4 = f
@1 1f
f
ff
f
ff
Let molar mass of AB4 = M2 g mol
M2

Molality of solution m = f
` a fff
f
ff
f
f
1f
f
ff
f
ff
f
ff
f
ff
f
∆Tf = 1.3 K
0.02M2

[ 1.3 = 5.1 B ff
fff
f
ff
f
ff
f
ff
f
ff
f
ff
f
ff
[ M2 = f
f
ff
f
ff
f
ff
f
ff
f
ff
f
ff
f
ff
f
ff
f
ff
f
ff
f
f
1 5.1 @1
∆Tf = Kf m = 196.15 g mol
0.02M2 1.3 B0.02
@@@@ @@@@ @@@@ @@@@ @@@@ @@@ @@@@ @@@@ @@@@ @@@@ @@@@ @
@1 @1
Let atomic mass of A = a g mol and atomic mass of B = b g mol
` a
# Molar mass of AB2 : a + 2b = 110.87 ……………… 1
` a
Molar mass ofAB4 : a + 4b = 196.15 ……………… 2
@1 @1
On solving Atomic mass of A = 25.59 g mol and Atomic mass of B = 42.64 g mol

(39) At 300 K, 36 g of glucose present in a litre of its solution has an osmotic pressure of
4.98 bar. If the osmotic pressure of the solution is 1.52 bars at the same temperature,
what would be its concentration?

jksahu74@gmail.com 57 | P a g e
Strength = Molar mass B Molarity [ Concentration of solution C1 = f
f
ff
f
ff
f
ff
f
ff
f
ff
f
ff
f
ff
f
ff
f
ff
f
ff
f
ff
f
ff
f
f 36
= ff
f
ff
f
ff
f
ff
f
b c Strength
= 0.2 M
Molar mass 180
` a
Osmotic pressure π1 = 4.98 bar

[ f
πf
f
ff
ff
iAe f
πf
f
f
1f
f
f πf
= f f
f
2f
f
ff
b c
1
π1 = C1 RT = RT $ At the same temperature
C1 C1 C2
` a b c
New osmotic pressure π 2 = 1.52 bar We have to calculate the concentration C2

ff
f
ff
f
ff
f
ff
f
ff
f 1.52
= f
f
ff
f
ff
f
ff
f
ff
f
[ C2 = ff
f
ff
f
ff
f
ff
f
ff
f
ff
f
ff
f
ff
f
ff
f
ff
f
ff
f
f
4.98 0.2 B 1.52
= 0.061 M
0.2 C2 4.98

(40) If the density of some lake water is 1.25g ml–1 and contains 92 g of Na+ ions per kg of
water, calculate the molality of Na+ ions in the lake.
ff
f
f
92f
f
ff
f
f
ff
f
ff
f
ff
f
ff
f
ff
f
ff
f
ff
f
ff
f
ff
f
ff
f
ff
f
ff
f
ff
f
ff
f
ff
f
ff
f
ff
f
ff
f
ff
f
ff
f
ff
f
ff
f
ff
f
ff
f
ff
f
ff
f
ff
ff
f
ff
f
ff
f
ff
f
ff
f 23
f
f
ff
ff
f
f
+
+ Number of moles ofNa ions
Molality ofNa ions = = =4m
Mass of water in kg 1

(41) If the solubility product of CuS is 6 × 10–16, calculate the maximum molarity of CuS in
aqueous solution.
@1
Maximum molarity of CuS in aqueous solution = Solubility of CuS in molL
B CB C
@1 2+ 2@ 2
If S is the solubility of CuS in molL , Then KSP = Cu S = S BS = S
w
w
w
w
ww
w
w
w
ww
w
w
w
ww w
w
w
w
ww
w
w
w
ww
w
w
w
ww
w
w
w
ww
w
w
w
ww
w
w
w
ww
w
w
w
ww
w
w
w
ww
w
[ S = qKSP = q6 B10
@ 16 @8 @1
= 2.45 B10 molL

(42) Calculate the mass percentage of aspirin (C9H8O4) in acetonitrile (CH3CN) when 6.5 g
of C9H8O4 is dissolved in 450 g of CH3CN.

f
f
ff
f
ff
f
ff
f
ff
f
ff
f
ff
f
ff
f
ff
f
ff
f
ff
f
ff
f
ff
f
ff
f
ff
f
ff
f
f
Massf
f
ff
f
ff
f
ff
f
f
off
f
ff
f
ff
f
ff
f
ff
f
ff
f
ff
f
ff
f
f
aspirinf
f
ff
f
ff
f
ff
f
ff
f
ff
f
ff
f
ff
f
ff
f
ff
f
ff
f
ff
f
ff
f
ff
f
f
Mass percent of aspirin = B 100
Mass of aspirin + Mass of acetonitrile
f
f
ff
f
ff
f
ff
f
ff
f
ff
f
f
6.5f
ff
f
ff
f
ff
f
ff
f
f
= B100 = = 1.424 %
6.5 + 450

(43) Nalorphene (C19H21NO3), similar to morphine, is used to combat withdrawal symptoms


in narcotic users. Dose of nalorphene generally given is 1.5 mg.Calculate the mass of
1.5 x 10–3m aqueous solution required for the above dose.
@1
Molar mass of Nalorphene = 311 g mol
@3 @3
Mass of 1.5 B10 mole of Nalorphene = 1.5 B10 B311 = 0.466 g = 466 mg
@3 ` a
1.5 B10 m aqueous solution [ 466 mg of Nalorphene present in 1000 g of water solvent

Mass of solution = 1000 + 0.466 = 1000.466g


iAe 466 mg of Nalorphene present in 1000.466 g of solution
f
f
ff
f
ff
f
ff
f
ff
f
ff
f
ff
f
f
1000.466f
f
ff
f
ff
f
ff
f
[ 1.5 mg of Nalorphene present in B1.5 = 3.22 g
466

jksahu74@gmail.com 58 | P a g e
(44) Calculate the amount of benzoic acid (C6H5COOH) required for preparing 250 mL of
0.15 M solution in methanol.
@1
Let the mass of benzoic acid = w g Molar mass of benzoic acid = 122 g mol

Number of moles of solute = f


f
ff
f
ff
f
ff
f
f
w
Volume of solution = 250 ml = 0.25 Litre
122

Molarity of the solution = f


f
ff
f
ff
f
ff
f
ff
f
ff
f
ff
f
ff
f
ff
f
ff
f
ff
f
fff
f
ff
f
ff
f
ff
f
ff
f
ff
f
ff
f
ff
f
ff
ff
f
ff
ff
f
ff
f
ff
f
ff
f
ff
f
ff
f
ff
f
ff
[ 0.15 = f
ff
f
ff
f
ff
f
ff
f
ff
f
ff
f
ff
f
ff
f
ff
f
ff
f
ff
f
ff
Number of moles of solute w
Volume of solution in litre 0.25 B 122
` a
# Amount of benzoic acid required w = 0.15 B0.25 B122 = 4.575 g

(45) Calculate the depression in the freezing point of water when 10 g of


CH3CH2CHClCOOH is added to 250 g of water. Ka = 1.4 × 10–3, Kf = 1.86 K kg mol–1.

@1 f
f
ff
f
ff
f
f
10f
f
ff
f
ff
f
ff
Molar mass of the acid = 122.5 g mol So number of moles present in 10 g =
122.5
` a f
f
ff
f
ff
f
ff
f
ff
f
ff
f
ff
f
f
10f
f
ff
f
ff
f
ff
f
ff
f
ff
f
ff
f
f
Mass of the solvent = 250 g = 0.25 kg # Molality of the solution m = = 0.3264 m
w v w
ww
w
w
ww
w
w
w
ww
w
w
ww
w
w
ww
w
w
ww
w
w
ww
w
w
ww
w
w
ww
w
w
ww
w
w
ww
w
w
ww
122.5 B0.25
v
ww
w
w
w
ww
w
w
w
ww u
u u
uKf
ff
f
fff f
ff
f
ff
f
ff
f
ff
f
ff
f
ff
f
ff
f
ff
f
ff
f
f
@3
f
f
ff
=t =t
a 1.4 B10
α = 0.065
C 0.3264

- +
CH3CH2CHClCOOH CH3CH2CHClCOO + H

Initial moles 1 0 0

Moles at equilibrium 1−α α α

`a f
f
ff
f
ff
f
ff
f
ff
f
ff
f
f
Numberf
f
ff
f
ff
f
ff
f
ff
f
ff
f
f
off
ff
f
ff
f
ff
f
ff
f
ff
f
f
molesf
f
ff
f
ff
f
ff
ff
f
ff
ff
f
ff
f
f
afterf
ff
f
ff
f
ff
f
ff
f
ff
f
ff
f
ff
f
ff
f
ff
f
ff
f
ff
f
ff
f
f
dissociationf
f
ff
f
ff
f
ff
f
f
Van. t Hoff factor i =
Number of moles before dissociation
1f
f
ff
f
f
@f
f
ff
f
ff
f
f
αf
ff
f
f
+f
f
ff
f
ff
f
f
αf
ff
f
f
+f
f
ff
f
ff
f
f
αf
ff
= = 1 + α = 1 + 0.065 = 1.065
1
b c
0
Depression in freezing point of water ∆Tf = i Kf m = 1.065 B 1.86 B 0.3264 = 0.65 C

(46) 19.5 g of CH2FCOOH is dissolved in 500 g of water. The depression in the freezing
point of water observed is 1.00 C. Calculate the van’t Hoff factor and dissociation
constant of fluoroacetic acid.
0 @1
∆Tf =1.0 C = 1.0K and k f for water = 1.86 K kg mol
Mass of solvent = 500g = 0.5 kg

Number of moles ofCH2 FCOOH = f


f
ff
f
ff
f
ff
f
ff
f
@1 19.5
Molar mass ofCH2 FCOOH = 78 g mol = 0.25
78

Molality of the solution = ff


f
ff
f
ff
f
ff
f
ff
f
ff
f
ff
f
ff
f
ff
f
ff
f
ff
ff
f
ff
f
ff
f
ff
f
ff
f
ff
ff
ff
ff
f
ff
f
ff
f
ff
ff
fff
ff
f
ff
ff
f
ff
ff
f
ff
f
f 0.25
= ff
ff
f
ff
f
ff
f
ff
ff
number of moles of solute
= 0.5 m
Mass of solvent in kg 0.5

[ Van. t Hoff factor i = f f


ffff
f
ff
f
ff
f
ff f
= f f
f
ff
f
ff
f
ff
f
ff
ff
f
ff
f
ff
f
ff
ff
f
ff
f
f
` a ∆T 1
∆Tf = i k f m = 1.0753
k f m 1.86 B 0.5
@@ @@ @ @@ @@ @@ @ @@ @@ @@ @ @@ @@ @@ @ @@ @@ @@ @ @@ @@ @@ @ @@ @@ @@ @ @
Suppose degree of dissociation at the given concentration α

jksahu74@gmail.com 59 | P a g e
- +
CH2FCOOH CH2FCOO + H

Initial moles 1 0 0

Moles at equilibrium 1−α α α

`a f
f
ff
f
ff
f
ff
f
ff
f
ff
f
f
Numberf
f
ff
f
ff
f
ff
f
ff
f
ff
f
f
offf
f
ff
f
ff
f
ff
f
ff
f
f
molesf
f
ff
f
ff
f
ff
ff
f
ff
ff
f
ff
f
f
afterf
ff
f
ff
f
ff
f
ff
f
ff
f
ff
f
ff
f
ff
f
ff
f
ff
f
ff
f
ff
f
f
dissociationf
f
ff
f
ff
f
ff
f 1f
f
ff
f
f
+f
f
ff
f
ff
f
αf
f
ff
Van. t Hoff factor i = = =1 + α
Number of molesbefore dissociation 1
α = i @ 1 = 1.0753 @ 1 = 0.0753

B CB C
+
f
f
ff
f
ff
f
ff
f
f2f
f
ff
f
ff
f
ff
f
ff
f
ff
f
ff
f
ff
f
ff
f
ff
f
ff
f
ff
f
ff
f
ff
f
ff
f
ff
f Cα
ff
f
ff
f
ff
f
ff
f
ff
f
ff
f
ff
f
ff
f
ff
f
ff
f
f
@
CH
b cFCOO H B Cα ` a` a2 @3
2
Dissociation constant Ka = B C = ` a = Cα = 0.5 0.0753 = 2.853 B10
CH2 FCOOH C 1@α

(47) Vapour pressure of water at 293 K is 17.535 mm Hg. Calculate the vapour pressure of
water at 293 K when 25 g of glucose is dissolved in 450 g of water.

0
P = 17.535 mm Hg P =?

Number of moles present in 25 g of glucose = ff


ff
f
ff
f
ff
f
f
25
= 1.39
180

Number of moles present in 450 g of water = ff


f
ff
f
ff
f
ff
ff
450
= 25 A
18

ff
f
f
∆Pf
f
fff
f f
f
ff
f
nf
ff
f
2f
f
ff
f
ff
f
ff nf
f
f
2f
ff 1.39
ff
f
ff
f
ff
f
ff
f
ff
0
= X2 @ = = = 0.055
P n1 + n2 n1 25
Pf
f
ff
f
ff
f
ff
f
ff
ff
f
ff
f
ff
f f
f
ff
f
ff
f
Af
f
ff
f
ff
f
ff
f
ff
f
ff
f
ff
f
ff
f
ff
f
ff
f
ff
f
0
@ P 17 535 @ P
0
= 0.055 [ = 0.055 [ P = 16.57 mm Hg
P 17.535
(48) Henry’s law constant for the molality of methane in benzene at 298 K is 4.27 × 105 mm
Hg. Calculate the solubility of methane in benzene at 298 K under 760 mm Hg.

[X = f
f
ff
f
ff
f f
= f
f
ff
f
ff
f
ff
f
ff
f
ff
f
ff
f
ff
f
ff
f
ff
f
ff
ff
P 760 @3
According to Henry. s law P = KH X 5
= 1.78 B10
KH 4.27 B10

(49) 100 g of liquid A (molar mass 140 g mol–1) was dissolved in 1000 g of liquid B (molar
mass 180 g mol–1). The vapour pressure of pure liquid B was found to be 500 torr.
Calculate the vapour pressure of pure liquid A and its vapour pressure in the solution
if the total vapour pressure of the solution is 475 Torr.

ff
ff
f
f
100f
f
ff
f
f
Number of moles present in 100 g of A = = 0.71
140
f
ff
f
ff
f
f
1000f
f
ff
f
ff
f
f
Number of moles present in 1000 g of B = = 5.55
180

f
f
ff
f
ff
f
ff
f
ff
f
ff
f
ff
f
f
0.71f
f
ff
f
ff
f
ff
f
ff
f
ff
f
f
Mole fraction of A = = 0.113 # Mole fraction of B = 1 @ 0.113 = 0.887
0.71 + 5.55

jksahu74@gmail.com 60 | P a g e
0 0
PB = 500 torr PA = ?
PTotal = PA + PB = X A PA0 + XB PB0 = 0.113 PA0 + 0.887 PB0
0
[ 475 = 0.113PA + 0.887 B 500

# PA = f
f
ff
f
ff
f
ff
f
ff
f
ff
f
ff
0 0 32
[ 0.113PA = 475 @ 443 = 32 = 283 torr
0.113
0
# PA = X A PA = 0.113 B283 = 32 torr

(50) Benzene and toluene form ideal solution over the entire range of composition. The
vapour pressure of pure benzene and naphthalene at 300 K are 50.71 mm Hg and
32.06 mm Hg respectively. Calculate the mole fraction of benzene in vapour phase
if 80 g of benzene is mixed with 100 g of naphthalene.
b c
0
Vapour pressure of pure benzene PB = 50.71 mm Hg
b c
Mole fractrion of benzene in vapour phase yB = ?
b c
0
Vapour pressure of pure naphthalene PN = 32.06 mm Hg

Number of moles present in 80 g of benzene nB = f f


f
ff
f
ff
b c 80
= 1.02
78

Number of moles present in 100 g of naphthalene nN = f f


f
ff
f
ff
f
ff
f
b c 100
= 0.78
128

PTotal = PB XB + PN XN = 50.71 B f f
f
ff
f
ff
f
ff
ff
ff
f
ff
f
ff
f
ff
f
ff
f
ff
ff
f
ff
f
ff
+ 32.06 B f f
ff
f
ff
f
ff
f
ff
f
ff
f
ff
f
ff
f
ff
f
ff
f
ff
f
ff
f
ff
f
0 0 1.02 0.78
= 42.65 mm Hg
1.02 + 0.78 1.02 + 0.78

Partial pressure of benzene PB = XB PB = f ff


f
ff
f
ff
f
ff
ff
f
ff
f
ff
f
ff
f
ff
f
ff
f
ff
f
ff
f
ff
b c 1.02 B C
0
B50.71 = 28.73 mm Hg In liquid phase
1.02 + 0.78
b c B C
Partial pressure of benzene PB = yB PTotal In vapour phase

# Mole fraction of benzene yB in vapour phase = f


f
ff
f
ff
f
fff
f
ff
f 28.73
= f
f
ff
f
ff
f
ff
f
ff
f
ff
f
ff
f
b c P B
= 0.674
PTotal 42.65

(51) The air is a mixture of a number of gases. The major components are oxygen and
nitrogen with approximate proportion of 20% is to 79% by volume at 298 K. The water
is in equilibrium with air at a pressure of 10 atm. At 298 K if the Henry’s law constants
for oxygen and nitrogen at 298 K are 3.30 × 107 mm and 6.51 × 107 mm respectively,
calculate the composition of these gases in water.
Total pressure of air in equlibrium with water = 10 atm

20% oxygen means partial pressure of oxygen pO2 = f ff


f
ff
ff
f
ff
f
b c 20
B10 = 2 atm = 2 B760 = 1520 mm Hg
100

79% nitrogen means partial pressure of nitrogen pN2 = f f


ff
f
ff
ff
f
ff
b c 79
B10 = 7.6 atm = 7.9 B 760 = 6004 mm Hg
100

[ XO2 = f
pff
ff
fff
f f
= fff
ff
f
ff
f
ff
ff
f
ff
f
ff
f
ff
ff
f
ff
f
ff
fff
O 1520 @5
Applying Henry. s law pO2 = kH XO2 2
7
= 4.61 B10
kH 3.30 B10

[ XN2 = f
pf
ff
ff
ff
f f
= f f
ff
f
ff
ff
f
ff
f
ff
f
ff
ff
f
ff
f
ff
f
ff
ff
ff
N 6004
pN2 = kH XN2 2
7
= 9.22 B10@ 5
kH 6.51 B10

jksahu74@gmail.com 61 | P a g e
(52) Determine the amount of CaCl2 (i = 2.47) in 2.5 L of water such that its osmotic
pressure is 0.75 atm at 27°C.
@1
Let mass ofCaCl2 = w g Molar mass ofCaCl2 = 111 g mol
f
f
f
wf
f
ff
f
ff
f
# Number of moles of CaCl2 = Volume of solution = 2.5 litre
111
f
f
f
wf
f
ff
f
ff
f
f
ff
f
ff
f
ff
f
ff
f
ff
f
f
Number
` a f
f
ff
f
ff
f
ff
f
ff
f
f
off
f
ff
f
ff
f
ff
f
ff
f
ff
f
f
molesf
f
ff
f
ff
f
ff
ff
f
ff
f
ff
f
ff
f
ff
f
f
ofCaClf
f
ff
f
ff
f
f
2f
ff 111
f
f
ff
f
ff
f
ff f
f
ff
f
ff
f
ff
f
ff
f
f
wf
f
ff
f
ff
f
ff
f
ff
f
ff f
f
ff
f
ff
f
f
wf
f
ff
f
ff
f
ff
# Concentration of solution C = = = =
Volume of solution in litre 2.5 111 B2.5 277.5
0
Applying osmotic pressure equation: π = i CRT Temperature = 27 C = 300K

[ 0 A 75 = 2.47 B f
f
ff
f
ff
f
ff
f
ff
f
ff
f
ff
[w= ff
f
ff
f
ff
f
ff
f
ff
f
ff
f
ff
f
ff
f
ff
f
ff
f
ff
f
ff
f
ff
f
ff
f
ff
f
ff
f
ff
f
ff
f
ff
f
ff
f
ff
f
ff
f
ff
f
w 0.75 B277.5
B0.0821 B300 = 3.42 g
277.5 2.47 B 0.0821 B300
(53) Determine the osmotic pressure of a solution prepared by dissolving 25 mg of K2SO4
in 2 litre of water at 25° C, assuming that it is completely dissociated.

Number of moles of K2 SO4 present in 25 mg i A e 0.025 g = f


f
ff
f
ff
f
ff
f
ff
f
ff
f
ff
b c 0.025
@4
= 1.44 B10
174
Since K2 SO4 dissociates completely as : K2 SO4 @@ @Q 2 K + + SO42 @
`a
So Van. t Hoff factor i = 3

f
f
ff
f
ff
f
ff
f
ff
f
ff
f
ff
ff
f
ff
f
ff
f
ff
f
ff
f
ff
f
f
@4
1.44
` a B10 @5
Concentration of solution C = = 7.18 B10
2
` a 0
Temperature of solution T = 25 C = 25 + 273 = 298K
Applying Osmotic pressure equation : π = i CRT
@5 @3
= 3 B7.18 B10 B0.0821 B 298 = 5.27 B10 atm
(54) A sample of drinking water was found to be severely contaminated with CHCl3. The
level of contamination was 15 ppm ( by mass). (i) Express this in percent by mass
(ii) Determine the molality of the sample
`a 6
i 15 ppm means ; 15 g ofCHCl3 is present in10 g of solution
f
f
ff
f
ff
f
ff
f
f
Massf
f
ff
f
ff
f
ff
f
ff
f
f
off
ff
f
ff
f
ff
f
ff
f
f
CHClf
f
ff
f
ff
f
3f
f
ff
f f
ff
f
f
15f
ff
fff @3
# % by mass = B100 = 1.5 B10
B100 =
6
Mass of solution 10
` a ff
f
ff
f
f
15f
f
ff
f
ff
f
ff
f
f
ii Number of moles present in 15 g ofCHCl3 =
119.5
6 6 3
Mass of solvent =10 @ 15 ≈10 g =10 kg
ff
f
ff
f
ff
f
ff
f
ff
f
f
Numberf
f
ff
f
ff
f
ff
f
ff
f
f
off
f
ff
f
ff
f
ff
f
ff
f
ff
f
f
molesf
f
ff
f
ff
f
ff
ff
f
f
off
ff
f
ff
f
ff
f
ff
f
ff
f
ff
f
f
solutef
f
ff ff
f
ff
f
ff
f
ff
f
ff
f
ff
f
f
15f
f
ff
f
ff
f
ff
f
ff
f
ff
fff @4
# Molality of the solution = = 3
= 1.25 B10 m
Mass of solvent in kg 119.5 B10

(55) 75.2 g of phenol is dissolved in solvent of kf =14. If the depression in freezing point
is 7 K , then find the % of phenol that dimerise.
@1
Let experimental molar mass of phenol = M g mol

Number of moles of phenol = ff


f
ff
f
ff
f
ff
f
ff
f
75.2
; ∆Tf = 7 K k f = 14 means 14 K kg mol@ 1
M

jksahu74@gmail.com 62 | P a g e
# Molality of the solution m = f f
f
ff
f
ff
f
ff
f
ff
f
` a 75.2
Hence mass of solvent is taking as 1 kg
M

[ 7 = 14 B f
f
ff
f
ff
f
ff
f
ff
f
f
[ M= f f
f
ff
ff
ff
f
ff
f
ff
f
ff
f
ff
f
ff
f
ff
f
ff
f
75.2 14 B 75.2 @1
We know that ∆Tf = k f m = 150.4 g mol
M 7
@1
Theoretical molar mass of phenol = 94 g mol

# Van. t Hoff factor i = f


f
ff
f
ff
f
ff
f
ff
f
ff
f
ff
f
ff
f
ff
f
ff
f
ff
f
ff
f
ff
f
ff
f
ff
f
ff
f
ff
f
ff
f
ff
f
ff
f
ff
f
ff
f
ff
f
ff
f
ff
f
ff
f
ff
f f
= f
f
ff
f
ff
f
ff
f
ff
f
ff
f
f
` a Theoretical molar mass 94
= 0.625
Observed molar mass 150.4
If α is the degree of association, then
2 C 6 H 5 OH ( C 6 H 5 OH ) 2
Initial moles 1 0

Moles at equilibrium 1−α α


-
2

αf
f
ff
f αf
f
ff
f
Total moles after association = 1 @ α + =1@
2 2
`a f ff
f
ff
f
ff
f
ff
f
ff
f
ff
f
f
Numberf
f
ff
f
ff
f
ff
f
ff
f
f
off
f
ff
f
ff
f
ff
f
ff
f
ff
f
f
molesf
f
ff
f
fff
f
ff
f
ff
f
ff
f
ff
f
f
afterf
ff
f
ff
f
ff
f
fff
f
ff
f
ff
f
ff
f
ff
f
ff
f
ff
f
f
associationf
f
ff
f
ff
f
ff
f
ff
f
f αf
f
ff
f
Van. t Hoff factor i = =1@
Number of moles before association 2
0.625 = 1 @ f
αf
ff
f
[ α = 0.75 [ % of association = 75 %
2
(56) What mass of 95% sulphuric acid ( density 1.85 g / cm3) and what mass of water must
be taken to prepare 100 cm3 of 15% solution of sulphuric acid ( d= 1.10 g/cc)

ff
f
ff
f
f
100f
f
ff
f
ff ff
f
f
1f
f
ff
f
ff
f
ff
Molarity of 95% Sulphuric acid density 1.85 g cm@ 3 [ Volume of solution = ml = L
1.85 18.5
ff
f
f
95f
f
ff ff
f
ff
ff
f
0.97f
ff
f
ff
f
Number of moles of acid = = 0.97 # Molarity of solution = 1 = 0.97 B18.5 = 17.94 M
98 f
f
ff
f
ff
f
ff
f
ff
f
ff
18.5

[ Volume of solution = f f
f
ff
f
ff
f
ff
f
ff
ml = f
100 1f
f
ff
f
f
Molarity of 15% sulphuric acid density 1.10 g cm@ 3 L
1.10 11

Number of moles of acid = f


f
ff
f
ff
f
# Molarity of solution = f
f
ff
f
ff
f
fff
f
ff
f
ff
f
f
15 0.153
= 0.153 = 0.153 B 11 = 1.68 M
98 1f
f
ff
f
ff
11
b c b c
Applying molarity equation : M1 V1 95% H2 SO4 = M2 V 2 15% H2 SO4

[ V1 = f f
f
ff
f
ff
ff
f
ff
f
ff
f
ff
f
ff
f
ff
f
ff
f
ff
f
ff
f
f
1.68 B 100
17.94 BV1 = 1.68 B 100 = 9.37 ≈ 9.4 cc
17.94
# Mass of 9.4 cc of 95% H2 SO4 = 9.4 B 1.85 = 17.4 g
Mass of 100 cc of 15% H2 SO4 to be prepared = 100 B 1.10 = 110 g
# Mass of water taken = 110 @ 17.4 = 92.6 g

jksahu74@gmail.com 63 | P a g e
Electrochemistry deals with the relationship between electrical energy and chemical changes
taking place in redox reactions.

Metallic conduction Electrolytic conduction


Movement of electrons results in the flow of Movement of cations and anions results in the
electric current flow of electric current.
No ionization of the metallic conductor takes Ionisation of the electrolyte occurs.
place.
There is no transfer of matter Ions move from one terminal to another
Decrease with increase in temperature Increase with increase in temperature.

The non-metallic conductors are :Carbon black ; Graphite ; Polyaniline ; Polypyrrole ;


Polythiophene ; Polyacetylene when exposed to iodine.

Resistance (R) : It measures the obstruction to the flow of current. It is not an additive property.

It is measured in ohm ( Ω )
In SI system 1 ohm = 1 kg. m2 . s-3 A-2
Resistance is measured with help of Wheat stone bridge.
Resistance of an object depends on :
1) Nature of the conductor

2) Length of conductor ( l ) : R ∝ l
1f
f
ff
3) Cross sectional area of conductor (a) : R ∝
a
So Ohm’s law is define as the electrical resistance of any conductor is directly proportional to the
length and inversely proportional to the area of cross – section.
lf
f
ff lf
f
ff
So that: R ∝ = > R = ρ
a a
Where, ρ (rho) is constant of proportionality and is known as resistivity.

If l = Unit length ( 1 cm or 1m ) and a = Unit area ( 1 cm2 or 1 m2 ), Then R = ρ

Measurement for resistance of a solution of an electrolyte with help of Wheatstone bridge


The conductivity cell having unknown resistance (R2) is connected to one
arm of the Wheatstone bridge containing two fixed ( R3 & R4 ) and one
variable (R1) resistance as shown.
The Wheatstone bridge is fed by an oscillator (O) { a source of a.c power in
the audio frequency range 550 to 5000 cycles per second} and a suitable
detector (P) { a headphone or other electronic device} . The bridge is
balanced when no current pass through the detector.

jksahu74@gmail.com 64 | P a g e
At equilibrium R 1 R 4 = R 2 R 3

So the unknown resistance (R2) of conductivity cell = f


Rf
f
f
1f
ff
f
f
Rf
f
f4f
f
ff
R3

Resistivity ( ρ ) : It may be defined as the resistance of a solution held between electrodes of unit
area of cross section and unit distance apart.

ff
f
f
Raff
f
f
ρ =
l
Unit of ρ is ohm.cm or ohm.m

1 ohm .m = 100 ohm. Cm

Conductance (G) : It is a measure of the ease with which current flow through a conductor. It is
an additive property.

G= f
1f
f
ff
It is reciprocal of the resistance, i.e.
R
Unit of G is ohm-1 or mho or Ω-1
In SI system, unit of G is Siemens, (S) 1 S = 1 Ω -1

Conductivity ( κ = kappa ) : It is the reciprocal of resistivity.


h ih i
1f
f
ff j 1f
f
ff
f
kj f
lf
f
f ` a`
k = G GC = Conductance Cell constant
a
κ = =
ρ R a

C lf
f
ff
G = is called cell constant because for a cell its value is fixed.
a
If l = Unit length ( 1 cm or 1m ) and a = Unit area ( 1 cm2 or 1 m2 ), Then κ = G
Thus Conductivity may be defined as the conductance of a solution held between electrodes of
unit area of cross section and unit distance apart.
Conductance of unit volume of a solution is also called conductivity.

f
f
Gf
f
f
κ= ( V: Volume of solution )
V
In CGS unit, conductivity is the conductance of one cm3 solution. ( S cm-1 )
In SI unit , conductivity is the conductance of one m3 ( 1000 L) solution . ( S.m-1 )
1S cm-1 = 100 S.m-1
Conductivity of electrolyte depend upon :
1. Nature of Electrolyte : Conductivity of an electrolyte depends upon the nature of electrolyte on
the following points :
(i) Weak or strong electrolyte : A weak electrolyte furnishes fewer ions therefore it has lower
conductivity than a strong electrolyte of same concentration.
jksahu74@gmail.com 65 | P a g e
(ii) Valency of the ions : The ions with higher valency carry more charge and therefore they
conduct more charge than the ion of lower valency. Thus higher the
valency of the ion greater is the conducting power.
(iii) Speed of the ion : The ion which can move faster will carry the charge also faster and
therefore has more conducting power.
2.Size of ions produced and their solvation: It decreases with increase in size and solvation of
ions.
3. The nature of the solvent and its viscosity : It decreases with increase of viscosity of the
solvent.
4. Concentration of the electrolyte: It decreases with dilution because number of ions per unit
volume decreases
5. Temperature - it increases with the increase of temperature

Super conductor: Super conductors have zero resistivity or infinite conductivity


e.g. Ceramic materials and mixed oxides having super conductivity at temperature above 150 K

Molar conductivity ( Λ m ) : The conductance of all ions produced from one mole of electrolyte
is called molar conductivity.
Let molarity of the solution is C . i.e. 'C' moles of electrolyte is present in 1000 ml of solution

ff
f
ff
ff
f
f
1000f
ff
1 mole of electrolyte is present in ml of solution.
C
Let conductivity of the solution is κ Ω-1 cm-1
=> Conductance of 1 ml solution is κ Ω-1

ff
f
ff
ff
f
f
1000f
ff ff
f
fff
f
f
1000f
ff
f
Therefore conductance of ml of solution = κ B
C C
ff
ff
ff
f
f
1000f
ff
f @1 @1
λm = κ B Unit : Ω cm 2 mol
C
The molar conductivity increases with increase of dilution as degree of ionization increases.

κ Ω
@1
cm@ 1 Ω
@1
m@ 1 Ω
@1
m@ 1
C mol L@ 1 mol m@ 3 mol L@ 1

λm ff
ff
ff
f
1000f
ff
ff
f κfffff f
f
fff
ff
ff
f
κf
ff
ff
ff
ff
ff
ff
f
κB
C C 1000 B C

Infinite dilution / Zero concentration


The infinite dilution or zero concentration means the electrolytes undergoes complete dissociation
and attain maximum molar conductivity. After infinite dilution, conductivity will not change with
dilution.
jksahu74@gmail.com 66 | P a g e
b c
0
Limiting molar conductivity λ m

The molar conductivity of an electrolyte at infinite dilution is known as limiting molar conductivity.
Case - 1
1f
f
ff
0
For strong electrolyte : λ m = λ m @ AC 2
λ m : Molar conductivity at any concentration
0
λ m : Limiting molar conductivity
C : Concentration
A : Constant which depends upon nature of solvent,electrolyte and temperature.
ff
f
f 1
` a 0
The equation can be written as λ m = @ A C 2 + λm
Straight line equation : y = mx + c
For strong electrolyte, limiting molar conductivity can be measured by graphical method.
Case - 2
For weak electrolyte, since degree of dissociation is very less as
compared to strong electrolyte, so molar conductivity increases very
slowly with dilution.
At very dilution, the mobility of ions increases, so molar conductivity
increases rapidly in such a manner that it remains almost parallel with
the λ m axis for which limiting molar conductivity cannot be measured
by graphical method.
Kohlrausch law of independent migration of ions.
The law states that limiting molar conductivity of an electrolyte can be represented as the sum of
the individual contributions of the anion and cationb of cthe electrolyte.
` a + ` @a
λ m AB = λ m A + λm B
Applications

[1] The law is used to calculate limiting molar conductivity of weak electrolyte CH3COOH & NH4OH
o o o
λm CH = λm - + λom H+
o
= λm CH3COONa + λmo HCl λm
3COOH CH3COO NaCl
o o o
λm NH4OH = λm NH+4 + λm OH - = λom NH4Cl + λom NaOH λom NaCl

[2] To calculate the degree of dissociation

λcm
α = o
λm

jksahu74@gmail.com 67 | P a g e
Electrolysis
A process in which on passing electricity electrolyte split into ions and ions are deposited at
opposite electrode is called electrolysis.
Faraday’s First law of electrolysis:
The amount of substance deposited or liberated during electrolysis is directly proportional to the
quantity of electricity passed through the solution.

W ∝ Q

> W ∝ I t
=> W = Z I t
Where Q : Quantity of electricity in coulomb.
W : Amount of substance liberated / deposited
I : Current in ampere
t : Time in second
Z : Proportionality constant called electrochemical equivalent (ECE)

f
f
f
Wf
f
ff
f
Z=
It
If I = 1 ampere and t = 1 second ; then Z=W
So, electrochemical equivalent is defined as the amount of substance deposited or liberated by
the passage of one ampere of current for one second i.e. one coulomb charge.
It is experimentally found that by passage of 1F = 96487 C electricity, the amount of substance
deposited/ liberated is equal with its chemical equivalent.( E )

f
f
ff
f
ff
f
f
Ef
f
ff
f
ff
f
ff
f
ff f
f
ff
f
ff
f
f
Ef
f
ff
f
ff
f
ff
f
ff ` a
Z= or Z= If 1 F is taken as 96500 C
96487 96500

Chemical equivalent of an element E = f


f
ff
f
ff
f
ff
f
ff
f
ff
f
ff
f
ff
f
ff
f
ff
f
ff
f
ff
f
ff
f
ff
f
ff
f
ff
f
ff
f
fff
f
ff
f
ff
f
ff
f
ff
f
ff
f
ff
f
ff
f
ff
f
ff
f
ff
f
ff
f
ff
f
ff
f
f
` a Atomic mass of the element
Valency of that element

Faraday's Second law of electrolysis

When same amount of current is passed through


different electrolytes which are connected in
series, the amount of substance deposited/
liberated from different electrolytes is directly
proportional to their equivalent weight.

That is W ∝ E

f
f
Wf
f
ff
f
Af
f
ff
f W
ff
f
ff
f
ff
f
f W
f
ff
f
ff
f
ff
f
f
= B = C
EA EB EC

jksahu74@gmail.com 68 | P a g e
f
ff
f
ff
f
ff
ff
f
= f
ff
f
ff
f
fff f
ff
f
ff
f
ff
ff
f Ef
= ff
f
ff
ff
f
W A W B W A A
=>
EA EB WB EB

Zf
f
ff
f
fff
f
f
If
f
ftf
f
f Ef
f
ff
ff
ff
f
=> f
Zf
f
ff
f
ff
f Ef
=> A
= A A
= ff
f
f
Af
ff
f
So Z ∝ E
ZB I t EB ZB EB

Thus the electrochemical equivalent( Z) of an element is directly proportional to its equivalent


weight (E).

Current Efficiency: The ratio of amount deposited or liberated and the amount expected from
Faraday’s law is called current efficiency.

ff
f
ff
f
ff
f
ff
f
f
Amounf
f
ff
f
ff
f
ff
f
tf
f
ff
f
ff
f
f
off
ff
ff
f
ff
f
ff
f
f
ionsf
f
ff
f
ff
ff
f
ff
f
ff
f
ff
f
ff
f
ff
f
ff
f
ff
f
f
depositedf
f
ff
f
ff
f
ff
f
ff
f
f
orf
f
ff
ff
ff
f
ff
f
ff
f
ff
f
ff
f
ff
f
ff
f
f
liberatedf
f
ff
f
ff
Current efficiency =
Amount of ions expected

Current efficiency also define as the ratio of current available to an ion to the total current passed
by the solution.

f
f
ff
f
ff
f
ff
f
ff
f
ff
f
ff
f
ff
f
ff
f
ff
f
ff
f
f
Currentf
f
ff
f
ff
f
ff
f
ff
f
ff
f
ff
f
ff
f
ff
f
ff
f
ff
f
f
availablef
f
ff
f
ff
ff
f
f
tof
ff
f
ff
f
ff
f
f
thef
f
ff
f
ff
f
ff
f
f
ionf
f
ff
f
ff
f
ff
f
ff
f
ff
f
ff f
f
ff
f
ff
f
ff
f
ff
f
ff
f
ff
f
f
Actualf
f
ff
f
ff
ff
f
ff
f
ff
f
ff
f
ff
f
ff
f
f
currentf
f
ff
f
ff
f
ff
Current efficiency = =
Current passed through the solution Ammeter current

Application of Electrolysis:

Extraction of metals
Purification of metals
Electroplating
Electro printing

Factors affecting the product of electrolysis:

Products of electrolysis depend on:

Nature of electrolyte : Molten / Aqueous solution


Nature of electrode : Reactive / Inert
Electrode potential :

Between two competing ions, that ion will discharge which have lower discharge potential
(External force)
On discharge cation reduces. So discharge potential will less if electrode potential ( driving force /
internal force) will high.
On discharge anion oxidizes. So discharge potential will less if oxidation potential ( driving force /
internal force) will high. Or in other word, that anion will discharge which have lower electrode
potential.
Electrodes are two types:
1) Inert electrode: Electrode which does not participate in the chemical reaction and acts only as
source or sink of electrons. e.g. Platinum (Pt) , Gold(Au).
2) Reactive electrode: Electrode which participates in the electrode reaction. e.g. Cu, Zn , Al , etc.

jksahu74@gmail.com 69 | P a g e
Over potential:
Some electrochemical reactions are thermodynamically feasible but kinetically very slow and
hence do not take place. These reactions are carried out using some extra potential. This is called
over voltage or over potential.
[1] Electrolysis of molten NaCl

NaCl Na + + Cl -
Anode 2 Cl - Cl2 + 2e-
Cathode Na + + e- Na

[2] Electrolysis of aqueous NaCl [The product of electrolysis depend upon nature of electrolyte]

NaCl Na + + Cl -
H2O H+ + OH -

Anode { Cl- , OH- } :

Between OH- and Cl-, the oxidation potential of OH- is greater than that of Cl-, So OH- should
oxidize but it is found that Cl- gets oxidized. It is due to the over potential of oxygen which is
produce during oxidation of OH- .

2 Cl - Cl2 + 2e-

Cathode { Na+ , H+ } :

Between H+ and Na+, the reduction potential of H+ is greater than that of Na+

2H
+
+ 2e- H2

[3] Electrolysis of aqueous CuSO4 using Cu electrode

The product of electrolysis depend upon the nature of electrode

2+ 2-
CuSO4 Cu + SO4

-
H2O H
+
+ HO

Anode { SO42- , OH- } :

Between SO42- and OH- , OH- has a greater oxidation potential than SO42-. OH- should oxidize but
due to over potential of oxygen, the SO42- try to get oxidize. But again, the oxidation potential of
SO42- is so small that it can also not be oxidised . So Cu itself get oxidise.

2+
Cu Cu + 2e-

jksahu74@gmail.com 70 | P a g e
Cathode { Cu2+ , H+ } :

Between Cu2+ and H+ ion, Cu2+have higher reduction potential. So Cu2+ get reduced.
2+
Cu + 2e- Cu

[4] Electrolysis of aqueous CuSO4 using Pt electrode

2+ 2-
CuSO4 Cu + SO4

-
H2O H
+
+ HO

Anode { SO42- , OH- } :

-
4 HO 2 H 2O + O2 + 4e-

Cathode { Cu2+ , H+ } :

2+
Cu + 2e- Cu

[5] Electrolysis of dilute H2SO4 using Pt electrode

+ 2-
H2SO4 2H + SO 4

+ -
H2O H + HO

-
Anode 4 HO 2 H2 O + O2 + 4e-

+
Cathode 2H + 2e- H2

[6] Electrolysis of concentrated H2SO4 using Pt electrode

+ 2-
H2SO4 2H + SO 4

Anode 2 SO4 2- S2O8 2- + 2 e -


peroxodisulphate
+
Cathode 2H + 2e- H2

jksahu74@gmail.com 71 | P a g e
Electrolyte Electrode Cathode reaction Anode reaction
+ -
Water Pt 2H + 2e- H2 4 HO 2 H2 O + O2 + 4e-
+ +
Silver nitrate Ag Ag + e- Ag Ag Ag + e-

-
Pt Ag
+
+ e- Ag 4 HO 2 H 2O + O2
+ 4e-

-
Copper chloride Pt Cu
2+
+ 2e- Cu 2 Cl Cl 2 + 2e-
2+ 2+
Cu Cu + 2e- Cu Cu Cu + 2 e -

+ -
Sodium chloride Pt 2H + 2e- H2 2 Cl Cl 2 + 2e-

-
Hg 2 Na
+
+ 2e- 2 Na 2 Cl Cl 2 + 2e-

+ -
Dilute H2SO4 Pt 2H + 2e- H2 4 HO 2 H2O + O2 + 4e-
2- 2-
Concn H2SO4 Pt 2H
+
+ 2e- H2 2 SO4 S 2O 8 + 2e-

Electrochemical cell:
An electrochemical cell is a device capable of either deriving electrical energy from chemical
reactions, or facilitating chemical reactions through the introduction of electrical energy.

Electrochemical cells are two types- Galvanic cell and Electrolytic cell

Galvanic cell or Voltaic Cells


The cell in which chemical energy converts into
electrical energy is called Galvanic cell. The process
carried in this cell is spontaneous.

Daniel cell is an example of a galvanic cell - contains


zinc immersed in zinc sulfate solution and copper in
copper sulfate solution separated by a diaphragm or
salt bridge

In Galvanic cell, the indirect redox reaction takes place


in separate vessel and decrease of free energy appear
in form of electrical energy.

Anode: Electrode where oxidation takes place. Cathode: Electrode where reduction takes place.

Salt bridge is used to maintain electrical neutrality and to complete internal circuit.

Cations and anions of salt of salt bridge must have same velocity (ionic mobility)

Special case

If we apply external potential to the electrochemical cell, the flow of electron will disturb.

Case-1 External potential < EMF => The cell act as electrochemical cell.

jksahu74@gmail.com 72 | P a g e
Electrons flow from Zn to Cu rod hence current flows from Cu to Zn

Zn dissolves at anode and Cu deposits at cathode

Case-2 External potential = EMF => The set is not a cell.

There is no flow of current. and no chemical reaction will takes place.

Case-3 External potential > EMF => The cell act as electrolytic cell

Electrons flow from Cu to Zn rod hence current flows from Zn to Cu

Cu dissolves at Cu electrode and Zn deposits at Zn electrode.

Galvanic Cell ( Voltaic cell) Electrolytic cell


A Galvanic cell converts chemical energy into An electrolytic cell converts electrical energy
electrical energy. into chemical energy.
Here, the redox reaction is spontaneous and is The redox reaction is not spontaneous and
responsible for the production of electrical electrical energy has to be supplied to initiate
energy. the reaction.
The two half-cells are set up in different Both the electrodes are placed in a same
containers, being connected through the salt container in the solution of molten electrolyte.
bridge or porous partition.
Here the anode is negative as it act as source Here, the anode is positive as it connected to
of electron and cathode is the positive electrode the positive terminal of external potential and
as it act as sink of electron. cathode is the negative electrode.
The electrons are supplied by the species The external battery supplies the electrons.
getting oxidized. They move from anode to the They enter through the cathode and come out
cathode in the external circuit. through the anode.

Electrode potential:
When a metal is dipped into solution of its ion, then metal atoms in the rod tend to go into solution
as ions and the ions in solution tend to go onto rod as atoms. As a result, a potential develops at
the solid-liquid boundary and that potential is called electrode potential.
A potential difference developed between the electrodes and electrolyte is called electrode
potential.
[1] Reduction electrode potential : The tendency of an electrode to gain electron or to get reduced
. According to IUPAC it is known as positive electrode potential
[2] Oxidation electrode potential : The tendency of an electrode to lose electron or to get oxidized.
According to IUPAC it is known as negative electrode potential.

jksahu74@gmail.com 73 | P a g e
E Zn 2+ | Zn = Reduction potential = Electrode potential of Zn = x Volt
E Zn | Zn 2+ = Oxidation potential = Negative electrode potential of Zn = - x Volt
Cell potential ( EMF)
B C B C
EMF = Reduction potential of reduction half cell @ Reduction potential of oxidation half cell
= ECathode @ EAnode
B C B C
EMF = Oxidation potential of oxidation half cell @ Oxidation potential of reduction half cell
b c b c
= @ EAnode @ @ ECathode = ECathode @ EAnode

Standard Hydrogen Electrode ( Reference electrode )


The Standard Hydrogen Electrode consists of a platinum electrode
coated with platinum black. The electrode is dipped in an acidic
solution and pure hydrogen gas is bubbled through it. The
concentration of both the reduced and oxidised forms of hydrogen
is maintained at unity. This implies that the pressure of hydrogen
gas is one bar and the concentration of hydrogen ion in the
solution is one molar.

+
Anode - H2 2H + 2 e-

+
Cathode - 2H + 2 e- H2

E 2 H+ | H2 = 0

If it act as cathode: The maximum bubbling of hydrogen gas from the solution will evolve.
If it act as anode: The minimum bubbling of hydrogen gas from the solution will evolve
Nernst Equation
Electrode potential affected by:
Nature of metal
Concentration of its ions
Valency of metal
Temperature
This equation correlates the change of electrode potential with temperature and concentration.
And is applicable only for the reduction reaction:
` a
For this reaction M n + aq + n e@ @ @ @ @ @ Q M`Sa

0 f
ff
f
f
RTf
f
ff
f
f f f
f
ff
f
1f
f
ff
f
ff
f
ff
f
ff
The Nernst equation is : EMn + | = EM n+ @ ln @ n +A
M | M nF M
For Zn –CuSO4 cell , the cell representation is Zn | Zn2+ || Cu2+ | Cu
` a
For anode : Zn 2 + aq + 2 e@ @ @ @ @ @ Q Zn`Sa

jksahu74@gmail.com 74 | P a g e
f
ff
f
f
RTf
f
ff
f
f f ff
ff
f
ff
1f
f
ff
f
ff
f
fff
ff
f
The Nernst equation is : EZn 2 + | = E0Zn 2 + | @ ln B C
Zn Zn 2F Zn 2+

` a
For Cathode : Cu 2 + aq + 2 e@ @ @ @ @ @ Q Cu`Sa

0 f
ff
f
f
RTf
f
ff
f
f f f
f
ff
f
f1f
f
ff
f
ff
f
ff
ff
ff
The Nernst equation is : ECu2 + | = ECu 2+ @ ln B C
Cu | Cu 2F Cu 2+

The cell potential , ECell = ECu 2 + | Cu


@ E Zn 2 + | Zn
X Y X Y
\ 0 f
ff
f
f
RTf
f
ff
f
f f f
f
ff
f
f1f
f
ff
f
ff
f
ff
f
ff
f \
] f
ff
f
f
RTf
f
ff
f
f f f
f
ff
f
f1f
f
ff
f
ff
f
ff
f
ff
f
]
ECell = ZE 2+ @ ln B C @ E0 2 + @ ln B C
Cu | Cu 2F Cu2 + [ Z
Zn | Zn 2F Zn2 + [
X Y
R S
f
ff
f
f
RTf
f
ff
f
f ln B
\ ff
f
ff
f
ff
f
1f
f
ff
f
ff
f
ff
ff ff
f
ff
f
ff
f
1f
f
ff
f
ff
f
ff
f
]f
0
= ECu @ E0Zn 2 + | Zn @ C @ ln B C
2+ 2+ 2+
| Cu 2F Z Cu Zn [
B C B C
f
ff
f
ff
f
ff
f
f f
f
ff
f
ff
f
ff
f
ff
f
ff
f
ff
f
f ff
f
ff
f
ff
f
ff
f
ff
f
ff
f
ff
f
ff
f
ff
f
ff
f
ff f
f
ff
f
ff
f
ff
f
ff
f
ff
f
ff
f
f
2+ 2+
RT Zn 2.303 RT Zn
0 0
ECell = ECell @ ln B C = ECell @ log B C
2F Cu 2 + 2F Cu 2 +
B C
ff
f
ff
f
ff
f
ff
f
ff
f
ff
f
ff
f
ff
f
ff
f
ff
f
ff f
f
ff
f
ff
f
ff
f
ff
f
ff
f
ff
f
ff
f
ff
n
2.303 RT Oxid
In general E = E0 @ log B C
nF Red
n

By substituting the values R = 8.314 JK -1 mol -1


, T = 25 C = 298 K and F = 96487 C
The general Nernst equation can be written as :
B C
ff
f
ff
f
ff
f
ff
f
ff
f
ff
f
ff f
f
ff
f
ff
f
ff
f
ff
f
ff
f
ff
f
ff
f
n
0.059 Oxd
E = E0 @ log B C
n Red
n

Special case
At the point of equilibrium EMF of the cell = 0
B C
ff
f
ff
f
ff
f
ff
f
ff
f
ff
f
ff
f
ff
f
ff
f
ff
f
ff fff
f
ff
f
ff
f
ff
f
ff
f
ff
fff
ff
f
f
n
2.303 RT Oxid
So E0 @ log B C =0
nF Red
n

B C
ff
f
ff
f
ff
f
ff
f
ff
f
ff
f
ff
f
ff
f
ff
f
ff
f
ff f
f
ff
f
ff
f
ff
f
ff
f
ff
f
ff
f
ff
f
ff ff
f
ff
f
ff
f
ff
f
ff
f
ff
f
ff
f
ff
f
ff
f
ff
f
ff
n
2.303 RT Oxid 2.303 RT
[ E0 = log B C [ E0 = log K
nF Red
n nF

Electrical work = Total charge carried X EMF


Total charge carried by n moles of electrons = nF
So electrical work = nFE
In electrochemical cell, decrease of free energy appears in form of electrical work.

jksahu74@gmail.com 75 | P a g e
So ∆G =@ nFE
In standard form ∆G 0 = @ nFE 0
ff
f
ff
f
ff
f
ff
f
f
F 2.303f
f
ff
f
ff
f
ff
f
f
RTf
f
ff
f
ff G
∆G 0 =@ nF log K = @ 2.303RTlog K
nF

∆Go
0 0 0
∆G = @nFEcell ∆G = @ RT ln K

Eocell K
Ecell = f
f
ff
f
ff
f
ff
f
ff
f
ff
f
ff
f
ff
f
0 0.059
log K
n

Nernst equation for : Al – ZnSO4 cell

The Cell is : Al | Al 3+ || Zn 2+ | Zn

Anode reaction : Al Al 3+ + 3e- ] x 2


Cathode reaction : Zn 2+ + 2e- Zn ] x 3
Cell reaction : 2 Al + 3 Zn 2+ 2 Al 3+ + 3 Zn

B C2
3+
ff
f
ff
f
ff
f
ff
f
ff
f
ff
f
ff f
f
ff
f
ff
f
ff
f
ff
f
ff
f
ff
f
ff
f
ff
0.059 Al
E = E0 @ log B C3
6
Zn 2 +

Concentration cell
Concentration cell is a device in which electricity is produced due to difference in concentration of
electrolyte solution of same electrode.
Example: Zn | Zn2+ (C1) || Zn2+ ( C2 ) | Zn

E cell = ff
f
ff
f
ff
f
ff
ff
f
ff
f
ff
f
ff
f
ff
f
ff
f
ff
log f
2.303 RT Cf
f
ff
f
2ff
nF C1
0
Because E cell = 0 , where C2 > C1

E cell = f
f
ff
f
ff
f
ff
ff
f
ff
f
ff
f
log f
0.059 Cf
f
ff
f
2ff
At 298 K
n C1

When cell consists of two hydrogen electrodes operating at different pressures P1 and P2 having
same concentration of solution.
Pt,H2(P1) | H+(C1) || H+(C2) | H2(P2),Pt

Ecell = f
f
ff
f
ff
f
ff
ff
f
ff
f
ff
f
log f
0.059 Pf
f
f
1f
ff
At 298 K, Where P1 > P2
n P2

jksahu74@gmail.com 76 | P a g e
Combination of electrode potential of two electrodes

Electrode potential is an intensive property. Therefore, electrode potential of two half cells can not
be added or subtracted to give the electrode potential of third half cell whose reaction is obtained
by addition or subtraction of the given cell reactions. However , free energy change is an extensive
property. Hence Eo values of two half reactions is converted to ∆G o . These are combined to get
∆G of the new half reaction. And again the new ∆G is converted to corresponding Eo values.
o o

Example: ∆G3o = ∆G1o + ∆G2o


o o o
or @n3 FE3 = @n1 FE1 @ n2 FE2
o o o
or @ n3 E3 = @ n1 E1 @n2 E2
o o o
or n3 E3 = n1 E1 + n2 E2
nf
f
1f
f
ff
f
ff
f
1f
f
ff
f
f+f
f
ff
f
fnf
f
ff
2f
f
ff
f
ff
f
f2f
f
ff
o o
o E E
0r E =
3
n3
Where n1, n2 & n3 are number of electrons involved in first, second and the resulting half cell
reactions.

Batteries
Battery is a collection of galvanic cells connected in series that is used as a source of electrical
energy. Batteries are two types: (1) Primary & (2) Secondary

Primary cell
The cell which can not be recharged again is called primary cell. Examples are :-
LECLANCHE CELL
Anode Zinc container EMF = 1.5 V
Cathode Graphite rod surrounded by MnO2 and carbon powder.
Electrolyte NH4Cl and ZnCl 2

2+
Anode Reaction Zn Zn + 2e-
+
Cathode Reaction MnO 2 + NH4 + e- MnO ( OH ) + NH3 X 2
2+
Cell Reaction Zn + 2 MnO 2 + 2 NH4
+
Zn + 2 MnO ( OH ) + 2 NH3

MERCURY CELL
Anode Zinc-Mercury amalgam ( Used to increase reactivity )
Cathode Paste of mercuric oxide ( HgO ) and carbon.
Electrolyte KOH and ZnO
Anode Reaction -
Zn ( Hg ) + 2 HO ZnO + H2O + 2 e-
Cathode Reaction + H2O + 2 e- -
HgO Hg + 2 HO
Cell Reaction Zn ( Hg ) + HgO ZnO + Hg

jksahu74@gmail.com 77 | P a g e
Secondary battery
The battery which can be recharged again after discharging is called secondary battery.

LEAD STORAGE BATTERY

Anode: A number of plates joined in parallel, each plate is a grid of lead filled with finely divided
spongy lead.

Cathode: Equal number of plates joined in parallel, each plate is a grid of lead filled with lead
dioxide (PbO2 ).

Electrolyte => dilute H2SO4 ( 38% ) EMF = 6 to 12 V

Cathode +
Anode
Lead grids filled
Lead grids filled
with PbO2
with spongy Pb ::::::::::::::::::::::::::::::::::::::::::::::::::::::::::::::::
::::::::::::::::::::::::::::::::::::::::::::::::::::::::::::::::
:::::::::::::
:::::::::::::::::::::::::::::::::::::::::::::::::::::::::::
::::::::::::: Dilute H2SO4 ( 38 % )
::::::::::::::::::

Discharging reaction -Cell act as electrochemical cell


Oxd n React n Pb + SO 2-
4 PbSO4 + 2 e-

2- +
Red n React n PbO2 + SO4 + 4 H + 2 e- PbSO4 + 2 H2O

+
Cell React n Pb + PbO2 + 2 SO42- + 4 H 2 PbSO4 + 2 H 2O
Recharging reaction -Cell act as electrolytic cell
Red n React n 2-
PbSO4 + 2 e- Pb + SO 4
2- +
Oxd n React n PbSO4 + 2 H2O PbO2 + SO 4 + 4H + 2 e-

Cell React n 2 PbSO4 + 2 H2O Pb + PbO2 +2 2-


SO 4 + 4H
+

Recharging is possible in this case because PbSO4 formed during discharging is a sticky solid
which sticks to the electrode. Therefore it can either take up or give up electrons during recharge.

CADMIUM-NICKEL STORAGE CELL EMF = 1.4 V

Anode: Cadmium Cathode : Metal grid containing nickel dioxide Electrolyte: KOH
Discharging reaction:

Oxd n React n Cd + 2 HO- Cd (OH) 2 + 2 e-

Red n React n NiO 2 + 2 H2O + 2 e- Ni(OH) 2 + 2 HO


-

Cell React n Cd + NiO 2 + 2 H2O Cd (OH) 2 + Ni(OH) 2

jksahu74@gmail.com 78 | P a g e
Recharging Reaction:

-
Oxd n React n Ni(OH) 2+ 2 HO + 2 H2O + 2 e-
NiO 2
-
Red n React n Cd (OH)2 + 2 e- Cd
+ 2 HO
Cell React n Ni(OH)2
+ Cd (OH)2 Cd + NiO 2 + 2 H2O

This battery gives constant potential during its life because the net cell reaction does not involve
any ion whose concentration can change during its life time. It has longer life than lead storage
battery. It is more expensive.

Fuel Cell
Fuel cells are the devices which convert the energy of
fuel into electrical energy without the use of heat
engine. The efficiency of fuel cell is 70 %.
It is pollutant free whereas the thermal heat engine is
40 % efficient which create pollution.
H2 - O2 Cell
Anode: Porous carbon electrode filled with finely
divided platinum / palladium
Cathode: Porous carbon electrode filled with Pt / Pd
Electrolyte: Concentrated NaOH or KOH [ Pt and Pd act as catalyst.
-
Anode React n H2 + 2 HO 2 H2O + 2 e- X 2

Cathode React n O2 + 2 H2O + 4 e- 4 HO


-

2 H2 + O 2
Cell React n 2 H2O

The H2-O2 fuel cell was used for producing electricity in Apollo space programme.

Corrosion
The process of coating on surfaces of metallic objects with oxides or other compounds of the
metal as a result of its reaction with air or water surrounding it. In case of iron, corrosion is called
rusting. The mechanism of rusting is explained as:
[1]Creation of Acidic medium: Atmospheric carbon dioxide and water vapour combine to form
. carbonic acid.

CO 2 + H2O H2CO 3

[2] Iron will oxidize [ Anode- Oxidation half ]

2+
Fe Fe + 2 e- X 2

jksahu74@gmail.com 79 | P a g e
[3] In another spot, oxygen of air will take the two electrons with help of H+ ion and will be .
reduced to H2O [ Cathode - Reduction half ]
+
O2 + 4H + 4 e- 2 H2O

+
Cell React n 2 Fe + O2
+ 4H 2 Fe
2+
+ 2 H2O

In alkaline medium, atmospheric oxygen is unable to take electron which is given by the oxidation
of Fe .
[4] Atmospheric oxygen further oxidises ferrous ion into ferric oxide.

2+ 1 +
2 Fe + 2 H2O + 2 O2 Fe2O 3 + 4H

[5] Ferric oxide will hydrolyse with water to form rust.

Fe 2O 3 . x H 2O [ Rust ]
Fe 2O3 + x H 2O

Prevention of corrosion
1) By surface coating: By applying oil, grease, paint or varnish on the surface of the metal objects
to prevent come in contact with atmosphere.
2) By galvanization:Prevention of corrosion by zinc costing.
3) Sacrificial protection: In this method metal object is connected with metals of lower electrode
potential value which corrodes itself and protects the oxidation of metal. E.g. Iron objects are
protected by connecting with Zn & Mg.
Hydrogen economy
Hydrogen can be used as renewable and non-polluting source of energy. Water will split to
hydrogen and oxygen using solar energy.


1 O
H2O H2 + 2
2

Combustion of hydrogen in a fuel cell will produce water.

1 O
H2 + 2
2 H2 O

Both these technologies are based on electrochemical principles. This production of energy will be
very economical in comparison to fossil fuel and nuclear energy.

jksahu74@gmail.com 80 | P a g e
(1) Calculate the potential of hydrogen electrode in contact with a solution whose PH is 10.

PH = 10 means [H+] = 10-10 M

1
Now for the electrode; H+ + e− H2 (Here n = 1)
2
0 ⋅ 059 1 1
E H + H = E 0H + - log + = 0 − 0⋅059 log = − 0⋅059 log 1010 = − 0⋅059 × 10 = − 0⋅59 v
2 H2
n [ ]
H 10 −10

(2) Calculate the EMF of the cell in which the following reaction take place:

Ni + 2 Ag+ (0⋅002M) Ni2+ (0⋅160 M) + 2 Ag


Given that E 0Cell = 1 ⋅ 05 V

According to Nernst equation : E Cell = E 0Cell −


0 ⋅ 059
log
[ ]
Ni 2+
n [ ]
Ag +
2

0 ⋅ 059 0 ⋅160
= 1 ⋅ 05 − log = 0 ⋅ 9143V
2 (0 ⋅ 002)2

(3) The cell in which following reaction occurs : 2 Fe3+ + 2 I− 2 Fe2+ + I2


has E 0Cell = 0⋅236t 298 K. Calculate the standard Gibb’s energy and the equilibrium constant for
the cell reaction.

0 ⋅ 059 nE 0Cell 2 × 0 ⋅ 236


E 0Cell = log K C ⇒ log K C = = = 8⋅0 K C = anti log(8 ⋅ 0) = 1× 108
n 0 ⋅ 059 0 ⋅ 059

∆ G 0 = − nFE 0Cell = − 2 × 96487 × 0 ⋅ 236 = − 45541 j mol −1 = 45 ⋅ 541 kj mol −1

(4) The molar conductivity of 0⋅025 mol L−1 methanoic acid is 46⋅1 S cm2 mol−1. Calculate its degree
of dissociation and dissociation constant. Given λ0H + = 346⋅6 S cm2 mol−1 and λ0HCOO− = 54⋅6 S cm2 mol−1

λC = 46⋅1 S cm2 mol−1 C = 0⋅025 mol L−1

λ0HCOO− = λ0H+ + λ0HCOO = 349.6 + 54⋅6 = 404⋅2 S cm2 mol−1


λC 46 ⋅1
Degree of dissociation (α ) = 0
= = 0 ⋅114
λ 404 ⋅ 2
cα 2 0 ⋅ 025 × (0 ⋅114 )
2
Dissociation constant (K a ) = = = 0 ⋅ 0003667 = 3 ⋅ 67 ×10−4
1− α 1 − 0 ⋅114

(5) If a current of 0⋅5 ampere flows through a metallic wire for 2 hours, then how many electrons flow
through the wire ?

Quantity of electricity (Q) = Current (ampere) × time (second) = 0⋅5 × 2 × 60 × 60 = 3600 C (Coulombs) A flow
of 69487 C of electricity ≅ 6⋅022 × 1023 electrons

jksahu74@gmail.com 81 | P a g e
6 ⋅ 022 ×1023
∴ 3600 C of electricity = × 3600 = 2 ⋅ 246 ×1022 electrons
96487

(6) Consider the reaction: Cr2O 72− + 14 H+ + 6e− 2 Cr3 + 7H2O


What is the quantity of electricity in coulombs needed to reduce 1 mol of Cr2 O 72− ?

From the equation it is found that for the reduction of 1 mole of Cr2O 72−
6 mole of electrons are required
1 mole of electrons ≅ 96487 C of electricity
∴ 6 moles of electrons = 96487 × 6 = 5⋅78 105 C

(7) Represent the cell in which the following reaction takes place :
Mg + 2 Ag+ (0⋅0001M) Mg2+ (0⋅130M) + 2 Ag
Calculate Ecell, if E ocell = 3 ⋅17 V

The cell can be written as MgMg2+ (0⋅130M)  Ag+ (0⋅0001M)  Ag

E cell = E ocell −
0 ⋅ 059
log
[ ]
Ng 2+
= 3 ⋅17 −
0 ⋅ 059
log
0.130
= 3 ⋅17 − 0 ⋅ 21 = 2 ⋅ 96 V
n [ ]
Ag + 2 2 (0 ⋅ 0001) 2
(8) Calculate the equilibrium constant of the reaction: Cu + 2 Ag+ Cu2+ + 2 Ag;
E ocell = 0⋅46 V

0 ⋅ 059 0 ⋅ 059 0 ⋅ 059


E ocell = log K C = log K C ⇒ 0 ⋅ 46 = log K C
n 2 2

2 × 0 ⋅ 46
⇒ log K C = = 15 ⋅ 6
0 ⋅ 059

∴ KC = Antilog (15⋅6) = 3⋅92 × 1015

(9) The standard electrode potential for Daniell cell is 1.1 V. Calculate the standard Gibbs energy for
the reaction : Zn + Cu2+ Zn2+ + Cu

For the above reaction n – 2, F = 96487 C mol−1 and E ocell = 1⋅1 V

∆ G0 = − nFE ocell = − 2 × 1⋅1 × 96487 = − 21227 J mol−1 = −21⋅227 kl mol−1

(10) Resistance of a conductivity cell filled with 0⋅1 mol−1 KCI solution is 100Ω. If the resistance of the
same cell when filled with 0⋅02 mol−1 KCI solution is 520Ω, Calculate the conductivity and molar
conductivity of 0⋅02 mol−1 KCI solution. The conductivity of 0⋅1 mol−1 KCI solution is 1⋅29 S m−1.

0⋅01 M KCI Solution having Resistance (R1) = 100Ω and Conductivity (k1) = 1⋅29 S m−1.

0⋅02 M KCI solution having Resistance (R2) = 520Ω Conductivity (k2) = ?

And Molar conductivity (λ2) = ?

jksahu74@gmail.com 82 | P a g e
1 l
We know that conductivity (k) = Conductance   × Cell constant  
R a
x −1
Cell constant (G ) = k1 R1 = 1⋅29 × 100 = 129 m

Since cell is same, so 0⋅02 M KCI having same value of Gx

1 l 1
∴ Conductivity of 0⋅002 M KCl = G × Gx =   ×   = ×129 = 0⋅248 S m-1
 R   a  520
--------------------------------------------------------------------------------------------------------------------
0⋅02 M KCI ⇒ 0⋅02 moles present in 1 litre of solution (i.e. 1 dm3 soln or 10−3 m3 solution)

10 −3 3
∴ 1 mole present in m solution
0 ⋅ 02
--------------------------------------------------------------------------------------------------------------------
K = 0⋅248 S m−1 ⇒ 1 m3 solution having conductive = 0⋅248 S

10 −3 10 −3
∴ m3 Solution having conductance = 0⋅248 × = 124 ×10−4 S m2 mol−1
0 ⋅ 02 0 ⋅ 02

i.e. Molar conductivity of 0⋅02 M KCI solution (λ) = 124 × 10−4 S m2 mol−1

2
−4  100 cm 
2 −1 2 −1
= 124 × 10 S (1 m)   mol = 124 S cm mol
 1m 

(11) The electrical resistance of a column of 0⋅05 M NaOH solution of diameter 1 cm and length 50 cm
is 5⋅55 × 103 ohm. Calculate its resistivity, conductivity and molar conductivity.

Area of cross section (a) = πr2 = 3⋅14 × (0⋅5 cm)2 = 0⋅758 cm2 Length of solution (l) = 50 cm

1 Ra 5 ⋅ 55 × 103 Ω × 0 ⋅ 785cm 2
Resistance (R) = ρ ⇒ Resistivity (ρ) = = cm = 87⋅135Ωcm
a I 50
 1m  −2
= 87⋅135Ω (1 cm)   = 87⋅135 ×10 Ωm
 100cm 
--------------------------------------------------------------------------------------------------------------------
1 1  1  100cm 
Conductivity (k) = = = 1⋅148 × 10−2 S cm−1 = 1⋅148 × 10−2 S    = 1⋅148 S m
−1
ρ 87 ⋅135  cm  1m 

0⋅05 M NaOH means 0⋅05 moles of NaOH present in1 litre i.e. 1000 cm3 solution

1000
∴ 1 mole of NaOH present in cm3 of solution.
0 ⋅ 05
K = 1⋅148 × 10−2 S cm−1 ⇒ 1 cm Solution having conductance = 1⋅148 × 10−2 S

1000 1000
∴ Conductance of cm3 solution = × 1⋅148 × 10−2 = 229⋅6 S cm2 mol−1
0 ⋅ 05 0 ⋅ 05
2
2 −1  1m  2
-1
i.e. Molar conductivity (λ) = 229⋅6 S cm mol = 229⋅6 S (1 cm )   mol
 100cm 

jksahu74@gmail.com 83 | P a g e
= 229⋅6 × 10−4 S m2 mol−1

(12) Calculate ∧ om for CaCl2 and MgSO4 from the data given in the table of Book.

∧ om (Cacl2 ) = ∧ oCa 2 + + 2 ∧ ocl- = 119 ⋅ 0 + 2 × 76 ⋅ 3 = 119 ⋅ 0 + 152 ⋅ 6 = 271 ⋅ 6 S cm 2 mol -1

∧ om (MgSO 4 ) = ∧ oMg 2+ + ∧SO


o
2- = 106 ⋅ 0 + 160 ⋅ 0 = 266 S cm
2
mol -1
4

(13) ∧ om for NaCI, HCI and NaAc are 126⋅4, 425⋅9 and 91⋅0 S cm2 mol−1 respectively. Calculate ∧0 for
HAC
0 0 0
λ HAc = λ H + + λ Ac

= ∧ oH + + ∧ oCl − + ∧ oAc − + ∧ oNa + - ∧ oCl − - ∧ oNa +

= ∧om (HCL ) + ∧om( NaAc ) - ∧om ( NaCl) = 425⋅9 + 91⋅0 − 126⋅4 = 390⋅5 S cm2 mol−1

(14) The Conductivity of 0⋅001028 mol L−1 acetic acid is 4⋅95 × 10−5 S cm−1. Calculate its
dissociation constant if ∧0 for acetic acid is 390⋅5 S cm2 mol−1.

k × 1000 4 ⋅ 95 × 10-5 1000


∧m = = = 48⋅15 cm2 mol−1
C 0 ⋅ 001028
∧ m 48 ⋅ 15
α= o = = 0 ⋅ 1233
∧ m 390 ⋅ 0
Cα 2 0 ⋅ 001028 × (0 ⋅1233)
2
Dissociation constant (Ka) = = = 1 ⋅ 78 ×10−5
1- α 1 − 0 ⋅1233

(15) A solution of CuSO4 is electrolysed for 10 minutes with a current of 1⋅5 amperes. What is the mass
of copper deposited at the cathode ?

Quantity of electricity (Q) = Current × time = 1⋅5 × 10 × 60 = 900 C

According to the reaction : Cu2+ + 2e− Cu

We required 2 F or 2 × 96487 C of electricity to deposit 1 mol or 63 g of Cu

63
∴ 900 C electricity will deposit = × 900 = 0⋅2938 g of Cu at the cathode
2 × 96487

(16) Calculate the standard cell potentials of galvanic cell in which the following reactions take
place:
(i) 2 Cr(s) + 3 Cd2+ (aq) 2 Cr3+ (aq) + 3 Cd

(ii) Fe2+ (aq) + Ag+ (aq) Fe3+ (aq) + Ag (s)

Calculate the ∆rG0 and equilibrium constant of the reactions.

jksahu74@gmail.com 84 | P a g e
(i) Oxidation Half : Cr Cr3+ + 3 e− ] × 2

Reduction Half : Cd2+ + 2e− Cd ] × 3


-----------------------------------------------------------------------------------------------------------------
Cell Reaction : 2 Cr + 3 Cd3+ 2 Cr3+ + 3 Cd

∴ Number of moles of electrons involved (n) = 6

Standard cell potential (E ocell ) = E oCd2+ Cd − E oCr3+ Cr = − 0⋅40 − (− 0⋅74) = + 0⋅34 V

∆ G0 = − nFE oCell = − 6 × 96487 × 0⋅34 = − 196833 J = − 196⋅833 kj mol−1

∆G 0 − 196833
∆G0 = − 2⋅303 RT log KC ⇒ log KC = = =34⋅501
− 2 ⋅ 303RT − 2 ⋅ 303 × 8 ⋅ 314 × 298

⇒ KC = Antilog (34.501) = 3⋅17 × 1034

(ii) Oxidation Half : Fe2+ Fe3+ + e−

Reduction Half : Ag+ + e− Ag


--------------------------------------------------------------------------------------------------------------------
Cell Reaction : Fe2+ + Ag+ Fe3+ + Ag

∴ Number of moles of electrons involved (n) = 1

Standard cell potential (E ocell ) = E oAg + Ag − E oFe3+ Fe2+ = 0⋅80 − 0⋅77 = 0⋅03 V

∆G0 = − nFE ocell = − 1 × 96487 × 0⋅03 = − 2894⋅6 J = − 2⋅8946 kj mol−1


∆G 0 − 2894 ⋅ 6
∆G0 = − 2⋅303 RT log KC ⇒ log KC = = = 0⋅507
− 2 ⋅ 303RT − 2 ⋅ 303 × 8 ⋅ 314 × 298

⇒ KC = Antilog (0⋅507) = 3⋅2

(17) Write the Nernst equation and find emf of the following cells at 298 K:

(i) Mg(s) Mg2+ (0⋅001 M) Cu2+ (0⋅0001 M) Cu(s)

(ii) Fe(s) Fe2+ (0⋅001 M) H+ (1M) H2(g) (1 bar) Pt(s)

(iii) Sn(s) Sn2+ (0⋅050 M) H+ (0⋅020 M) H2 (g) (1 bar) Pt(s)

(iv) Pt(s) Br2 ( ι )Br−(0⋅010 M)H+(0⋅030 M) H2 (g) (1 bar) Pt(s).

(i) Oxidation Half Mg Mg2+ + 2e−

Reduction Half Cu2+ + 2e− Cu

----------------------------------------------------------------------

jksahu74@gmail.com 85 | P a g e
Cell Reaction Mg + Cu2+ Mg2+ + Cu

Here number of moles of electrons (n) = 2

E ocell = E oCu 2+ Cu − E oMg 2+ Mg = 0⋅34 − (− 2⋅37) = 2⋅71 V


0 ⋅ 059 [Mg 2+ ]
The Nernst equation for the cell : E cell = E ocell − log
2 [Cu 2+ ]
0 ⋅ 59 0 ⋅ 001
E cell = 2 ⋅ 71 − log = 2⋅71 − 0⋅0295 log 10 = 2⋅71 − 0⋅0295 = 2⋅6805 V
2 0 ⋅ 0001

(ii) Oxidation Half Fe Fe2+ + 2e−

Reduction Half 2 H+ + 2e− H2

-------------------------------------------------------------------------

Cell Reaction Fe + 2 H+ Fe2+ + H2

Here number of moles of electrons (n) = 2

E ocell = E o2 H + H − E oFe2+ Fe = 0 − (− 0⋅44) = 0⋅44 V


2

0 ⋅ 059 [Fe2+ ] 0 ⋅ 059 0 ⋅ 001


The Nerst equation for the cell : E cell = E ocell − log + 2 = 0⋅44 − log
2 [H ] 2 (1) 2

= 0⋅44 − 0⋅0295 ×log 10−3 = 0⋅44 − 0⋅0295 (−3) = 0⋅44 + 0⋅0885 = 0⋅5285 V

(iii) Oxidation Half Sn Sn2+ + 2e−

Reduction Half 2H+ + 2e− H2


---------------------------------------------------------------------------------------
Cell Reaction Sn + 2H+ Sn2+ + H2

Here number of moles of electrons (n) = 2

E ocell = E o2 H + H − E Sn
o
2+
Sn
= 0 − (− 0⋅14) = 0⋅14 V
2

0 ⋅ 059 [Sn 2 + ] 0 ⋅ 059 0 ⋅ 05


The Nerst equation for the cell : E cell = E ocell − log + 2 = 0⋅14 − log
2 [H ] 2 (0 ⋅ 02) 2

0 ⋅ 059 0 ⋅ 059
= 0⋅14 − log 125 = 0⋅14 − × 2⋅0969 = 0⋅078 V
2 2

(iv) Oxidation Half 2 Br- Br2 + 2e−

Reduction Half 2H+ + 2e− H2


---------------------------------------------------------------------------------------
Cell Reaction 2 Br− + 2H+ Br2 + H2

Here number of moles of electrons (n) = 2


jksahu74@gmail.com 86 | P a g e
E ocell = E o2 H + H − E oBr Br −
= 0 − 1⋅09 = − 1⋅09 V
2 2

0 ⋅ 059 1
The Nerst equation for the cell : E cell = E ocell − log
2 [Br ] [H + ]2
− 2

0 ⋅ 059 1 0 ⋅ 059
= − 1⋅09 − log 2 2
= − 1⋅09 − log (1⋅111 × 107)
2 (0 ⋅ 01) (0 ⋅ 03) 2

0 ⋅ 059
= − 1⋅09 − × 7⋅0457
2

Negative emf prove non spontaneous process i.e. oxidation will occur at the hydrogen electrode

and reduction at the bromine electrode. So that we will get E cell = 1⋅298 V

(18) In the button cells widely used in watches and other devices the following reaction takes
place :

Zn(s) + Ag2O(s) + H2O( ι ) Zn2+(aq) + 2Ag(s) + 2OH−(aq)

Determine ∆rG0 and E0 for the reaction.

In the cell : Oxidation Half Zn Zn2+ + 2e−

Reduction Half Ag+ + e− Ag ]×2


---------------------------------------------------------------------------------------
Cell Reaction Zn + 2 Ag+ Zn2+ + 2 Ag

Here number of moles of electrons (n) = 2

E ocell = E oAg + Ag − E oZn 2+ Zn = 0⋅80 − (− 0⋅76) = 1⋅56 V

∆ G0 = − nFE ocell = − 2 × 96487 × 1⋅56 = 301039 J

(19) The conductivity of 0⋅20 M solution of KCI at 298 K is 0⋅0248 S cm−1. Calculate its molar
conductivity.

0⋅2 M ⇒ 0⋅2 moles KCI present in 1 litre i.e. 1000 cm3 of solution
1000
∴ 1 mole KCI present in cm3 of solution
0⋅2
K = 0⋅0248 S cm−1 ⇒ Conductance of 1 cm3 solution = 0⋅0248 S
1000 1000
∴ Conductance of cm3 solution = 0⋅0248 × = 124 S cm2 mol−1
0⋅2 0⋅2
So Molar conductivity (λ) = 124 S cm2 mol−1

(20) The resistance of a conductivity cell containing 0⋅001 M KCI solution at 298 K is 1500 Ω. What is
the cell constant if conductivity of 0⋅001 M KCI solution at 298 K is 0⋅146 × 10−3 S cm−1.
1 l
Conductivity (k) = Conductance   × Cell constant  
R a
jksahu74@gmail.com 87 | P a g e
l
⇒ Cell constant   = k R = 0⋅146 × 10−3 × 1500 = 0⋅219 cm−1
a

(21) Conductivity of 0⋅00241 M acetic acid is 7⋅896 × 10−5 S cm−1. Calculate its molar conductivity and
if ∧ 0m for acetic acid is 390⋅5 S cm2 mol−1,what is its dissociationconstant ?

0⋅00241 M Acetic acid ⇒ 0⋅00241 moles of acetic acid present in 1000 cm3 solution
1000
∴ 1 mole of acetic acid present in cm3 solution
0 ⋅ 00241
Conductivity (k) = 7⋅896 × 10−5 S cm−1 ⇒ 1 cm3

solution having conductance = 7⋅896 × 10−5 S cm−1


1000 1000
∴ Conductance of cm3 solution = 7⋅896 × 10−5 S cm−1 ×
0 ⋅ 00241 0 ⋅ 00241
= 32⋅76 S cm2 mol−1

i.e. Molar conductivity (λm) = 32⋅76 S cm2 mol−1


λ 32 ⋅ 76
degree of dissociation of acid (α) om = = 0⋅084
λ m 390 ⋅ 5
Cα 2 0 ⋅ 00241× (0 ⋅ 084) 2
∴ Dissociation constant of acid (Ka) = = = 1⋅85 × 10−5
1− α 1 − 0 ⋅ 084

(22) How much charge is required for the following reductions:


(i) 1 mol of Al3+ to Al. (ii) 1 mol of Cu2+ to Cu. (iii) 1 mol of MnO4− to Mn2+.

(i) Al3+ + 3e− Al

For reduction of 1 mole of Al3+ 3 mole of electron i.e. 3 F or (3 × 96487 C) charge is required.

(ii) Cu2+ + 3e− Cu

For reduction of 1 mole of Cu2+ ,2 mole of electron i.e. 2 F or (2 × 96487 C) charge is required.

(iii) MnO4− + 8 H+ + 5e− Mn2+ + 4 H2 O

For reduction of 1 mole of MnO4− ,5 mole of electron i.e. 5 F or (5 × 96487 C) charge is required.

(23) How much electricity in terms of Faraday is required to produce

(i) 20⋅0 g of Ca from molten CaCl2. (ii) 40⋅0 g of Al from molten Al2O3.

(i) Ca2+ + 2e− Ca

To produce 40 g Ca, electricity needed = 2F


2
∴ To produce 20 g Ca, electricity needed = × 20 = 1F
40
(ii) Al3+ + 3e− Al

To produce 27 g Ca, electricity needed = 3F

jksahu74@gmail.com 88 | P a g e
3
∴ To produce 40 g Al, electricity needed = × 40 = 4⋅44F
27

(24) How much electricity is required in coulomb for the oxidation of


(i) 1 mol of H2O to O2. (ii) 1 mol of FeO to Fe2O3.

1
(i) H2O 2H+ + O2 + 2e−
2
Quantity of electricity required = 2F

(ii) Fe2+ Fe3+ + e−

Quantity of electricity required = 1F

(25) A solution of Ni (NO3)2 is electrolysed between platinum electrodes using a current of


5 amperes for 20 minutes. What mass of Ni is deposited at the cathode ?

Quantity of electricity passed (Q) = Current in amperes × time in second = 5 × 20 × 60 = 6000 C

Ni2+ + 2e− Ni

i.e. 2 × 96487 C of electricity deposit Ni = 58⋅5 g

58 ⋅ 5
∴ 6000 C of electricity deposit Ni = × 6000 = 1⋅82 g
2 × 96487

(26) Three electrolytic cells A, B, C containing solutions of ZnSO4, AgNO3 and CuSO4, respectively are
connected in series. A steady current of 1⋅5 amperes was passed through them until 1⋅45 g of silver
deposited at the cathode of cell B. How long did the current flow ? What mass of copper and zinc were
deposited?

Cell B contains AgNO3 and reaction may be represented as : Ag+ + e− Ag

According to the equation, 108 g of silver deposited b 96487 C of electricity

96487
∴ 1⋅45 g of silver is deposited by = × 1⋅45 = 1295⋅6 C
108
Q 1295 ⋅ 6
Now Q = It ⇒ Time (t) = = = 863 sec.
I 1⋅ 5

The weight of copper and Zinc can be calculated by using Faraday’s second law of electrolysis

wt. of Ag Eq. wt. of Ag 1⋅ 45 108


In the cell B and C ⇒ = ⇒ =
wt. of Cu Eq. wt. of Cu wt. of Cu 31⋅ 75

1 ⋅ 45 × 31 ⋅ 75
⇒ Weight of copper deposited = = 0⋅426 g
108

wt. of Ag Eq. wt. of Ag 1 ⋅ 45 108


Similarly in cells A and B : = ⇒ =
wt. of Zn Eq. wt. of Zn wt. of Zn 32 ⋅ 7
jksahu74@gmail.com 89 | P a g e
1 ⋅ 45 × 32 ⋅ 7
⇒ Weight of zinc deposited = = 0⋅44 g
108

(27) Using the standard electrode potentials given, predict if the reaction between the following is feasible
:

(i) Fe3+ (aq) and I- (aq) (ii) Ag+ (aq) and Cu(s) (iii) Fe3+ and Br− (aq)

(iv) Ag(s) and Fe3+ (aq) (v) Br2(aq) and Fe2+ (aq)

(i) 2Fe3+ + 2I− 2Fe2+ + I2

E ocell = E oFe 3+ Fe 2+ − E oI I−
= 0⋅77 − 0⋅54 = 2⋅23 V
2

The emf of the reaction is positive, thus the reaction between Fe3+ and I− is feasible.

(ii) 2Ag+ + Cu Cu2+ + 2 Ag

E ocell = E oAg + Ag − E oCu 2+ Cu = 0⋅80 − 0⋅34 = 0⋅46 V

The emf of the reaction is positive, thus the reaction between Ag+ and Cu is feasible.

(iii) 2Fe3+ + 2Br− 2Fe2+ + Br2

E ocell = E oFe 3+ Fe 2+ − E oBr Br −


= 0⋅77 − 1⋅09 = − 1⋅32 V
2

The emf of the reaction is positive, thus the reaction between Fe3+ and Br− is not feasible.

(iv) Fe3+ + Ag Fe2+ + Ag+

E ocell = E oFe 3+ Fe 2+ − E oAg + Ag = 0⋅77 − 0⋅80 = − 0⋅03 V

The emf of the reaction is positive, thus the reaction between Fe3+ and Ag is not feasible.

(v) Br2 + 2Fe2+ 2Fe3+ + 2 Br−

E ocell = E oBr Br −
− E oFe 3 Fe 2+ = 1⋅08 − 0⋅77 = 0⋅31 V
2

The emf of the reaction is positive, thus the reaction between Fe2+ and Br2 is feasible.

(28) Calculate the standard electrode potential of Cu+ Cu. Given that the standard electrode
potential of Cu2+ Cu+ are 0⋅34 V and 0⋅15 V respectively.

Cu2+ + 2e− Cu; E0 = 0⋅34V ⇒ ∆G10 = − 2 × F × 0⋅34 J

Cu2+ + e− Cu+; E0 = 0⋅15 V ⇒ ∆G 02 = − 1 × F × 0⋅15 J

jksahu74@gmail.com 90 | P a g e
Aim : Cu+ + e− Cu; ∆G 30 = ?

But ∆G 30 = ∆G10 − ∆G 02 = − 0⋅53 F

∴ − nF E 0Cu + Cu = − 0⋅53 F Since n = 1 So E 0Cu + Cu = 0⋅53 V

(29) The E0 values corresponding to the following two electrode processes are :
(i) Cu+ Cu = 0⋅52 V and (ii) Cu2+ Cu+ = 0⋅16 V Formulate the galvanic cell for their
combination. What will be the standard cell potential for it.

For emf to be + ve, oxidation should take place on electrode (ii) i.e. half reactions will be

Cu+ + e− Cu (Reduction half)

Cu+ Cu2+ + e− (Oxidation half)

The overall cell reaction will be 2 Cu+ Cu + Cu2+

Hence the cell will be represented as : Cu+Cu2+  Cu+  Cu

E ocell = E oCu + Cu − E Cu 2+ Cu + = 0⋅52 − 0⋅16 = 0⋅36 V

jksahu74@gmail.com 91 | P a g e
The branch of physical chemistry which deals with the rate at which the chemical reactions occur,
the mechanism by which the chemical reactions take place and the influence of various factors
such as concentration, temperature, pressure, catalyst etc., on the reaction rates is called
chemical kinetics. Rate of reaction

With time concentration of reactants decreases and concentration of


products increases.

Reactant Product

The rate of the reaction is the change in concentration of one of the

reactants or products in unit time.

f
ff
f
ff
f
ff
f
ff
f
ff
f
ff
f
ff
f
f
Decreasef
f
ff
f
ff
f
ff
f
fff
f
f
inf
ff
f
ff
f
ff
f
ff
f
ff
f
ff
f
ff
f
ff
f
ff
f
ff
f
ff
f
ff
f
ff
f
f
concentrationf
f
ff
f
ff
f
ff
f
ff
f
f
off
ff
f
ff
f
ff
f
ff
f
ff
f
ff
f
ff
f
ff
f
f
reactantf
f
ff
f
f Increase
f
f
ff
f
ff
f
ff
f
ff
f
ff
f
ff
f
ff
f
ff
f
ff
f
fff
f
f
inf
ff
f
ff
f
ff
f
ff
f
ff
f
ff
f
ff
f
ff
f
ff
f
ff
f
ff
f
ff
f
ff
f
f
concentrationf
f
ff
f
ff
f
ff
f
ff
f
f
off
ff
f
ff
f
ff
f
ff
f
ff
f
ff
f
ff
f
f
productf
f
ff
f
ff
Rate = =
Time interval Time interval

When the time interval is large, h i At a particular instant of time,


h i
jf
∆f
f
fxf
f
ff
f
k
df
f
ff
f
f
xf
f
f
the rate calculated is called average rate the rate is called instantaneous rate j k
∆t dt

f
f
f
dxf
f
ff
f ∆x
f
f
ff
f
ff
f
f
= ,∆t Q 0
dt ∆t
Rate of reaction is always positive quantity.

Since concentration of reactant decreases with time, hence rate of reaction w.r.t reactant is

multiplied with a negative sign (-) to make rate of reaction positive.


@ A @ A
df
f
ff
f
ff
f
f
Af
f
ff
f
f df
f
ff
f
ff
f
f
Bf
f
ff
f
f
Rate = @ => ‘A’ is reactant Rate = + => ‘B’ is Product
dt dt

Reaction Rate and Stoichiometry


For the reaction: a A + bB cC + dD

∆[ A] d[A] ∆[ C] d[C]
Rate of consumption of 'A' = −
∆t
= −
dt
Rate of production of 'C' = + ∆t
= + dt

∆[ B] d[B] d[D]
Rate of consumption of 'B' = − =− Rate of production of 'D' = + ∆[ D] = +
∆t dt ∆t dt

jksahu74@gmail.com 92 | P a g e
1 d[A] 1 d[B] 1 d[C] 1 d[D]
Instantaneous rate = − −
a dt
=− −
b dt = + −
c dt = + −
d dt

1 ∆[ A] 1 ∆[ B] 1 ∆[ C] 1 ∆[ D]
a ∆t = − −
Average rate = − − b ∆t = + −
c ∆t = + −
d ∆t

Graphical representation of Instantaneous and average rate of a reaction

Product
C2
Concentration

C1 Slope = Instantaneous rate

Concentration
Slope = Instantaneous rate
at time 't' at time 't'

Reactant
C2 C1

t1 t t2
t1 t t2
Time
Time
∆X = − ( C2 - C1 ) = C1 − C2 ∆X = C2 − C1 ∆t = t2 − t1
∆t = t2 − t1
C1 − C2 ∆X C2 − C1
∆X Average rate = = t − t
Average rate = = t − t ∆t
∆t 2 1
2 1

Law of mass action


The rate of reaction is directly proportional to the product of molar concentration of reactants with
each term raised to some power, which may or may not be same as the stoichiometric co-efficient
of the reacting species in the balanced chemical equation.
@ A
df
f
ff
f
ff
f
f
Rff
f
ff
f @ Ax @ Ay
For a reaction a A + bB Product Rate = @ ∝ A B
dt

=> r = k [A]x [B]y k : constant of proportionality , known as rate constant.

x and y are experimentally determined quantity , may or may not be equal with a & b.

Rate constant is defined as the rate of the reaction when the concentrations of reactants are all
unity. Rate constant is also known as specific reaction rate.

Characteristics of rate constant


1: Rate constant (k) is independent of concentration and time, for a given reaction.
2: Rate constant is a measure of the rate of the reaction. Higher the value of ‘k’ faster is the
reaction, and vice versa.
3: The value of ‘k’ does not change during the progress of the reaction.
4: The value of ‘k’ is a constant at a constant temperature for a given reaction. K values are
different for different reactions. For a given reaction, k value increases with rise in temperature.
5: The value of ‘k’ changes when a catalyst is used.
6: Unit of rate constant depends upon the order of the reaction.

jksahu74@gmail.com 93 | P a g e
Order of the reaction

Order of the reaction in the rate law indicates how sensitive the rate is to changes in the
concentration of reactants. For the reaction a A + b B Product

Rate = k [A]x [B]y x: order w.r.t ‘A’ y: order w.r.t ‘B’ x+y : Overall order of reaction

Hence, the sum of the power of the concentration of reactants in the rate law expression is called
order of that chemical reaction. Order of reaction can be zero,1,2,3 or even fractional number.
Unit of rate constant
dx k 0
For zero order reaction ( k0 ) dt = 0
[R] = k0 So unit of k0 = unit of rate = mol.L-1Sec-1

dx k dx 1
For first order reaction ( k1 )
dt = 1 [R]
1
k1 = = time = Sec
-1
So unit of k1 = Sec-1
dt [R]

dx k 2 dx 1 -1
= L. mol Sec
-1
For second order reaction ( k 2) k2 = dt 2 =
dt = 2 [R] [R] time . Conc.

dx k 3 1
For third order reaction ( k 3) k3 = dx = = L2. mol-2 Sec-1
dt = 3 [R] dt [R]3 time . Conc.2
n-1 1-n
For 'n' th order reaction unit of kn = L M Sec−1

Examples of zero order reactions


Combination of hydrogen and chlorine gas on the hν
surface of water, in presence of light. H2
+ Cl2 2 HCl

Decomposition of ammonia on a hot molybdenum or 2 NH3 Mo or W


tungsten.
N2 + 3 H2

Thermal decomposition of HI on gold surface 2 HI Au
H2 + I2

Decomposition of phosphine on the surface molybdenum 2 PH Mo or W
or tungsten, at high pressure. 3
∆ high pressure
2P + 3 H2

Examples of First order reactions


Decomposition of nitrogen pentoxide 2N O 2 2N O
5 2 4 + O2

Decomposition of hydrogen peroxide 2 H 2O 2 2 H 2O + O2

Acid hydrolysis of esters +


CH3COOCH 3 + H2O H CH3COOH + CH3OH

Inversion of cane sugar +


C12H22O11 + H2O H C6H12O6
+ C6H12O6

Radioactive decay All radioactive disintegration processes


Decomposition of thionyl chloride SO2
SO2Cl2 + Cl2

Decomposition of azoisopropane (H3C)2HCN NCH(CH3)2 N2


+ C6H14

Examples of Second order reactions

jksahu74@gmail.com 94 | P a g e
Thermal decomposition of HI ∆
2 HI H2
+ I2

Saponification of esters CH3COOC 2H5 + NaOH CH3COONa + C2H5OH

Thermal decomposition of nitrous oxide 2 N2O 2 N2 + O2

Examples of Fractional order reactions


Photochemical combination of H2 + Br 2 2 HBr
Rate = k H2
B C1 B
Br 2
C1fffff
2
hydrogen and bromine
CH3CHO 400°C CH4
Decomposition of acetaldehyde + CO B
Rate = k CH3 CHO
C3fffff
2

Molecularity of reaction

Number of reacting species which must collide simultaneously in order to bring about a chemical
reaction is called molecularity.

Unimolecular Decomposition of ammonium nitrite NH4NO2 N2 + 2 H2 O

Bimolecular Dissociation of hydrogen iodide 2 HI H2 + I2

Trimolecular / Oxidation of nitric oxide 2 NO + O2 2 NO2


Termolecular
Molecularity is always a whole number i.e. 1,2,3........ It may not be zero or fraction.
Molecularity is a theoretical value. It is derived for each step of the mechanism of the
reaction.
Termolecular and more are rare.
Molecularity has no significance in case of complex reactions.

Elementary reaction

The reaction taking place in one step is called elementary reaction.From elementary reaction one
can determine the molecularity and order of reaction.
I −
Complex reaction
2 H2O2
Alkaline medium
2 H 2O + O2

When a single reaction follows a sequence of Mechanism


elementary reaction to give product is called complex
reaction. In complex reaction there is no meaning of
H2O2 + I− H2O + IO− [ Slow]
H2O2 + IO − H2O + I − + O2 [ Fast]
molecularity but the slowest step is the rate
determining step. From this step we can calculate 2 H2O2 2 H2O + O2

order of reaction. Rate α H2O2 I− Order = 2

Pseudo first order reaction

In this type of reaction, the rate of reaction depends upon only the concentration of one species,
whereas the concentration of other species remains almost constant.
+
H
Acid hydrolysis of ester: CH3COOC 2H5 + H 2O CH 3COOH + C2H5OH

jksahu74@gmail.com 95 | P a g e
Rate α [ Ester] [ Water] - - - - - - - - - It should

But Rate = k CH3COOC2H5 - - - - - - - - - - - - - - In this case k = k' [H2O]

Inversion of cane sugar:


+
H
C12H22O 11
+ H2O C6H12O 6 + C6H12O 6 Rate = k C12H22O 11
Glucose Fructose

Integrated rate equation

Zero order First order


Consider the zero order
@ A
reaction, A P Consider the first order @reaction,
A
A P
df
f
ff
ff
f
f
Af
ff
f
ff @ A0 df
f
ff
ff
f
f
Af
ff
f
ff @ A
Rate = @ = k0 A = k0 Rate = @ = k1 A
dt dt
@ A @ A
d A = @k 0 dt df
f
ff
ff
f
f
Af
ff
f
ff
ff
@ A = @k1 dt
On integrating from t=0, when the concentration A
of A is [A]0 , to the time t, when the molar On integrating from t=0, when the concentration
concentration of A is [A], we obtain of A is [A]0 , to the time t, when the molar
B C concentration of A is [A], we obtain
A
t

Z d A = @k 0 Z dt
@ A B C
A @ A
Z f
dff
f
ff
ff
f
ff
ff
f
t

@ A = @k1 Z dt
B C
0
A
A
0
B C A 0
A
@ A @ A 0
A @ A 0 = @k 0 t
@ A @ A @ A @ A
[ k0 t = A 0 @ A [ ln A @ ln A 0 = @k1 t
@ A @ A @ A @ A
f
f
ff
ff
f
f
0f
ff
f
ff
ff
f
ff
ff
f
ff
ff
f
f
A @ A [ ln A = ln A 0 @k1 t
[ k0 =
t The exponential form of the equation can be
@ A @ A
The rate proceeds with constant rate which is written as A = A 0 e@ k t 1

@ A
independent of concentration. @ A @ A ff
f
Af
ff
f
ff
ff
The rate constant is equal to the rate of k1 t = ln A 0 @ ln A = ln @ A0
A
reaction. @ A @ A
For 100% completion of zero order reaction, 1f
f f f
f
Af
ff
f
ff
ff ff
f
ff
ff
ff
2.303f
f
ff
ff
f
f ff
f
Af
ff
f
ff
ff
[ k1 = ln @ A0 = log @ A
0

Initial concentration = [A]0 t A t A


Final concentration [A] = 0 If concentration of reactants are [A]1 and [A]2 at
@ A @ A times t1 and t2 respectively, then rate equation
f
f
f
Af
f
ff
f
0f
f
ff
f
f
@f
f
ff
f
0f
f
ff
f f
f
f
Af
ff
f
f
0f
f
f can be written as: @ A
k0 = [ t100% =
t100% k0 f
f
ff
ff
f
ff
f
2.303f
f
ff
ff
f f
f
f
Af
ff
f
ff
ff
k1 = log @ A1
t 2 @ t1 A2
Half life of reaction ( )

Half life is the time in which the concentration of reactant is reduced to half of its initial

concentration. When t = [A] =

Zero order First order

jksahu74@gmail.com 96 | P a g e
@ A @ A @ A
@ A f
f
ff
f
f
ff
f
f
ff
f
f
0f
f
ff
f
fff
f
ff
ff
f
f ff
f
Af
ff
f
ff
ff 2.303
f
f
ff
ff
f
ff
ff
f
ff
ff
f f
f
f
Af
ff
ff
ff
f 2.303
f
f
ff
ff
f
ff
ff
f
ff
ff
f ff
ff
f
ff
ff
f
ff
ff
f
ff
ff
f
ff
ff
f
ff
ff
f
ff
ff
f
ff
ff
f
ff
ff
f
ff
f
@ A @ A A2.303 2.303 B0.3010
A 0@
f
f
ff
ff
f
f
0f
ff
f
ff
ff
f
ff
ff
f
ff
ff
f
f f
f
ff
ff
f
ff
ff
fff f f
fff
f
f
2f
ff
f
ff
ff
0
A @ A k1 = log @ A = log @ A 0 = log 2 =
k0 = [ k0 = t A t 1ffff A f
f
ff
f
ff
f
0f
f
ff k1 k1
t t 1ffff 2 2
ff
ff
ff
ff
f
0.693f
ff
ff
ff
2
@ A t 1ffff=
f
f
f
Af
ff
f
f
0f
ff k1
t 1fffff = 2
2
2 k0
The half-life period is independent of initial concentration of the
The half-life period is directly
proportional to initial reactant.
concentration of the reactant.
It is found that half-life of nth order reaction t 1fffff ∝ `f
f
ff
f
ff
f
ff
f
ff
f
ff
f
ff
f
ff
f
ff
f
ff
f
ff
f
ff
f
ff
f
ff
f
ff
f
ff
f
ff
f
ff
f
ff
f
ff
f
ff
f
ff
f
ff
f
ff
f
ff
f
ff
f
ff
f
ff
f
ff
f
ff
ff
ff
ff
ff
ff
ff
f
1
an @ 1
2 Concentration of reactant

Graphical representation of Zero order reaction

Versus Derivation Plot

@ A @ A
f
f
f
Af
f
ff
f
0f
f
ff
f
f
@f
f
ff
f
ff
f
f
Af
f
ff
f
f @ A @ A

Concentration [A]
k0 = [ k0 t = A 0 @ A Slope = -k0
t
Concentration & time @ A b c @ A
[ A = @k 0 t + A 0 [A]0
y = mx + c

Time (t)

@ A
Rate & Concentration df
f
ff
ff
f
f
Af
ff
f
ff @ A0
Rate = @ = k0 A = k0
Rate

dt k0

Concentration [A]

@ A f
f
1f
f
ff
f
ff
f
Slope =
f
f
f
Af
ff
f
f
0f
ff 2k 0
Half-life & Initial t 1fffff =
2 k0
concentration 2
h i

t 1fffff = jf
f
f
1f
ff
f
ff@ A
k A + 0
t 1fffff
0
2k 0
2
2
y = m x + c
[A]0
Graphical representation of first order reaction

jksahu74@gmail.com 97 | P a g e
@ A
log [A] & time ff
ff
f
ff
ff
f
f
2.303f
ff
f
ff ff
f
Af
ff
f
f
0f
ff f
f
fkf
f
ff
1f
ff
tf
f
ff
ff
f @ A @ A
k1 = log @ A [ = log A 0 @ log A
t A 2.303
Slope = @ f
f
ff
f
h i kf
f
ff
1f
f
ff
f
ff
f
ff
@ A f
f
ff
kf
f
ff
1f
f
ff
ff
f
ff @ A
[ log A =j@ k
t + log A 0 2.303
2.303
y = m x + c
log [A]0

log [A]
Time (t)

@ A
df
f
ff
ff
f
f
Af
ff
f
ff @ A
Rate & Concentration Rate = = k1 A
dt

Rate
y=mx + c Slope = k 1

[A]

ff
ff
ff
ff
f
0.693f
ff
ff
ff
t 1ffff=
Half-life & Initial concentration 2 k1

t 1fffff ff
f
ff
f
ff
f
ff
f
f
0.693f
f
ff
f
f
k1
2
[A]0

@ A
f
f
ff
ff
f
ff
2.303f
f
ff
ff
f
f ff
f
Af
ff
f
f
0f
ff
@ A k1 = log @ A
ff
f
ff
f
ff
0f
f
f
A t A
log & time @ A h i @ A
log @f
@ A
A ff
f
Af
ff
f
ff
ffj f
f
ff
kf
f
ff
1f
f
ff
ff
f
ff Af
f
ff
f
ff
fff
f
f
ff
f
kf
f
ff
1f
f
ff
f
ff
f
ff
0 kt 0
log @ A = A
A 2.303 A Slope =
y = mx + c 2.303

time

ff
f
ff
f
ff
h i dx @ A
jff
f
ff
f
ff
th dx k
@ An # log = n log A + log k n
For ‘n’ order reaction, Rate = kn A dt
dt
y = m x + c

jksahu74@gmail.com 98 | P a g e
n)
ctio
Second order re a
of
Third order
r d er
o
= n(
l o pe
Slope = k 3
Rate

Rate
Slope = k 2 ff
f
f
dxf
f
ff
log
dt log k n
[A]2 [A]3 @ A
log A

Half life of ‘n’th order reaction t 1fffff ∝ `f


f
ff
f
ff
f
ff
f
ff
f
ff
f
ff
f
ff
f
ff
f
ff
f
ff
f
ff
f
ff
f
ff
f
ff
f
ff
f
ff
f
ff
ff
ff
ff
ff
ff
f
1 P Q
an @ 1 Where n: order of reaction
2 Concentration

For 2 nd order reaction For 3 rd order reaction

t1 t1
2
2

1 1
[R]o [R]2o

Reaction Profile

When two reactant molecules possessing the necessary energy of activation, come together, they
form an activated complex or transition state.
Et T.S
The minimum amount of energy which must be
associated with molecules so that their mutual collisions
Potential energy

result in chemical reaction is called threshold energy.(Et ) E1


E2
The excess energy that the reactant molecules must
acquire in order to react to yield product is known as ER ∆r H
activation energy ( Ea) EP
Energy of activation for forward direction ( E1 ) = Et - ER
Reaction co-ordinate
The energy given out when the activated complex
decomposes into product ( E2 ) = Et - E p

The energy difference (∆E) between ER and EP

∆E = E1 - E2 = ( Et - ER ) - ( Et - EP ) = EP - ER

At constant pressure, ∆E is known as enthalpy of reaction (∆rH).

jksahu74@gmail.com 99 | P a g e
If E2 > E1 => Exothermic reaction and

Fraction of molecules
Most probable K:E
If E2 < E1 => Endothermic reaction
Energy of
t0C activation Et
The peak of Maxwell's distribution of energy
curve represents the kinetic energy
possessed by the maximum fraction of ( t + 10 ) 0 C
molecules and is called most probable kinetic
energy.
Kinetic energy
With increase of temperature

- Most probable K:E increases

- Fraction of molecules possessing most probable K:E decreases.


` a0
ff
ff
ff
f
Ratef
ff
ff
ff
ff
ff
ff
ff
ff
ff
ff
f
constantf
ff
ff
ff
ff
ff
f
atf
ff
ff
ff
f
tfff
f
+f
ff
ff
f
10f
ff
ff
ff
ff
ff
f
Cf
ff
f
Temperature coefficient = 0
=2
Rate constant at t C
When temperature increased by 100C the fraction of
molecules having K:E greater than threshold value Absence of
catalyst
becomes double. For which, for a chemical reaction with

Potential energy
rise in temperature by 10 0C, the rate constant is nearly
doubled.
Presence of
A catalyst increases the rate of reaction by lowering the catalyst
activation energy.
R
A catalyst does not alter Gibb's energy ( ∆G ) of a
P
reaction and equilibrium constant of a reaction, but Reaction co-ordinate
helps in attaining the equilibrium faster.

Collision Theory

According to collision theory, the rate of a chemical reaction depend upon :

(i) Activation energy [ Ea ]

(ii)Collision frequency [Z]

(iii) Probability or steric factor [p]

jksahu74@gmail.com 100 | P a g e
-Number of collisions per second per unit volume of the reaction mixture is known as collision
frequency (Z)

- The proper orientation of reactant molecules leads to bond formation.

H H
H
-
-
HO
C Br HO C Br HO C
H
+ Br
H H H H
H

- The improper orientation makes reactant molecules simply bounce back and no products are
formed.
H
δ+ δ− -
C Br HO No product
H
H

Effective collisions: - The collisions in which molecules collide with threshold energy and proper
orientation, so as to facilitate breaking and making of bonds to form products is called effective
collision.

Arrhenius equation for Activation energy

The temperature dependence of the rate constant (k) is directly proportional to the fraction of
molecules having energy equal to or greater than the activation energy (Ea).
Ef
f
faf
f
ff
f Ef
f
faf
f
ff
f
@ @
k ∝e RT [ k = Ae RT

A - Frequency factor, is a constant for a particular reaction.

R- 8.314 JK-1mol-1
Ea
Ea :- Activation energy J mol -1 Slope=
R
A & Ea known as Arrhenius parameters ln k
ln A

jksahu74@gmail.com 101 | P a g e
1
T
f
f
Ef
f
faf
f
ff
f
ln k = ln A @
RT
h i

[ ln k =j@ f
Ef
ff
af
f
f
kf
1f
f
ff
+ ln A
R T
y = mx + c

If k1 and k2 are the values of rate constants at temperature T1 and T2 , then:

lnk1 = ln A @ f
Ef
f
ff
af
ff
ff
f ` a
@ @@@@ @@@@ 1
RT1

lnk 2 = ln A @ f
f
ff
f
f
E af
f
ff
f
f ` a
@@@@ @@@@ @ 2
RT2

Subtracting equation (1) from (2) , we get


H I
f
f
ff
ff
fff
f
f f
f
ff
f
faf
f
ff
f
f Ef
f
faf
ff
fJ 1
ff
f
ff
f
@ f
E a E 1f
f
ff
ffK
ln k 2 @ lnk1 = @ =
RT1 RT2 R T1 T2
H I
kf
f
f
2f
f
f
= f
Ef
ff
af
f
fJ 1
ff
f
ff
f 1
[ ln @f
f
ff
ff
fK
k1 R T1 T2
H I
kf
f
f
2f
f
f f
f
ff
f
ff
f
f
Ef
f
ff
af
f
ff
f
ff
f
ff
f
ffJ 1f
f
ff
f
f f
f
1f
f
ff
fK
[ log = @
k1 2.303 R T1 T2

(1) The decomposition of N2O5 in CCL4 at 318 K has been studied by monitoring the concentration of
N2O5 in the solution. Initially the concentration of N2O5 is 2⋅33 mol L−1 and after 184 minutes, it is
reduced to 2⋅08 mol L−1. The reaction takes place according to the equation 2 N2O5 (g) → 4 NO2 (g) +
O2 (g) Calculate the average rate of this reaction in terms of hours, minutes and seconds. What is the
rate of production of NO2 during this period ?

1  ∆[ N 2O5 ]  1  (2 ⋅ 08 − 2 ⋅ 33) mol L-1 


Average Rate =  −  = −
2 ∆t  2  184 min 

1 60 min
= 6⋅79 × 10−4 mol L−1 min−1= 6⋅79 × 10−4 L−1 × = 4⋅07 × 10−2 mol L−1 h−1
min 1h
−4 −1 1 1 min
= 6⋅79 × 10 mol L × = 1⋅13 × 10 mol L−1 S−1
−5
min 60 sec

1  ∆[ NO 2 ]  ∆[ NO 2 ]
Rate of reaction =   ⇒ Rate of production of NO2 = = 4 × Rate of reaction
4  ∆t  ∆t

= 4 × 6⋅79 × 10−4 mol L−1 min−1 = 2⋅72 ×10−3 mol L−1 min−1

(2) Calculate the overall order of a reaction which has the rate expression
(a) Rate = k [A]1/2 [B]3/2 (b) Rate = k [A]3/2 [B]−1

(a) Rate = k [A]x [B]y order = x + y So order = ½ + 3/2 = 2, i.e., second order

jksahu74@gmail.com 102 | P a g e
(b) order = 3/2 + (−1) = ½, i.e., half order.

(3) Identify the overall order of a reaction which has the rate expression
(i) k = 2⋅3 × 10−5 L mol−1 S−1 (ii) k = 3 × 10−4 S−1

(i) The unit of second order rate constant is L mol−1 S−1, therefore

k = 2⋅3 × 10−5 L mol−1 S−1 represents a second order reaction.

(ii) The unit of a first order rate constant is S−1 therefore

k = 3 × 10−4 S−1 represents a first order reaction.

(4) The initial concentration of N2O5 in the following first order reaction
N2O5(g) 2NO2(g) + 1/2O2 (g) was 1.24 × 10−2 mol L−1 at 318 K. The concentration of N2O after 60
minutes was 0.20 × 10−2 mol L−1. Calculate the rate constant of the reaction at 318 K.

k=
2.303 [R ]
log 1 =
2.303
log
1.24 × 10 2 mol L-1
For a first order reaction :
t 2 − t1 [R ]2 (60 min − 0 min ) 0.20 × 10-2 mol L-1
2.303
= log 6.2 min −1 = 0.0604 min −1
60
(5) The following data were obtained during the first order thermal decomposition of N2O5 (g) at
constant volume : 2N2O5 2N2O4 + O2
S.No. Time / s Total Pressure / atm
1 0 0.5
2 100 0.512
Calculate the rate of constant.

Let the pressure of N2O5 (g) decrease by 2x atm. As two moles of N2O5 decomposes to give two moles
of N2O4 (g) and one mole of O2 (g), the pressure of N2O4 (g) increases by 2x atm and that of O2 (g) increases
by x atm.

2 N2 O5 2 N2 O4 + O2
Start t = 0 0⋅5 atm 0 atm 0 atm

At time t = 100 s (0⋅5 − 2x) atm 2x atm x atm

PTotal = PN2O5 + PN2O4 + PO2 [At time 100 s]

0⋅512 = (0⋅5 − 2x) + 2x + x = 0⋅5 + x

x = 0⋅512 − 0⋅5 = 0⋅012

PN 2O5 after time 100 s = 0⋅5 − 2x = 0⋅5 − 2 × 0⋅012 = 0⋅476 atm

2 ⋅ 303 p 2 ⋅ 303 0⋅5 2 ⋅ 303


k= log = log = × 0 ⋅ 0216 = 4 ⋅ 98 ×10−4 S-1
t p 100 0 ⋅ 476 100

(6) A first order reaction is found to have a rate constant, k = 5⋅5 × 10−14 S−1. Find the half-life of the
reaction.
jksahu74@gmail.com 103 | P a g e
0 ⋅ 693 0 ⋅ 693
Half-life for a first order reaction is t 1 = = −14 −1
= 1 ⋅ 26 ×10−14 S
2
k 5 ⋅ 5 × 10 S
(7) Show that in a first order reaction, time required for completion of 99⋅9% is 10 times of half-life
(t1/2) of the reaction.

When reaction is completed 99⋅9%, [R] = [R]0 = − 0⋅999[R]0 = 0⋅001 [R]0 = 1 × 10−3 [R]0

2 ⋅ 303 [ R ]0 2 ⋅ 303 [R ] 2 ⋅ 303 6 ⋅ 909


t= log = log −3 0 = log 103 =
k [R ] k 10 [ R ]0 k k

0 ⋅ 693
For half life of the reaction : t 1 =
2 k

t 6 ⋅ 909 k
= × = 10 => t = 10 t 1fff
t1 k 0 ⋅ 693 2
2

(8) Hydrolysis of methyl acetate in aqueous solution has been studied by titrating the liberated

acetic acid against sodium hydroxide. The concentration of the ester at different times is given
below.
t/min 0 30 60 90
C / mol L−1 0⋅8500 0⋅8004 0⋅7538 0⋅7096

Show that it follows a pseudo first order reaction, as the concentration of water remains
nearly constant (55 mol L−1), during the course of the reaction. What is the value of k’ in
this equation ? Rate = k’ [CH3COOCH3][H2O]

For pseudo first order reaction, the reaction should be first order with respect to ester when [H2O] is
2 ⋅ 303 C
constant. The rate constant k for pseudo first order reaction is : k = log O {Q k = k ' [H 2O] }
t C
From the above
t/min C/mol L−1 K/ mol L−1
0 0.8500 -
30 0.8004 2.004 × 10−1
60 0.7538 2.002 × 10−1
90 0.7096 2.005 × 10−1

It can be seen that k[H2O] is constant and equal to 2.004 × 10−3 min−1 and hence, it is pseudo

first order reaction. We can now determine k’ from:



K’[H2O] = 2.004 × 10−3 min −1 k’ [55 mol L−1] = 2.004 × 10−3 min−1

k’= 3.64 × 10−5 mol−1 L min−1

(9) The rate constants of a reaction at 500K and 700K are 0.02 s−1 and min−1 respectively. Calculate
the values of Ea and A.

jksahu74@gmail.com 104 | P a g e
k2 E a  T2 − T1  0.07  Ea   700 − 500 
log = ⇒ log = 
k1 2.303R  T1T2  0.02  2.303 × 8.314JK −1   700 × 500 

E a × 5.714 ×10−4 19.15


0.544 = ⇒ E a = 0.544 × × 10−4 = 18230.8J
19.15 5.714

Ea 18230.8
0.02
Since k = Ae RT ⇒ 0.02 = Ae 8.314 ⇒A= = 1.61
0.012

(10) The first order rate constant for the decomposition of ethyl iodide by reaction
C2H5I(g) C2H4(g) + HI (g) at 600K is 1.60 × 10-5 s−1. Its energy of activation is 209
kJ/mol. Calculate the rate of constant of the reaction at 700K.
Ea  1 1 
We know that : log k 2 − log k1 = −
2 ⋅ 303R  T1 T2 
Ea  1 1 
log k2 = log k1 = −
2 ⋅ 303R  T1 T2 

209000 J mol L-1  1 1 


= log (1⋅60 × 10−5) + −
2 ⋅ 303 × 8 ⋅ 314 J mol L K  600 700 
-1 -1 

log k2 = − 4⋅796 + 2⋅599 = −2⋅197

k2 = 6⋅36 × 10−3 S−1

(11) For reaction R → P, the concentration of a reactant changes from 0⋅03 M to 0⋅02 M in 25 minutes.
Calculate the average rate of reaction using units of time both in minutes and seconds.
− ∆[ R ] − (0 ⋅ 02 − 0 ⋅ 03) 0 ⋅ 01
Average rate of reaction = = = = 4 × 10−4 M min -1
∆t 25 25

 1  1 min 
(4 ×10 −4
M) 
−6
 = 6 ⋅ 66 ×10 M s
-1

 min  60 sec 

(12) In a reaction, 2A → Products, the concentration of A decreases from 0⋅5 mol L−1 to 0⋅4 mol L−1 in
10 minutes. Calculate the rate during this interval ?

For the reaction 2A Product

1 ∆[ A ] 1 (0 ⋅ 4 − 0 ⋅ 5) 0 ⋅1
Average rate of the reaction = = × = =0⋅005 M min−1
2 ∆t 2 10 20

(13) For a reaction, A + B → Product; the rate law is given by, r = k [A]1/2 [B]2. What is the order of
the reaction ?
1
Order of the reaction = + 2 = 2⋅5
2
(14) The conversion of molecules X to Y follows second order kinetics. If concentration of X is
increased to three times how will it affect the rate of formation of Y ?

For the reaction X Y The rate of reaction (r1) = k[X]2


jksahu74@gmail.com 105 | P a g e
If concentration of X increased by 3 times; then rate of reaction (r2) = k[X]2 = 9k[X]2 = 9r1

Since rate of consumption of X = Rate of formation of y; So rate of formation of Y increases by 9 times

(15) A first order reaction has a rate constant 1⋅15 × 10−3 s−1. How long will 5 g of this reactant take to
reduce to 3 g ?
2 ⋅ 303 [ R ]O 2 ⋅ 303 5
For a first order reaction t = log = −3
log = 444 s
k [ R ] 1 ⋅15 × 10 3

(16) Time required to decompose SO2CI2 to half of its initial amount is 60 minutes. If the
decomposition is a first order reaction, calculate the rate constant of the reaction.

 
The half life of the reaction  t 1  = 60 min
 2
0 ⋅ 693 0 ⋅ 693 0 ⋅ 693
For first order reaction, Rate constant of the reaction (k) = = =
t1 60 60 × 60
2
−4 −1
= 1⋅925 × 10 S

(17) The rate of the chemical reaction doubles for an increase of 10K in absolute temperature from
298K. Calculate Ea.
k Ea  1 1 
According to Arrhenius equation, log 2 = −
k1 2 ⋅ 303 R  T1 T2 
k2
=2, T1 = 298 K and T2 = 308 K, R = 8⋅314 Jk−1 mol−1
k1
Ea  1 1  10 E a
log 2 =  −  ⇒ 0⋅3010 =
2 ⋅ 303 × 8 ⋅ 314  298 308  2 ⋅ 303 × 8 ⋅ 314 × 298 × 308

0 ⋅ 3010 × 2 ⋅ 303 × 8 ⋅ 314 × 298 × 308


∴ Ea = = 528977 J mol−1 = 52⋅898 kj mol−1
10

(18) The activation energy for the reaction 2 HI(g) → H2 + I2 (g) is 209⋅5 kJ mol−1 at 581 K. Calculate
the fraction of molecules of reactants having energy equal to or greater than activation energy ?

Fraction of molecules having energy equal to or greater than the activation energy,

Ea Ea Ea
⇒ In x = - ⇒ 2⋅303 log x = - ⇒ log x = -
RT RT 2 ⋅ 303 RT

− 209500 J mol -1
⇒ log x = = − 18⋅8323 = 19 ⋅1677
2 ⋅ 303 × 8 ⋅ 314 J K -1 × 581 K

⇒ x = antilog (19 ⋅1677) = 1⋅471 × 10−19

(19) For the reaction: 2 A + B → A2B the rate = k[A][B]2 with k = 2⋅0 × 10−6 mol−2 L2 s−1.
Calculate the initial rate of the reaction when [A] = 0⋅1 mol L−1, [B] = 0⋅2 mol L−1.
Calculate the rate of reaction after [A] is reduced to 0⋅06 mol L−1.

jksahu74@gmail.com 106 | P a g e
Rate of reaction (r) = k [A][B]2 = (2⋅0 × 10−6) × (0⋅1) × (0⋅2)2 = 8⋅0 × 10−9 mol L−1 s−1

When [A] = 0⋅06 mol L−1 Then rate of reaction (r) = (2⋅0 × 10−6) × (0⋅06) × (0⋅2)2 = 4⋅8 × 10−9

(20) The decomposition of NH3 on platinum surface is zero order reaction.


What are the rates of production of N2 and H2 if k = 2⋅5 × 10−4 mol−1 L s−1 ?

2 NH3 N2 + 3 H2

1 d[NH 3 ] d[N 2 ] 1 d[H 2 ]


Rate of reaction = − = =
2 dt dt 3 dt

k0 = 2⋅5 × 10−4 mol−1 L s−1 As Rate = ko[NH3]0 ⇒ Rate = k0 = 2⋅5 ×10−4

d[N 2 ]
Rate of production of N2 = = Rate = 2⋅5 × 10−4
dt
d[H 2 ]
Rate of production of H2 = = 3 × 2⋅5 × 10−4 = 7⋅5 × 10−4
dt
(21) The decomposition of dimethyl either leads to the formation of CH4, H2 and CO and the reaction
rate is given by Rate = k [CH3OCH3]3/2 The rate of reaction is followed by increase in pressure in a
closed vessel, so the rate can also be expressed in terms of the partial pressure of dimethyl either, i.e.,
Rate = kpCH3 O CH3.
If the pressure is measured in bar and time in minutes, then what are the units of rate and rate
constants ?

Pr essure
Unit of rate of reaction = = bar⋅ min−1
time
1
3 Rate bar ⋅ min −1 −
Unit of rate constant: Rate = k [Pr essure]2 ⇒k= 3
= 3
= (bar ) 2
min−1
[Pr essure] 2 [bar ]2

(22) A reaction is second order with respect to a reactant. How is the rate of reaction affected if the
concentration of the reactant is (i) doubled (ii) reduced to half ?

Rate (r) = k[A]2

(i) When the concentration of reactant is doubled, then the rate (r1) = k[2A]2 = 4k[A]2 = 4r

Thus, rate of reaction becomes four times.


2
1  1 1
(ii) When the concentration of reactant reduced to half, then the rate (r2) = k  A  = k[ A]2 = r
2  4 4
Thus, rate of reaction is reduced to one fourth.

(23) In a pseudo first order hydrolysis of ester in water, the following results were obtained:

t/s 0 30 60 90

[Ester]/mol L−1 0⋅55 0⋅31 0⋅17 0⋅085

(i) Calculate the average rate of reaction between the time interval 30 to 60 seconds.
(ii) Calculate the pseudo first order rate constant for the hydrolysis of ester.

jksahu74@gmail.com 107 | P a g e
− (0 ⋅17 − 0 ⋅ 31) 0 ⋅14
(i) Average rate of reaction = = =4⋅67 × 10−3 Ms−1
60 − 30 30

2 ⋅ 303 a 2 ⋅ 303 0 ⋅ 55
(ii) k[H2 O] = log = log = 1⋅91 × 10−2 s−1
t a−x 30 0 ⋅ 31

(24) A reaction is first order in A and second order in B.


(i) Write the differential rate equation.
(ii) How is the rate affected on increasing the concentration of B three times ?
(iii) How is the rate affected when the concentrations of both A and B are doubled ?

(i) Differential rate equation (r) = k [A][B]2

(ii) When concentration of B increase by 3 times, then rate (r1) k[A][3B]2 = 9k [A][B]2 = 9 r

Thus, rate becomes 9 times.

(iii) When concentration of both A and B are double, then rate (r2) = k[2A][2B]2 = 8k [A][B]2 = 8r

Thus, rate becomes 8 times.

(25) In a reaction between A and B, the initial rate of reaction (r0) was measured for different initial
concentrations of A and B as given below:

I II III
−1
A/mol L 0⋅20 0⋅20 0⋅40
B/mol L−1 0⋅30 0⋅10 0⋅05
r0/mol L−1s−1 5⋅07 × 10−5 5⋅07 × 10−5 1⋅43 × 10−4
What is the order of the reaction with respect to A and B ?

Let the rate equation is represented as r = k[A]x[B]y

In first experiment : 5⋅07 × 10−5 = k[0⋅20]x[0⋅30]y

In second experiment : 5⋅07 × 10−5 = k[0⋅20]x[0⋅10]y

5 ⋅ 07 × 10−5 k[0 ⋅ 20] [0 ⋅ 30] y


x
First experiment
= = ⇒ 1 = 3y ⇒ 30 = 3y i.e. y = 0
k[0 ⋅ 20] [0 ⋅10]
−5 x y
Second experiment 5 ⋅ 07 ×10

In third experiment : 1⋅43 × 10−4 = k[0⋅40]x[0⋅05]y


1 ⋅ 43 ×10−4 k[0 ⋅ 40] [0 ⋅ 05] y
x
Third experiment
= = ⇒ 2⋅82 = 2x[0⋅5]y ⇒ 2⋅82 = 2x (Q y = 0)
Second experiment 5 ⋅ 07 ×10−5 k[0 ⋅ 20] x[0 ⋅10] y

3
3
x
i.e 2 = 2 2
⇒ x=
2
3
∴ The rate equation is r = k[A] [B]0 2

3
The order of reaction w. r. t A is and w. r. t B is zero.
2
jksahu74@gmail.com 108 | P a g e
(26) The following results have been obtained during the kinetic studies of the reaction:
2A+B C + D Determine the rate law and the rate constant for the reaction.

Experiment [A]/mol L−1 [B]/mol L−1 Initial rate of formation of D/mol L−1 min−1

I 0⋅ 1 0⋅ 1 6⋅0 × 10−3
II 0⋅ 3 0⋅ 2 7⋅2 × 10−2
III 0⋅3 0⋅ 4 2⋅88 × 10−1
IV 0⋅4 0⋅ 1 2⋅40 × 10−2

Let the rate equation is r = k[A]x[B]y

In first experiment 6⋅0 × 10−3 = k[0⋅1]x[0⋅1]y

In second experiment 7⋅2 × 10−2 = k[0⋅3]x[0⋅2]y

In third experiment 2⋅88 × 10−1 = k[0⋅3]x[0⋅4]y

In fourth experiment 2⋅40 × 10−2 = k[0⋅4]x[0⋅1]y

2 ⋅ 88 × 10 −1 k[0 ⋅ 3] [0 ⋅ 4] y
x
Third experiment
= = ⇒ 4 = 2y ⇒ 22 = 2y ⇒ y = 2
Second experiment 7 ⋅ 2 × 10 − 2 k[0 ⋅ 3] x [0 ⋅ 2]
y

Fourth experiment 2 ⋅ 40 × 10 −2 k[0 ⋅ 4] [0 ⋅ 1] y


x
= = ⇒ 4 = 4x ⇒ x = 1
6 ⋅ 0 × 10 −3 k[0 ⋅ 1] x [0 ⋅ 1]
y
First experiment

∴ Rate equation can be written as r = k[A][B]2

So rate equation can be written as : 6⋅0 × 10−3 = k [0⋅1][0⋅1]2 = k × 0⋅1 × 0⋅01 = k × 0⋅001 = k × 10−3

6 ⋅ 0 ×10−3
⇒k= = 6⋅0 M−2 min−1
10−3

(27) The reaction between A and B is first order with respect to A and zero order with respect to B.
Fill in the blanks in the following table:
Experiment [A]/mol L−1 [B]/mol L−1 Initial rate/mol L−1 min−1

I 0⋅ 1 0⋅ 1 2⋅0 × 10−2
II − 0⋅2 4⋅0 × 10−2
III 0⋅4 0⋅ 4 −
IV − 0⋅2 2⋅0 × 10−2

The rate equation is r = k [A][B]0

In First experiment : 2⋅0 × 10−2 = k[0⋅1][0⋅1]0 =0⋅1k = k × 10−1 ⇒ (Rate constant) k = 2⋅0 × 10−1

4 ⋅ 0 ×10−2
In Second experiment : 4⋅0 × 10−2 = (2⋅0 × 10−1) [A][0⋅2]0 ⇒ [A] = −1
= 0⋅2 mol L−1
2 ⋅ 0 ×10

jksahu74@gmail.com 109 | P a g e
In Third experiment : Rate (r) = (2⋅0 × 10−1) [0⋅4][0⋅4]0 = 8⋅0 × 10−2 mol L−1 min−1.

2 ⋅ 0 ×10−2
In fourth experiment : 2⋅0 × 10−2 = (2⋅0 × 10−1) [A][0⋅2]0 ⇒ [A] = −1
= 0⋅1 mol L−1
2 ⋅ 0 ×10

(28) Calculate the half-life of a first order reaction from their rate constants given below:
(i) 200 s−1 (ii) 2 min−1 (iii) 4 years−1

  0 ⋅ 693 0 ⋅ 693
Half – life  t 1  = (i) Half – life = = 3⋅46 × 10−3 s
 2  Rate constant (k) 200s −1

0 ⋅ 693 0 ⋅ 693
(ii) Half – life = = 0⋅346 min (iii) Half – life = = 0⋅173 year
2 min −1 4 year −1

(29) The half – life for radioactive decay of 14C is 5730 years. An archaeological artifact containing
wood had only 80% of the 14C found in a living tree. Estimate the age of the sample.

  0 ⋅ 693
Half life  t 1  =5730 years ⇒ Rate constant (k) = year−1
 2 5730

Let intial concentration of C14 = a Final concentration = 0⋅80 a

2 ⋅ 303 a 2 ⋅ 303 × 5730 a 10


∴ Age of the sample (t) = log = log = 19042 log
k a−x 0 ⋅ 693 0 ⋅ 80a 8

= 19042 (log 10 − log 8) = 19042 (1 − 0⋅9030) = 19042 × 0⋅097 = 1847 years

(30) The rate constant for a first order reaction is 60 −1. How much time will it take to reduce the initial
concentration of the reactant to its 1/16th value ?
a
Rate constant (k) = 60 −1 Let initial concentration = a Final concentration =
16

2 ⋅ 303 C 2 ⋅ 303 16a 2 ⋅ 303 2 ⋅ 303 2 ⋅ 303 × 4 × log 2


∴ Required time (t) = log O = log log 16 = log 24 =
k C 60 a 60 60 k

2 ⋅ 303 × 4 × 0 ⋅ 3010
= = 0⋅0462 s = 4⋅60 × 10−2 seconds
60
(31) During nuclear explosion, one of the products is 90Sr with half-life of 28⋅1 years. If 1 µg of 90Sr was
absorbed in the bones of a newly born baby instead of calcium, how much of it will remain after 10
years and 60 years if it is not lost metabolically.

  0 ⋅ 693
Half life  t 1  = 28⋅1 year ⇒ Rate constant (k) = year−1
 2 28 ⋅1

Intial concentration of Sr90 i.e. CO = 1 µg Let final concentration = C after 10 years

2 ⋅ 303 C 0 ⋅ 693 2 ⋅ 303 1 1 0 ⋅ 693 × 10


k= log O ⇒ = log ⇒ log = = 0 ⋅107
k C 28 ⋅1 10 C C 28 ⋅1× 2 ⋅ 303

jksahu74@gmail.com 110 | P a g e
1 1
⇒ = antilog (0⋅107) = 1⋅27 ⇒ C= = 0⋅7874 µg
C 1 ⋅ 27

Similarly after 60 years the final amount will be 0⋅228 µg

(32) For a first order reaction, show that time required for 99% completion is twice the time required
for the completion of 90% of reaction.

2 ⋅ 303 C
For a first order reaction : t = log O
k C

2 ⋅ 303 100 2 ⋅ 303 × log 100 2 ⋅ 303 × 2


For 99% completion : t 99% = log = =
k 100 − 99 k k

2 ⋅ 303 100 2 ⋅ 303 2 ⋅ 303


For 90% completion : t 90% = log = log 10 =
k 100 − 90 k k

2 ⋅ 303 × 2
t k
∴ 99% = = 2 ⇒ t99% = 2 × t90%
t 90% 2 ⋅ 303
k

(33) A first order reaction takes 40 min for 30% decomposition. Calculate t1/2.

30% complete in 40 min means: Time (t) = 40 min.

Intial concentration (CO) = 100 Final concentration (C) = 100 − 30 = 70


2 ⋅ 303 C O 2 ⋅ 303 100 2 ⋅ 303
Rate constant (k) = log = log = × 0 ⋅1549 = 0 ⋅ 00829 min −1
t C 40 70 60

  0 ⋅ 693 0 ⋅ 693
Half life  t 1  = = = 77 ⋅ 7 min
 2 k 0 ⋅ 00892

(34) For the decomposition of azoisopropane to hexane and nitrogen at 543 K, the following data are
(CH3)2CHN =NCH(CH3)2 N2 + C6H14
t (sec) 0 360 720
Pressure (atm) 35⋅0 54⋅0 63⋅0

Obtained Calculate the rate constant.

The reaction is A B + C type

Initial pressure Pi 0 0

Final pressure Pi − x x x

So total pressure (Pf) = Pi − x + x + x + = Pi + x

x = Pt − Pi ⇒ Final pressure of the reactant (Pr) = Pi − x = P1 − (Pt − Pi)= 2Pi − Pt

jksahu74@gmail.com 111 | P a g e
2 ⋅ 303 P 2 ⋅ 303 Pi
So rate constant of the reaction (k) = log i = log
t Pt t 2Pi − Pt

2 ⋅ 303 35 2 ⋅ 303
∴Rate constant after 360 s : k = log = log 2 ⋅1875 = 2 ⋅17 × 10−3 s −1
360 70 − 54 360

2 ⋅ 303 35 2 ⋅ 303
Rate constant after 720 s : k = log = log 5 = 2 ⋅ 24 ×10−3 s −1
360 70 − 54 720

(35) The following date were obtained during the first order thermal decomposition of SO2Cl2 at a
constant volume. SO2Cl(g)→SO2(g)+Cl2(g)
t(sec) 0 100
Total pressure (atm) 0⋅5 0⋅6
Calculate the rate of the reaction when total pressure is 0⋅65 atm.

For this reaction initial pressure (Pi) = 0⋅5 atm

2 ⋅ 303 Pi 2 ⋅ 303 0⋅5


For first order reaction, Rate constant (k) = log = log
t 2Pi − Pt 100 2× 0⋅5 − 0⋅6

2 ⋅ 303 2 ⋅ 303
= log1 ⋅ 25 = × 0 ⋅ 0969 = 2 ⋅ 23 × 10 −3 s −1
100 100

If total pressure is 0⋅65 atm, then partial pressure of SO2 Cl2 = 2Pi − Pt = 2 × 0⋅5 − 0⋅65 = 0⋅35 atm

Rate of reaction ∝ Partial pressure of SO2 Cl2

∴ Rate of reaction = Rate constant × Partial pressure of SO2 Cl2

= 2⋅23 × 10−3 × 0⋅35 = 7⋅8 × 10−4 atm s−1

(36) The rate constant for the decomposition of hydrocarbons is 2⋅418 × 10−5 s−1 at 546 K. If the energy
of activation is 179⋅9 kJ/mol, what will be the value of pre-exponential factor.

k = 2⋅418 × 10−5 s−1; Ea = 17900 J mol−1; R = 8⋅314 JK−1 mol−1; T = 546 K

Ea
According to Arrhenius equation : log k = log A −
2 ⋅ 303RT

Ea  179900 
⇒ log A = log k + = log (2⋅418 × 10−5) +   = − 4⋅6134 + 17⋅21
2 ⋅ 303RT  2 ⋅ 303 × 8 ⋅ 314 × 546 
= 12⋅5916

∴ A = antilog (12⋅5916) = 3⋅9 × 10−12

(37) Consider a certain reaction A Products with k = 2⋅0 × 10−2 s−1. Calculate the
concentration of A remaining after 100 s if the initial concentration of A is 1⋅0 mol L−1.

For first order reaction : k =


2 ⋅ 303 C C
log O ⇒ log O =
kt
=
[
2 ⋅ 0 × 10−2 × 100 ]
= 0 ⋅ 8684
t C C 2 ⋅ 303 2 ⋅ 303
jksahu74@gmail.com 112 | P a g e
CO
= antilog (0⋅8684) = 7⋅3858 ⇒ Final concentration of A
C
CO 1
i.e. C = = = 0 ⋅135M
7 ⋅ 3858 7 ⋅ 3858

(38) Sucrose decomposes in acid solution into glucose and fructose according to the first order rate
law, with t1/2 = 3⋅00 hours. What fraction of sample of sucrose remains after 8 hours ?

0 ⋅ 693 0 ⋅ 693
For first order reaction; Rate constant (k) = = = 0 ⋅ 231 h -1
Half life 3

2 ⋅ 303 C 2 ⋅ 303 C
k= log O ⇒ 0 ⋅ 231 = log O
t C 8 C

CO 8 × 0 ⋅ 231 C
⇒ log = = 0 ⋅ 8024 ⇒ O = antilog (0⋅8024) = 6⋅3445
C 2 ⋅ 303 C

CO 1
∴ Fraction of sucrose remains = = = 0 ⋅1576 M
C 6 ⋅ 3445
(39) The decomposition of hydrocarbon follows the equation k = (4⋅5 × 1011s−1) e−28000K/T Calculate Ea.
Ea
-
According to Arrhenius equation k = A e RT

- 28000 J
According to the given data k = (4⋅5 × 1011s−1) e T

On comparison

Ea 28000
− =− ⇒ Ea = 28000R = 28000 × 8⋅314 = 232792 J mol−1
RT T

(40) The rate constant for the first order decomposition of H2O2 is given by the following equation:
Log k = 14⋅34 − 1⋅25 X 104 K/T Calculate Ea for this reaction and at what temperature will is half-
period be 256 minutes ?
E
− a Ea
According to Arrhenius equation : k = Ae RT
⇒ log k = log A −
2 ⋅ 303RT

1 ⋅ 25 ×104 K
The given equation: log k = 14⋅34 −
T
Ea 1 ⋅ 25 ×104
On comparison: = ⇒ Ea = 1⋅25 × 104 × 2⋅030 × R
2 ⋅ 303RT T

⇒ Ea = 1⋅25 × 104 × 2⋅303 × 8⋅314 = 239339 J mol−1 = 239⋅339 kJ mol−1

f
f
ff
f
ff
f
ff
f
ff
f
ff
f
f
0.693f
f
ff
f
ff
f
ff
f
f
Half life = 256 minutes ⇒ Rate constant (k) = = 4⋅51 × 10−5 s−1
256 B 60

1 ⋅ 25 ×104
According to Arrhenius theory : log k = 14⋅34 −
T

jksahu74@gmail.com 113 | P a g e
1⋅ 25 ×104
⇒ log (4⋅51 × 10−5) = 14⋅34 −
T

1⋅ 25 ×104
⇒ − 4⋅35 = 14⋅34 −
T
1 ⋅ 25 × 10 4
⇒ = 14⋅34 + 4⋅35 = 18⋅69
T
1 ⋅ 25 × 10 4
So the temperature (T) = = 669 K
18.69
(41) The decomposition of A into product has value of k as 4⋅5 × 103 s−1 at 100C and energy of
activation 60 kJ mol−1. At what temperature would k be 1⋅5 × 104 s−1 ?

k1 = 4⋅5 × 103 S−1 at T1 = 100C = 10 + 273 = 283 K

k2 = 1⋅5 × 104 S−1 at T2 = ?

Ea = 60 kJ mol−1 = 60000 J mol−1

k2 E a  T2 − T1 
According to Arrhenius equation: log =
k1 2 ⋅ 303R  T1 T2 

1 ⋅ 5 × 10 4 60000  T2 − 283 
log 3
=  
4 ⋅ 5 × 10 2 ⋅ 303 × 8 ⋅ 314  283 T2 

 T − 283 2 ⋅ 5288 × 283 T2 − 283


Log 3⋅33 = 3133⋅62  2  ⇒ = ⇒ T2 = 297⋅19 K = 24⋅190 C
 283 T2  3133 ⋅ 62 T2

(42) The time required for 10% completion of a first order reaction at 298 K is equal to that required
for its 25% completion at 308 K. If the value of A is 4 × 1010s−1. Calculate k at 318K and Ea.

2 ⋅ 303 C
For a first order reaction : t = log O
k C

2 ⋅ 303 100 2 ⋅ 303 100 2 ⋅ 303 × 0 ⋅ 0458 0 ⋅1055


At 298 K 10% completion ⇒ t = log = log = =
k1 100 − 10 k1 90 k1 k1

2 ⋅ 303 100 2 ⋅ 303 100 2 ⋅ 303 × 0 ⋅ 125 0 ⋅ 2879


At 308 K 25% Completion ⇒ t = log = log = =
k2 100 − 25 k2 75 k2 k2

0 ⋅1055 0 ⋅ 2879 k 0 ⋅ 2879


At the two temperature time is equal ⇒ = ⇒ 2= = 2 ⋅ 7289
k1 k2 k1 0 ⋅1055
Applying Arrhenius equation at temperature T1 = 298 K and T2 = 308 K

k2 E a  T2 − T1  Ea  308 − 298 
log =   ⇒ log 2⋅7289 =
k1 2 ⋅ 303R  T1 T2  2 ⋅ 303 × 8 ⋅ 314  298 × 308 

jksahu74@gmail.com 114 | P a g e
Ea  308 − 298 
⇒ 0⋅4360 =   ⇒ Ea = 76⋅623 kl mol−1
2 ⋅ 303 × 8 ⋅ 314  298 × 308 

Ea
log k = log A −
2 ⋅ 303 RT
[Q A = 4 ×1010 −1
s ]
76623
= log (4 × 1010) − = 10⋅6021 − 12⋅5843 = − 1⋅9822
2 ⋅ 303 × 8 ⋅ 314 × 318

⇒ k = antilog (− 1⋅9822) = antilog ( 2 ⋅0178) = 1⋅042 × 10−2 s−1

(43) The rate of a reaction quadruples when the temperature changes from 293 K to 313 K. Calculate
the energy of activation of the reaction assuming that it does not change with temperature.

T1 = 293 K k1 = k

T2 = 313 K k2 = 4k Ea = ?

k2 E a  T2 − T1 
log =
k1 2 ⋅ 303R  T1 T2 

4k Ea  313 − 293  E a × 20
⇒log =   ⇒ 0⋅6020 =
k 2 ⋅ 303 × 8 ⋅ 314  313 × 293  2 ⋅ 303 × 8 ⋅ 314 × 313 × 293

⇒ Ea = 52854⋅55 J mol−1 = 52⋅85 kJ mol−1

Shortest method for Half Life – Numericals [Having Limitation]


Let initial concentration of reactant = CO
Half life = t 1
2
Number of half life (n) Final concentration (C)
1
CO 1
After 1st = CO  
2  2
2
CO 1
2nd = CO  
4 2
3
CO 1
3rd = CO  
8 2
n
1
n th Final concentrat ion (C) = CO  
2
−1
(30) The rate constant for a first order reaction is 60 s . How much time will it take to reduce the
initial concentration of the reactant to its 1/16th value ?
  0 ⋅ 693
Rate constant (k) = 60s−1 ∴ Half – life  t 1  = sec Let initial concentration = CO
 2 60
4
CO 1
Final concentration (C) = = CO   ⇒ Number of half life (n) = 4
16 2
0 ⋅ 693 0 ⋅ 693
∴ Total time required (t) = n × t 1 = 4 × = = 0⋅0462 sec
2 60 15

jksahu74@gmail.com 115 | P a g e
The branch of physical chemistry, which deals with the nature of surfaces and also with the
chemical and physical processes which takes place on the surfaces, is called surface chemistry.

Catalysis
Colloidal
Adsorption properties

Surface
chemistry

Adsorption

The phenomenon of attracting and retaining the molecules of a substance (immiscible phase) on
the surface of a liquid or solid resulting into higher concentration of the molecules on the surface is
called adsorption.
Cause: Unbalanced forces of attraction or free valencies which is present at the solid or liquid
surfaces, have the property to attract and retain the molecules of a gas or dissolved substances to
their surfaces with which they come in contact.

Some basic terms used in adsorption:


(1) Interface: The plane which separates any two phases.
(2) Adsorbate: The substance which gets adsorbed on any surface.

(3) Adsorbent: The substance, on the surface of which adsorption takes place.

(4) Desorption: The removal of adsorbate from adsorbent.


(5) Absorption: It is the phenomenon in which a substance is uniformly distributed throughout the
bulk.
(6) Sorption: The phenomenon in which adsorption and absorption occur simultaneously.

(7) Occlusion: When adsorption of gas occur on the surface of metals.

Adsorption Absorption
It is a surface phenomenon It concerns with the whole mass of the
adsorbent
In it, the substance is only retained on the It implies that a substance is uniformly
surface and does not go into the bulk or interior distributed through the bulk of the solid or liquid.
of the solid or liquid.
Water vapors adsorbed by silica gel. Water vapors absorbed by anhydrous CaCl2.
Ammonia adsorbed by charcoal. NH3 is absorbed in water forming NH4OH

jksahu74@gmail.com 116 | P a g e
Consequences of adsorption
(1) The pressure of a gas in the enclosed vessel containing powdered charcoal decreases, as gas
molecules are adsorbed by charcoal.
(2) The solution of an organic dye, say methylene blue, turns colorless when animal charcoal is
added.
(3) Aqueous solution of raw sugar, when passed over beds of animal charcoal, becomes
colourless as the colouring substances are adsorbed by the charcoal.
(4) The air becomes dry in the presence of silica gel because the water molecules get adsorbed
on the surface of the gel.
(5) Due to the difference in degree of adsorption of gases by charcoal, a mixture of noble gases
can be separated by adsorption on coconut charcoal at different temperatures.
(6) A low grade sulphide ore is concentrated by separating it from silica and other earthy matter by
this method using pine oil and frothing agent.
(7) Adsorption indicators: Surfaces of certain precipitates such as silver halides have the property
of adsorbing some dyes like eosin, fluorescein, etc. and thereby producing a characteristic
colour at the end point.
(8) When a solution of acetic acid in water is shaken with charcoal, a part of the acid is adsorbed
by the charcoal and the concentration of the acid decreases in the solution.
(9) The litmus solution when shaken with charcoal becomes colourless.
(10) The precipitate of Mg(OH)2 attains blue colour when precipitated in presence of magneson
reagent.
Application of adsorption
[1] Production of high vaccum.
[2] Used as gas mask for breathing in coal mines to adsorb poisonous gases.
[3] Controlling humidity by removing moisture.
[4] For the removal of coloured material from solution.
[5] Used as heterogeneous catalysis.
[6] For the separation of inert gases from each other.
[7] Used as drugs to kill germs by the phenomenon of adsorption.
[8] Concentration of sulphide ores by froth floatation process.
[9] Used as indicator.
[10] For the chromatographic analysis.
Mechanism of adsorption
On the surface, the particles are not surrounded by atoms or molecules of their own kind on all
sides, and hence they possess unbalanced or residual attractive forces. These forces on the
adsorbent are responsible for attracting the adsorbate particles on its surface. The extent of

jksahu74@gmail.com 117 | P a g e
adsorption increased with the increase of surface area per unit mass of the adsorbent at a given
temperature and pressure.
Adsorption is accompanied by decrease in enthalpy as well as decrease in entropy of the system.
On the basis of equation, ∆G = ∆H – T∆S,
∆G can be negative if ∆H has sufficiently high negative value as – T∆S is positive.
Thus adsorption is favored by low temperature.

Physisorption Chemisorption
[1] It arises because of vander waal’s forces [1] It is formed by chemical bond formation.
[2] It is not specific in nature. [2] It is highly specific in nature.
[3] It is reversible in nature. [3] It is irreversible in nature.
[4] It depends on the nature of gas. More [4] It also depends on the nature of gas. Gases
easily liquefy gases are adsorbed readily. which can react with the adsorbent show
chemisorptions.

[5] Enthalpy of adsorption is low [5] Enthalpy of adsorption is high


( 20-40 KJ/mol.) ( 80-240 KJ/mol.)
[6] Low temperature is favourable for [6] High temperature is favourable for adsorption.
adsorption. It decreases with increase of It increases with increase of temperature.
temperature.
[7] No appreciable activation energy is needed. [7] High activation energy is sometimes needed.
[8] It depends on the surface area. It [8] It also depends on the surface area. It too
increases with increase of surface area. increases with increase of surface area.

[9] It results into multimolecular layers on [9] It results into unimolecular layer on adsorbent
adsorbent surface under high pressure. surface.
[10] The rate of adsorption increases with [10] There is no effect of pressure.
increase in pressure, at constant
temperature.

244 K At constant T
xf
f
ff
f
f
m
xf
f
ff
f
f
p
m
p
[11] The rate of adsorption decreases with [11]Rate of adsorption initially increases then
increase in temperature, at constant decreases with increase in temperature, as at
pressure high temperature breaking of bond occurs.

Constant P Constant P

xf
f
ff
f
f xf
f
ff
f
f
m m

T T
.

jksahu74@gmail.com 118 | P a g e
Freundlich adsorption isotherm
(The plot at constant temperature is known as isotherm)

According to Freundlich at a particular temperature, the Adsorption isotherm


quantity of gas adsorbed by unit mass of solid adsorbent is
xfffff 1f
f
ff ` a 195 K
related to the applied pressure as: = kPn n>1
m
xf
f
ff
f
f
xfffff 244 K
Where quantity of gas adsorbed per unit mass of m
m
273 K
adsorbent and p - Applied pressure

k and n are constants depend on the nature of the


p
adsorbent and the gas adsorbed at a particular
temperature.

xfffff 1ffff
Taking logarithm on both sides of the equation log = log P + log k
m n
1ffff
The factor can have values between 0 and 1 (probable range 0.1 to 0.5).
n
1ffff xfffff
When =0 [ = k (constant), the adsorption is independent of pressure.
n m
1ffff xfffff xfffff 1ffff
When =1 [ = kP iA e A ∝P slope =
n m m n b
i.e. The adsorption varies directly with pressure. xfffff θ
a
log
This isotherm is also generally used for adsorption of m
solutes from solution and has been found satisfactory. log k
The pressure in the expression is replaced by
concentration ( C ) of solution as : log P
xfffff 1f
f
ff
= kC
n
m
xfffff 1ffff
log = log k + log C
m n
Limitations of Freundlich adsorption isotherm

The experimental values, however, show some deviation from linearity, especially at lower
pressure and lower temperature. This relation is hence considered as an approximate one and it is
suitable for small range of pressure. The experimental isotherms always approach saturation at
high pressure. This cannot be explained by Freundlich isotherm. Thus it fails at high pressure.

jksahu74@gmail.com 119 | P a g e
Graphical representation of Freundlich adsorption

xf
f
ff
f
f
1f
f
f
ff

= k pn
m

Versus Derivation Plot


If f
1f
f
= 0 then
n
xf
f
ff
ff
= kp 0 = k
Between f
xf
f
ff
f m
xf
f
ff
f
f
Thus f
and p xf
f
ff
f
m = constant m
1f
f
f
m k
when = 0 independent of pressure
n
p

If f
1f
f xf
f
ff
f
f
=1 then = kp
n m
Slope = k
Thus f
xf
f
ff
f
∝ p
xf
f
ff
f
f m
Between and p y = mx + c xf
f
ff
f
f
m
m
when n = 1
p

xf
f
ff
f
f
If n = 1 then = kp
m 1
p e=
Taking logarithim: Sl o
f g
f
xf g xf
f
ff
f
f f g
f
ff
f
f log = log p + log k log
xf
f
ff
f
f
Between log and log p m
m m log k
y = mx + c
when n = 1
log p
1f
f
f xf
f
ff
f
f 0
If = 0 then = kp = k
n m
f g Taking logarithm:
Between log f
xf
f
ff
f f g
and log p xf
f
ff
f
f
m log = log k f g
m xf
f
ff
f
f
1f
f
f f g log
when = 0 xf
f
ff
f
f m log k
n Thus, log = constant
m
log p
independent of pressure

jksahu74@gmail.com 120 | P a g e
Catalysis
Substance which alters the rate of a reaction and themselves remain chemically and quantitatively
unchanged after the reaction is known as catalyst, and the phenomenon is known as catalysis.
Catalyst alter the rate by changing the activation energy.

Catalysis

Positive Negative Auto

Positive Catalysis: The substance that accelerated rate of the reaction by decreasing activation
energy (By stabilizing the activated complex i.e. TS )

Decomposition of KClO3 Absence of


catalyst
MnO 2
+

Potential energy
2 KClO 3 2 KCl 3 O2
270 °C
Presence of
Oxidation of SO2 catalyst
2 SO 2 V2O 5 R
+ O2 2 SO 3
P
Reaction co-ordinate is the function of collision number Reaction co-ordinate

Negative Catalysis: The substance that retard rate of the reaction by increasing activation
energy (By destabilizing the activated complex i.e. TS ) called negative
catalysis or inhibitors.
Presence of
Oxidation of sodium sulphite by air retarded by alcohol. catalyst
Alcohol
2 Na2SO3 + O2 2 Na2SO4
Potential energy

Oxidation of benzaldehyde retarded by Diphenyl amine.


2 C6H5CHO + O2 Diphenyl amine 2 C6H5COOH Absence of
catalyst
Tetra ethyl lead is added to petrol to retard the ignition
R of
petrol vapours on compression in an internal P
combustion engine and thus minimize the knocking
effect.

Auto Catalysis: In certain reactions, one of the products acts as a catalyst. In the initial stages the
reaction is slow but as soon as the products come into existences the reaction rate increases. This
type of phenomenon is known as auto-catalysis.
For example the ester hydrolysis slows in the beginning and becomes faster after sometime. Ester
on hydrolysis gives carboxylic acid and alcohol. Carboxylic acid produced in this reaction act as
auto catalyst for this reaction, thus as soon as acid is produced the rate of reaction starts
increasing.
+
H
Acid hydrolysis of ester: CH3COOC 2H5 + H 2O CH3COOH + C2H5OH

jksahu74@gmail.com 121 | P a g e
Catalysis

On the basis of physical state

Homogeneous Heterogeneous

Homogeneous catalysis

When the reactants and the catalyst are in the same phase (i.e., liquid or gas), the process is said
to be homogeneous catalysis.

2 SO 2 (g) NO (g)
+ O 2 (g) 2 SO 3 (g)

CH3COOCH3 (l) HCl (l)


+ H2O (l) CH3COOH (aq) + CH3OH (aq)

C12H22O 11 (aq) H2SO 4 (l)


+ H2O (l) C6H12O 6 (aq)
+ C6H12O 6 (aq)
Glucose Fructose

Theory:
Theory of homogeneous catalysis is also known as intermediate compound formation theory.
Can be explained with help of this reaction.

2 SO2 (g) NO (g)


+ O2 (g) 2 SO 3 (g)

1. Catalyst combines with one of the reactants and forms an unstable intermediate.

2 NO + O2 2 NO2
Catalyst

2. The intermediate reacts with another reagent or decomposes itself to give product.

SO2
+ NO2 SO3
+ NO

Heterogeneous catalysis

The catalytic process in which the reactants and the catalyst are in different phases is known as

heterogeneous catalysis.

Pt (s)
2 SO 2 (g)
+ O2 (g) 2 SO 3 (g)

N2 (g) Fe (s) 2 NH3 (g)


+ 3 H 2 (g)

4 NH3 (g) + 5 O 2 (g) Pt (s)


4 NO (g) + 6 H2O (g)

Vegetable oils (l) Ni (s)


+ H2 (g) Vegetable ghee (s)

jksahu74@gmail.com 122 | P a g e
Theory:
Theory of heterogeneous catalysis is also known as adsorption theory.
According to this theory, reaction involves following five steps:
(i) Diffusion of reactants to the surface of the catalyst.
(ii) Adsorption of reactant molecules on the surface of the catalyst.
(iii) Occurrence of chemical reaction on the catalyst’s surface through formation of an
intermediate.
(iv) Desorption of reaction products from the catalyst surface, and thereby, making the surface
available again for more reaction to occur.
( v) Diffusion of reaction products away from the catalyst’s surface.
During adsorption, some energy had been released, which is used to break the old bond of
reactant and an intermediate will produce, then the intermediate will give the product.
Adsorptio of A A Desorption
A + B reacting
+Reacting molecules of product +A B
B B
molecules Product
Catalyst molecules Adsorptio of Catalyst
Intermediate
surface Reacting molecules
having free
valencies [Adsorption of reacting molecules, formation of intermediate and desorption of products]

Catalysis

On the basis of doping

Promoter Poison

Promoter: A substance which is used to activate the catalyst is called promoter. e.g In Haber’s
process for manufacture of ammonia, molybdenum acts as promoter for iron catalyst.

Poison: A substance which is used to reduce the activity of a catalyst is called catalyst poison. e.g
presence of sulphur or quinoline reduces the activity of Lindlar’s catalyst. ( Pd + BaSO4 )

Catalysts in Industry

Process Catalyst
[1] Haber's process: [1] Finely divided iron, molybdenum as promoter
N2 + 3 H2 2 NH3

[2] Ostwald's process: [2]Platinised asbestos


4 NH3
+ 5 O2 4 NO
+ 6 H 2O

2 NO + O2 2 NO2

4 NO2 + 2 H 2O + O2 4 HNO3

[3] Contact process: [3] Platinised asbestos or vanadium pentoxide


[ V 2O 5 ]
2 SO2
+ O2 2 SO3

SO3
+ H2SO4 H 2S 2 O 7 Oleum

H2 S 2 O 7
+ H2O 2 H2SO4

jksahu74@gmail.com 123 | P a g e
Activity of catalyst
Activity of a catalyst is its ability to increase the rate of reaction.
The ability of a catalyst depends upon strength of chemisorptions but, reactants must not get
adsorbed so strongly that, they remain on the surface of catalyst.
For example reaction between H2 and O2 to form water in presence of platinum as catalyst takes
place with explosive violence.
In absence of catalyst, H2 and O2 can be stored indefinitely without any reaction.
2 H2 (g) Pt
+ O2 (g) 2 H2O (l)

Selectivity
The selectivity of a catalyst is its ability to direct a reaction to yield a particular product.
Ni
CO (g) + 3 H2 (g) CH4 (g) + H2O (g)

Cu / ZnO - Cr 2 O 3
CO (g) + 2 H2 (g) CH3OH (g)
Cu
CO (g) + H2 (g) HCHO (g)

Catalysts are highly selective in nature.


Shape-Selective Catalysis
The catalytic reaction that depends upon the cavities on the surface of the catalyst and the size of
the reactant and product molecules is called shape-selective catalysis.
[1] Zeolites are micro porous aluminosilicates with three dimensional network of silicates in which
some silicon atoms are replaced by aluminium atoms giving Al–O–Si framework.
[2] Zeolites are used as catalysts in petrochemical industries for cracking of hydrocarbons and
isomerisation.
[3] ZSM-5 ( Zeolite siever of molecular porosity ) converts alcohols directly into gasoline (petrol) by
dehydrating them to give a mixture of hydrocarbons.
Enzymes
Enzymes are complex nitrogenous organic compounds which are produced by living beings. They
are actually protein molecules of high molecular mass and form colloidal solutions in water.
[1 ] C12H22O11 (aq) Invertase
+H2O (l) C6H12O6 (aq) C6H12O6 (aq)
+
Cane sugar Glucose Fructose
Zymase
[2 ] C6H12O6 (aq) 2 C2H5OH (aq)
Ethyl alcohol
+ 2 CO2 (g)
Glucose
Diastase
[3 ] 2 (C6H10O5 )n (aq) + n H2O (l) n C12 H22 O 11 (aq)
Starch Maltose
Maltase
[4 ] C12 H22 O 11 (aq) + H2O (l) 2 C6 H12O6 (aq)
Maltose Glucose
Urease 2 NH 3 (g)
[5 ] NH 2CONH 2 (aq)
+ H2O (l) + CO 2 (g)
Urea
[6] Lacto bacilli enzyme converts milk into curd.
[7] The pepsin enzyme ( Present in stomach ) converts proteins into peptides
[8] The pancreatic trypsin ( present in intestine ) converts proteins into amino acids by hydrolysis.
Characteristics of enzyme catalysis

(i) Most highly efficient.


(ii) Highly specific in nature.

jksahu74@gmail.com 124 | P a g e
(iii) Highly active under optimum temperature. The optimum temperature range for enzymatic
activity is 298-310K.
(iv) Highly active under optimum PH. Optimum pH range for enzymatic activity is 5 to 7.
(v) Increasing activity in presence of activators and co-enzymes.
(vi) The inhibitors or poisons interact with the active functional groups on the enzyme surface and
often reduce or completely destroy the catalytic activity of the enzymes.
Mechanism of enzyme catalysis
The enzyme-catalysed reactions may be considered to proceed in two steps.
Step 1: Binding of enzyme to substrate to form an activated complex.

E + S @@ Q ES ≠
Step 2: Decomposition of the activated complex to form product.

ES ≠ @@ Q E + P

Colloidal state

The state of a substance in which solute and solvent float with each other is called colloidal state.
It is a heterogeneous solution having particle size 1 nm – 1000 nm. It passes through ordinary
filter paper but not through animal membrane. e.g. Soap solution, milk , ink.
Colloids consists of two parts : - [1] Dispersed phase and [2] Dispersion medium.
[1] Dispersed phase –The component present in small amount, which act as the solute.
[2] Dispersion medium - The component present in excess amount, which act as the solvent.
On the basis of physical state, colloids are classified as:

Sl.No. Dispersed Dispersion Types of Examples


phase medium colloids
1 Solid Solid Solid sol Some coloured glasses and gem stones
2 Solid Liquid Sol Paints, cell fluids
3 Solid Gas Aerosol Smoke, dust
4 Liquid Solid Gel Cheese, butter, jellies
5 Liquid Liquid Emulsion Milk, hair cream
6 Liquid Gas Aerosol Fog, mist, cloud, insecticide sprays
7 Gas Solid Solid sol Pumice stone, foam rubber
8 Gas Liquid Foam Froth, whipped cream, soap lather

On the basis of dispersion medium, Colloidal solutions are classified as:

Hydrosol/ Aquosol Alcosol Benzosol


Dispersion medium is water Dispersion medium is alcohol Dispersion medium is benzene

jksahu74@gmail.com 125 | P a g e
Types of colloidal solution on the basis of particle size
Multimolecular colloids Macromolecular Associate Colloids
colloids
On dissolution, a large number of They consists of These are the substances which

atoms or smaller molecules of a large molecules when dissolved in a medium behave

substance (having diameter less than generally as normal electrolytes at low

1 nm) aggregate together to form polymer. They concentration but behave, as colloidal

species having size in the colloidal have lyophobic particles at higher concentration due

range . In these colloids, the particles character. to the formation of aggregate

are held together by Vander Waal’s e.g. Starch, particles. The aggregate particles

forces. They have usually lyophilic cellulose, protein, thus formed are called micelles. They

character. enzymes. have both lyophilic and lyophobic

e.g- Gold sol , sulphur sol. character. e.g Soaps and detergents.

Mechanism of micelle formation

Polar part ( hydrophilic ) soluble in water

Oil
droplet Water Non-Polar part ( hydrophobic ) soluble in oil

Soaps and detergents when dissolved in water form micelles.

Soap is sodium or potassium salt of a higher fatty acid and may be represented as RCOO–Na+

(e.g., sodium stearate CH3(CH2)16COO–Na+, which is a major component of many bar soaps).
When dissolved in water, it dissociates into RCOO– and Na+ ions.

In case of detergents, e.g., sodium laurylsulphate, CH3(CH2)11SO4–Na+, the polar group is – SO4–
along with the long hydrocarbon chain.

Kraft temperature ( Tk ) :- The temperature above which formation of micelles take place is
called Kraft temperature.

Critical micelle concentration ( CMC ) :- The concentration above which formation of micelle
takes place is called critical micelle concentration.

For soaps, the CMC is 10 -4 to 10 -3 mol L-1

Micelles may contain as many as 100 molecules or more.

jksahu74@gmail.com 126 | P a g e
Classification Based on Nature of Interaction between Dispersed Phase and Dispersion Medium

Sl. Property Lyophilic sols Lyophobic sols


No.
1 Mode of Prepared easily by direct mixing Prepared only by specific method.
preparation
2 Nature Reversible Irreversible
3 Visibility The particles are not visible under The particles are easily visible under
ultra microscope ultra microscope

4 Stability Self stabilized Unstable and require some amount of


stabilizer.
5 Action of Large amount of electrolytes are Small amount of electrolytes are
electrolyte required for coagulation required for coagulation

6 Viscosity Much higher than the dispersion About same as that of dispersion
medium medium.
7 Tyndall Do not exhibit Exhibit
effect
8 Surface Lower than that of dispersion Nearly same as that of dispersion
tension medium medium.

Preparation of Colloids

(1) Chemical methods

(a) Double Decomposition=> Preparation of colloids from inorganic salt.


Double decomposition
As2O 3 + 3 H 2S AS2S3 (sol) + 3 H 2O

(b) Oxidation=> Preparation of colloids of non-metals.


Oxidation
SO2 + 2 H 2S 3 S (sol) + 2 H 2O

(c) Reduction => Metal sols are prepared by this method.


Reduction
2 AuCl3
+ 3 HCHO + 3 H2 O 2 Au (sol)
+ 3 HCOOH + 6 HCl

(d) Preparation of hydroxide sol.


Hrdrolysis
FeCl 3
+ 3 H 2O Fe(OH) 3 (sol) + 3 HCl

(2) Electrical disintegration or Bredig’s Arc method

In this method, electric arc is struck between electrodes of the metal immersed in the dispersion
medium. The intense heat produced vaporises the metal, which then condenses to form
particles of colloidal size.

jksahu74@gmail.com 127 | P a g e
(3) Peptization

Peptization may be defined as the process of converting a precipitate into colloidal sol by
shaking it with dispersion medium in the presence of a small amount of electrolyte.
The electrolyte used for this purpose is called peptizing agent.
During peptization, the precipitate adsorbs one of the ions of the electrolyte on its surface.
This causes the development of positive or negative charge on precipitates, which ultimately
break up into smaller particles of the size of a colloid.

++ +
+
+ Peptizing agent +
++ ++
+

Fe ( OH ) 3
( ppt ) Colloidal particles

Purification of Colloidal Solutions

(i) Dialysis: It is a process of removing a dissolved substance from a colloidal solution by means of
diffusion through a suitable membrane. The apparatus used for this purpose is called dialyser.

(ii) Ultra filtration: This is done by decreasing the size of porous of filter paper by using colloidion
solution to stop the flow of colloidal particles.

The usual colloidion is a 4% solution of nitrocellulose in a mixture of alcohol and ether.

Properties of Colloidal Solutions

[1] Colligative properties: Colloidal particles being bigger aggregates, the number of particles in a
colloidal solution is comparatively small as compared to a true solution. Hence, the values
of colligative properties (osmotic pressure, lowering in vapour pressure,depression in freezing
point and elevation in boiling point) are of small order as compared to values shown by true
solutions at same concentrations.

[2] Tyndall effect: When a strong beam of light pass through a colloidal solution, scattering of light
takes place and the colloidal particles are visible as dust particles is called Tyndall effect. If the
beam of light before passing through the colloidal solution is passed through a convex lens a
bright cone of light will observed, known as Tyndall cone.

Tyndall effect is observed only when the following two conditions are satisfied:

(i) The diameter of the dispersed particles is not much smaller than the wavelength of the light
used; and

(ii) The refractive indices of the dispersed phase and the dispersion medium differ greatly in
magnitude.

[3] Colour: The colour of colloidal solution depends on the wavelength of light scattered by the
dispersed particles. The wavelength of light further depends on the size and nature of the
particles. The colour of colloidal solution also changes with the manner in which the observer
receives the light.
jksahu74@gmail.com 128 | P a g e
For example, (i) A mixture of milk and water appears blue when viewed by the reflected light
and red when viewed by the transmitted light.
(ii) Finest gold sol is red in colour; as the size of particles increases; it appears
purple, then blue and finally golden.
[4] Brownian movement: When colloidal solutions are viewed under a powerful ultra microscope,
the colloidal particles appear to be in a state of continuous zigzag motion all over the field of
view.This motion is known as Brownian movement. Smaller the size and lesser the viscosity,
faster is the motion. The Brownian movement has a stirring effect which does not permit the
particles to settle and thus, is responsible for the stability of sols.
[5] Electrophoresis: The movement of colloidal particles under an applied electric potential is
called electrophoresis. With help of electrophoresis we can confirm about the charge of colloidal
particles. Osmosis by electrolysis is called electro osmosis.

[6] Charge on colloidal particles: The sol particles acquire positive or negative charge by
preferential adsorption of +ve or –ve ions. When two or more ions are present in the dispersion
medium, preferential adsorption of the ion common to the colloidal particle usually takes place.
[a] When silver nitrate solution is added to potassium iodide solution, the precipitated silver
iodide adsorbs iodide ions from the dispersion medium and negatively charged colloidal
solution results.

AgNO3 + KI AgI / I
Dispersion medium

However, when KI solution is added to AgNO3 solution, positively charged sol results due to
adsorption of Ag+ ions from dispersion medium.
+
KI + AgNO3 AgI / Ag
Dispersion medium

[b] If FeCl3 is added to excess of hot water, a positively charged sol of hydrated ferric oxide is
formed due to adsorption of Fe3+ ions.

FeCl3 + x H 2O Fe2O3 . x H 2O / Fe 3+

when ferric chloride is added to NaOH a negatively charged sol is obtained with adsorption of
OH- ions.
FeCl3 + NaOH Fe2O3 . x H 2O / OH

Positively charged sol Negatively charged sol


Hydrated metallic oxides, Metals, e.g., copper, silver, gold sols.
e.g., Al2O3.xH2O, CrO3.xH2O and Fe2O3.xH2O
Basic dye stuffs, e.g., methylene blue sol. Metallic sulphides, e.g., As2S3, Sb2S3, CdS sols.
Haemoglobin (blood) Acid dye stuffs, e.g., eosin, congo red sols.
Oxides, e.g., TiO2 sol. Sols of starch, gum, gelatin, clay, charcoal, etc.

jksahu74@gmail.com 129 | P a g e
Helmholtz electrical double layer
+
K Diffused layer I
+
AgI / I Fixed layer AgI / Ag

The combination of the two layers of opposite charges around the colloidal particle is called
Helmholtz electrical double layer
Zeta potential: The potential difference between the fixed layer and the diffused layer of opposite
charges is called the electro kinetic potential or zeta potential.
Stability of colloidal solution
The presence of equal and similar charges on colloidal particles is largely responsible in providing
stability to the colloidal solution, because the repulsive forces between charged particles having
same charge prevent them from coalescing or aggregating when they come closer to one another.
Coagulation
The process of settling of colloidal particles is called coagulation or precipitation of the sol.
The coagulation of the lyophobic sols can be carried out in the following ways.
By electrophoresis
By mixing two oppositely charged sols
By boiling
By persistent dialysis
By addition of electrolytes
Flocculating ion
The electrolyte which is added for coagulation is known as coagulating agent and the ions of the
electrolyte which causes the coagulation is known as Flocculating ion .
Coagulating value or flocculating value
The minimum concentration of an electrolyte in millimoles per litre required to cause precipitation
of a sol in two hours is called coagulating value. The smaller the quantity needed, the higher will
be the coagulating power of an ion.
Hardy-Schulze rule
The greater the valence of the flocculating ion added, the greater is its power to cause
precipitation.
In the coagulation of a negative sol, the flocculating power is in the order: Al3+ > Ba2+ > Na+
The coagulation of a positive sol, the flocculating power is in the order:
[Fe(CN)6]4– > PO4 3– > SO4 2– > Cl–
Protective colloids
Lyophilic colloids have a unique property of protecting lyophobic colloids. When a lyophilic sol is
added to the lyophobic sol, the lyophilic particles form a layer around lyophobic particles and thus
protect the latter from electrolytes. Lyophilic colloids used for this purpose are called protective
colloids.
Emulsions
These are liquid-liquid colloidal systems, i.e., the dispersion of finely divided droplets in another
liquid. There are two types of emulsions.

[1] Oil dispersed in water (O/W type) - Milk and vanishing cream.

In milk, liquid fat is dispersed in water.

[2] Water dispersed in oil (W/O type).- butter and cream.

jksahu74@gmail.com 130 | P a g e
For stabilisation of an emulsion, emulsifying agent is usually added. The emulsifying agent forms
an interfacial film between suspended particles and the medium.

Examples of colloids:

i) Blue colour of the sky

ii) Fog, mist and rain:

iii) Blood: It is a colloidal solution of an albuminoid substance.


The styptic action of alum and ferric chloride solution is due to coagulation of
blood forming a clot which stops further bleeding.

iv) Formation of delta:

Applications of colloids

[1] Purification of drinking water: Alum is added to water to coagulate the suspended impurities

[2] Cleansing action of soaps and detergents:

[3] Tanning: Animal hides are colloidal in nature. When a hide, which has positively charged
particles, is soaked in tannin, which contains negatively charged colloidal particles,
mutual coagulation takes place. This results in the hardening of leather. This process
is termed as tanning. Chromium salts are also used in place of tannin

[4] Medicines: Most of the medicines are colloidal in nature.

For example, Argyrol is a silver sol used as an eye lotion.

Colloidal antimony is used in curing kalaazar.

Colloidal gold is used for intramuscular injection.

Milk of magnesia, an emulsion, is used for stomach disorders.

Colloidal medicines are more effective as they have large surface area and are therefore easily
assimilated

[5] Electrical precipitation of smoke:

Smoke is a colloidal solution of solid particles


such as carbon, arsenic compounds, dust, etc., High voltage electrode
( 30000 volts or more ) Gases free from carbon
in air. The smoke, before it comes out from the
particles
chimney, is led through a chamber containing
plates having a charge opposite to that carried
by smoke particles. The particles on coming in
contact with these plates lose their charge and
get precipitated.The particles thus settle down Precipitated ash
on the floor of the chamber. The precipitator is Smoke
called Cottrell precipitator .
[ Cottrell smoke precipitator ]
jksahu74@gmail.com 131 | P a g e
GENERAL PRINCIPLES AND PROCESSES OF
ISOLATION OF ELEMENTS
The mode of occurrence of a metal is largely dependent on its chemical nature. Those metals,
which are relatively inert, occur in free or native state (i.e. in uncombined state) but most of the
metals are reactive and hence are found in combined state.

Mineral: It is the combined state in which a metal occurs naturally in the crust of the earth.
The mineral has a definite composition.

Ore: The minerals from which the metals can be conveniently and economically extracted are
known as ores.
Thus, all ores are minerals, but all minerals are not ores.

e.g. Cinnabar (HgS) is an ore as well as mineral of mercury .


But iron pyrites (FeS2) is a mineral of iron but not an ore.

Gangue / Matrix: Earthly or undesired materials present in mineral are called gangue

Ore mineral. The valuable mineral contained in an ore . Ore = Ore mineral + Gangue

(a) Native Ores :


These ores contain metal in free state. For example : Silver, gold, platinum, mercury, copper etc.
Sometime lumps of pure metals are found known as nuggets.

(b) Combined Ores:


These ores contain metal in combination with oxygen or sulphur or halides etc.
The most abundant elements in the earth’s crust: Oxygen > Silicon > Aluminum > Iron > Calcium
Reactivity decreases

Li K Ca Na Mg Al Mn Zn Fe Ni Pb Cu Ag Hg Au
Highly reactive elements Moderately reactive elements Least reactive elements
Ore Ore Ore
Electrometallurgy Pyrometallurgy Thermometallurgy

Crude metal Crude metal Crude metal

Electrometallurgy: [For highly reactive metals]


The strongest possible reducing agent is an electron; any ionic material can be reduced to their
respective metal by electrolysis. Electrolytic reduction can be performed by applying a potential
difference, E which is related to ∆G as follows: E applied ≥ f
b c ∆ff
f
f
Gf
f
ff
f
f
nF
For a reaction having ∆G value 100 kj /mol,n= 1 and F = 96487 C Then E(applied) is nearly 1.03 V
But for this type of reduction, in terms of pyrometallurgy, require high temperature, at which the
ore become volatile.

jksahu74@gmail.com 132 | P a g e
Pyrometallurgy: ( Pyro means fire )- [For moderately reactive metals]
The process of converting metal oxide in to metallic form upon strong heating with a suitable
reducing agent, is known as pyrometallurgy.
Moderately reactive metals can be isolated from their ores by the process of pyrometallurgy.
Thermite reaction: Reduction of metal oxide with aluminum.

Cr 2O 3 + 2 Al Al 2O 3 + 2 Cr

Thermo metallurgy :( For least reactive metal )


The metals can be isolated by simply heating the oxide of the metals.
2 HgO 2 Hg + O2

1 O
Ag2O 2 Ag + 2
2

Hydrometallurgy
The isolation of a metal present in the soluble complex by more reactive metal is called
hydrometallurgy. Zinc is more reactive than copper, So Zn can displace Cu from solution of Cu2+
2+ 2+
Zn (s) + Cu (aq) Zn (aq) + Cu (s)

To displace Zn from solution of Zn2+ ions, we need more reactive elements such as Al, Mg, Ca
and K. But all these metals react with water forming their corresponding ions with the evolution of
H2 gas. Thus Al, Mg, etc. cannot be used to displace Zn from solution of Zn2+ ions. Thus, copper
can be extracted by hydrometallurgy but not zinc.
SMELTING
The process of extracting metal from its fused (molten) state is called smelting. The roasted or
calcinated ore containing metal oxide is missed with a reducing agent and heated to a high
temperature. In this case, a less electropositive metal ore of Pb, Zn, Fe etc. are treated with
powerful reducing agent such as C, H2, CO etc.
Flux and Slag
Flux is a substance which combines with the non fusible impurities ( gangue ) of the ore to form
easily fusible mass called slag.
Gangue + Flux Slag
Fluxes are two types: (1) Acidic Flux ( SiO2 ) (2) Basic flux ( CaO )
In the extraction of copper from copper pyrites, FeO is present as basic impurity and to remove it,
acidic flux like silica is used.

FeO + SiO 2 FeSiO 3


Basic gangue Acidic flux Ferrous silicate (Slag ) )
In the extraction of iron from haematite ore, calcium oxide is used as a basic flux to remove the
acidic gangue ( SiO2)
SiO 2 + CaO CaSiO3
Acidic gangue Basic flux Calcium silicate (Slag ) )
The slag formed has lower melting point than that of the metal. It is lighter than the molten metal
and does not affect the lining of the furnace.
jksahu74@gmail.com 133 | P a g e
Iron Copper Tin Lead Magnesium Aluminium Zinc Silver
Iron pyrite; Cuprite; Cu2O Cassiterite or Galena; PbS Carnalite; Bauxite; AI2O3.2H2O Zinc blende; Horn silver
FeS2 Tin stonel KCI.MgCI2.6H2O ZnS (Chlorapatite);
SnO2 AgCI
Siderite; Copper pyrites; Stannite; Cerussite; Dolomite; Corundum; AI2O3 Zincite; ZnO Lunar caustic;
FeCO3 CuFeS2 Cu2S.FeS.SnS2 PbCO3 MgCO3.CaCO3 AgNO3

Red Copper glance; Anglessite; Magnesite; Feldspar; KAISi3O6 Smithsonite, Silver glance or
haematite; Cu2S PbSO4 MgCO3 ZnCO3 argentite; Ag2S
Fe2O3
Magnetite; Malachite; White lead; Epsomite; Clay silicate; Hemimorphite Ruby silver or
Fe3O4 CuCO3.Cu(OH)2 2Pb(OH)2.PbCO3 MgSO4.7H2O AI2O3.2SiO2.2H2O or Calamine, pyrogyrite;
Zn2SiO4.H2O 3Ag2S.Sb2S3
Limonite or Azurite; Kiesserite; Cryolite;
brown 2CuCO3.Cu(OH)2 MgSO4.H2O 3NaF.AIF3.(Na3AIF6)
haematite;
2Fe2O3.3H2O
Asbestos;
Alum;
CaMg(SiO3)4
K2O.3AI2(SO4)3.24H2O
Diaspore; AI2O3.H2O
Mica;
K2O.3AI2O3.6SiO2.2H2O
Metallurgy: The entire scientific and technological process used for isolation of the metal from its ores is known as metallurgy.
Concentrated Ore

Concentration Conversion
Metal oxide

Crude metal
Reduction Refining

Pure metal
Powdered Ore

of ore to oxide
Crushing -Reduction with ' C ' - Liquation
Ore

-Hydraulic washing
and -Magnetic separation -Calcination -Reduction with ' CO ' -Distillation
grinding -Froth Floatation -Roasting -Reduction with ' Al ' - Electrolysis
-Leaching -Electolitic Reduction - Zone refining
- Vapour phase refining
- Chromatographic method

jksahu74@gmail.com 134 | P a g e
Crushing and Grinding of Ore:
The ore lumps are broken into small pieces with help of jaw crusher and the small pieces of the
ore are converted into powder form using either ball mill or stamp mill. This process is known as
pulverization.
Hydraulic washing / Levigation : This method is used when the ore particles are heavier than
the gangue particles.e.g. Oxide ores
Magnetic separation :This is based on difference in magnetic properties of the ore component.
Froth floatation method : It is based on the difference in the wetting qualities of the gangue and
sulphide ore particles with water and oil. Whereas the ore particles are
wetted by oil, the gangue particles are wetted by water.
Collectors enhance non-wet ability of the mineral particles. e.g. Pine oil, fatty acid, xanthates, etc.
Froth stabilizers stabilizes the forth e.g. Cresol, aniline, etc.
Depressants are reagents used to suppress or depress the formation of forth.
For example, ZnS and PbS can be separated from each other in the forth floatation process by the
addition of small amounts of NaCN which act as depresent. NaCN selectively prevents ZnS from
coming to the froth by forming soluble complex. But allows PbS to come with the froth.

4 NaCN + ZnS Na2 [ Zn ( CN )4 ] + Na2S


Depressant Ore
Soluble complex

Leaching of alumina from Bauxite


This process is followed if ore is soluble in suitable solvent. Bauxite contains SiO2, TiO2 and iron
oxide as impurities. When bauxite ore is treated with Concn. NaOH, alumina dissolves as sodium
meta aluminate and silica as sodium silicate leaving behind the impurities i.e. Iron oxide.

Al2O3 + 2 NaOH + 3 H2O 2 Na [ Al (OH )4]


Alumia Sodium meta aluminate

SiO2 + 2 NaOH Na2SiO3 + H2O


Silica Sodium silicate

The solution is filtred and neutralised by passing CO2 gas, when hydrated alumina gets
precipitated leaving sodium silicate in the solution.

2 Na [Al (OH)4 ] + CO2 Al2O3.xH2O + 2 NaHCO3


Hydrated alumina

Hydrated alumina on heating give pure alumina.


Al2O3.xH2O Al2O3 + x H2 O

Leaching of Ag and Au from their ore.

Silver and gold ores are treated with dilute solution of NaCN or KCN in presence of air to form
soluble complex.

jksahu74@gmail.com Page | 135


4 Au + 8 NaCN + 2 H2O + O2 4 Na [ Au (CN)2 ] + 4 NaOH
4 Ag + 8 NaCN + 2 H2O + O2 4 Na [ Ag (CN)2 ] + 4 NaOH

After filtration the solution treated with zinc, which displace Ag and Au as precipitate.

2 Na [ Au (CN)2 ] + Zn Na 2[ Zn (CN)4 ] + 2 Au
2 Na [ Ag (CN)2 ] + Zn Na 2[ Zn (CN)4 ] + 2 Ag
( soluble complex ) ( Precipitate )

Cyanide process of extraction of gold and silver involves both oxidation and reduction processes.
Formation of cyanide complex involves oxidation of gold and silver. Recovery of the metal by the
addition of zinc involves reduction of gold and silver ions from their complex.

Calcination Roasting

The process of converting an ore into its The process of converting an ore into its oxide by
oxide by heating below its melting point in heating below its melting point in presence of air.
absence of air.
2 ZnS + 3O2 2 ZnO + 2SO2
ZnCO3 ZnO + CO2

During calcination the following changes During roasting the following changes occur:
occur:
a) Moisture and water from hydrated oxides a) Moisture is driven out.
or hydroxides are removed. Organic
matter, if any, is also expelled. b) Non-metallic impurities such as sulphur, arsenic
and phosphorous are removed as volatile oxides.
b) Metal carbonates decomposes to form c) Sulphide ores are converted into metallic
their corresponding oxides. oxides.

c) The mass becomes porous which can be d) The roasted mass becomes porous which can
conveniently reduced. be conveniently reduced.

Reduction of the oxide to the metal

The roasted or calcinated ore contains the metal in the form of its oxide. To obtain the metal in its
free state, metal oxide has to be reduced. Generally Carbon, Carbon monoxide, Aluminium or
electrolytic reduction process are used.
Refining

The process of removing the impurities from the crude metal is refining or purification.

Liquation : This method is used when the M.P of the metals are lower than those of the
impurities. e.g. low melting metal – Sn

Distillation :This method is employed for purification of volatile metals ( Zn, Hg, Cd ) by heating
the ore followed by condensation.

jksahu74@gmail.com Page | 136


Electrolytic refining ( Zn, Cu )

Anode : Impure metal ( Cu ) Cathode: Pure metal ( Cu )

Electrolyte : Soluble salt of same metal ( acidified CuSO4 solution )


The more basic metal remains in the solution and the less basic ones go to the anode mud.
2+ - 2+ -
Anode: Cu Cu + 2e Cathode Cu + 2e Cu

Antimony, Selenium, Tellurium, Silver, Gold and Platinum, which are less basic in nature present
as impurities in blister copper, collected as anode mud.

Zone refining ( Ge, Si, B, Ga, In )

This method is based upon the principle that the impurities are more soluble in the molten state
( melt ) than in the solid state of the metal. i.e. When a molten solution of the impure metal is
allowed to cool, the pure metal crystalises out while the impurities remain in the melt.

Vapour phase refining ( Ni, Zr, Ti )

In this method, the impure metal is converted into a volatile compound by suitable method leaving
behind the impurities. The volatile compound formed being unstable decomposes at an elevated
temperature to give pure metal.

330 - 350 K 450 - 470 K


Mond process Ni +
4 CO
Impure Nickel
Ni(CO)4 Ni + 4 CO
Volatile Pure Nickel
( Nickel tetra carbonyl )

Van Arkel Method ' W ' Filament


Zr
Impure Zirconium
2 I2 + ZrI4
1800 K
Zr +
2 I2
Volatile Pure Zirconium
Ti
Impure Titanium
+ 2 I2 Ti I4 Ti + 2 I2
Volatile
Pure Titanium

Chromatographic Method

(i) This method is based upon the principle that different components of a mixture are differently
adsorbed on an adsorbent
(ii) The zone thus formed due to adsorption is called chromatogram.
(iii) The process of separation of the different components of the mixture from the adsorbent and
their recovery with the help of a suitable solvent ( eluant ), is called elution.
(iv) The sample is dissolved in a mobile phase and then forced through the stationary phase, to
form chromatogram
(v) The stationary phase is selected in such a way that the impurities are more strongly adsorbed
in the stationary phase than element to be purified.

jksahu74@gmail.com Page | 137


Ellingham diagram

Ellingham diagrams are the graphs which represent the variation of standard free energy change (
0
∆G ) with temperature for the formation of oxides of various elements.

∆G = T ∆S
∆G = Ve ( Less )

∆G
∆G = ∆S T + Phase change

∆G = Ve ( More )

T
0
A reaction that proceeds with decrease of entropy ( ∆S = - ve ) must have + ve slope.
0
The slope becomes - ve for those reactions which proceed with increase of entropy ( ∆S = +ve )
0 0 0
∆H and ∆S of a reaction remain nearly constant, so the value of ∆G changes with
temperature.
Characteristics of Ellingham diagram
0
(i) The curves always slope upwards because ∆G value for formation of metal oxide increases
with the rise in temperature.
(ii) The sudden change in the slope indicates the phase change. This happens at the melting point
and boiling point.
0
(iii) For the formation of CO the ∆G values gradually becomes more negative with rise in
temperature. i.e. the graph slopes downward.
0 0
(iv) Any metal oxide with lower ∆G is more stable than the metal oxide with higher ∆G

(v) The element involved in the formation of oxides placed lower in the diagram can reduce the
oxide of the element placed higher in the diagram.

(vi) Ag2O and HgO, placed at the top of Ellingham diagram, are quite unstable and they
decompose of their own and no reducing agents are required to carry their reduction.

2 Ag2O 4 Ag + O2

2 HgO 2 Hg + O2

Limitation Ellingham diagram give no idea about the kinetics of reaction.

jksahu74@gmail.com Page | 138


Reducing nature of carbon ( coke )
[1] C (S)
+ O2 ( g ) CO2 ( g ) ; ∆S = 0 O2
2C
n R
=1 n P
=1
So ∆G remain same. O2
O+
2C C + O2 CO2

------------------
.

[2] 2 C + O2 ( g ) 2 CO ( g ) ; ∆S 0 2C
(S) .
∆G +
n =1 O
R 2
n P
=2
2C
So ∆G becomes increasingly negative. 983 K O

T
.

[3] 2 CO (g)
+ O2 ( g ) 2 CO2 ( g ) ; ∆S 0
.

n R
=3 n P
=2 So ∆G becomes positive.

So it is concluded that carbon ( coke ) act as reducing agent by changing to carbon monoxide
above 983 k
Extraction of Zinc
Ore Zinc blende / Sphalerite - ZnS; Calamine - ZnCO3 ; Zincite - ZnO
1673 K
ZnO + Coke
C Zn + CO

The metal is purified by fractional distillation


0
Zinc boils at 1180 K, so the curve ∆G ( Zn, ZnO ) increases rapidly at this temperature.
The metal is distilled off and collected by rapid chilling.
.

O
Above 1270 K ∆G ( Zn, ZnO ) ∆G ( C, CO ) Zn
.
2

O2
--------------------
Thus zinc oxide is reduced by coke above 1270 K. + 2C
Zn + O
2
∆G 2
------------

Above 1270 K. 2C
ZnO + C O
1270 K

+
1180 K

Zn CO

T
Extraction of Copper

Ore = Roasting
Copper Pyrites : CuFeS2
2 CuFeS 2 + O2 Cu2S + 2 FeS + SO2

Copper glance : Cu2S


2 FeS + 3 O2 2 FeO + 2 SO2
FeO + SiO 2 FeSiO 3
Malachite : CuCO3.Cu(OH)2 Impurity Flux Slag
Partial oxidation
Cuprite : Cu2O 2 Cu2S
+ 3 O2 2 Cu2O + 2 SO 2
2 Cu2O
+ Cu2S 6 Cu SO 2
The molten copper is poured

jksahu74@gmail.com Page | 139


off and is allowed to cool. During cooling dissolved sulphur dioxide escapes out from the molten
metal causing blister on the surface of the metal. The copper thus obtained is known as blister
copper. It is 98% pure. It is purified further by poling process and by electrolytic method.
Flow chart for the extraction of Copper
Copper Pyrites ( CuFeS )

Crushed and seived

Concentration by froth floatation process


Powdered ore suspended in water + Pine oil + air

Agitation

Sulphide ore collected on the forth

Roasting in reverbaratory furnace in the presence of air

S + O2 SO 2

4 As + 3 O2 3 As2O 3

4P + 5 O2 P 4O 10

2 CuFeS 2 Cu2S + 2 FeS + 2 SO 2

Smelting in blast furnace in presence of air


Roasted ore + Coke + Silica
2 FeS + 3 O2 2 FeO + 2 SO 2

FeO + SiO 2 FeSiO 3

Copper 'mate' ( Cu2S + FeS + Silica )

Bessmerisation in Bessemer's converter in presence of air

2 FeS + 3 O2 2 FeO + 2 SO 2

FeO + SiO 2 FeSiO 3 ( Slag )


Auto reduction
2 Cu 2 S + Cu2 O 6 Cu + SO 2

Blister copper 98% pure

jksahu74@gmail.com Page | 140


Ellingham diagram for the formation of oxides of copper and carbon
Thus , cuprous oxide can be easily reduced to
Metallic copper by heating with coke.
Cu2O + C
coke
2 Cu + CO

Copper is extracted by hydrometallurgy from low


grade ores. It is leached out using acid or bacteria
in presence of air. The solution containing Cu2+ is
treated with scrap iron or H2 to get Cu metal.

O2 2 CuSO4 H2 O
2 Cu
+ 2 H2SO 4 + +
2+ 2+
Cu + Fe Fe + Cu

2+ +
Cu + H2 Cu + 2H

Extraction of iron in the blast furnace

Ore = Haematite : Fe2O3 Magnetite : Fe3O4

Temperature = 2170 K Additive = Coke (Reducing agent and fuel)


and Lime stone (Basic Flux )

jksahu74@gmail.com Page | 141


Role of coke
(i) Zone of combustion (2173 K)
C + O2 CO2
Fuel
= - 393.3 Kj/mol

(ii) Zone of heat absorption ( 1673-1473 K)

CO2 + C 2 CO
Reducing agent = + 163.2 Kj / mol

Role of lime stone


(iii) Zone of slag formation ( 1473 - 1123 K)

CaCO3 1123 K
CaO + CO2 ∆ Η = + 179.9 kJ/mol
CaO
Flux
+ SiO 2
Impurity
CaSiO3
Calcium silicate ( Slag )

Role of carbon monoxide


(iv) Zone of reduction ( 973 - 500 K)
Oxides of iron from the ores are reduced by carbon monoxide

573 - 673 K
3 Fe2O 3 + CO 2 Fe3O 4 + CO2

Fe3O 4 + 4 CO 773 - 873 K 3 Fe + 4 CO 2

773 - 873 K
Fe2O3
+ CO 2 FeO + CO2

FeO + CO Fe + CO 2
Pig iron

Unreduced oxides of iron are directly reduced by coke in the region above 1973 K.

(v) Zone of fusion ( 1423- 1673K)

In this zone , the iron and the solid slag both melt and collect in the hearth of the furnace.

The iron obtained from Blast furnace contain about 4% carbon is known as pig iron.

jksahu74@gmail.com Page | 142


Flow chart for the extraction of iron
Iron Ore

Concentration

Gravity separation and


Magnetic separation

Calcination

To drive away moisture


expell volatile impurities

Calcinated ore + Coke + lime stone


subjected to smelting in blast furnace. Iron
oxides reduced by coke above 1073 K and
by carebon monoxide below 1073 K. Silica
present as acidic impurity is removed as a
slag i.e., ( calcium silicate)

Fe formed by reducing action of CO is spongy iron.

Spong iron + C, Si , Mn ( impurities )

Pig iron 4% carbon

O2 is passed through molten


Pig iron heated with pig iron + lime
scrap iron + coke then alloyed with Cr and Mn

Cast iron Steel


3% of Carbon
hard, brittle

Heated in reverberatory
furnace lined with haematite

Wrought iron
Purest form of iron ( 0.2 to 0.5 % carbon)
Soft , Malleable and Ductile
jksahu74@gmail.com Page | 143
Ellingham diagram for the formation of oxides of iron and carbon
.

[1] Below 1073 K, ∆G ( Fe, Fe2O3 ) ∆G ( CO, CO2 )


.

CO
2
O
So Haematite is reduce by CO 2 2 Fe 2 3
O
4 Fe + 2

-----------------
Fe2O 3 + 3 CO Below 1073 K 2 Fe + 3 CO 2 +
O2 2C
+
3

∆G CO O
2

1073 K
2
.

[2] Above 1073 K , ∆G ( Fe, Fe2O3 ) 2C


.
∆G ( C, CO ) O

So Haematite is reduce by coke. T


Fe2O 3 + 3C Above 1073 K
2 Fe + 3 CO

Extraction of Aluminium ( Hall - Heroult Process )

Ore Bauxite - Al2O3 . x H2O Cryolite – Na3AlF6

Cathode -Steel; Anode – Graphite

Pure Al2O3 mixed with Cryolite ( Na3AlF6 ) or CaF2:

(i) To make alumina a good conductor of electricity.

(ii)To lowers the melting point of the mix.

Al 2O 3 2 Al 3+ + 3 O 2-
3+ -
Cathode - Al + 3e Al

2- -
Anode - C + O CO + 2e

2- -
C + 2O CO2 + 4e

The role of graphite as anode is to prevent the liberation of O2 which may otherwise oxidize some
of the liberated Aluminium back to Al2O3.

Explain overall reaction for extraction of aluminium by Hall – Heroults process is the
carbon reduction.

Al2O3 2 Al3+ + 3 O2- ---------------[1]


Anode -(Graphite) O2- + C CO + 2e-
CO + O 2- CO2 + 2e-

C + 2 O 2- CO2 + 4e-

Cathode - [ Steel ] Al 3+ + 3e- Al

jksahu74@gmail.com Page | 144


[Anode reaction X 3 ] + [Cathode reaction X 4 ] = Cell reaction

C + 2 O 2- CO2 + 4e- ]X 3
Al 3+ + 3e- Al ]X 4
4 Al 3+ + 6 O 2- + 3C 4 Al + 3 CO2 -------------------[2]
Adding eqn(1) with (2) by multiplying suitable integer.

Al2O3 2 Al3+ + 3 O2- ] X 2


4 Al 3+ + 6 O 2- + 3C 4 Al + 3 CO2

2 Al2O3 + 3C 4 Al + 3 CO2

Explain pyrometallurgical carbon reduction process is not applicable for the extraction of
aluminium from bauxite ore.

4 2
Al (S) + O2 (g) 3
Al2 O3 (S) , ∆G1 ( ∆S = -ve since gas solid)
3

2 C (s) + O2 (g) 2 CO (g), ∆G2 ( ∆S = + ve since 1 mol 2 mol gas)

For reduction of bauxite by carbon:

2 C (s) + O2 (g) 2 CO (g), ∆G2


2 4
3
Al 2O 3 3 Al + O2 ∆G1

2 Al O 4
3
2 3 + 2C
3
Al + 2 CO ∆G = ∆G 2 ∆G 1

.
∆G1
To get ∆G negative , ∆G1 ∆G2
.
.
.

∆G2 ∆G1 ∆G1 ∆G2


--------------

∆G
.

∆G2
2000 C
T

i.e the reduction of bauxite by carbon will feasible above 2000 C . But at this
temperature bauxite becomes volatile. Therefore bauxite can not reduce by carbon.

jksahu74@gmail.com Page | 145


15 - - 18

Distance of Bond pair


H

Decreases
Decreases
Increases

increases

Increases
Decreases
Electron density
H

from central atom

B.P - B.P Repulsion

Bond strength
Bond length
Atomic size

H
H
H

Anomalous properties of second period elements


These elements exhibit anomalous behavior and differs from other members of their group on
account of the following reasons:
1) Small atomic size 2 ) Large ( charge/radius) ratio 3) High electro negativity
4) High ionization enthalpy 5) Absence of d- orbital in its valence shell.
A) The first element in p-block element has four valence orbitals i.e. one 2s and three 2p, Hence
maximum covalency of the first element in limited to four. The other elements of the p-block
elements have vacant d-orbitals in their valence shell, Hence these show maximum covalence
greater than four.
B) The first member of p-block elements displays greater ability to from pπ- pπ bond (s) with
itself, (e.g., C = C, C ≡ C , N = N, N ≡ N) and with the other elements of second period
(e.g., C = O, C ≡ N, N = O) compared to the subsequent members of the group. This is because
p-orbitals of the heavier members are so large and diffuse that they cannot have effective
sideways overlapping. Heavier members can form pπ- dπ bonds with oxygen. Nitrogen rarely
forms pπ- dπ bonds with heavier elements as in case of trisilylamine (SiH3)3N.
C) Due to small size and high electronegativity and presence of lone pair(s) of electrons, elements
N, O, F when bonded to hydrogen atom, forms intermolecular hydrogen bonds which are
stronger than other intermolecular forces. This results in exceptionally high m.p. and b.p. of the
compounds having N–H / O–H / F–H bonds.

Isostructural species
These have same number of bond pairs and lone pairs if present around the central atom in a
molecule/ion. Thus, they have the same geometry/shape/structure and the same hybridisation
scheme. For example ICl4– / XeF4, BrO3– / XeO3, BH4– / NH4+ are the pairs of isostructural
species.

jksahu74@gmail.com Page | 146


Bond length
Bond length get affected if resonance involved. Average bond length will consider by considering
! "! '
bond order. Bond order = Bond length ∝
#$ % $ &$ (
Bond angle
Explain with help of VSEPR theory.
In regular structures (where no lone pairs are present in the valence shell of the central atom in a
molecule/ion), the bond angle does not depend upon the size / electronegativity of the central or
terminal atoms. In presence of lone pair(s) on the central atom, the geometry is distorted and
the bond angle in changed.
Comparison of HNH and HPH bond angles
Since N is more electronegative than P, the bonding electron pair of N–H bond will shift more
towards N atom than the bonding electron pair of P–H bond would shift towards P atom. This
results in more bond pair-bond pair repulsion in NH3 molecules than PH3 molecule. Because of lp-
bp repulsion the N–H are pushed closer to a less extent than in PH3, Consequently, HNH bond
angle is greater than HPH angle.
Boiling and melting
Boiling and melting points of hydrides depends upon the molar mass (or surface area) of
molecules. More the molar mass, the higher in the m.p. and b.p. Hydrides forming intermolecular
hydrogen bonds have exceptionally high m.p. and b.p. since intermolecular hydrogen bonds are
stronger than the Van der waals forces.
Increasing order of melting point and boiling point of hybrides is as given below :
PH3 < AsH3 < SbH3 < NH3 ; Melting point

PH3 < AsH3 < NH3 < SbH3 ; Boiling point

H2S < H2Se < H2Te < H2O ; Melting point and Boiling point

HCl < HBr < HI < HF ; Boiling point

HCl < HBr < HF < HI ; Melting point

Thermal stability, reducing power and acid strength of hydrides

Thermal stability, reducing power and acid strength of hydrides depend upon bond dissociation
enthalpy of E - H bond (E = group 15, group 16, and group 17 element). Due to the increase in
size down the group, bond dissociation enthalpy of E - H bond decreases. Consequently, thermal
stability, reducing power and acid strength of hydrides increases down the group.
Oxo acids
Strength of oxoacid depends upon the polarity of O–H bond which in turn, depends on the electron
with drawing power (or electronegativity) of the element E. Strength of oxoacids increase if the
number of oxygen atom bonded with E increases. Strength of oxoacid of halogens in the same
oxidation state depends on the electronegativity of the halogen. The more the electrongeativeity,
jksahu74@gmail.com Page | 147
stronger is the oxoacid. Stength of oxoacid of a halogen in different oxidation state increases with
the increase in oxidation state. This is because the stabilisation of the oxoanion increases with the
number of the oxygen atoms bonded to the halogen atom. More the number of oxygen atoms, the
more the dispersal of –ve charge present on the oxoanion and stronger will be the oxoacid.
Inert pair effect
The tendency of nS2 electron pair to participate in bond formation decreases with increase in
atomic number in a group. This effect is known as inert pair effect.
Group oxidation state (G ) is the most stable state for lighter elements.
( G-2 ) oxidation state becomes more stable for heavier elements in each group.
Bond energy decreases with inrease in size of atom. For this reason, the energy required to unpair
the nS2 electrons is not compensated by the energy released during the formation two additional
bonds.

Covalent character in ionic compounds (Fajan’s rule)


When anion and cation approach each other, the valence shell of anion is pulled towards cation
nucleus and thus shape of anion is deformed i.e the degree of polarisation of the bond
increases. This phenomenon of deformation of anion by a cation is known as polarisability of
anion and the ability of cation to polarize a near by anion is called as polarizing power of cation.

Greater the polarizing power of cation and greater the polarizability of anion ,greater is the degree
of polarisation of the bond and hence greater will be covalent nature.
Degree of polarisation ∝ Covalent character
'
Polarizing power of cation ∝ , ∝ Charge
)*
Polarisability of anion ∝ Size , ∝ Charge

Disproportionation Reaction
The same chemical substance undergoes both oxidation and reduction.
The element undergo Disproportionation Reaction should have intermediate oxidation state.
+3 +5 +2
3 HNO2 HNO3
+ 2 NO + H 2O
Nitrous acid Nitric acid Nitric oxide

+3 ∆ +5 -3
4 H3PO3 3 H3PO4
+ PH3
Phosphorus acid Phosphoric acid Phosphine

Species NH3 H2N – NH2 H2N - OH N2 N2O NO N2O3 NO2 N 2O 5


O.S -3 -2 -1 0 +1 +2 +3 +4 +5

jksahu74@gmail.com Page | 148


jksahu74@gmail.com Page | 149
- + + - Nitrogen dioxide or Nitrogen ( IV ) oxide
N N O N N O P
xx x xx xx x xx x
x O N O O N O x
x
xx xx xx xx P P
+1 Oxidation state N 2O Nitrous oxide
xx
x
x O NO 2 x
x
xx
O xx
P
White Phosphorus
Dinitrogen trioxide or Nitrogen (III) oxide N N
xx
O
xx
xx
O
N 2O 3 x
x
xx
O
x
x
xx
O
x
x O xx
xx
O
xx
x
x
xx xx
N N N N
xx xx Dinitrogen pentoxide or Nitrogen (V) oxide
xx
x Ox
x x
+3 Oxidation state O xx xx xx xx
xx x O O x
xx xO O xx x x
x xx
xx xx
xx
N O N N O N
xx
P Inter electronic repulsion xx
P NO xx x
xx
Ox
P P P P Nitric oxide
x
x xOx O
xx
x x O
x xx
N 2O 5 xx x

P P x
x
N O O N N O
Red Phosphorus x

Cl Cl
+ Cl SP
3
3
sp .
Cl
Cl Cl P
P Cl. P P P Cl
Cl Cl Cl
Cl
. Cl
Cl Cl Cl
H
H SP3d Cl Cl
H Solid PCl5 ( Ionic) [ Phosphorus trichloride ]
[ Phosphorus pentchloride ]
Phosphine

Hypophosphorus Acid Orthophosphorous Acid Orthophosphoric Acid Pyrophosphorus Acid


( Phosphinic Acid ) ( Phosphonic Acid ) O
O
H 4P 2O 5 O
O sp3 O sp 3
H 3 PO 4 Basicity = 3 Basicity = 2
H 3 PO 2 Basicity = 1 Basicity = 2 P P P
H 3PO 3 OH H
P P HO O
H OH
OH OH OH OH
H H
H OH
jksahu74@gmail.com Page | 150
Hypophosphoric Acid Basicity = 3
Pyrophosphoric Acid Metaphosphoric Acid ( HPO 3 )n HO O
O H 4 P 2O 6 O O
H 4P 2O 7
O O
O OH O P P
Basicity = 4 O OH
P P
Basicity = 4 P O O
P . P P P .
OH O
OH O P
HO HO O O O
OH OH OH
OH
OH HO O
OH O
Polymetaphosphoric acid Cyclotrimetaphosphoric acid ( HPO 3 )3

F F S
S6
F S8 S S
F F S S S
S S
S S
S
F F F S S
S

F F Cl Cl
3 2 3 3 O
Octahedral SP d See-Saw SP d Bent shape SP
H Cl

Sulphurous Acid Hypochlorous acid


Sulphuric acid Peroxodisulphuric acid
Hybridisation - SP3 O Hybridisation - SP3 O O
xx
H 2S 2 O 8 O O
S
H 2 SO 3 H 2 SO 4
S S S H Cl
OH Basicity = 2 OH Basicity = 2 O O
O O O
OH
Pyramidal O
Tetrahedral OH HO
Chlorous acid
OH

Pyrosulphuric Acid- Oleum H


O O Sulphur dioxide H O F
xx
H 2 S 2O 7 xx O
S+ S+ BrF3
x x- xx Cl x
S S xx
xx
- Cl x
O
xx

O O
xx

O xx
O xx xx O O Br F
xx O
O O O

xx
OH HO O
Chloric acid Perchloric acid
F F
F
IF5 IF7
F
F
T- Shaped
F F F I
I
F F F F
xx
F
Square pyramidal Pentagonal
bipyramidal

jksahu74@gmail.com Page | 151


Fact Explanation
Nitrogen does not form pentahalide although it Due to absence d-orbitals N cannot extend its valency beyond four
exhibit +5 oxidation state
PH3 has lower B.pt than NH3 N is more electronegative than P so in NH3 there is intermolecular H-bonding hence it has
high b.pt
NH3 acts as Lewis base Because N has a lone pair electron so NH3 acts as a Lewis base
NO2 dimerises NO2 has an odd electron so it dimerises to pair up electron and to achieve octet configuration
NH3 is stronger base than PH3 Due to smaller size of nitrogen there is high electron density on nitrogen so electron pair is
easily available
PCl3 fumes in moisture
PCl3 undergoes hydrolysis and gives fumes of HCl.
PCl3 + 3 H2O H3PO3 + 3HCl
All the five P-Cl bonds are not equal in PCl5 The two axial bonds suffer more repulsion from equatorial bonds and hence are elongated
H3PO2 has reducing character Since it has two P-H bonds
H3PO3 is dibasic (diprotic) but H3PO4 is In H3PO3 only two H atoms are linked to O which are ionisable the third H is attached to P
tribasic and not ionisable because P is less electronegative. In H3PO4 all the three H atoms are with
O and ionisable
PCl5 is ionic in solid state It is due to the following conversion : 2PCl5 [PCl4]+[PCl6]-
Formation of PH5 is energetically unfavorable Amount of energy required for promotion of of electrons from 3s to 3d is greater than energy
released in two extra bond formation. Thus, formation of PH5 is energetically unfavorable
Basic strength decreases from NH3 to BiH3 The basic character of these hydrides is due to presence of lone pair of electron on the
central atom. As the size of the central atom increases the electron density on the central
atom decreases and consequently its tendency to donate a pair of electron decreases.

Nitrogen forms a number of oxides. Nitrogen has a great tendency to form Pπ- Pπ multiple bonds between N & O atoms, where
as other elements of this group do not form such type of bonds.

jksahu74@gmail.com Page | 152


Fact Explanation
In NCl3 , Cl has vacant d-orbitals to accept the lone pair from H2O but in NF3 , F has
NCl3 hydrolysed but NF3 does not no d-orbitals NCl3 + 3H2O NH3 + 3 HOCl

Nitrogen shows little catenation but phosphorous distinctly Due to smaller size of N there is repulsion between the lone pairs and N-N single
shows catenation property bond is weaker than P-P

+5 oxidation state of Bi is less stable than +3 Because inert pair effect is very prominent in Bi , so +5 oxidation state is not stable

Bi in +5 oxidation state is strong oxidizing agent Because inert pair effect is very prominent in Bi so Bi5+ can be easily converted into
Bi3+

NO(nitric oxide) becomes brown when released to air It oxidizes to NO2

NH3 is a good complexing agent/ NH3 acts as a ligand It has lone pair of electron on N-atom and can be donated for the coordination bond.

Bi2O3 is not acidic The size of Bi3+ is very large and so there is very weak +ve electric field around it so
it does not interact with water to release H+

BiH3 is the strongest reducing agent among the group-15 Since Bi-H bond is the weakest among pr-15 hydrides so H2 gas is evolved which is
hydrides reducing
N2 is less reactive at room temperature Due to having triple bond and hence high bond dissociation energy (946 kJ/mol)
In PH3 there is lp-bp repulsion so bond angle is less where as in PH4+ there is no lp-
Bond angle in PH4+ higher than in PH3 bp repulsion

As we move from NH3 to BiH3, the size of the central atom goes on increasing and
The bond angle gradually decreases from NH3 to BiH3 electronegativity goes on decreasing. As a result, the electron pairs ( LP & BP )
tends to lie away from the central atom. There is a gradual decrease in the force of
repulsion among the electron pairs on the central atom resulting in decreased bond
angles in the same order.

R3P=O exists but R3N=O does not The absence of d-orbitals in the valence shell of nitrogen make it not to form
Pπ –dπ multiple bonds. As a result nitrogen cannot expand its covalency beyond 4

jksahu74@gmail.com Page | 153


Fact Explanation
NO is paramagnetic in gaseous state but diamagnetic in NO(g) has odd number of electrons so it is paramagnetic but in liquid and solid
liquid and solid state state it exists as dimmer so there is no unpaired electron and it will be
diamagnetic
N exists as N2 but P exists as P4 Due to smaller size N can form pπ-pπ multiple bonding and exists as discrete N2
molecule but P cannot form pπ-pπ multiple bonding as their atomic orbitals are
so large and diffused that they cannot have effective overlapping.
PCl5 cannot act as reducing agent In PCl5 P has +5 oxidation state. P has five valence electron in its valence shell
so it can not increase its oxidation state beyond +5, so it cannot act as reducing
agent.
Phosphorous is kept under kerosene It is highly reactive and easily catches fire due to air oxidation
H3PO3 is syrupy liquid Due to intermolecular H-bonding
PH3 bubbles but NH3 dissolves in water NH3 forms H-bonding with water but PH3 cannot form so NH3 dissolves but PH3
bubbles out
Only a small increase in radius is observed from As to Bi Due to poor shielding effect of d and f orbital.
Nitrogen is gas where as phosphorous is solid at room Nitrogen is diatomic molecule having weak van der Walls attraction where as
temperature.
phosphorous is tetra atomic so it has strong van der Walls attraction.
N-N bond is weaker than P-P bond Due to greater repulsion between lone pairs in the single N-N bond
Maximum number of covalent bond formed by N is four since only 4 orbitals ( one 'S' and three 'P' ) are available for bonding.
P2O5 cannot be used for drying ammonia gas P2O5 is acidic it reacts with ammonia in presence of moisture and form
(NH4)3PO4
NO2 is coloured but its dimmer N2O4 is colourless Because NO2 has unpaired electron so it can absorb light from VR
Acidity of oxyacids of nitrogen increases with increase in Because non metallic character increases with oxidation number
oxidation number of N

jksahu74@gmail.com Page | 154


Fact Explanation
White phosphorous is more reactive than red White phosphorous consists of discrete P4 molecules which is tetrahedral so reactive
phosphorous due to angular strain but in red phosphorous the P4 molecules are linked in extended
chain structure so it is less reactive.
Phophinic acid ( H3PO2) is mono basic / mono protic Only one H atom is linked with O which is ionisable

N2 has higher bond dissociation energy than NO Because N2 has higher bond order

N2 and CO have same bond order but CO is more CO is polar molecule


reactive
(CH3)3N is pyramidal but (SiH3)3N is planar (CH3)3N is pyramidal due to sp3 hybridisation and one lone pair on N but (SiH3)3N is
planar due to sp2 hybridisation and its lone pair is donated to vacant d orbital of Si for
pπ-dπ overlap
The first IE of N is greater than that of O It is due to half filled and hence stable electronic configuration of N
HNO2 disproportionates In HNO2 the N is in +3 oxidation state which may increase as well as decrease.
PCl5 cannot act as reducing agent In PCl5 phosphorous is in +5 oxidation state that is the highest oxidation state of P.
Nitrogen dioxide is a mixed anhydride As it react with water to give two different oxide.
2 NO2 + H2O HNO2 + HNO3
Nitrogen pentoxide is N2O5 while phosphorous Due to small size , N able to complete its octet via pπ – pπ bond formation with
pentoxide is P4O10 oxygen. Hence Nitrogen pentoxide is N2O5. Whereas P cannot complete its octet via
π-bond formation. So phosphorous pentoxide as dimer is P4O10
Cl2 can be prepared from HCl and HCl from Cl2 MnO2 + 4 HCl MnCl2 + Cl2 + 2 H2O
H2 + Cl2 2 HCl ( in presence of light )
Dimeric form of NO will not exist in gaseous phase because bond become weaker
NO undergo dimerisation only in solid and liquid phase
due to inter electronic repulsion of lone pair present on nitrogen atom. But it is found
that NO undergo dimerisation in liquid and solid state.

jksahu74@gmail.com Page | 155


Fact Explanation
Group 16 elements have lower I.E with compare to Because group 15 elements have stable half filled p-sub shell(ns2np3) so electron cannot
corresponding group 15 elements be removed easily
H2S is less acidic than H2Te In H2Te there is lower bond energy of H-Te bond due to larger size of Te
H2S acts as reducing agent while SO2 acts as both. In H2S , S has its minimum oxidation state - 2 where as in SO2 it is +4 so it can be
decreased up to -2 or increased up to +6 , So H2S is only reducing but SO2 is both.
H2S is acidic while H2O is neutral H-S bond is weaker due to larger size of S so proton release easier in H2S
SF6 is known but SH6 does not exist Fluorine is the strongest oxidizing agent so it can oxidizes S to its maximum oxidation
state +6 , H can not
Compound of F & O is fluoride of oxygen not oxide of F is more electronegative than O
fluorine
SCl6 is not known but SF6 is known F is strongest oxidizing agent so it can oxidizes S to its maximum oxidation state +6. Cl
cannot. Again Cl has larger size so steric repulsion is there in SCl6
SF6 is used as gaseous electrical insulator It is thermally stable and chemically inert
SF6 is not easily hydrolyzed It is sterically protected by six F atoms hence does not allow H2O molecules to attack the
S atom
S exhibits catenation properties but not Se Due to smaller size of S than Se. S – S bond is much stronger than Se – Se bond.
S disappears when boiled with Na2SO3 It forms sodium thiosulphate . Na2SO3 + S Na2S2O3 ( soluble)
H2O is liquid but H2S is gas Due to small size and high electronegativity of oxygen, water molecules form inter
molecular H-bonding, for which H2O is liquid whereas H2S is gas at room temperature.
Ozone is powerful oxidizing agent It decomposes to form nascent oxygen
SO2 act as reducing agent. Since +6 oxidation state of sulphur is more stable than +4
HNO3 cannot be used for the preparation of H2S from HNO3 being strong oxidising agent oxidises H2S to sulphur.
metal sulphides.

jksahu74@gmail.com Page | 156


Fact Explanation
Ka2 is less than Ka1 , for H2SO4 in water The 2nd proton releases from HSO4- which is difficult. So Ka2 is less than Ka1
Due to smaller size, Oxygen can form pπ-pπ multiple bond and exists as
O2 is gas but sulphur is solid discrete diatomic molecule. But sulphur is polyatomic. The large difference
between the melting point and boiling point of oxygen and sulphur is due to their
atomicity, oxygen exist as O2 whereas sulphur exist as S8.
Group 16 elements are called chalcogens Chalcogen means ore forming elements. They form several ores
Positive oxidation states of O are generally not found Due to high electro negativity of Oxygen
Thermal stability decreases from H2O to H2Te in group 16 Due to increase in atomic size from O to Te the bond dissociation energy
decreases
Oxygen does not show +4 & +6 oxidation states like Sulphur Due to absence of d-orbital in oxygen
The magnitude of electron gain enthalpy of oxygen is less Due to very small size of O there is inter electronic repulsion
than that of sulphur
Among the hydrides of group 16 water shows unusual Due to H-bonding in water the molecules get associated
properties
Sulphur exhibits catenation but not Oxygen Because S-S bond is stronger than O-O bond
Tendency to show -2 oxidation state diminishes from O to Po Due to decreases in electronegetivity moving down the group
in group 16
O2 is paramagnetic although it has even number of electrons Due presence of unpaired electrons in anti bonding molecular orbital
Sulphur in vapour state exhibit paramagnetism In vapour stare sulphur partly exists as S2 molecule and like O2 it has unpaired
electrons in π* orbitals
SF6 is less reactive than SF4 In SF6 sulphur atom is sterically hindered due to six F atoms
Although electron gain enthalpy of fluorine is less than that of
-low enthalpy of dissociation of F-F bond
chlorine, but fluorine is a stronger oxidising agent than-high hydration enthalpy of F -
chlorine. -high lattice enthalpy
Nitrogen shows +1 to +5 oxidation state in the formation The range of oxidation state of nitrogen from – 3 to + 5. Since oxygen is more
of oxide. electronegative than nitrogen so nitrogen will not show –ve oxidation state.

jksahu74@gmail.com Page | 157


Fact Explanation
Halogens have maximum negative electron gain Because they have smallest size in their respective periods
enthalpy (∆egH)
Fluorine exhibits only -1 oxidation state , other halogen Fluorine is most electronegative element and due to absence of d-orbitals it cannot
shows +1, +3, +5, +7 oxidation states expand its octet so it does not exhibit positive oxidation state.
Iron reacts with HCl gives Fe(II)chloride and not Fe + 2 HCl FeCl2 + H2
Fe(III)chloride H2 liberated prevents the oxidation of FeCl2 to FeCl3
Bond dissociation energy of F2 is less than Cl2 Due to very small size of F there is interelectronic repulsion in F2 so it has low bond
dissociation energy
Fluorine does not undergo disproportionation Disproportination means simultaneous oxidation-reduction. F being the most
electronegative element undergoes only reduction but not oxidation
NO dimerises but Cl2O does not NO is odd electron species so it complete its octet by dimerisation
Bleaching by Cl2 is permanent but by SO2 is temporary Cl2 bleaches by oxidation while SO2 does it by reduction. The reduced product gets
oxidized again in air and the colour returns.
HF has lower acid strength than HI Due to larger size of I , the H-I bond is weaker than H-F bond so HI is stronger.
I2 is more soluble in KI than in water I2 forms complex with KI i.e K+I3-
HClO4 is stronger than H2SO4 Because the conjugate base ClO4- is stable due to resonance

Fluorine is stronger oxidizing agent than chlorine Fluorine has higher electrode potential value due to its low bond dissociation energy
though it has lower electron gain enthalpy and high hydration enthalpy with compare to chlorine.
SO3 is not absorbed directly in water to form H2SO4 by The reaction becomes difficult to handle as the reaction is highly exothermic and mist
contact process will form.
HClO is stronger acid than HIO ClO- is more stable than IO- because Cl is more electronegative, so HClO is stronger
HClO4 is stronger acid than HClO3 ClO4- is more stable than ClO3- due to more resonance

jksahu74@gmail.com Page | 158


Fact Explanation
Interhalogens are more reactive than halogens They are polar
HF is stored in wax coated glass bottle HF reacts with alkali present in glass.
MF is more ionic than MCl ( M is alkali metal) Because F- is smaller than Cl- and hence it is less polarisable
Chlorine being stronger oxidizing agent than iodine, first oxidizes KI into I2 which
imparts brown colour to the solution.
Cl2 + KI brown, 2KI + Cl2 2 KCl + I2
but excess Cl2 turns it colourless But when chlorine is passed in excess, the I2 so formed gets further oxidized to HIO3
(Colourless)
5 Cl2 + I2 + 6 H2O 10 HCl + 2 HIO3
ClF3 exists but FCl3 does not F is smaller in size and cannot accommodate three chloride ions due to steric factor.
HF is less volatile than HCl In HF there is intermolecular H-bonding so the HF molecules get associated
F form only one oxo acid , HOF Due to absence of d-orbital it cannot exhibit higher oxidation states
Oxygen form hydrogen bonding , Chlorine does not Oxygen is more electronegative and small in size than Chlorine
Iodine forms I3- but fluorine does not form F3 -
Due to small size of fluorine
Acid strength increases in the order HF< HCl < HBr < HI As size increases from F to Cl the bond dissociation energy decreases from HF to HI
HI cannot be prepared by heating KI with conc. H2SO4 H2SO4 is an oxidizing agent. Instead of forming HI, it oxidizes I- into I2
Impure PH3 is inflammable Due to presence of impurities such as P2H4 or P4 vapour.
- They have completely filled valence shell.
The noble gases are chemically inert
- They have high ionisation enthalpies.
- They have more positive electron gain enthalpies

jksahu74@gmail.com Page | 159


Fact Explanation
The magnitude of electron gain enthalpy It is due to small size of fluorine atom. As a result, there are stronger inter electronic repulsion in the
of F is less than that of Cl relatively small 2P-orbitals of fluorine and thus, the incoming electron does not experience much
attraction
He , Ne do not form compound with F Due to high IE
Noble gases have very low b.pt Because there is only weak dispersion force between their atoms
Hydrolysis of XeF6 is not a redox Because in the products formed XeOF4 and XeO2F2 the Xe has the same oxidation state (+6) as in
reaction XeF6
Neon used as warning signal Because Ne – light has high fog penetration power
Noble gases form compounds only with Because F & O are the most electronegative elements
fluorine and oxygen
Xe does not form XeF3 or XeF5 Xe has all paired electrons so promotion of one, two or three electrons will give rise to two, four or six
unpaired electrons hence cannot form XeF3 and XeF5
Out of noble gases only Xe forms Because Xe has comparatively low IE and vacant orbitals for promotion of electrons
compounds
Noble gases are mostly inert Because they have completely filled valence orbitals i.e octet configuration
Helium is used as diving apparatus Because it is less soluble in blood with compare to nitrogen
It is difficult to study the chemistry of Rn Because Rn is radioactive and hence very unstable
Noble gases have comparatively large They are mono atomic so their van der Walls radii measured which is longer than covalent/ionic or
atomic size metallic radii
Halogens are coloured Due to absorption of radiation in visible region which results in the excitation of outer electrons to
higher energy level. As halogens absorb different wave lengths, they show different colours. F2 –
Yellow , Cl2 – Greenish yellow , Br2 – Brown , I2 – Violet

Acid strength As the stability order of conjugate bases is


HClO4 > HClO > HClO > HClO
3 2 −
ClO4 >

ClO3
> ClO2−
> −
ClO

jksahu74@gmail.com Page | 160


Fact Explanation

NH3 PH3 AsH3 SbH3 BiH3 * Due to decrease in bond ( H-E )


Acidic character H2O < < H2S <
H2Se H2Te
Stability Decreases dissociation enthalpy from top to bottom in
a group.
Thermal stability H2O > 2 > 2 > H2Te
H S H Se
Reducing character Increases ** Electron density decreases top --> bottom
Reducing character H2S < H2Se < H2Te Basic character Decreases *** BP - BP repulsion gradually decreases
Bond angle Decreases top --> bottom
( H2O is not reducing agent )
- They have completely filled valence shell.
The noble gases are chemically inert - They have high ionisation enthalpies.
- They have more positive electron gain
enthalpies.

PtF6 is a powerful oxidising agent PtF6 oxidises O2 and Xe

The first ionisation enthalpy of Xe ≈ O2


As PtF6 react with O2, so PtF6 should react with
Xe
O+ -
PtF6 should react with Xe
O2 + PtF 6 2 [PtF 6]

Dioxygenyl hexafluoro platinate(V)


[ Red]
Xe + PtF 6 Xe [PtF ] -
+
6

[ Red]
Noble gases being monoatomic, have no
interatomic forces except weak dispersion forces
Noble gases have low boiling point and therefore, they are liquified at very low
temperature. Hence they have low boiling point.

jksahu74@gmail.com Page | 161


NH3 PH3
In air and soil, ammonia is formed by the decay of nitrogenous organic
matter i.e. urea.
Ca 3P 2 + 6 H 2O 3 Ca ( OH ) 2 + 2 PH 3
Calciumphosphide
Preparation

NH2CONH2
Urea
+ 2 H 2O ( NH4 )2 CO 3

Ca3P2 + 6 HCl 3 CaCl2 + 2 PH3


( NH4 ) 2 CO 3 2 NH3 + H2O + CO2

2 NH4Cl + Ca (OH)2 2 NH3 + 2 H2O + CaCl2 P4 + 3 NaOH + 3 H2 O PH3 + 3 NaH2PO2


( White )
2 NH3 + 2 H2O + Na2SO4
(NH4)2SO4 + 2 NaOH ( Sodiumhypophosphite )

Aqueous solution of NH3 is basic due to the formation of OH- ion. PH3 act as reducing agent, due to weak P – H bond.
+ -
NH3 + H2O NH4 + HO

NH3
+ HCl NH4Cl PH3
+ HI PH4I
Properties

Impure Phosphonium iodide


NH3 + H2SO 4 (NH4)2SO4
PH4I
KOH
Acid Salt
KI + H2O + PH3
Pure
2 FeCl 3 + 3 NH4OH Fe2O 3.xH 2O + 3 NH4Cl
( Brown ppt. ) PH3 + H2O Light
P4 ( Red ) + H2

ZnSO 4 + 2 NH4OH Zn (OH) 2 + (NH4)2SO 4


( White ppt. )

Cu
2+
+ 4 NH3 [ Cu (NH3)4 ]
2+ 3 CuSO4 + 2 PH3 Cu3P2 + 3 H2SO4
( Blue ) Ligand ( Deep Blue )
3 HgCl2 + 2 PH3 Hg3P2 + 6 HCl
AgCl + 2 NH3 [ Ag (NH3)2 ]Cl
( White ppt. ) Ligand ( Colour less )

jksahu74@gmail.com Page | 162


Cl

Cl e a
e Heat
e P Cl P
Cl Cl Cl
a Cl

Cl

PCl5 PCl3
P4
P4
+ 10 Cl2 4 PCl5 + 6 Cl 2 4 PCl3
Preparation

( white )

P4
+ 10 SO2Cl2
Sulphuryl chloride
4 PCl5
+ 10 SO2 P4 + 8 SOCl 2 4 PCl 3 + 4 SO 2 + 2 S 2Cl 2
( white ) ( white ) Thioinyl chloride

H2O
PCl 5 POCl 3 + 2 HCl 3 H 2O

POCl 3
H2O
H3PO4 + 3 HCl
PCl 3 H3PO 3 + 3 HCl
( Fumes )

CH3COOH 3 CH 3 COOH
PCl 5 CH3COCl + POCl 3
+ HCl PCl 3 H3PO3 + 3 CH3COCl
Properties

C 2H 5OH 3 C 2H 5OH
PCl 3
PCl5 C2H5Cl
+ POCl3 + HCl H3PO3 + 3 C2H5Cl

Sn
PCl5 SnCl4 + 2 PCl 3

2 Ag
PCl 5 2 AgCl + PCl 3

jksahu74@gmail.com Page | 163


Name Compounds of Nitrogen
Preparation
N2O ∆
NH 4NO 3 N 2O + 2 H2O
Amm. nitrate Nitrous oxide
NO 2 NaNO2 + 2 FeSO 4 + 3 H2SO4 Fe 2(SO 4)3 + 2 NaHSO4 + 2 H 2O + 2 NO
Sodium nitrite Ferrous sulphate Ferric sulphate Sodium bisulphate Nitric oxide
N2O3 250 K 2 N 2O 3
2 NO + N2 O 4

Nitrogen (III) oxide


NO2 Cool
673 K
2 Pb(NO3)2 2 PbO + 4 NO2
Heat
2 N2O 4

Nitrogen ( IV ) oxide
N2O5 4 HNO3 + P4O 10 4 HPO 3 +
2 N 2O 5
Phosphorous pentoxide Metaphosphoric acid
Nitrogen (V) oxide

Compounds of Phosphorous
Hypophosphorus Acid
(Phosphinic Acid)
P4 + 3 NaOH + H2 O PH3 + NaH2PO2 H 2SO 4
H3PO 2 + Na2SO4

H 3PO 2
4 AgNO3 + 2 H2O + H3PO2 4 Ag + 4 HNO2 + H3PO4
Orthophosphorous Acid
+3 +5 -3
(Phosphonic Acid) ∆
P 4O 6 + 6 H 2O 4 H3PO 3 4 H3PO3 3 H3PO4 + PH3
H 3PO 3
Orthophosphoric acid

jksahu74@gmail.com Page | 164


Orthophosphoric Acid
H 3PO 4
P4O10 + 6 H2 O 4 H3PO4

Pyrophosphorus Acid PCl 3 + 5 H3PO 3 3 H 4P2O 5


+ 3 HCl
H4P2O 5
2 H3PO 4 523 K
Pyrophosphoric Acid H4P2O 7 + H2O
Phosphoric acid
H4P2O7
Hypophosphoric Acid + 2 H 4P2O 6 + 8 NaCl
P4
Red
8 NaOCl + 4 H 2O
H4P2O6
Metaphosphoric Acid ∆
3 H3PO3 + 3 Br 2 ( HPO3 ) 3
+ HBr
( HPO 3 )n Sealed tube

White Phosphorus 573 K Red Phosphorus

It is a softy, waxy solid and can be cut with a knife Hard and crystalline solid.
Have garlic odour Odourless
Poisonous in nature Non-poisonous in nature
Shows phosphorescence Doesn't exhibit phosphorescence
React with caustic alkalies to produce phosphine. Doesn't react with caustic alkali
P4 + 3 NaOH + 3 H 2O PH3 + 3 NaH2PO2
Sodium hypophosphite
Burns in oxygen with greenish glow to form Only upon strong heating burns in oxygen to
phosphorous pentoxide. form P2O5
P4 + 5 O2 P4O10

Highly reactive due to angular strain in tetrahedral Weakly reactive due to polymeric structure.
P P
discrete molecule.
P P P .
P . P
P P
P P
P
P4 molecules held by weak Vanderwaal's forces. P4 molecules are linked by covalent bonds in
the polymeric form.
Ignition temperature is very low due to angular Ignition temperature is very high
strain.
White phosphorus, when heated to 573 K and Red phosphorus, when heated to 803 K
high pressure produce β- black phosphorus. produce α- black phosphorus.
White 473 K Red 803 K
β − black α− black
phosphorus High pressure phosphorus. phosphorus phosphorus

jksahu74@gmail.com Page | 165


Rhombic sulphur 369 K Monoclinic sulphur

α - Sulphur β - Sulphur
Yellow Colourless
S8 rings fit snugly into each other S8 rings are stacked on top of each other.
. .
.
.
. .
. ..
. . .
. .
.
.
Octahedral shape crystal Fine needle shaped crystal

Stable below 369 K Stable above 369 K

Oxidising nature of Ozone


Ozone act as an oxidising agent because it liberates nascent oxygen. O3 O2
+ [O]
[1] Ozone oxidises lead sulphide to lead sulphate.
O3 O2 + [O] Χ 4
PbS + 4[O] PbSO 4

PbS + 4 O3 4 O2 + PbSO 4

[2] Ozone oxidises potassium iodide to iodine


O3 O 3 can be estimated by titrating liberated
O2
+ [O]
iodine against a standard solution of sodium
2 KI + H 2O + [O] 2 KOH + I2 thiosulphate.
2 KI + H2 O + O3 2 KOH + I2 + O2

[3] Ozone oxidises nitric oxide to nitrogen dioxide


O3 O2 + [O] Nitric oxide emitted from supersonic jet
NO + [O] NO 2 aeroplane depletes the ozone layer.
NO + O3 NO 2 + O2

[4] Ozone oxidises potassium manganate ( green) to potassium permanganate ( pink)


O3 O2 + [O]

2 K2MnO 4 + H 2O + [O] 2 KMnO 4 + 2 KOH

2 K2MnO 4 + H 2O + O3 2 KMnO 4 + 2 KOH + O2


potassium manganate potassium permanganate
( green) ( pink)

Oxidising Nature of Nitric acid


Nitric acid act as an oxidising agent because it provide nascent oxygen. The product of oxidation
depend upon:
[1] The concentration of acid
[2] Temperature
[3] Nature of the material undergoing oxidation.
jksahu74@gmail.com Page | 166
Concn 2 HNO3 2 NO 2 + H2O + [O]

Dilute 2 HNO3 2 NO + H2O + 3[O]

Dilute nitric acid is better oxidising agent because it provides more number of nascent oxygen.

HNO 3
HNO 3 + [H] NO 2 + 2 [H] NO + 5 [H] NH3 NH 4NO 3 ∆
N2O
H2O H2O H2O 2 H 2O

Oxidation of non-metal with concentrated HNO3

2 HNO 3 2 NO 2 + H2O + [O] X 2


C + 2 [O] CO 2
4 HNO 3 + C 4 NO 2 + 2 H 2O + CO 2

2 HNO3 2 NO2 + H2O +[O] X 5


I2 + H2O + 5 [O] 2 HIO3

10 HNO3 + I2 10 NO2 + 4 H2O + 2 HIO3

2 HNO3 2 NO2 + H2O + [O] X 24


S8 + 8 H2O
+ 24 [O] 8 H2SO4
48 HNO3 + S8 48 NO2
+ 16 H2O + 8 H2SO4

2 HNO3 2 NO2 + H2O + [O] X 10


P4 + 10 [O] + 6 H2O 4H3PO4
20 HNO3 + P4 20 NO2 + 4 H2O + 4 H3PO4

Oxidation of metal with concentrated / dilute HNO3

Oxidising power Dilute HNO3 > Concn HNO3


Metal + HNO3 Metal nitrate + H
Reactivity of Zn > Cu

Zinc with concentrated HNO3


2 HNO3 2 NO2 + H2O + [O]
Zn+ 2 HNO3 + [ O ] Zn (NO 3)2 + H2O

Zn + 4HNO3 2 NO2 + Zn (NO3)2 + 2 H 2O

jksahu74@gmail.com Page | 167


Copper with dilute HNO3
2 HNO3 2 NO + + 3[O]
H2O

Cu + 2 HNO3 + [O] Cu (NO3)2 + H2O X3


3 Cu + 8 HNO 3 2 NO + 3 Cu (NO3)2 + 4 H2O

Zinc with dilute HNO3


2 HNO3 N2 O + H2O + 4 [O]

+ + X4
Zn + 2 HNO3 [O] Zn (NO 3)2 H2O

10 HNO 3 + 4 Zn N2 O + 4 Zn (NO 3)2 + 5 H 2O

Copper with concentrated HNO3


2 HNO 3 2 NO 2 + H2O + [O]
Cu + 2 HNO3 + [ O ] Cu (NO3)2 + H2O

Cu + 4HNO3 2 NO 2 + Cu (NO3)2 + 2 H 2O

Brown ring test for Nitrate


When dilute ferrous sulphate solution is added to an aqueous solution containing nitrate ion,
Then ferrous ion reduces nitrate ion to nitric oxide.

+
NO3 + 4H +2+ 3 e- NO + 2 H 2O
3+
Fe Fe + e- X 3
2+ + 3+
NO3 + 3 Fe + 4H 3 Fe + 2 H2O + NO

When concentrated H2SO4 is added along the sides of the test tube a brown ring will form at the
interface between the solution and H2SO4 layer.

2+ 2+
[ Fe ( H2O )6 ] + NO [ Fe ( H2O ) 5 NO ]
Brown ring

The brown ring appears due to charge transfer complex i.e. one electron is transferred.

Oxidising properties of Chlorine Cl2

Cl 2+ H2 O HCl + HOCl 2 HCl + [O]


Yellow Colourless

Coloured substance + [O] Colourless substance [ Bleaching action]

jksahu74@gmail.com Page | 168


(1) Chlorine water oxidises acidified ferrous to ferric
Cl 2 + H2 O 2 HCl + [O]
2 FeSO 4 + H2SO4 + [O] Fe2(SO4)3 + H2 O

2 FeSO 4
+ H2SO4 + Cl2 Fe2(SO 4)3 + 2 HCl

(2) Chlorine water oxidises sulphite to sulphate

Cl2 + H 2O 2 HCl + [O]


Na2SO3 + [O] Na2SO4
Na2SO3 + Cl2 + H2O Na2SO4
+ 2 HCl

(3) Chlorine water oxidises sulphur dioxide to sulphuric acid

Cl2 + H2 O 2 HCl + [O]


SO2 + H2 O + [O] H2SO4
SO2 + Cl2 + 2 H2 O H2SO4 + 2 HCl
(4) Chlorine water oxidises iodine to iodic acid
Cl 2 + H2 O 2 HCl + [O] Χ5
I2
+ H2O + 5 [O] 2 HIO 3
I2
+ 5 Cl2 + 6 H2O 2 HIO 3 + 10 HCl

Acidic Nature of Nitric acid


+
HNO3+ H 2O H 3O + NO3
CaO + 2 HNO3 Ca (NO3)2 + H 2O
NaHCO3
+ HNO3 NaNO3 + CO2 + H2 O

Reducing Character of Sulphur dioxide


Moist sulphur dioxide behave as reducing agent, because with water sulphur dioxide produce
nascent hydrogen.

SO2 + 2 H2O H2SO4


+ 2 [H]
It reduces ferric to ferrous salt
2-
2 e−
+
SO2 + 2 H2O SO4 + 4H +
SO2
+ 2 H2O H2SO4
+ 2 [H]
Fe
3+
+ e−
2+
Fe Χ2
Fe2(SO4)3
+ 2 [H] 2 FeSO4 + H2SO4 3+ 2- 2+ +

Fe2(SO4)3 + SO2 + 2 H2O 2 FeSO4 + 2 H2SO4


SO2 + 2 Fe + 2 H2O SO4 + 2 Fe + 4H

jksahu74@gmail.com Page | 169


It decolourises acidified KMnO4 solution [ Test for SO2 ]

2 KMnO 4+ H2SO4 K2SO 4


+ 2 MnSO4 + 3 H2O + 5 [O]
SO 2 + 2 H2O H2SO 4
+ 2 [H] Χ 5
2 [H] + [O] H2O Χ5
2 KMnO 4
+ 5 SO 2 + 2 H2O K2SO 4
+ 2 MnSO4 + 2 H2SO 4
[ Pink] [Colourless]

2-
+ 4 H + 2 e− Χ 5
+
SO 2
+ 2 H 2O SO 4
MnO−4 + 2+
+ 8H + 5 e− Mn + 4 H 2O Χ 2

2 MnO−4 + 2- 2+ +
5 SO 2
+ 2 H 2O 5 SO 4
+ 2 Mn + 4 H
[ Pink] [Colourless]

Oxidizing power of F2 > Cl2 > Br2 > I2 Explain.

F2 oxidises water to oxygen 2 F2 + 4 H+ 4 F−


2 H 2O
+ + O2

Cl2 and Br2 react with water to produce acid. Cl


2 + H 2O HCl + HOCl

Br 2
+ H2O HBr + HOBr

Reaction of iodine with water is not spontaneous. I2 + H2O No reaction

But I- can be oxidised by oxygen in acidic medium. 4 I − + +


4H + O2 2 I2
+ 2 H2O

A halogen oxidises halide ions of higher atomic number.



F2 + 2 X− 2F + X2
∴ X Cl, Br. I
Cl 2
+ 2 X− 2 Cl− + X2
∴ X Br. I
Br2 + 2 I− 2 Br− + I2

Hydrogen chloride
Preparation Properties
NH3
NaCl+ H2SO4 420 K NaHSO4 + HCl NH4Cl

NaHSO4 + NaCl 823 K Fe


Na2SO4 + HCl FeCl 2
+ H2
Liberated H2 prevents the formation of FeCl 3
HCl Na2CO3
CO 2
+ H2 O
+ 2 NaCl
NaHCO3
CO 2
+ H2 O + NaCl
Na2SO3
SO 2 + H2 O + 2 NaCl

jksahu74@gmail.com Page | 170


O2
2 SO 3 C12H22O 11
V2O 5 12 C + 11 H2O

Cu
H2O
CuSO4 + SO2 + 2 H2 O
H2SO 3
S
SO 2 + 2 H 2O

H2SO4
(Sulphurous acid) 3 SO 2

NaOH SO2
H2O
+ Na2SO3 2 NaHSO3 C CO 2 + 2 SO2 + 2 H 2O

MX
HX + M2SO 4
Cl 2 ∴
SO2Cl 2 (Sulphuryl chloride)
Charcoal X F, Cl, NO3

Ca
CaO Al
Al AlCl 3
Al 2O 3
P4 Na
P4O 10 NaCl
C
CO2 Fe FeCl 3
ZnS P4
ZnO + SO2
PCl 3
O2 CH4

SO2
CO2 + H2O Cl2 S8 S2Cl 2
SO3 H2 HCl
V2O 5
H 2S
HCl
CuCl2
Cl 2 + H2O
HCl +S
C 10H 16
HCl + C
Au
No reaction CH 4
Pt CH 3Cl + HCl
No reaction
C 2H 4
C 2H 4Cl 2

Interhalogen compounds
Preparation Properties
437 K Inter halogen compounds undergo hydrolysis
Cl 2 + F2 2 ClF
( Equal volume) giving halide ions derived from smaller halogen.
573 K
Cl 2 + 3 F2 2 ClF 3
( Excess ) ClF + H2O HF + HOCl
[Hypochlorous acid]
I2 + 3 Cl 2 2 ICl 3
( Excess ) ClF 3 + H2O HF + HOClO
[Chlorous acid]
5 F2 2 BrF 5
Br2
+ ( Excess ) BrF 5
+ H2O HF + HOBrO 2
[Bromic acid]

jksahu74@gmail.com Page | 171


Preparation Properties
I2 +
+ Cl2 2 ICl IF 5 H2O HF +[Iodic
HOIO 2
acid]
( Equimolar)
IF 7 + H2O HF + HOIO 3
Br2 + 3 F2 2 BrF 3 [Periodic acid]
( Diluted with water )

ClF3 and BrF3 are used for the production of UF6 in the enrichment of U235
U (s) + 3 ClF 3 (l) UF 6 (g) + 3 ClF (g)

Xenon- Fluorine compounds


Preparation Properties
XeF2 + PF 5 +
[XeF] [PF 6]
-
Fluoride ion acceptor

+
XeF4 + SbF 5 [XeF3] [SbF 6] -

+
XeF6 + MF M [XeF7] -
673 K 1 bar
Xe
Excess
+ F2 XeF2 (s) ∴ M Na, K, Rb, Cs

873 K 7 bar Hydrolysis of Xenon florine compounds are not a


Xe + 2 F2
1: 5 ratio
XeF4 (s)
redox reaction, because the hydrolysis products
having same oxidation states.
Xe + 3 F2 573 K 60-70 bar XeF 6 (s)
1: 20 ratio 2 XeF2 + 2 H2O 2 Xe + 4 HF + O2
XeF2 + O2F 2 XeF6 + O2
6 XeF4 + 12 H2O 4 Xe + 24 HF+ 3 O2 + XeO3
Partial hydrolysis XeOF4 + 2 HF
XeF6 + H2O
Partial hydrolysis
XeF6 + 2 H2O XeO2F 2 + 4HF

XeF 6 + 3 H2O XeO3 + 6 HF

Chile salt peter- Sodium nitrate- NaNO3


Indian salt peter- Potassium nitrate- KNO3

The general formula of apatite family is Ca9 (PO 4)6.CaX2 Where X F, Cl, OH

Fluorapatite Ca9 (PO 4) 6.CaF2

Chlorapatite Ca9 (PO 4) 6.CaCl2

Hydroxyapatite Ca9 (PO 4)6.Ca (OH)2

Phosphorite Ca3 (PO 4)2

jksahu74@gmail.com Page | 172


Process Description

On a large scale ammonia is manufactured by Haber's process


N2 + 3 H2 2 NH3 ∆f H0 = - 46.1 Kj/ mol

Iron oxide act as catalyst, K2O & Al2O3 act as promoter


Haber's process According to Le-chatelier's principle, the reaction is favoured by high pressure and low temperature.

Water

Co n
N2 Co C
mp
res ch atal
am yti

dens
sor N2 + 3 H2 2 NH 3 Unreacted
be c gases
r

er
H2
NH3

Iron oxide K 2O & Al 2O 3

Starting material- Ammonia Catalyst- Platinum and Rhodium

Ammonia undergoes catalytic oxidation to produce nitric oxide.


Pt/Rh gauge catalyst
4 NH3 + 5 O2 4 NO + 6 H2 O
Ostwald's process 500 K, 9 bar
Nitric oxide combines with oxygen to produce nitrogen dioxide
2 NO + O2 2 NO2

Nitrogen dioxide hydrolysed to give nitric acid which is 68 % pure by mass.


3 NO2 + H2O 2 HNO3 + NO

Nitric acid is further concentrated by dehydration with concentrated H2SO4

jksahu74@gmail.com Page | 173


Process Description

Burning of sulphur or sulphide ores in air to generate SO2


S + O2 SO 2

4 FeS2 + 11 O 2 2 Fe 2O3
+ 8 SO 2

Conversion of SO2 into SO3 by the reaction with oxygen in the presence of catalyst V2O5 [ Slow step]

V 2 O5
2 SO 2 + O2 2 SO 3 , ∆H0 = −196.6 KJ/mol.
Contact process
Low temperature and high pressure are favourable conditions for maximum yield

Absorption of SO3 in H2SO4 to give oleum.


SO 3 + H2SO 4 H2S2O 7 [Oleum]

Dilution of oleum with water give 96-98 % sulphuric acid.

H2 S 2 O 7 + H2O 2 H2SO 4

Species Laboratory Method Industrial Method

Nitrogen NH4Cl + NaNO2 N2


+ 2 H2O
+ NaCl
( NH4 ) 2 Cr2O 7 ∆ N2 + 4 H 2O + Cr2O 3
Amm.chloride (aq) Sodium nitrite Ammonium dichromate
N2
Ba ( N3 ) 2 ∆
Barium azide
Ba + 3 N2

jksahu74@gmail.com Page | 174


Species Laboratory Method Industrial Method
Ostwald's process
Nitric Acid
NaNO3 + H2SO 4 NaHSO4 + HNO3

HNO3 KNO3 + H2SO 4 KHSO4 + HNO3

2 KMnO 4 ∆ ∆
K2MnO 4 + MnO 2 + O2 2 HgO 2 Hg + O2
2 Ag2O ∆
2 KClO 3 ∆ 2 KCl + 3 O2
4 Ag
+ O2
Potassium chlorate ∆
Oxygen 2 PbO2 2 PbO + O2
O2 Na2O2 + H2SO4 Na2SO4 H 2O 2
+ Water Electrolysis 2 H2 + O2
MnO 2
H2 O 2 2 H2O + O2
Sulphur
dioxide
Na2SO 3 + H2SO 4 Na2SO 4 + H2 O + SO 2 4 FeS 2 + 11 O 2 2 Fe 2O 3
+ 8 SO 2
Pyrites
SO2
CuCl 2
MnO 2 + 4 HCl MnCl 2 + 2 H 2O + Cl 2 4 HCl + O2
Deacon's process 2 H 2O + 2 Cl 2

Chlorine Brine solution Electrolysis Cl 2 as by product


Cl2 4 NaCl + MnO 2 + 4 H2SO4 MnCl 2 + 4 NaHSO 3 + 2 H2O + Cl2
2 KMnO 4 + 16 HCl 2 KCl + 2 MnCl 2 + 8 H 2O + 5 Cl2

The composition of bleaching powder is Ca(OCl)2.CaCl2.Ca(OH)2. 2 H2O


2 NaOH
Cold-dilute
+ Cl 2 NaCl +
NaOCl +
H 2O
Sodium hypochlorite
which is prepared when chlorine is treated with dry slaked lime. Disproportionation
6 NaOH + 3 Cl 2 5 NaCl + +
NaClO 3 3 H 2O
2Ca(OH)2 + 2 Cl2 Ca(OCl)2 + CaCl2 + 2 H 2O Hot-concentrated Sodium chlorate

8 NH3
Excess
+ 3 Cl 2 6 NH4Cl + N2
Phosgene - COCl2

+ 3 Cl 2 Tear gas - CCl3NO2


NH3
Excess
NCl3 +
Nitrogen trichloride
3 HCl

( Explosive) Mustard gas -( ClCH2CH2)2S

jksahu74@gmail.com Page | 175


Simple oxide Metal or non-metal having single oxidation state in their oxide.

Acidic Oxide Oxides of non-metals are acidic.These oxides produce acidic solution when combine with water.
SO 2 , Cl 2O 7 ,CO 2 ,N 2O 5 Water H2SO3 ,HClO 4 ,H2CO3 ,HNO 3

Basic Oxide Oxides of reactive metals are basic.These oxides produce alkaline solution when combine with water.
Na2O , CaO , BaO Water NaOH , Ca(OH) 2 , Ba(OH) 2
OXIDES

Neutral Oxide Neither acidic nor basic. CO , NO , N 2O , H2O

Amphoteric Oxide React both with aciods and alkalies. Al 2O3 , ZnO , PbO , SnO
3+
Al 2O 3
Base
+ 6 HCl + 9 H2O 2 [ Al ( H 2O ) 6 ] + 6 Cl-
Acid

Al 2O 3 + 6 NaOH + 3 H2O 2 Na3 [ Al ( OH ) 6 ]


Acid Base

Metal in higher oxidation state also have acidic nature. Mn O , CrO ,V O


2 7 3 2 5

Combination of two simple oxides with metal in different oxidation states.


Mixed oxide
Red lead Pb3O4 i.e. PbO 2. 2 PbO

Magnetic oxide Fe3O4 i.e. Fe 2O3. FeO

jksahu74@gmail.com Page | 176


Periods

Group-13 Group-14 Group-15 Group-16 Group-17 Group-18

B-5 C-6 N-7 O-8 F-9 Ne - 10


2
[He] 2s2 2p1 [He] 2s2 2p2 [He] 2s2 2p3 [He] 2s2 2p4 [He] 2s2 2p5 [He] 2s2 2p6

Al - 13 Si - 14 P - 15 S - 16 Cl - 17 Ar - 18
3
[Ne] 3s2 3p1 [Ne] 3s2 3p2 [Ne] 3s2 3p3 [Ne] 3s2 3p4 [Ne] 3s2 3p5 [Ne] 3s2 3p6

Ga - 31 Ge - 32 As - 33 Se - 34 Br - 35 Kr - 36
4
[Ar] 4s2 3d10 4p1 [Ar] 4s2 3d10 4p2 [Ar] 4s2 3d10 4p3 [Ar] 4s2 3d10 4p4 [Ar] 4s2 3d10 4p5 [Ar] 4s2 3d10 4p6

In - 49 Sn - 50 Sb - 51 Te - 52 I - 53 Xe - 54
5
[Kr] 5s2 4d10 5p1 [Kr] 5s2 4d10 5p2 [Kr] 5s2 4d10 5p3 [Kr] 5s2 4d10 5p4 [Kr] 5s2 4d10 5p5 [Kr] 5s2 4d10 5p6

Tl - 81 Pb - 82 Bi - 83 Po - 84 At - 85 Rn - 86
6
[Xe] 6s2 4f145d106p1 [Xe] 6s2 4f145d106p2 [Xe] 6s2 4f145d106p3 [Xe] 6s2 4f145d106p4 [Xe] 6s2 4f145d106p5 [Xe] 6s2 4f145d106p6

Number of vacant orbitals in Bismuth = { 52 - ( s - 1, p - 3, d - 5 ) } + { 62 - ( s - 1 , p - 3 ) } = 48

Number of vacant orbitals in Arsenic = 42 - ( s - 1 , p - 3 ) = 12

jksahu74@gmail.com Page | 177


The elements in which the last electron enters the d-orbital of the penultimate shell known as
d-block elements .
The general electronic configuration of d-block elements is (n−1) d 1–10 ns 0–2.
The general electronic configuration of the four series of d-block elements are :
3d series: [Ar] 3d1−10 4s1−2

4d series: [Kr] 4d1−10 5s0−2

5d series: [Xe] 5d1−10 6s2

6d series: [Rn] 6d1−10 7s2

d-block elements are called transition elements as they shows transition in the properties from the
most electropositive s-block elements to the less electropositive p-block elements.

Characteristics of Transition elements

Transition elements are defined as those elements which have partly or incompletely filled (n−1)d

orbitals in their elementary state or in any of their common oxidation states.

Facts Explanations
They exhibit several oxidation states due to low energy difference between ns and
(n-1)d subshell
They form coloured ions due to presence of unpaired electrons and d – d
transition
The elements and many of their compounds are due to presence of unpaired electrons
paramagnetic
The metal and their compounds act as catalyst due to their ability to exhibit multiple oxidation
state
They have the ability to form complexes due to highly charged ions and contain vacant
d-orbitals
They form alloys with each other due to similar sizes

Oxidation States

a) In transition element +1 is the minimum oxidation state (Shown by Cu, Ag and Hg) and +8 is the
maximum oxidation state (shown by Ru and Os). Zero oxidation state is found only incomplexes

i.e. [Ni(CO)4]

b) In first transition series , if in any oxidation state, d-sub shell has 0,5 or 10 electrons , that
oxidation state will be more stable. e.g. Ti4+(3d0) is more stable than Ti3+(3d1), Mn2+ is more
stable than Mn3+.

c) Cu2+ is more stable than Cu+ because of lower reduction potential which is due to higher
hydration energy.
jksahu74@gmail.com Page | 178
d) Highest oxidation state is shown in case of fluoride or oxide because fluorine and oxygen are
strong oxidising agent.
e) The maximum oxidation state increases down the group.
f) The maximum oxidation state shown by element which occurs in the middle of the series(Mn)
and lesser number of oxidation state shown by elements of extreme.
Ionization Energy

Ionisation energy increases from Sc to Zn. Cu and Cr have high IE due to stable electronic
configuration. The first ionisation energy of 5d element is higher than those of 3d and 4d element.
This is due to the greater effective nuclear charge acting on outer valence electron because of
poor shielding of the nucleus by 4f electron in 5d element.

Magnetic Property

The elements and their compounds of transition elements are generally paramagnetic due to

` a www
ww
ww
w
w
ww
w
`w
w
w
w
ww
w
w
ww
w
ww
w
ww
w
w
w
w
presence of unpaired electrons. The magnetic moment is due to spin angular moment and orbital
angular moment. Magnetic moment µ = q n n + 2 , Unit – Bohr Magneton, n- Number of
a

unpaired electrons.

Magnetic moment of Fe3+ ion ( 1s2 2s2 2p6 3s2 3p6 3d5 ), Number of unpaired electron = 5
` a w w
w
ww
w
w
w
ww
w
w
`w
w
w
w
ww
w
w
ww
w
w
w
ww
w
w
ww
w
So µ = q5 5 + 2 = 5.9 BM
a

Melting point

The melting point and boiling point of transition elements are very high due to stronger inter
particle bonds. In a period from left to right the melting point of the metal first increases to
maximum then gradually decreases towards the end of the period. ( Mn , Tc posses anomalous
MP) . The strength of inter particle bonds in transition elements is roughly related to the number of
half filled d-orbitals. In the beginning the numbers of half filled d-orbital increases till the middle of
the period then decreases. So the strength of bonds initially increases and at the end it is
minimum. So melting point and enthalpy of atomisation increases from left to middle and then
decreases to the right end in a period. As there are no unpaired electrons in Zn, Cd and Hg, So
they are soft and have low melting point.

Potassium dichromate (K2Cr2O7)


The oxidation state of chromium in chromate ( CrO4 2-) and dichromate ( Cr2O7 2-) is the same
(+6) They are interconvert able in aqueous solution depending upon PH of the solution.

2− Acid 2−
2 CrO4 + 2 H+ Cr2O7 + H 2O
Alkali
Yellow Orange

2− 2−
2 CrO 4 + 2 H+ Cr2O7
+ H2O

2− − 2−
Cr2O7 + 2 OH 2 CrO 4 + H2O

jksahu74@gmail.com Page | 179


-
O −- O
−-
O

Cr Cr Cr
-
O O O O
O
O O O

[ Chromate ion (Yellow Colour) ] [ Dichromate ion ( Orange colour ) ]


{Tetrahedral} { Two tetrahedral sharing one corner with
Cr – O – Cr bond angle 1200 }

Preparation of Potassium dichromate (K2Cr2O7)


K2Cr2O7 is prepared from chromite ore ( FeCr2O4 )
(1) Chromite ore is converted into sodium chromate by fusing with sodium carbonate in presence
of air.

4 FeCr2O4 + 8 Na2CO3
+ 7 O2 8 Na2CrO4 + 2 Fe2O3 + 8 CO2

(2) After filtration the yellow solution of sodium chromate is treated with concentrated sulphuric
acid to get sodium dichromate.

2 Na2CrO4
+ H2SO4 Na2Cr2O7 + Na2SO4 + H2O
Sodium chromate Sodium dichromate

(3) Hot concentrated solution of sodium chromate is treated with potassium chloride.
Then potassium dichromate , which is less soluble than sodium chromate,
crystallizes out on cooling as orange crystal.

Na2Cr2O 7
+ 2 KCl K 2Cr2O 7
+ 2 NaCl

Oxidising action of Potassium dichromate (K2Cr2O7)


2−
Cr2O7 + 14 H+ + 6 e− 2 Cr3+ + 7 H2O

Acidified potassium dichromate will oxidise

[1] Iodide in to Iodine



2I I2
+ 2 e− X3
2−
14 H+ 6 e− 2 Cr3+
[ Ionic ]
Cr 2O 7 + + + 7 H2O

Cr2O 2−
7 + 6 I− + 14 H + 2 Cr 3+ + 3 I2 + 7 H2O

K2Cr 2O 7 + 4 H2SO 4 K 2SO 4+ Cr2(SO4)3 + 4 H2O + 3 [O]


[Molecular] 2 KI + H2SO 4
+ [O] K2SO 4 + I 2
+ H2O X 3
K2Cr2O 7 + 7 H2SO 4
+ 6 KI 4 K2SO 4 + Cr 2(SO 4) 3 + 7 H2O + 3 I 2

jksahu74@gmail.com Page | 180


[2] Sulphides to sulphur

H2S 2 H+ + 2 e−
S + X3
2−
Cr2O7 + 14 H+ + 6 e− 2 Cr3+ + 7 H 2O [ Ionic ]
Cr2O 72−
+ 8 H+ + 3 H 2S 2 Cr 3+ + 7 H2O + 3 S

K2Cr2O 7 + 4 H2SO4 K2SO 4 + Cr2(SO 4)3


+ 4 H 2O + 3 [O]
[Molecular] +
H2S [O] H2O + S X3
K2Cr2O7
+ 4 H2SO4 + 3 H 2S K2SO 4
+ Cr2(SO4)3
+ 7 H 2O + 3S
[3] Tin (II) salt to Tin (IV) salt
2+
Sn4+

2−
Sn + 2e X3
Cr2O 7 + 14 H+ + 6 e− 2 Cr3+ + 7 H 2O
[ Ionic ]
2−
Cr2O 7 + 2+
3 Sn + 14 H+ 2 Cr
3+
+ 3 Sn4+ + 7 H 2O

K2Cr2O7 + 8 HCl 2 KCl + 2 CrCl 3 + 4 H 2O + 3 [O]


[Molecular]
SnCl 2 + 2 HCl + [O] SnCl4 + H2O X3
K2Cr2O7
+ 3 SnCl2 + 14 HCl 2 KCl + 2 CrCl 3 + 3 SnCl4 + 7 H2O
[4] Iron (II) salts to Iron (III) salts
2+ 3+
Fe Fe + e− X 6
Cr2O 2−
7 + 14 H+ + 6 e− 3+
2 Cr + 7 H2O [ Ionic ]
Cr2O 72− + 3+
14 H+ + 6 Fe
2+
2 Cr
3+
+ 6 Fe + 7 H 2O
K2Cr2O 7 + 4 H2SO 4 K2SO 4 + Cr2(SO 4)3
+ 4 H2O + 3 [O]
[Molecular] 2 FeSO 4 + H2SO4 + [O] Fe2(SO4)3 + H2O X 3

K2Cr2O 7 + 7 H2SO 4 + 6 FeSO 4 K2SO 4 + Cr2(SO 4)3 + 3 Fe2(SO 4)3 + 7 H2O

Potassium Permanganate (KMnO4)


2 KMnO 4 Heat
513K
K2MnO 4 + MnO 2 + O2

O O

Mn Mn
− −
O O O O

O O

[Permanganate ion ] [Manganate ion]


( Purple ) [ Diamagnetic ] ( Green )
[ Paramagnetic with one unpaired electron ]

jksahu74@gmail.com Page | 181


Preparation of Potassium Permanganate (KMnO4)
KMnO4 is prepared from pyrolusite ( MnO2 )
The mineral is fused with KOH in presence of air or an oxidising agent ( KNO3 , KClO3 , etc ) to
produce potassium mangante.
2 MnO 2
+ 4 KOH + O2 2 K 2MnO 4
+ 2 H 2O

K2MnO4 is converted into KMnO4 by any of the following methods:

(1) By acidification of K2MnO4 by H2SO4


3 K 2MnO4 + 2 H2SO4 2 K 2SO 4
+ 2 KMnO 4 + MnO 2
+ 2 H2 O

2− −
3 MnO 4
+ 4 H+ 2 MnO 4 + MnO 2
+ 2 H 2O (Ionic )
(2) By electrolytic oxidation of K2MnO4
+ −
H2O H + OH

2−
K2MnO 4 2 K+ + MnO4

2−
At anode:- MnO4 MnO 4− + e− X2

+ −
At cathode:- 2H + 2e H2

2− + −
2 MnO 4 + 2H 2 MnO4 + H2
Green Purple

2 K2MnO 4 + 2 H 2O 2 KMnO 4
+ H2
+ 2 KOH ( Molecular )

Laboratory Preparation of KMnO4


A manganese (II) ion salt is oxidised by peroxodisulphate to permanganate.
2+ 2− − 2− +
2 Mn + 5 S 2O 8 + 8 H 2O 2 MnO 4
+ 10 SO 4
+ 16 H

Oxidising action of Potassium Permanganate (KMnO4)



2+
MnO 4
+ 8 H+ + 5 e− Mn + 4 H 2O

Acidified Potassium permanganate will oxidise:-


[1] Iodide to iodine
2 I− I2
+ 2 e− X5 It is not possible to titrate
− HCl with KMnO4, because
MnO 4 + 8 H+ + 5 e− Mn2+ + 4 H 2O X2
HCl will oxidise to Cl2
2 MnO 4−
+ 10 I − + 16 H + 2 Mn2+ + 5 I2 + 8 H 2O

[2] Ferrous to ferric 2+ 3+


Fe Fe + e− X5

MnO 4 + 8 H+ + 5 e− Mn2+ + 4 H 2O
MnO−4 + 5 Fe2+ + 8 H+ 5 Fe
3+
+ Mn2
+
+ 4 H2 O

jksahu74@gmail.com Page | 182


[3] Oxalate ion or oxalic acid into carbondioxide

COO
10 CO 2 + 10 e −
5 −
COO

MnO 4 + 8 H+ + 5 e− Mn2+ + 4 H 2O X2
COO−
5 − +2 MnO 4

+ 16 H+ 10 CO 2 + 2 Mn2+ + 8 H2O
COO

[4] Sulphide into sulphur


2−
S S + 2 e− X 5
MnO 4

+ 5 e−
+ 8 H+ Mn 2+ + 4 H2O X2
5 S2 −
+ 2 MnO4− + 16 H + 5 S + 2 Mn2+ + 8 H2O
[5] Sulphurous acid / sulphite into sulphuric acid / sulphate
2−
+ H2O SO23 − SO 4 + 2 H+ + 2 e− X5

MnO 4 + 8 H + + 5 e − Mn2+ + 4 H2O X2
2−
+ 6 H+ 2+ 2−
5 SO 3 + 2 MnO−4 2 Mn + 5 SO 4 + 3 H 2O
[6] Nitrite to nitrate
− − +
NO 2 −

+ H2O NO3 + 2H + 2e X5
MnO 4 +
+ 8 H + 5 e− Mn2+ + 4 H2 O X2
5 NO 2− 2+
+ 2 MnO−4 + 6 H + 2 Mn + 5 NO 3

+ 3 H 2O

Neutral / Faintly alkaline Solution ( MnO4 − MnO 2 ) oxidises:


[1] Iodide to iodate:
− − −
I + 6 OH IO 3 + 3 H 2O + 6 e−

MnO 4 + 2 H 2O + 3 e− MnO 2 + 4 OH − X2
I− + 2 MnO 4− + H2O IO 3− + 2 MnO 2 + 2 OH −

[2] Thiosulphate to sulphate:


2− − 2−
S 2O 3
+ 10 OH 2 SO 4
+ 5 H 2O + 8 e − X 3

MnO 4 + 2 H 2O + 3 e− MnO 2 + 4 OH − X 8
2− 2−
8 MnO−4 + 3 S 2O 3 + H2O 8 MnO 2 + 6 SO 4 −
+ 2 OH
[3] Manganous salt is oxidised to MnO2 in presence of ZnSO4 or ZnO


Mn 2+ + 2 H2 O MnO 2 + 4 H+ + 2 e − X3
4 H+

MnO 4 + + 3e MnO 2 + 2 H 2O X 2
2+
3 Mn + 2 MnO 4− + 2 H2 O 5 MnO 2 + 4 H+

jksahu74@gmail.com Page | 183


The Inner Transition elements ( f – Block )
Lanthanoids ( 4f – Block ) 4 f 1 – 14 5 d 0 , 1 6 s2
Elements Ce Pr Nd Pm Sm Eu Gd Tb Dy Ho Er Tm Yb Lu

Z 58 59 60 61 62 63 64 65 66 67 68 69 70 71

M3+ =[Xe]4 F1 F2 F3 F4 F5 F6 F7 F8 F9 F10 F11 F12 F13 F14

All elements having no d – electrons in their penultimate shell , except the following:

Ce : [Xe]4f1 5d1 6s2 Gd : [Xe] 4f7 5d1 6s2 Lu : [Xe] 4f14 5d1 6s2

Lanthanum ( La : [Xe] 5d1 6s2 ) though a d – block element,

is included in the lanthanoid ( Ln ) series because it closely resembles lanthanoids.

The more stable oxidation state of lanthanoids is +3 .

So aqueous solution of Sm2+ , Eu2+ and Yb2+ are good reducing agents and aqueous solution of
Ce4+ and Tb4+ are good oxidising agents.

Lanthanoids show limited number of oxidation states because the energy gap between 4f and 5d
subshell is large.

All the lanthanoids except La3+ and Lu3+ contain unpaired electrons and hence are paramagnetic.

Misch metal is an alloy ,consists of lanthanoid (~95%) and Iron (~5%) and traces of S, C ,Ca and
Al. It is used to produced bullets, shell and lighter flint.

Lanthanoid Contraction
The regular decrease in the size of the atoms and ions with increasing atomic number is known as
lanthanoid contraction.

Cause : As the new electron is added to the f – subshell , there is imperfect shielding of one
electron by another in this subshell due to the shapes of these f – orbitals. This imperfect
shielding is unable to counterbalance the effect of the increased nuclear charge. Hence,
the net results is the contraction of atomic size.

jksahu74@gmail.com Page | 184


Consequences :

(1)The valence shell configuration of lanthanoids remains the same, because the electrons are
added into the 4f – subshell. Hence , they show similar chemical properties. Due to lanthanoid
contraction, the change in atomic or ionic radii of these elements is very small. This makes the
separation of anthanoid elements difficult.

(2) As the size of the lanthanoid ions decreases from La3+ to Lu3+, the covalent character of the
hydroxides increases ( Fajan rule ). Hence, the basic strength decreases from La(OH)3 to
Lu(OH)3.

(3)Similarity in the size of the atoms belonging to same group of second and 3rd transition series.

1st Transition series Sc21 (144 Ti22(132pm) V23(122pm)


pm)

2nd Transition series Y39 (180pm) Zr40(160pm) Nb41(146pm)

3rd Transition series La57 (187pm) LANTHANOIDS Hf72(159pm) Ta73(146pm)

Actinoids ( 5f – Block ) 5 f 1 – 14 6 d 0 , 1, 2 7 s2
Elements Th Pa U Np Pu Am Cm Bk Cf Es Fm Md No Lr

Z 90 91 92 93 94 95 96 97 98 99 100 101 102 103

M3+ =[Rn]5 F1 F2 F3 F4 F5 F6 F7 F8 F9 F10 F11 F12 F13 F14

All elements having no d – electrons in their penultimate shell , except the following:

Pa : [Rn]5f2 6d1 7s2 U : [Rn] 5f3 6d1 7s2 Np : [Rn] 5f4 6d1 7s2

Cm : [Rn]5f7 6d1 7s2 Lr : [Rn] 5f14 6d1 7s2

Th90 : [Rn] 6d2 7s2 , Last electron enter into d – orbital of (n – 1)d subshell. But ‘Th’ is included in

f – block, because Th3+ contain electron in (n – 2)f subshell. Th3+ : [Rn] 5f1

Actinium ( Ac89 : [Xe] 6d1 7s2 ) though a d – block element, is included in the Actinoid series
because it closely resembles Actinoids.

The chemistry of actinoids is much more complicated, beause they show a wide range of
oxidation states and they are radioactive in nature.

jksahu74@gmail.com Page | 185


Application of d – and f – block elements

Group – 11 elements are called coinage metal


UK copper coin – Copper coated steel
Silver UK coin – C / Ni alloy
Ziegler catalyst – TiCl4 with Al(CH3)3 for manufacture of polythene
PdCl2 – Wacker’s process for the oxidation of ethylene to ethanol
AgBr – Photography industry MnO2 – Dry battery cells
TiO – Pigment industry V2O5 – Manufacture of H2SO4
With reference to structural variability and chemical reactivity, write the differences
between lanthanoids and actinoids.

Lanthanoids Actinoids
The size of lanthanoids is smaller The size of actinoids is larger than that of
than that of actinoids. They also lanthanoids. They also exhibit actinoid
exhibit lanthanoid contraction which contraction which is larger than lanthanoids
Structural
is lesser than actinoids contraction. contraction. The valence electron present in
variability
The valence electron present in 4f – 5f – subshell.
subshell.
Atomic / ionic radii does not show Atomic / ionic radii show much variation /
much variation / +3 is the most Besides +3 oxidation state, they exhibit +4,
common oxidation state, in few +5, +6 and +7 also
cases +2 & +4
Lanthanoids are less reactive than Actinoids are more reactive than
actinoids because shielding effect of lanthanoids. They exhibit more number of
Chemical
reactivity 4f is stronger than 5f subshell. Also oxidation states than lanthanoids because
they exhibit lesser number of of less energy gap in 5f, 6d and 7s
oxidation state than actinoids due to subshells.
more energy gap in 4f , 5d and 6s
subshell.
They are quite reactive Highly reactive in finely divided state

jksahu74@gmail.com Page | 186


Property Lanthanoids Actinoids

Size The size of lanthanoids is smaller than that of actinoids. The size of actinoids is larger than that of lanthanoids.
Extent of They exhibit lanthanoid contraction which is lesser than They also exhibit actinoid contraction which is larger than
contraction actinoids contraction. lanthanoids contraction

Electronic The valence electron present in 4f – subshell. The valence electron present in 5f – subshell.
configuration

Radii Atomic / ionic radii does not show much variation Atomic / ionic radii show much variation

Reactivity Lanthanoids are less reactive than actinoids because Highly reactive in finely divided state
shielding effect of 4f is stronger than 5f subshell

Oxidation states Show mainly +3 Oxidation state except in a few cases Show wide range of oxidation states due to the fact that the
where it is +2 and +3 (They exhibit lesser number of 5f,6d and 7s energy levels are of comparable energies.
oxidation state than actinoids due to more energy gap in Therefore all these three sub shells can participate. Common
4f , 5d and 6s subshell.) Oxidation States is +3 Th(+4),Pa(+5), U (+6) ,Np(+7)

Binding energy Binding energies of 4f are higher Binding energies of 5f are lower

shielding effect 4f electrons have greater shielding effect as compared to 5f electrons have poor shielding effect as compared to 4f .
5f . Therefore , the contraction in ionic radii is less . Therefore , the contraction in ionic radii is more .

complexes Tendency to form complexes is less . They have greater tendency to form complexes .
Basic character Lanthanoid compounds are less basic Actinoid compounds are more basic

oxo ions They do not form oxo ions They form oxo ions such as UO2+ ,NpO2+ , PuO2+,UO2+

Radioactivity Except promethium , these are non-radioactive All the acinoids are radioactive

Colours Most of their ions are colourless. But exceptions are there The ions are coloured e.g. U3+(red) , U4+(green) ,UO2+(yellow)

Paramagnetic They are paramagnetic and their magnetic properties can They are also paramagnetic but their magnetic properties
character be easily explained cannot be easily explained

jksahu74@gmail.com Page | 187


Normal Salt It is formed by replacement of all replaceable hydrogen atoms of an acid by the
metal atoms. NaCl , K2SO4 , NaNO3

Double Salt These are molecular compounds which exist only in crystal lattice but break
down into their constituent components when dissolved in water or in any other
solvent.
K2SO4 . Al2(SO4)3 . 24H2O FeSO4 . (NH4)2SO4 . 6 H2O KCl . MgCl2 . 6 H2O

Potash alum Mohr Salt Carnalite

Coordination Compound A molecular compound resulting from the combination of two or


more stable molecular compounds and retains its identity in the solid state as well as in
solution and its properties are totally different from the constituents present.
Fe(CN) 2 Evaporation K4[Fe(CN) 6]
(aq) + 4 KCN
(aq) (aq)

Fe2+ + 2 CN − K+ + CN − 4 K+ + [ Fe (CN)6 ]4−

Terms in Coordination Compounds

Coordination entity or sphere

Counter ion Ligand ( Lewis base )


K4 [ Fe (CN) 6
]
Coordination number

Central metal atom or ion( Lewis acid )

Oxidation Number The residual charge remained on the central metal atom after removal of
all the ligands or counters ions as their respective ions, is known as
oxidation number.

Homoleptic : The central metal atom is bonded with only one kind of donor groups.
[Co(NH3)6]Cl2

Heteroleptic: The central metal atom is bonded with more than one kind of donor groups.
Co(NH3)4Cl2]Cl

Coordination Polyhedron The spatial arrangement (octahedral, square planar or tetrahedral )


of the ligands around the central metal ion is known as
coordination polyhedron.
CN
CN CN
K4 Fe

CN CN
CN

jksahu74@gmail.com Page | 188


L
L L
L L L
M M
M
L L L L
L L L
Tetrahedral Square planar Square pyramidal
L L
L L
L
L M M

L L L
L L

Trigonal bipyramidal Octahedral

Ligands : Ligands are Lewis bases ( ionic / neutral) form coordinate bond with central metal
atom (Lewis acid) Number of ligating groups ( No. of donor atoms) present in a
ligand is called denticity

Unidentate ligand:
xx xx
CN − , OH −
, CH3COO − , H2O
xx , NH3

Didentate Ligand :
xx xx
H2N CH2 CH2 NH2 COO−
2−
( ethane-1,2-diamine) i.e ( en) COO− i.e C2O4 ( Oxalato ) i.e. (ox)

Hexadentate ligand :
Ethylenediamine tetra acetate ion i.e. EDTA4-
CH 2 COO

xx
H2C N

CH 2 COO
xx

CH 2 COO
H2C N
CH 2 COO

Ambidentate ligand: Unidentate ligand which can ligate through two different atoms is called
ambidentate ligand

- SCN thiocyanato - NCS isothiocyanato

- NO2 nitrito – N - - ONO nitrito – O –

Chelate ligand : When a di- or polydentate ligand uses its two or more donor atoms to bind a
single metal ion, is called chelate ligand and the complex is called chelate
complex which is more stable.

COO

M

COO

jksahu74@gmail.com Page | 189


Formula Writing : Formula of the cation whether simple or complex is written first followed by that
of the anion.
Cation – Anion

Within the coordination sphere, the symbol of the metal atom is written first followed by the
ligands arranged alphabetically.

[Metal ( Ligands sorted alphabetically )]

K3[Co(CN)5(NO) ] [ Co(NH3)4Cl(NO2)]NO3 K [ Pt (NH3) Cl3] [ Cr (H2O)4Cl2 ] NO3

Nomenclature of Coordination compounds:

(i) Name should not start with a capital letter and should one word without any gap for complex .

(ii) Cation + Anion ; ( whether simple or complex )

(iii) For complex part Ligands alphabetically + Metal ( O.N )

(iv) If the anion part is complex, the metal ends with ' ate

Metal Cation / Neutral Anion Metal Cation / Neutral Anion


Cr Chromium Chromate Pt Platinum Platinate
Mn Manganese Manganate Ag Silver Argenate
Fe Iron Ferrate Au Gold Aurate
Co Cobalt Cobaltate Pb Lead Plumbate
Ni Nickel Nickelate Sn Tin Stannate
Cu Copper Cuprate Pd Palladium Palladate
Zn Zinc Zincate Ti Titanium Titanate

(v) Mono,di,tri are used to indicate the number of individual simple ligands.
Bis, tris and tetrakis are used inplace of di, tri and tetra for the ligands of complex name
i.e. name having more than one word.

(vi) Neutral ligands are named as :


H2O aqua NH3 ammine CO carbonyl CS Thiocarbonyl NO nitrosyl

P(Ph)3 triphenyl phosphine CH3NH2 methylamine

H2N – CH2 – CH2 – NH2 ethane – 1,2 – diamine (en)

(Vii) Anionic ligands are named as :


fluorido F- chlorido Cl- bromido Br- iodido I-
-
thicyanato SCN cyano CN- oxalato (ox) C2O42- imido NH2-
-
isothiocyanato NCS hydroxo OH- oxo O2- amido NH2-
-
nitrito – O - ONO nitrato NO3- carbonato CO32- sulphato SO42-
-
nitrito – N - NO2 nitrido N3- acetato CH3COO- sulphido S2-

jksahu74@gmail.com Page | 190


IUPAC names of the following coordination compounds
[Co(NH3)4(H2O)Cl]Cl2 tetraammineaquachloridocobalt(III) chloride

K2[Zn(OH)4] potassium tetrahydroxozincate(II)

K3[Al(C2O4)3] potassium trioxalatoaluminate(III)

[CoCl2(en)2]+ dichloridobis(ethane-1,2-diamine)cobalt(III)

[Pt(NH3)2Cl(NO2)] diamminechloridonitrito-N-platinum(II)

K3[Cr(C2O4)3] potassium trioxalatochromate(III)

[CoCl2(en)2]Cl dichloridobis(ethane-1,2-diamine)cobalt(III) chloride

[Co(NH3)5(CO3)]Cl pentaamminecarbonatocobalt(III) chloride

Hg[Co(SCN)4] mercury tetrathiocyanatocobaltate(III)

[Co(NH3)4(H2O)2]Cl3 tetraamminediaquacobalt(III) chloride

K2[Ni(CN)4] potassium tetracyanonickelate(II)

[Cr(en)3]Cl3 tris(ethane–1,2–diamine) chromium(III) chloride

[Pt(NH3)BrCl(NO2)]– amminebromidochloridonitrito-N-platinate(II)

[PtCl2(en)2](NO3)2 dichloridobis(ethane–1,2–diamine)platinum(IV) nitrate

Fe4[Fe(CN)6]3 iron(III) hexacyanoferrate(II)

[Co(NH3)6]Cl3 hexaamminecobalt(III) chloride

[Co(NH3)5Cl]Cl2 pentaamminechloridocobalt(III) chloride

K3[Fe(CN)6] potassium hexacyanoferrate(III)

K3[Fe(C2O4)3] potassium trioxalatoferrate(III)

K2[PdCl4] potassium tetrachloridopalladate(II)

[Pt(NH3)2Cl(NH2CH3)]Cl diamminechlorido(methylamine)platinum(II) chloride

[Zn(OH)4]2- tetrahydroxozincate(II)

K2[PdCl4] potassium tetrachloridopalladate(II)

[Pt(NH3)2Cl2] diamminedichloridoplatinum(II)

K2[Ni(CN)4] potassium tetracyanonickelate(II)

jksahu74@gmail.com Page | 191


[Co(NH3)5(ONO)]2+ pentaamminenitrito-O-cobalt(III)

[Co(NH3)6]2(SO4)3 hexaamminecobalt(III) sulphate

K3[Cr(C2O4)3] potassium tri(oxalato)chromate(III)

[Pt(NH3)6]4+ hexaammineplatinum(IV)

[CuBr4]2– tetrabromidocuprate(II)

[Co(NH3)5(NO2)]2+ Pentaamminenitrito-N-cobalt(III)

[Co(NH3)6]Cl3 hexaamminecobalt(III) chloride

[Pt(NH3)2Cl(NH2CH3)]Cl diamminechlorido(methylamine)platinum(II)chloride

[Ti(H2O)6]3+ hexaaquatitanium(III)

[Co(NH3)4Cl(NO2)]Cl tetraamminechloridonitrito-N-cobalt(III)chloride

[Mn(H2O)6]2+ hexaaquamanganese(II)

[NiCl4]2– tetrachloridonickelate(II)

[Ni(NH3)6]Cl2 hexaamminenickel(II)chloride

[Co(en)3]3+ tris(ethane–1,2–diamine)cobalt(III)

[Ni(CO)4] tetracarbonylnickel(0)

K[Cr(H2O)2(C2O4)2].3H2O potassium diaquadioxalatochromate(III) trihydrate

[Co(NH3)5Cl]Cl2 pentaamminechloridocobalt(III) chloride

[ CrCl3(py)3 ] trichloridotripyridinechromium(III)

Cs[FeCl4] caesium tetrachloridoferrate(III)

K4[Mn(CN)6] potassium hexacyanomanganate(II)

K3[Co(C2O4)3] potassium trioxalatocobaltate(III)

cis-[Cr(en)2Cl2]Cl cis - bis(ethane–1,2–diamine)dichloridochromate(III)


chloride
(NH4)2[CoF4] ammonium tetrafluoridocobaltate(II)

[Cr(NH3)3(H2O)3]Cl3 triamminetriaquachromium(III) chloride

[Co(H2NCH2CH2NH2)3]2(SO4)3 tris(ethane-1,2–diammine)cobalt(III) sulphate

[Ag(NH3)2][Ag(CN)2] diamminesilver(I) dicyanoargentate(I)

jksahu74@gmail.com Page | 192


Werner's theory of coordination compounds

According to this theory :


(i) In coordination compounds metals show two types of linkages (valences) i.e. primary and
secondary
(ii) The primary valences are normally ionisable and satisfied by negative ions. The primary
valence is represented by oxidation number.
(iii) The secondary valences are non-ionisable. These are satisfied by ligands ( neutral molecules
or negative ions). It is represented by cordination number and is fixed for a metal.
(iv) The secondary valence describe the characteristic spatial arrangement and shape of
compound.

Coordination compound (aq) + Silvernitrate (aq) Silver chloride Complex formula


1 Mole Excess No. of moles

CoCl3.6 NH3 AgNO3 3 [Co(NH3)6]Cl 3

CoCl3.5 NH3 AgNO3 2 [Co(NH3) 5Cl]Cl 2

CoCl3.4 NH3 AgNO3 1 [Co(NH3)4Cl2]Cl

CoCl3.3 NH3 AgNO3 No ppt. [Co(NH3)3Cl3]

Isomerism
Compounds having same molecular formula but different physical and chemical properties, which
is due to different arrangement of atoms is known as isomers and this phenomenon is known as
isomerism.

Stereo Geometrical + Optical


Isomerism

Structural Ionisation + Solvated + Coordination + Linkage

Ionisation isomerism
Isomers give different ions in solution.
This type of isomerism arises when counter ion is itself a potential ligand

[Co(NH3)5Br] SO 4 Aq. BaCl2


White ppt. of BaSO4
( Red-violet)
[Co(NH3)5SO 4]Br Aq. AgNO3
Yellow ppt. of AgBr
(Red)

jksahu74@gmail.com Page | 193


Solvated isomerism
Isomers having different number of solvent molecules in coordination sphere

[Cr(H2O) 5Cl]Cl 2.H2O Anhydrous CuSO4


Blue CuSO4. 5 H 2O
( Grey-green)
Anhydrous CuSO4
[Cr(H2O) 6]Cl3 No blue colour
( Violet)

Coordination isomerism
This type of isomerism arises due to inter change of ligands between cationic and anionic entities
present in the complex.
[ Co(NH3)6] [Cr(CN)6] and [ Cr(NH3) 6] [Co(CN)6]

[Pt(NH3)4] [PtCl 4] and [Pt Cl(NH 3) 3] [Pt Cl 3(NH3)]

Linkage isomerism
This arises due to presence of ambidentate ligands

[Co(NH3)5NO2]Cl 2
and [Co(NH3)5(ONO)]Cl2
pentaamminenitrito-N-cobalt(III) pentaamminenitrito-O-cobalt(III)
(Yellow) ( Red)

[Mn(CO)5SCN] [Mn(CO)5NCS]
and
pentacarbonylthiocyanatomanganese(I) pentacarbonylisothiocyanatomanganese(I)

Geometrical Isomerism
Heteroleptic complexes with coordination number equal to 6 or 4 ( square planar )show
geometrical isomeris.

[MX 2L2] type [PtCl 2(NH3)2]


X X X L Cl Cl Cl NH3

M and M Pt and M

L Cis− L L X H3 N Cis− NH3 H 3N


Trans− Trans− Cl
( Adjacent) ( Opposite ) ( Pale- yellow )
(Dark- yellow )

jksahu74@gmail.com Page | 194


[MX2L4] type [ Co(NH3)4Cl2]
X X Cl Cl
L L L X H3N H 3N
NH3 Cl
M M Co Co
L L L L H3N H3N
NH3 NH3

X L Cl NH3
Trans- Cis-
Trans- Cis-
Cl Cl

Cl

en Co en en Co
[CoCl2(en)2]

Cl en
Cis-
Trans-

[ M A 3 B3 ] type
A
A
NH3
B A NH3
B B O2N NH3
O2N NO2
M
M Co
Co
B A O2N
B NH3 O 2N
A NH3

B NO2
NH3
A
fac-isomer fac-isomer
mer-isomer mer-isomer
( Cis - form ) ( Trans - form ) [ Co(NH 3)3(NO 2)3]
Fac- means Facial ,that is three same type of ligands occupy adjacent positions at the corner
i.e. at the trigonal face of the octahedron.

Mer - means Meridional, that is three same type of ligands occupy position around the meridian of
the octahedron.
Geometrical isomerism is shown by the above mentioned types of complexes. But not shown by
the complexes with coordination number 4 ( tetrahedral), squre planar complexes of MA3B and
MA4 and coordination number 6 ( MA6 or MA5B) because the spatial arrangement of ligands are
equivalent.

Optical Isomerism ( C.N = 6 )


[1] Optical isomers are non-superimposable mirror images.

[2] Enantiomers- Optical isomers are mirror image of each other.

[3] Chiral species - The species that can not superimpose with its mirror image.
jksahu74@gmail.com Page | 195
[4] Dextrorotatory (d-) - The optical isomers rotate the plane of polarised light towards right in
the polarimeter.
[5] Laevorotatory ( l -) - The optical isomers rotate the plane of polarised light towards left in
the polarimeter.
Cl Cl
'Trans' will not show optical isomerism due to
Cl
presence of plane of symmetry.
en Co en en Co

Only 'cis' isomer will show optical activity.


Trans- Cis-
Cl en
Cl
Cl en en
Cl Cl

en Co Co en en Co
Co en

en en en en
dextro- laevo- dextro- [Co(en) 3] laevo-
Cis- [CoCl 2 (en) 2 ]

Valence Bond Theory ( VBT ) According to this theory :-

[1] The metal atom or ion under the influence of ligands can use its (n-1)d, ns, np OR ns, np, nd
orbitals for hybridisation to yield a set of hybrid orbitals.

[2] Metal hybrid orbitals (empty) overlap with the ligand orbitals (filled) containing the lone pair of
electrons and results in the formation of ligands-metal coordinate bond.

[3] If the inner d- orbitals i.e. (n-1)d- sub shell is used in hybridistion, the complex is called inner
orbital complex or low spin complex or spin paired complex. This type of complex involving
d2sp3 hybridisation.
If the outer d-orbital i.e. nd - subshell is used in hybridisation, the complex
is called outer orbital complex or high spin complex or spin free complex. This type of complex
involving sp3d2 hybridisation.

[4] The geometry of the complex can be predicted from its magnetic behaviour.

Explain the diamagnetic nature of [Co(NH3)6]3+ on the basis of VBT.


Co 3+ [Ar] 3d6 3d6 4S 4P
In absence of ligands
With the approach of ligands

These orbitals undergo d2sp3 hybridisation


x x x x x x
After hybridisation x x x x x x

Six NH3 will form six M L coordinate bonds NH3 NH3 NH3 NH3 NH3 NH3
Geometry - Octahedral Diamagnetic due to absence of unpaired electrons
jksahu74@gmail.com Page | 196
Explain the paramagnetic nature of [CoF6]3- on the basis of VBT.
F- is weak ligand and can not cause pairing of electrons.
Co 3+ [Ar] 3d6
In absence of ligands 3d6 4S 4P 4d

With the approach of ligands there is no pairing of electrons

These orbitals undergo sp3d2 hybridisation


After hybridisation
x x x x x x
x x x x x x

Six F - form six coordinate bonds. F− F −


F− F −
F− F −

Paramagnetic due to presence of four unpaired electrons Geometry - Octahedral

Limitations of VBT
This theory does not :-

[1] give quantitative interpretation of magnetic data

[2] explain the colour of the coordination compound

[3] explain thermodyanamic and kinetic stability.

[4] explain exact geometry of molecule

[5] distinguish between weak and strong ligands

Crystal Field Theory ( CFT )


It makes the following assumptions:

[1] Ligands are treated as point charges in case of anion and dipoles in case of neutral molecules

[2] The interactions between the metal ions and the ligands are purely electrostatic
i.e. ionic in nature.

[3] The five d- orbitals in an isolated gaseous metal atom or ion are degenerate

[4] The degeneracy is maintained if a spherically symmetrical field of negative charges surrounded
the metal atom or ion.

[5] In a complex , the field produced by the ligands is not spherically symmetrical, so the d-orbitals
are not equally affected by the field and hence it results in splitting of the d-orbital energies.

jksahu74@gmail.com Page | 197


Crystal Field splitting in Octahedral Coordination Entities
[1] In octahedral complex, the six ligands are approaching the central metal atom along x,y and z
axes.

[2] Since the lobes of the two 'eg' orbitals ( d x2 - y2, dz2 ) lie directly along the path of the
approaching ligands, so 'eg' orbitals experience more repulsion and will be raised in energy.

[3] The t2g orbitals ( dxy,dyz,dzx ) which are directed between the axes will be lowered in energy
relative to the average energy in the spherical crystal field.
(dx 2-y2, dz 2 )

eg
3 ∆ο
5
∆ο
Bary centre 2
( spherical field) ∆ο
Energy

5
t2g
Free metal ion ( dxy,dyz,dzx )
( Octahedral field )
Bary centre: The energy of d- orbitals of a transition metal ion in a spherical crystal field,
generated by a set of ligands. In bary centre the five d- orbitals are degenerate

Crystal Field Stabilisation Energy ( CFSE ) in Octahedral Coordination Entities

CFSE is the stability that results from placing a transition metal ion in the crystal field generated
by a set of ligands.

If CFSE in an octahedral field is ∆o , then 't2g' orbitals are stabilised relative to barycentre by
(2/3) ∆ o and 'eg' orbitals are destabilised (3/5) ∆ o .

For octahedral complex, putting electrons in 'eg' orbitals reduce the amount of CFSE

Case – 1 : d4 ion in strong field

∆ o > P ( P- Pairing energy, energy required for pairing electrons in an orbital )

Electrons will be paired

Electronic configuration : t2g4 eg0


f g
2f
ff
∆o = f
8f
f
CFSE = 4 ∆
5 5 o
This is energetically favorable

Case- 2: d4 ion in weak field ∆o < P

Electrons will jump to 'eg' orbital

Electronic configuration : t2g3 eg1


jksahu74@gmail.com Page | 198
f g
CFSE = 3 f
2f
f
∆ @f
3ff
∆ = f
3f
f

5 o 5 o 5 o
This is energetically less favorable

Strong field - Low spin which is known as spin paired.

Weak field - High spin which is known as spin free

d4 to d7 coordination entities are more stable for strong field as compared to weak field cases.

Because in weak field, number of electrons in 'eg' orbital increases and value of CFSE
decreases and stability of complex decreases.

Crystal Field splitting in Tetrahedral Coordination Entities


[1] The four ligands in tetrahedral complexes are approaching the central metal atom in between
the three axes i.e. x,y and z.

[2] t2g orbitals ( dxy, dyz, dzx ) will experience greater force of repulsion and destabilized by f
2ff

5 t
relative to bary centre and energy of 'eg' orbitals ( dx2-y2 , dz2 ) will stabilized by f
3f
f
∆ relative
5 t
to bary centre.
( dxy,dyz,dzx )
t2g
2
5
∆t

Bary centre ∆t 3 ∆t
( spherical field)
Energy

eg
Free metal ion
2 2 2
(dx -y , dz )
( Tetrahedral field )

In tetrahedral complex, since the ligands do not directly approaching the d-orbitals as in
octahedral complex and there are only four ligands in tetrahedral complex, while in octahedral
complex there are six ligands, so the tetrahedral orbital spliting ( ∆t ) is less than octahedral
orbital spliting ( ∆o )

4f
f
ff
∆o > ∆t That is ∆ t = ∆
9 o

In tetrahedral complex low spin configuration is rarely observed, because the orbital spliting
energy ( ∆t ) is not sufficiently large for forcing electrons to be paired. So electrons remain
unpaired and shows high spin configuration by decreasing CFSE.

jksahu74@gmail.com Page | 199


Spectrochemical series
A series of ligands arranged in the order of increasing field strength is termed as spectrochemical
series.

I-< Br-< SCN-< Cl-< F-< OH-< < H O < NCS-< EDTA < NH3 < en < CN-< CO
C 2O 2−
4 2
4−

Weak Strong
Halide donor < O- Donor < N- Donor < C- Donor

The ligands which cause only a small crystal field splitting are called weak field ligands and while
those responsible for large splitting are known as strong field ligands.
Strong field ligands form low spin complex. Weak field ligands form high spin complex.

Factors Affecting Magnitude of CFSE ( ∆ )


[1] Higher the oxidation state of the metal ion greater is the value of ∆

[2] ∆ Value of metal ion increases in the order 3d < 4d < 5d series.

[3] Geometry of coordination entity ∆o > ∆t

[4] Nature of ligand ( ∆ of stronger ligand ) > ( ∆ of weaker ligand)

Limitation of CFT
[1] It is considered that ligands are point charges, So anionic ligands should have the greatest
splitting effect. But anionic ligands are present at the low end of the spectriochemical series.
[2] It does not take into account the covalent character of bonding between the ligand and the
central atom.

Structure of Homoleptic Metal carbonyl


CO CO
CO
OC
CO CO
Ni OC Fe Cr
CO OC
OC CO CO
CO
CO CO
Ni(CO) 4
Fe(CO) 5 Cr(CO) 6
CO CO
CO CO CO
CO OC
CO
OC Mn Mn CO Co Co

OC OC OC CO
CO CO OC CO
[Mn 2(CO) 10] [Co2(CO) 8]

jksahu74@gmail.com Page | 200


Synergic bonding in metal carbonyl
The metal carbon bond in the metal carbonyl possess both sigma and pi character.
[1] Metal-carbon σ -bond is formed by the donation of the lone pair of electron on the carbonyl
carbon into a vacant orbital of the metal.
[2] Metal-carbon π -bond is formed by the donation of a pair of electrons from a filled d-orbital
of the metal into the vacant antibonding π * orbital of carbon monoxide.
π π π∗

M σ C O
π

Due to this synergic effect, the bond strength between metal and carbon monoxide become
stronger.

Stability constant ( β )
Stability of a complex in solution means the measure of resistance to the replacement of a ligand
by some other ligand. This stability can be expressed in terms of equilibrium constant of the
following reaction.
[ML]
M + L ML K1 =
[M] [L]

[ML2]
ML + L ML2 K2 =
[ML] [L]
[ML 3]
ML2 + L ML3 K3 =
[ML 2] [L]
-----

-----

[MLn]
MLn-1
+L MLn Kn =
[MLn-1] [L]

M + nL MLn βn=
[MLn]
[M] [L] n

Stability of a coordination compound [MLn] is measured in terms of its stability constant.


The stepwise and overall constants are related as β n = K1. K2. K3. ------- Kn
The successive stability constants decreases K1 > K2 > K3 > -------- > Kn
The higher the oxidation state of the metal, the more stable are the complexes that it form with a
given ligand.
The instability constant or the dissociation constant of the coordination compounds is defined as
the reciprocal of the formation constant.
1f
f
ff
Instability constant =
β
Gem stones
Emerald : Cr3+ ions occupy octahedral sites in the mineral beryl ( Be3Al2Si6O18)

Ruby : Cr3+ ions occupy octahedral sites in alumina (Al2O3 )


jksahu74@gmail.com Page | 201
Application of coordination compounds
[1] [Rh Cl (PPh3)3] Chloridotris(triphenylphosphine)rhodium(I) is known as Wilkinson catalyst.
It is used as homogeneous catalyst for the hydrogenation of alkene.

[2] Electroplating :-Ag, Au, and Cu can be used for plating from solutions of their cyano complex
ions

Anode - Plate of Ag / Au / Cu Cathode - Object to be plated

At anode : At cathode:
2 CN −

Ag + [Ag(CN)2] + e− [Ag(CN)2] − + e− Ag + 2 CN

Au + 2 CN − [Au(CN)2] − + e− −
[Au(CN)2]
+ e− Au + 2 CN −
4 CN − [Cu(CN)4] 3- + e− [Cu(CN)4] 3- + e −
Cu + Cu + 4 CN −

[3] In photography: In black and white photography, the developed film is fixed by washing with
hypo ( sodiumthiosuphate) solution, which dissolves the undecomposed silverbromide (AgBr) to
form complex ion [Ag (S2O3)2]3-
2- 3- -
AgBr (s) + 2 S 2O 3 [Ag(S 2O 3)2]
+ Br

[4] In metallurgy: Gold and silver are extracted by converting them into their soluble cyanide
complex.
− −
4 Au + 8 CN + O2 + 2 H2O 4 [Au(CN)2] + 4 OH

Ag2S + 4 NaCN 2 Na[Ag(CN)2] + Na2S

The metals are then obtained from the soluble complex by treating with zinc metal
− 2-
4 [Au(CN)2] + Zn [Zn(CN) 4]
+ 4 Au

2 Na[Ag(CN)2]
+ Zn Na2[Zn(CN)2] + 2 Ag

[5] Purification of metal : A metal can be purified through formation of coordination compounds
followed by decomposition.
Ni + 4 CO Ni(CO) 4 ∆ Ni + 4 CO

[6] For determining hardness of water: For determing the hardness of water , the titrations are
performed using Na2EDTA2- in buffer solution to estimate the amount of Ca2+ and Mg2+ ions. The
difference in the stability constants are permit the selective estimation of different ions.

[7] In medicine: Vitamin B12 (Cyanocobalamine) is a complex of cobalt and is used to prevent
Anaemia. Cisplatin ( cis-diamminedichloridoplatinum is used for the treatment of Cancer

Cl NH 3

Pt

Cl NH 3
cisplatin

EDTA is used in the treatment of lead poisoning


jksahu74@gmail.com Page | 202
Excess of metals such as copper and iron in plants and animal system are recovered by
chelating ligands D-Penicillamine and desferrioxime-B via the formation of coordination
compounds.
2x = 3y
x + y = ∆ο
x
2ffff
=> x + x =∆o
∆ο 3
y 5ffff
=> x = ∆o
3

=>
3ffff
x = ∆o = 0.6 ∆o
5
The spin only magnetic moment of [MnBr4]2– is 5.9 BM. Predict the geometry of the complex
ion ?
2
n (n+2) = 5.9 n (n+2) = ( 5.9 )2 n2 + 2n = 34.81

n2 + 2n −35 = 0 n2 + 7n −5n −35 = 0 (n+7) ( n-5 ) = 0

n = -7 or + 5 n = - 7 is not possible, so number of unpaired electrons is five ( n = 5 )

Mn 2+
[Ar] 3d5 3d5 4S 4P
In absence of ligands
With the approach of ligands there is no pairing of electrons, because there should 5 unpaired electrons

These orbitals undergo sp3 hybridisation

After hybridisation x x x x
x x x x

Four Br− will form four coordinate bonds. Br −


Br − Br −
Br −

Since hybridisation is SP3 , so the geometry is tetrahedral

Since the coordination number of Mn2+ ion in the complex ion is 4, it will be either tetrahedral (sp3
hybridisation) or square planar (dsp2 hybridisation). But the fact that the magnetic moment of the
complex ion is 5.9 BM, it should be tetrahedral in shape rather than square planar because of the
presence of five unpaired electrons in the d orbitals.
Explain on the basis of valence bond theory that [Ni(CN)4]2– ion with square planar
structure is diamagnetic and the [NiCl4]2– ion with tetrahedral geometry is paramagnetic.
In [Ni(CN)4]2– , Ni in +2 oxidation state . CN- ion is a strong ligand and cause pairing of unpaired
electrons.

jksahu74@gmail.com Page | 203


Ni 2+ [Ar] 3d8

3d8 4S 4P
In absence of ligands

With the approach of ligands

These orbitals undergo dsp2 hybridisation


x x x
After hybridisation x
x x x x

Four CN- will form four coordinate bonds. CN −


CN −
CN − CN −

Diamagnetic due to absence of unpaired electrons Geometry -Square planar


In [NiCl4]2– Ni in +2 oxidation state . Cl- ion is a weak ligand and cause no pairing of unpaired
electrons.
Ni 2+ [Ar] 3d8
3d8 4S 4P
In absence of ligands

With the approach of ligands there is no pairing of electrons

These orbitals undergo sp3 hybridisation

x x x x
After hybridisation x x x x

Four Cl- will form four coordinate bonds. Cl −


Cl −
Cl −
Cl −

Paramagnetic due to presence of two unpaired electrons Geometry -Tetrahedral


Discuss the nature of bonding in the following coordination entities on the basis of valence
bond theory: (i) [Fe(CN)6]4– (ii) [FeF6]3– (iii) [Co(C2O4)3]3– (iv) [CoF6]3–

(i) CN- is strong ligand, pairing of electrons will occur.

Fe 2+ [Ar] 3d6 3d6 4S 4P


In absence of ligands
With the approach of ligands

These orbitals undergo d2sp3 hybridisation


x x x x x x
After hybridisation x x x x x x

Six CN − will form six M L coordinate bonds L L L L L


L
Geometry - Octahedral Diamagnetic due to absence of unpaired electrons
jksahu74@gmail.com Page | 204
(ii) F- is weak ligand and can not cause pairing of electrons.

Fe 3+ [Ar] 3d5
In absence of ligands 3d5 4S 4P 4d

With the approach of ligands there is no pairing of electrons

These orbitals undergo sp3d2 hybridisation


After hybridisation
x x x x x x
x x x x x x

Six F - form six coordinate bonds . F −


F −
F −
F −
F −
F −

Paramagnetic due to presence of five unpaired electrons Geometry - Octahedral

(iii) C2O42- is strong ligand, pairing of electrons take place.

Co 3+ [Ar] 3d6 3d6 4S 4P


In absence of ligands
With the approach of ligands

These orbitals undergo d2sp3 hybridisation


x x x x x x
After hybridisation x x x x x x

Three ox 2− will form six M L coordinate bonds L L L L L


L
Geometry - Octahedral Diamagnetic due to absence of unpaired electrons

(iv) F- is weak ligand and cannot cause pairing of electrons.

Co 3+ [Ar] 3d6
In absence of ligands 3d6 4S 4P 4d

With the approach of ligands there is no pairing of electrons

These orbitals undergo sp3d2 hybridisation


After hybridisation
x x x x x x
x x x x x x


Six F - form six coordinate bonds. F F− F −
F −
F −
F −

Paramagnetic due to presence of four unpaired electrons Geometry - Octahedral

jksahu74@gmail.com Page | 205


[Cr(NH3)6]3+ is paramagnetic while [Ni(CN)4]2– is diamagnetic. Explain why?
(i) [Cr(NH3)6]3+ is paramagnetic because:
Cr 3+ [Ar] 3d3 3d3 4S 4P
In absence of ligands

With the approach of ligands


These orbitals undergo d2sp3 hybridisation

x x x x x x
After hybridisation x x x x x x

Six NH3 will form six M L coordinate bonds NH3 NH3 NH3 NH3 NH3 NH3

Geometry - Octahedral Paramagnetic due to absence of unpaired electrons


(ii) [Ni(CN)4]2– is diamagnetic because:

Ni 2+ [Ar] 3d8
3d8 4S 4P
In absence of ligands
With the approach of ligands

These orbitals undergo dsp2 hybridisation


x x
After hybridisation x
x
x
x x x

Four CN- will form four coordinate bonds. CN −


CN −
CN − CN −

Diamagnetic due to absence of unpaired electrons Geometry -Square planar

jksahu74@gmail.com Page | 206


Stereochemistry The study of various atoms or groups of atoms in a molecule, how they
arranged relative to one another in three-dimensional space, is called
stereochemistry.

Ordinary light Light coming from any source is called ordinary light. It consists of light
of various wavelength propagating in all directions perpendicular to the
direction of the propagation.

Monochromatic light Light of single wavelength propagating in all possible directions.

Plane polarized light The monochromatic light vibrating in a single plane is called plane
polarized light. It can be produced by passing monochromatic
lightthrough a Nicol prism.

Nicol prism It is made from calcite, a particular crystalline form of calcium


carbonate.

Polariser The device which produces plane polarized light is called polarizer.

Optically active Substances which rotate the plane of polarized light is known as
substance optically active substance.

Polarimeter The angle of rotation by which the plane of polarized light is rotated can
be measured by means of an instrument called polarimeter.

Dextro rotatory A substance that rotates the plane of polarized light in a clockwise
direction (i.e. towards right) is known as dextrorotatory or d-form or (+)-
form.

Laevorotatory A substance that rotates the plane of polarized light in anticlockwise


direction(i.e towards left) is known as laevorotatory or l-form or (-)-form.

Meso form The isomer in which half of the molecule is dextrorotatory and other half
is laevorotatory. It has asymmetric carbon atoms in pairs and has plane
of symmetry. This type of isomers are optically inactive due to internal
compensation.

Racemic mixture / A mixture containing equimolar amounts of two enantiomers is known


Racemic as racemic mixture or racemic modification.This type of mixtures are
modification(± or dl ) optically inactive due to external compensation. i.e., the rotation due to
one isomer will be cancelled by the rotation due to the other isomer.

Racemisation The process of conversion of enantiomer into a racemic mixture is


known as racemisation.

jksahu74@gmail.com Page | 207


Optical Two or more compounds having same molecular formula, same structural
isomerism
formula, same chemical properties, same physical properties; but differ from
each other to rotate the plane of polarized light, arecalled optical isomers
and this phenomenon is called optical isomerism.

Conditions of a >The molecule should have chiral centre


molecule to be > The molecule should not superimposable with its mirror image.
optically active >The molecule should not possess any type of symmetry elements, such as
plane of symmetry or centre of symmetry.

Chiral centre A carbon atom to which four different groups are attached is known as
asymmetric carbon or stereocentre or chiral centre.
Chirality The objects which are non-superimposable with their mirror image are said to
be chiral and this property is known as chirality

Achiral The objects which are superimposable with their mirror image are said to be
achiral.

Enantiomers The stereo isomers related to each other as non-superimposable mirror


images are called enantiomers.
Enantiomers possess identical physical properties, they differ w.r.t the
rotation of plane polarised light. If one of the enantiomer is dextrorotatory
then the
other will be laevorotatory.

Diastereoisomers The stereo isomers which are neither super imposable nor mirror images of
each other.

Retention of If during a chemical reaction,the product will have the same general
configuration configuration of groups around the stereocentre as that of reactant,then that
reaction is said to proceed with retentionof configuration. During such
reaction , no bond to the stereo centre is broken.
CH3 CH3

H CH2 OH H CH2 Cl

CH2
+ H Cl
CH2 + H2O

CH3 CH3

Inversion of If during a reaction, the products have opposite configuration of groups


configuration around the stereocentre as that of reactant,such a reaction is said to proceed
with inversion of configuration.
C 2H 5 C 2H 5

Y
H H
X Y
H3C CH3

jksahu74@gmail.com Page | 208


Alkylidene / Alkylenedihalides

Geminal halides Vicinal halides


( Alkylidene halides) (Alkylenedihalides)

Halogenatoms are present on the same Halogen atoms are present on the
carbon atom. adjacent carbon atoms.

Structure Cl Cl Cl

H3C CH Cl H2 C CH2
Common Ethylidene chloride Ethylene dichloride

IUPAC 1,1 - Dichloroethane 1,2 –Dichloroethane

Markovnikov’s rule
According to Markovnikov rule , when an unsymmetrical alkene added with an unsymmetical
addendum, the negative part of the addendum goes to that doubly bonded carbon atom which
contain lesser number of hydrogen atoms.

Br
+ −
H 3C CH CH2
+ H Br H 3C CH CH3
2-Bromopropane

Anti-Markovnikov’s rule
According to Anti-Markovnikov rule , when an unsymmetrical alkene added with an unsymmetical
addendum in presence of peroxide, the negative part of the addendum goes to that doubly bonded
carbon atom which contain more number of hydrogen atoms.

Br
(C 6 H 5 CO) 2O 2
+ −
H 3C CH CH2 + H Br H 3C CH 2 CH 2

Zaitsev Rule ( Saytzeff rule)


According to Saytzeff'srule during elimination reaction, highly branched alkene is the major
product.

H Br Alc. KOH
H3C CH CH CH3
H3C CH CH CH3

jksahu74@gmail.com Page | 209


Preparation of Haloalkane
1. Free radical Halogenation of Alkanes
Br2 AlBr3
R-Br + HBr

Halogenation Cl 2
RH
Sun light
R-Cl + HCl
Alkane
I2
HIO 3 or HNO 3
R-I + HI

( Oxidising agent )

Order of reactivity of halogens F2> Cl2 > Br2> I2

Order of reactivity of the different types of hydrogen atoms of alkane 30> 20> 10

The reaction of iodine with alkanes is reversible as hydrogen iodide formed during the reaction
reduces iodo derivative to parent hydrocarbon. Hence iodination carried in presence of oxidizing agent
such as HgO , HIO3 , or HNO3

Reaction of F2 with alkane is explosive therefore they are prepared by halogen exchange methods.

2. Halogen exchange methods


(a) Finkelstein reaction:
Acetone
RCH 2Br NaI RCH2I NaBr
+ Heat +
(b) Swarts reaction( AgF , Hg2F2 , CoF2 , or SbF2 )

RCH2Br AgF RCH2F AgBr


+ +
If chloro alkanes are used, yield is less as C – Cl bond energy is higher than C – Br

3.Hydrohalogenation of Alkene
Br
Markovnikov's rule
R CH CH3

R CH CH2 HBr

Alkene Peroxide R CH2 CH2 Br


Anti-Markovnikov's rule

jksahu74@gmail.com Page | 210


4.Nucleophilic substitution of Alcohol

PCl 5
R-Cl + POCl 3 + H Cl

PX 3
3 R-X + H 3 PO 3
Red P + X 2
ROH
H Cl
ZnCl 2
R-Cl + H 2O

SOCl 2
R-Cl + SO 2
+ H Cl

Preparation of Halo alkanes from alcohols using Thionyl chloride is preferred because the twoby -
products are escapable gases.
SOBr2 is less stable & SOI2 does not exist and thus bromides & iodides cannotbe prepared by this
method.
Preparation of Halo alkanes from alcohols using hydrogen halides, the order of reactivity of
alcohols with a given halo acid is :
Allyl , Benzyl >3°>2°>1°. As polarity of C – O bond is in this order.
Reactivity of halogen acid HI >HBr>HCl> HF
The above method is not applicable for the preparation of aryl halides because the carbon-oxygen
bond in phenols has a partial double bond character and is difficult to break being stronger than a
single bond.
The formation of secondary and tertiary bromides and iodides by using HX from respective alcohol
is not favoured as they readily undergo dehydration forming alkenes.
PBr3 and PI3 being less stable thus phosphorous and bromine or iodine mixture is used . PBr3 and
PI3 are produced in situ by heating red phosphorous with bromine or iodine.
PBr5 and PI5 are highly unstable due to steric hindrance, so only chlorides are prepared by this
method.
Preparation of fluoro alkanes from alcohol by using HX is not in practice as HF is least reactive
Mixture of ( 1: 1) dry HCl and anhydrous ZnCl2 is called Lucas reagent.
Reaction of 10 and 20 alcohols with HCl requires a catalyst like ZnCl2 but 30 alcohols react with
HCl at room temperature.
Anhydrous ZnCl2 is added to help the cleavage of C – O bond. It act as Lewis acid and readily

coordinates with oxygen atom of the alcohol. As a result C – O bond weakens to yield carbocation

which readily reacts with chloride ion to form chloroalkane. In addition anhydrous ZnCl2 acts as

dehydrating agent, absorbs water molecules formed during reaction and prevent reverse reaction.

jksahu74@gmail.com Page | 211


Boiling Points

Alkyl halides having more boiling point as compare with hydro carbons of equal molecular mass, it
is due to dipole- dipole attractions present in it.
For the same alkyl group, the boiling points of alkyl halides decrease in the order: RI>RBr>RCl>
RF. This is because with the increase in size and mass of halogen atom, the magnitude of van der
Waal forces increases.
With the increase in the size of alkyl group boiling point increases and with the increasing in
branching boiling point decreases.
The boiling points of isomeric halo alkanes decrease with increase in branching due to decrease in
surface area which in turn decreases vanderwaal’s force.
Boiling points of isomeric di halo benzenes are very nearly the same. However, the para-isomers
are high melting as compared to their orthoand meta-isomers. It is due to symmetry of para-
isomers that fits in crystal lattice better as compared to ortho- and meta-isomers.
The halo alkanes are only very slightly soluble in water, Energy released during dissolution is not
sufficient to break the hydrogen bond between water molecules.

Solubility

The halo alkanes are only very slightly soluble in water, Energy released during dissolution is not
sufficient to break the hydrogen bond between water molecules.
Haloalkanestend to dissolve in organic solvents because the new intermolecular attractions
between haloalkanes and solvent molecules have much the same strength as the ones being
broken in the separate haloalkane and solvent molecules.
Fluoro and chloro compounds are lighter than water, where as bromo and iodo compounds are
heavier than water. Density decreases with increase in size of alkyl group. The density also
increases with increasing number and atomic mass of the halogen.
Dipolemoment of CH3Cl > CH3F > CH3Br > CH3I As dipolemoment not only depend upon
electronegativity but also on bond length.

Chemical properties of Alkyl halides

Halo alkanes undergo following reactions:

1. Nucleophilic substitution 3.Reduction

2. Elimination reaction4. Reaction with metals

jksahu74@gmail.com Page | 212


Nucleophilic substitution

The nucleophilic substitution may follow SN1 or SN2 mechanism.

SN1 SN2

Substitution nucleophilic unimolecular Substitution nucleophilic bimolecular

H3C H CH3
Slow C 2H 5
+ Slow
C Br H3C C - C Br Nu C Br
H5 C2 Nu + H3 C
H7 C3 C3 H7 H D
D
Nu (d) TS
C 2H 5 - H
Nu
H3C C+
Fast C2 H Nu C
C 3H 7 H3C CH3
C3 H7 D
(l )

Carbocation dl

Two steps reaction One step reaction

Rate ∝ [Alkyl halide] Rate ∝ [Alkyl halide] [ Nucleophile]


First order, Unimolecular Second order , Bimolecular

Racemisation of configuration Inversion of configuration

Favour by polar solvent Favour by non-polar solvent

Tertiary halide Primary halide

3o Halide > 2o Halide > 1o Halide 3o Halide < 2o Halide < 1o Halide

Weak nucleophile Strong nucleophile

The energy, obtained through solvation of halide The incoming nucleophile interacts with alkyl
ion with the proton of protic solvent, causing the halide causing the carbonhalidebond to break.
carbonhalidebond to break.

Reaction rate determined by electronic factor Reaction rate determined by steric hindrance
( Stability of R)

Catalysed by Lewis and Bronsted acids. Not catalyzed by any catalyst ( phase transfer)
( AlCl3, ZnCl2 , HA )
Rearraged products can be formed No rearranged product is formed ( except for
allylic )

For a given alkyl group, the reactivity of the halide, R-X, follows the same order in both the

mechanisms R–I>R–Br>R–Cl>>R–F.

jksahu74@gmail.com Page | 213


Aqueous KOH
ROH [Alcohol]

H2O
ROH [Alcohol]

AgOH
ROH [Alcohol]
Moist Ag2O

R ONa
ROR [ Ether ]

-+
RNa
RR [ Alkane ]

LiAlH4
RH [ Alkane ]

RCOOAg
RCOOR [ Ester ]

R-X AgNO2 xx
RNO2 [ Nitroalkane ] Ag N
O O

KNO2
RONO [ Alkylnitrite ] xx
+ - N
K O O
Alcoholic AgCN
RNC [ Isonitrile ]

Alcoholic KCN
RCN [ Nitrile ]

Alcoholic NH3
RNH2 [ Primary Amine ]
R
RNH2 R NH [ Secondary Amine ]
R
R
R NH
R N R [ Tertiary Amine ]

The alkyl halides are highly reactive due to the high electronegativity difference between carbon &

halogen atom which provides polarity in C –X bond& thus carbon atom of C-X bond is easily

attacked by a nucleophile to show nucleophilic substitution.

In halo alkanes as size of halogen increases C—X bond strength decreases hence reactivity

increases. Thus the reactivity order of alkyl halide is : R – I>R – Br> R – CI> R – F

jksahu74@gmail.com Page | 214


The reactivity order for given alkyl halide also shows the order :
3o halide > 2o halide > 1o halide
This has been explained in terms of +ve inductive effect of alkyl groups which increases the
polarity of C-X bond & thereby making it more reactive.
Groups like cyanides and nitrites possess two nucleophilic centres and are called
ambidentnucleophiles.
Halo alkanes react with KCN to form alkyl nitrile as main product while AgCN forms isonitrile as
the chief product. It is because KCN is predominantly ionic and provides cyanide ions in solution.
Although both carbon and nitrogen atoms are in a position to donate electron pairs, the attack
takes place mainly through carbon atom and not through nitrogen atom since C—C bond is more
stable than C—N bond. However, AgCN is mainly covalent in nature and nitrogen is free to donate
electron pair forming isonitrile as the main product.

Elimination reaction

H
B- β− Elimination
-
C C C C + B-H + X

B- is the base and X is the leaving group ( Br, I )

In general elimination reaction occurs:


(i) In the presence of a strong and/or bulkier base,
(ii) At high temperature and
(iii) With bulkier haloalkanes
CH3CH2 Br Alc. KOH
H2C CH2
+ KBr + H2O

The alcoholic KOH results in the formation of a strong base, ethoxide ion.

-+
H3C CH2 OH + KOH CH3CH2 O K + H2O

The much stronger ethoxide ion abstracts a proton to form alkene


H
CH3CH2 -
O
H2 C CH2 H2C CH2 + H3C CH2 OH

Br
Bromoethane Ethene

jksahu74@gmail.com Page | 215


Reduction
Alkyl halides are reduced to alkanes in the presence of a reducing agent like ( Zn – Cu / EtOH ) , (
Na + alcohol ) , ( Zn + HCl ) , ( Sn + HCl ) , LiAlH4 , ( Red P + HI ) , ( H2 / Ni or Pd at 470 K )

Zn - Cu / C 2H5OH
CH3 CH2 Br + 2[H] CH3 CH3 + HBr

Reaction with metals

Alkyl halides react with sodium in presence of dry ether to give higher alkanes.
Dry ether
2 RX + 2 Na R R + 2 NaX ( Wurtz reaction )

Dry ether
2 Cl + 2 Na + 2 NaCl ( Fittig reaction )

Dry ether
Cl + 2 Na + R Cl R + 2 NaCl ( Wurtz - Fittig reaction )

Alkyl halides react with Zinc in presence of dry ether to give higher alkanes.
Dry ether
2 RX + Zn R R + ZnX2 ( Frankland reaction )

Alkyl halides react with magnesium in presence of dry ether to give Grignard’s reagent.
( Reactivity order is RI >RBr>RCl )
Dry ether
RX + Mg RMgX

Alkyl halides react with Lithium in presence of dry ether to give alkyl lithium
Dry ether
RX + 2 Li R Li + Li X

Ethyl chloride reacts with an alloy of sodium and lead to form TEL ( Tetra Ethyl Lead ) which is
used as anti-knocking agent in petroleum industry.
4 C 2H5 Cl
+ 4 Na - Pb ( C 2H5 ) 4Pb + NaCl + 3 Pb

Preparation of Haloarenes
Cl
[1] From benzene Cl2
I I2 FeCl3

HIO 3
Br
Br 2
FeBr 3
[2] From Benzene diazonium chloride
X
Cu2X2
+- I
N2Cl
KI
x = Cl , Br ∆

Cl
+-
N2BF 4
Cu F
NaBF4

HCl + N2 + BF3
Benzenediazoniumfluoroborate

jksahu74@gmail.com Page | 216


Preparation of aryl halides from phenol is not applicable by nucleophilic substitution because the
carbon-oxygen bond in phenols has a partial double bond character and is difficult to break being
stronger than a single bond

Chemical properties of Aryl halides


2
SP
Aryl halides are extremely less reactive towards nucleophilic substitution
reactions due to the following reasons:
Cl
1. C—Cl bond acquires a partial double bond character due to resonance.
As a result, the bond cleavage in haloarene is difficult.
2. The sp2hybridised carbon with a greater s-character is more electronegative and can hold the
electron pair of C—X bond more tightly .
3. The phenyl cation formed as a result of self-ionisation will not be stabilized by resonance and
therefore, SN1 mechanism is ruled out.
4.Because of the possible repulsion, it is less likely for the electron rich nucleophile to approach
electron rich arenes.
Nucleophilic substitution
The presence of an electron withdrawing group (like nitrogroup )at ortho and para position of
chlorobenzene increases the reactivity.This effect is not shown when nitro group is present at
meta position.
xx
Cl OH
ortho
xx

xx

Cl OH
(i) NaOH, 443 K
(i) NaOH, 623 K, 300 atm +
+
(ii) H
(ii) H
NO2 NO2
Cl para OH Cl OH
NO2 NO2 O 2N NO2 O 2N NO2
(i) NaOH, 368 K
+
Warm
(ii) H H2O

NO2 NO2 NO2 NO2

The presence of nitro group at ortho- and para-positions withdraws the electron density from the
benzene ring and thus facilitates the attack of the nucleophile on haloarene. The carbanion thus
formed is stabilisedthrough resonance. The negative charge appeared at ortho- and para-
positions with respect to the halogen substituent is stabilised by –NO2 group while in case of meta
nitro benzene, none of the resonating structures bear the negative charge on carbon atom bearing
the –NO2group. Therefore, the presence of nitro group at meta- position does not stabilise the
negative charge and no effect on reactivity is observed by the presence of –NO2group at meta-
position.
jksahu74@gmail.com Page | 217
Electrophilic substitution reaction
Cl
Cl
H 3C Cl 2
Anhydrous AlCl3
+ Cl Cl
Cl
CH3Cl
Cl CH 3 + AlCl 3
Cl
Cl
NO2

O
Conc. HNO3
+ Cl NO2
R H2SO4
C

O Cl Cl
AlCl 3
Cl C + RCOCl
SO3H
R Conc. H2SO4 Cl SO3H
+

Although chlorine is an electron withdrawing group, yet it is ortho-, para- directing in electrophilic
aromatic substitution reactions. As reactivity is controlled by the stronger inductive effect and
orientation is controlled by resonance effect.

Polyhalogen compounds
Compounds Uses Harmful effects
Methylene chloride - As solvent for paint remover. -Harms the human central nervous
CH2Cl2 - As propellant in aerosols. system
- As metal cleaning solvent. -Impaired hearing and vision
-Causes dizziness, nausea, tingling and
numbness in the fingers and toes
-Can burn the cornea of eye by direct
contact
Chloroform -As solvent for fats, alkaloids -Inhaling depresses the central nervous
CHCl3
and iodine
system.
- For the production of the -Causes dizziness fatigue and headache.
freon refrigerant R-22

Iodoform As antiseptic ( due to liberation --------------------


CHI3 of free iodine )
Carbon tetrachloride -In the manufacture of -Causes liver cancer
CCl
refrigerants
4 -Damages to nerve cells.
-As feed stock in the synthesis -Depletes the ozone layer
of chlorofluorocarbon
-Cleaning fluid
-As fire extinguisher

jksahu74@gmail.com Page | 218


Compounds Uses Harmful effects

Freon-12CCl2F2 -In aerosol propellants, Upset the natural ozone balance


[Chlorofluorocarbon refrigeration and air
compounds of methane and conditioning purposes.
ethane ]

p,p'-Dichlorodiphenyl As insecticide -Have a high toxicity towards fish


trichloroethane -It is not metabolised by animals
Cl

Cl
Cl Cl

Cl H DDT

Compounds Composition Application

Chloramphenicol Chlorine containing Effective for the treatment of

(Produced by soil microorganisms ) antibiotic typhoid fever.

Thyroxine Iodine containing hormone Deficiency of which causes a

(Produced in our body ) disease called goiter.

Chloroquine Synthetic halogen Used for the treatment of

compounds malaria.

Halothane Anesthetic Used during surgery

Certain fully fluorinated compounds Considered as potential Used in surgery

blood substitutes

C2Cl6 ( hexachloro ethane ) A solid As artificial camphor.

jksahu74@gmail.com Page | 219


Alkyl halides burn on Cu wire with green edge flame. This is Beilstein test for halogens. The green
or blue colour to flame is due to interaction of halogens with Cu wire.
Freons are prepared by tetrachloromethane through Swarts reaction.

CCl4 + AgF CF2Cl2

Alkyl iodides are less stable and darken their colour on exposure to light on standing due to
adsorption of liberated iodine.

2RI R R + I2

Chloroform is slowly oxidized by air in the presence of light to an extremely poisonous gas,
carbonyl chloride, also known as phosgene. It is therefore stored in closed dark colored bottles
completely filled so that air is kept out.
Light
2 CHCl 3
+ O2 2 COCl 2
+ 2 HCl
Phosgene

Sulphuric acid not used during the reaction of alcohol with KI .


As sulphuric acid converts KI to corresponding HI and then oxidises it to I2

Dipolemoment of CH3Cl > CH2Cl2 > CHCl3 > CCl4


µ = 1.87 1.55 1.02 0

Grignard reagents are very reactive. These are readily decomposed by compounds containing
acidic hydrogen (Protic solvent i.e. water ) Therefore Grignard reagent is prepared in dry ether
under anhydrous conditions.

RMgX + H2O RH + Mg (OH)X


Grignard reagent Alkane

p-Dichlorobenzene has higher m.p and lower solubility than those of o- & m-isomers
Cl
Cl
Cl

Cl Cl
Cl
Ortho- Meta- Para-

Due to symmetrical nature of p-dichlorobenzene, in the crystal, it is more closely packed as


compared to the other isomers. As a result , it has a higher melting point
In p-dichlorobenzene, the net dipolemoment is zero, so it is non-polar, hence has lower solubility
as compared to other isomers.

jksahu74@gmail.com Page | 220


ALCOHOLS - PHENOLS - ETHERS
OH
Monohydric alcohols Containing one −OH group C2H5OH
Ethanol Phenol

OH
H2C CH2
Catechol
Dihydric alcohols Containing two −OH groups OH OH
Ethyleneglycol
OH
HO OH

H2 C CH CH2
Trihydric alcohols Containing three −OH groups
OH OH OH OH
Glycerol Benzene- 1, 2 , 4- triol
Monohydric alcohols having C SP3 − OH group
Types 1o 2o 3o
CH3
Alkyl alcohols H3C CH2 OH H3C HC OH
H3C C OH
CH3
CH3
H2C CH CH2 OH H2C CH CH OH CH3
Allylic alcohols
CH3 H2C CH C OH
CH3

Benzylic alcohols CH2 OH


CH3
CH OH
C OH
CH3
CH3

Monohydric alcohols having C − OH group OH OH


SP2
H2C CH OH
CH3
Preparation of alcohols
[1] From alkene ( acid catalysed hydration ) OH
+
H
H2C CH CH3 + H2O H3C CH CH3

[2] From alkene ( Hydroboration )


H2C CH CH3
H3C CH CH2 + (BH3)2 H3C CH2 CH2 BH2 ( CH3CH2CH2 ) 2BH

H2O 2 OH

H3BO3 + 3 CH3CH2CH2 OH ( CH3CH2CH2 ) 3B
H2C CH CH3
( Highly pure )
H2O 2
OR +
H3C CH CH2 B 2 H6
OH−
3 CH3CH2CH2 OH + H3BO3

[3] From Aldehyde and Ketone ( Reduction by H2 / Pt or H2/Ni or H2/Pd or NaBH4 )

O O
NaBH4 R CH2 OH H2 / Ni
R C H R C H
Aldehyde Primary alcohol Aldehyde

O OH O
H2 / Pt H2 / Pd
R C R R CH R R C R
Ketone Ketone
Secondary alcohol
jksahu74@gmail.com Page | 221
[4] From Aldehyde and Ketone ( By using Grignard Reagent )
O − +
O Mg Br HO
− + H2O
R Mg Br + C C R C OH + Mg Br
R
Carbonyl compound
Alcohol
Formaldehyde will give primary alcohol
− + H
HO
O O Mg Br H2O
− +
CH 3 Mg Br + H C H H C H H3C C OH + Mg Br

CH3 H
Formaldehyde
Primary alcohol
Aldehydes ( except formaldehyde ) will give secondary alcohol.

O − + CH3
+ O Mg Br H O HO
CH 3 Mg Br + H C CH3
H C CH3
2
H3C C OH
+ Mg Br
Acetaldehyde H
CH3
Secondary alcohol
Ketones will give tertiary alcohol. − + CH3
O Mg Br HO
O H2 O
− +
CH 3 Mg Br + H3 C C CH3
H3C C CH3 H3C C OH + Mg Br
CH3 CH3
Ketones
Tertiary alcohol
[5] From Carboxylic acid O
( 1 ) LiAlH 4
R C OH R CH 2 OH Primary alcohol
( 2 ) H 2O
Carboxylic acid
O
[6] From Esters H2
R C OR
Catalyst
R CH 2 OH + R OH
Primary alcohol
Esters
LiAlH 4 is an expensive reagent, so acid should convert into ester before cnversion into
alcohol.
Properties of alcohols
[1] When alcohols act as nucleophile, the O − H bond breaks.
H H
+
+ + H
R O C
C R O C
+
..

R O
..
[2] Protonated alcohols act as electrophile, the C − O bond breaks.
.. H
+
+ −
Nu
R CH 2 OH
.. R CH 2 OH 2 R CH 2 Nu + H2 O

[3] Acidic nature


.. − −
+ B + BH
..

R O H R O
..
Base Conjugate base Conjugate acid
Acid
The acidic character of alcohol is due to polarity of O−H bond. Electron releasing groups
decrease the polarity of O−H bond , so acid strength of alcohol is:
H R R

R C OH R C OH R C OH
o o
H 1 H 2 R
3o
jksahu74@gmail.com Page | 222
2 ROH + 2 Na 2 RONa + H2

6 (CH 3)3C OH + Al [ (CH 3)3 C O] 3 Al


+ 3 H2
tert.butylalcohol Aluminiumtert.butoxide

[4] Esterification +
H
RCOOH + HOR' RCOOR' + H2O
Acid Alcohol Ester
O O +
H
R C O C R + HOR' RCOOR' + RCOOH
Acid
Alcohol Ester
Acid anhydride
O
Pyridine
RCOOR'
H3C C Cl + HOR'
Ester
[ Pyridine neutralises the HCl
Alcohol produced in the reaction]
Acid chloride

[5] Reaction with Lucas reagent ( concn HCl & ZnCl2 )


ZnCl 2
ROH
+ HCl RCl + H2O
Alcohol
Insoluble with each other
Soluble with each other

[6] Reaction with PCl3


3 ROH + PCl 3 3 RCl + H3PO 3

[7] Dehydration
Protic acid i.e concn. H2SO4, H3PO4 or catalyst such as Al2O3 or ZnCl2 used as dehydrating
agents.
Fo
H OH
rd
Conc. H 2SO 4
H2 C CH2
443 K
H2C CH2 + H2O
eh
Primary alcohol ( Ethanol ) Alkene ( Ethene ) yd
ra
OH H tio
85% H3PO 4
n
H3C CH CH2 + H2O
3
H3C CH CH2 o
440 K Propene
>
Secondary alcohol ( Propan - 2-ol )
2
o
OH H
>
1
20% H 3PO 4
o a

H3C C CH2
358 K
H3C C CH2 + H2O
lc

2 - Methylpropene
oh

CH3 CH3
Oxi

ol

Tertiaryalcohol ( 2 - Methylpropan - 2 - ol )
dati

[8] Oxidation Oxidising power of HNO3 or KMnO4 > CrO3


on o

+
Anhydrous CrO 3 KMnO 4 / H
Aldehyde / Ketone Alcohol Acid
f1

+
KMnO 4 / H
o > 2

Anhydrous CrO 3
RCHO RCH2OH RCOOH
Adehyde Primary
o > 3

O OH
+
Anhydrous CrO 3 KMnO 4 / H
o alco

R C CH3 R CH CH3 RCOOH + H2O + CO 2


Ketone Secondary
OH
hol

+
Anhydrous CrO 3 KMnO 4 / H
No reaction R C CH3 RCOOH + H2O + CO 2
CH3 Tertiary
jksahu74@gmail.com Page | 223
[9] Dehydrogenation Cu / 573 K
RCH2OH RCHO ( Aldehyde )
Primary Dehydrogenation
OH O
Cu / 573 K
R CH CH3 R C CH3
Dehydrogenation ( Ketone )
OH Secondary
Cu / 573 K
R C CH3 R C CH2 ( Alkene )
Dehydration
CH3 Tertiary CH3

PHENOL ( Carbolic Acid )


Preparation
-+
[1] From haloarenes Cl O Na
OH
+
623 K H
+ NaOH
300 atm.

Chlorobenzene Sodiumphenoxide Phenol


[2] From Benzenesulphonic acid SO 3H OH
NaOH
+
H Phenol
SO 3 H SO 3 Na
SO 3 + H 2SO 4 ONa + OH
NaOH Fused NaOH H
- H2O - NaHSO3

[3] From Benzenediazonium chloride + −


N2 Cl OH
Warm H 2O
+ N2 + HCl

[4] From Cumene ( Isopropyl benzene )


CH3 CH3 OH O
O2 Dilute Acid
HC
CH3
C O O H + H3C C CH3
Acetone
CH3

Properties
[1] Acidic nature
OH + Na ONa
+ H2
Sodium phenoxide

OH + NaOH ONa + H2O

[2] Esterification
The introduction of acetyl group in alcohols or phenols is known as acetylation. Acetylation
of salicylic acid produces aspirin.
COOH COOH
+
+ ( CH3CO ) 2 O H + CH3COOH

OH OCOCH3
Salicylic acid Acetylsalicylic acid ( Aspirin )

jksahu74@gmail.com Page | 224


[3] The electrophillic sustitution
OH
[a] Nitration
OH OH
OH
NO2
O2N NO2 Conc. HNO3 Dilute HNO 3
+
2,4,6-Trinitrophenol NO2
( Picric acid ) Phenol Ortho Para
NO2
nitrophenol

[b] Halogenation OH
OH OH OH

Br Br Br2 Br2 Br

Water CS2 +
( Solvent of high polarity ) ( Solvent of low polarity )
( Minor ) Br
Br Phenol
( Major )
2,4,6-tribromophenol
[4] Reaction with Zinc dust
OH + Zn + ZnO

[5] Oxidation Na2Cr2O7


OH O O
H2SO 4
Benzoquinone

Name reactions
−+ OH
Kolbe's Reaction OH O Na

CO2 COOH
NaOH
+ 2-Hydroxybenzoic acid
H
( Salicylic acid )

Reimer-Tiemann Reaction −+
−+ O Na OH
OH O Na
NaOH CHO CHO
+
CHCl3 CHCl2 H
aq. NaOH
2-Hydroxybenzaldehyde
( Salicylaldehyde)
Role of PCC
N Cl Cr O-3
+
Pyridiniumchlorochromate ( C5H5NH )+ ClCrO3−
H
PCC oxidises CH2− OH group upto aldehyde without affecting other groups.

H3C CH CH CH2 OH PCC H3C CH CH CHO


Role of sodiumborohydride ( NaBH4 ) and Lithiumaluminiumhydride ( LiAlH4 )
Both will reduce carbonyl group into alcohol group, but NaBH4 can reduce carbonyl group
without affecting C= C in conjugation with carbonyl group.
O
LiAlH4 NaBH4
H3C CH2 CH2 CH2 OH H3C CH CH C H H3C CH CH CH2 OH

jksahu74@gmail.com Page | 225


Explanations
C bond angle in alcohol is less than normal tetrahedral angle. Because Lp-Lp repulsion
O H is greater than Lp-Bp repulsion
xx

xx

C bond angle in phenol is slightly more than alcohol, because the lone pair electron
O H density on oxygen of phenol is slightly decreases due to resonance. So Lp-Lp
xx

xx
repulsion of phenol is less than alcohol.
C bond angle in ether is slightly greater than normal tetrahedral angle due to
O C repulsive interaction between the two bulky alkyl groups.
xx

xx

C O bond length in phenol is slightly less than C O bond length of ethanol. This is due
to partial double bond character of C O bond of phenol due to resonance
C O is polar towards oxygen. The dipolemoment of phenol is smaller than methanol,
because the electrons on oxygen of phenol get delocalised towards benzene ring, but in
case of methanol, due to electron donating effect of -CH3 group, C O bond becomes
more polar.
The high boiling point of alcohol and phenol is due to presence of intermolecular
hydrogen bonding ,which is not possible incase of hydrocarbons and ethers.
R H H
O O
Ether is soluble in water by forming hydrogen bonds
O
R H H
ether water
The boiling point of alcohols decrease with increase of branching in carbon chain,
because the van der waals forces decrease with decrease in surface area.
The solubility of alcohols and phenols in water is due to their ability to form hydrogen
bonds with water molecules.
The solubility decreases with increase in size of alkyl or arene (hydrophobic ) groups
i.e.the solubility of methyl alcohol is greater than ethyl alcohol in water.
Alcohol is less acidic than water because conjugate base of alcohol is less stable than that
of water.
- +
R OH R O + H
Alcohol Alkoxide ion ( less stable, due to high electron density on oxygen atom )
- +
H2O H O + H Hydroxide ion, more stable as compared to RO −

Phenols are stronger acid than alcohol and water, because of conjugate base of phenol
i.e. phenoxide ion is more stable due to resonance.
+
−H O O O O
OH O

PKa = - log Ka = log 1/ Ka Acid strength α Ka


i.e. PKa α 1/Ka α 1/ PKa
i.e. greater the PKa value weaker will be the acid.
Ethanol mixed with methanol is called denatured alcohol. Inside the human body methanol
is oxidised to methanal and then to methanoic acid which may cause blindness or death.
Methanol poisoned patient is treated by giving intravenous injection of dilute ethanol.
Orthonitrophenol is steam volatile due to intramolecular hydrogen bonding ,paranitrophenol
is less volatile due to intermolecular hydrogen bonding , which causes assocation of molecules.
H
O H O H O
O
+ + +
N O N O N
- - -
O O O
O-nitrophenol P-nitrophenol

jksahu74@gmail.com Page | 226


Explain why dehydration of alcohol to form alkene is always carried out with conc. sulphuric acid and
not with conc. HCl or conc.HNO3 .
For the formation of alkene :
+ +
H +
R CH2 CH2 OH R CH2 CH2 OH2 R CH2 CH2
− H2O
( Carbocation )

If HCl is used Cl ( Nucleophile) give substitution product. If H2SO4 is used ( non nucleophile )
give ellimination product.
H
+ HSO4

R CH CH2 R CH CH2 [ Ellimination reaction ]

Cl−
R CH2 CH2 Cl [ Substitution reaction ]
If HNO3 ( Oxidising agent ) is used, alcohol will oxidise to aldehyde or ketone and next
oxidise to acid.
Explain bromination of benzene takes place in presence of FeBr3 but bromination of phenol require
no FeBr3.
Bromination of benzene takes place in presence of lewis acid ( FeBr3 ) which polarises
Br - Br molecule.
Incase of phenol the polarisation of bromine molecule occur in absence of lewis acid,
because highly activating effect of OH group attached to benzene ring make the bromine
molecule polar. Br
FeBr3
+ Br2
OH OH

Br
+ Br2
+ Br OH

Explain phenol becomes dark when exposed to air.


In presence of air phenols are slowly oxidised to a dark coloured mixture containing
quinones.
O2
OH O O
( Benzoquinone )
Test for primary, secondary and tertiary alcohols
This can be done by Lucas test ( Lucas reagent - conc.HCl & ZnCl2 )
R R
* Tertiary alcohol produce cloudiness ZnCl2
within one minute. R C OH + HCl
Room temperature
R C Cl
R R
R R
* Secondary alcohol produce cloudiness ZnCl2
within five minutes. R CH OH + HCl
Room temperature
R CH Cl

* Primary alcohol do not produce ZnCl2


cloudiness at room temperature R CH2 OH + HCl
Room temperature
No reaction
due to less polarity. OH

Test for alcohol containing H3C CH group.


This test is performed by iodoform test. These alcohols give yellow precipitate of iodoform
when treated with alkaline solution of iodine.

CH3CH2OH
+ 4 I2 + 6 NaOH CHI3 + HCOONa + 5 NaI + 5 H 2O
Yellow precipitate

jksahu74@gmail.com Page | 227


Test for phenol
Litmus test Phenol turns blue litmus red.
Ferric chloride test Phenol react with neutral ferric chloride solution to give violet
colouration.
Azo dye test Phenol react with diazonium salt in weakly basic medium to form yellow
or orange coloured due.
Bromine water test With bromine water phenol give a white precipitate of
2,4,6-tribromophenol.
METHANOL (Wood Sprit )
Destructive distillation
Wood Methanol
Catalytic hydrogenation of carbon monoxide give methanol
ZnO - Cr 2O 3
CO + 2 H2
200 - 300 atm. , 573 - 673 K
CH3OH

Methanol is used as a solvent in paints and varnishes and for making formaldehyde.

ETHANOL
It is prepared by fermentation of sugar which takes place in absence of air (anaerobic
condition )with production of carbon dioxide gas.
As grapes ripen , the quantity of sugar increases and yeast grows on the outer skin.
When grapes are crushed, sugar and enzyme comes in contact and fermentation starts.
Invertase
C12H22O 11
+ H2O C6H12O6
Sugar Glucose & Fructose

Zymase
C6H12O 6 2 C 2H5OH + CO2
The action of enzyme inhibits if alcohol formed exceeds 14%
If air gets into fermentation mixture, the oxygen of air oxidises ethanol to ethanoic acid and
it destroys the test of alcoholic drink.

ETHER
R O R1
R = R1 simple ether R=
/ R1 mixed ether

Preparation
[1] Dehydration of Alcohol H2SO4
2 C2H5OH H5 C2 O C 2H 5 Ethoxyethane
413 K
It is not possible to prepare mixed ether, because when we take different alcohols, as starting
material, we will get a mixture of ethers, which is difficult to to separate.

Alcohols give ether by SN2 mechanism at low temperature, So for the formation of
ether , the reactivity of alcohol is 1o > 2o > 3o
Alcohols give alkene by ellimination reaction at high temperature, So for the
formation of alkene, the reactivity of alcohol is 3o > 2o > 1o
[2] Williamson synthesis
This is used to prepare both simple and mixed ether
-+
R O Na + R1 X R O R1 + NaX

The reaction involves SN2 attack of an alkoxide ion on primary alkyl halide.
If alkyl halide used is tertiary then alkene will be the main product.

jksahu74@gmail.com Page | 228


CH3 CH3
- +
H3C C O Na + H3C Br H3C C O CH3

CH3 Ether
CH3

H CH3 CH3
- + CH2 C Br H2C C Alkene
H3C O Na +
CH3 CH3

NaOH - + H3C Br
OH O Na O CH 3
Anisole

Reaction of ether with HX


Reactivity of HI > HBr > HCl
R O R1 + HX R X + R1 OH

Case-1 R1 OH + HX R1 X + H2O
xx
O
xx
CH3 + HBr OH + CH3Br

stronger due to partial double


bond character
Case-2
H3C O CH2 CH3 + HI CH3I + CH3CH2OH

Since by SN2 mechanism I− ( nucleophile ) attack to the smaller alkyl group ( − CH3 )

CH3 CH3
Case-3
H3C C O CH3 + HI H3C C I + H3C OH

CH3 CH3

By SN1 mechanism C − O bond of t-butyl group break to produce stable ( CH3 )3C+
OCH3
Electrophilic substitution Br

Br2 in ethanoic acid + Br OCH3

OCH3 O

C O
OCH3 CH3COCl CH3
OCH3
AlCl 3 + C
H3C

OCH3
CH3
CH3Cl
AlCl 3
+ H3C OCH3

OCH3

NO2
H2SO 4
HNO3
+ O2N OCH3

jksahu74@gmail.com Page | 229


ALIPHATIC PORTION

[1] Preparation of Aldehydes and Ketones


(a) From alcohols

Primary alcohols in presence of oxidizing agents such as anhydrous CrO3 or PCC , or by


catalytic dehydrogenation with Cu at 573 K get oxidised to aldehydes whereas secondary
alcohols results in the formation of ketones.

Cu / 573 K Anhydrous CrO 3 / PCC


R CHO R CH2 OH R CHO
Dehydrogenation Oxidation
Aldehyde 1o Alcohol Aldehyde

Details of dehydrogenation 2 Cu + O2 2 CuO


R CH 2 OH
+ CuO R CHO + Cu + H2O

(b) From alkenes

The alkenes on treatment with ozone followed by reductive cleavage (by Zn dust and water)
results in the formation of aldehydes whereas substituted alkenes form ketones.

R O H O O
Zn / H 2O
R CH CH2 + O3
H
C C
H
R C H + H C H
Alkene
O O Aldehydes
Ozonide
R O H
O O
R CH CH2 + O3 C C
H
Zn / H 2O
R R R C R + H C H
Aldehyde
O O Ketone

(c) From alkynes

Ethyne reacts with water in the presence of mercuric sulphate and sulphuric acid to yield
aldehyde i.e. ethanal. Other alkynes give ketones.
O H O
HgSO 4 Tautomerisation
HC CH + H OH
H 2SO 4 H2C CH H3C C H
Ethyne enol ( Unstable ) Ethanal
O H O
+ − HgSO 4 Tautomerisation
R C CH + H OH
H 2SO 4 R C CH2 R C CH3
Ketone

[2] Preparation of Aldehydes only

(a) From acid chlorides [ Rosenmund reduction ]


O O
H2
R C Cl R C H
Pd - BaSO 4
Acid chloride Aldehyde
jksahu74@gmail.com Page | 230
(b) From Nitriles [ Stephen Reduction ]
The reaction of alkanenitrile with HCl in the presence of anhydrous stannous chloride
yields corresponding imine, which on hydrolysis give corresponding aldehyde.
+
SnCl 2 H3O
R C N R CH NH R CHO
HCl
imine
(c) From Acid derivatives
LiAlH 4 LiAlH
Acid Aldehyde 4
1o Alcohol
In order to prevent formation of alcohol, we use a derivative of carboxylic acid ( Acid
chloride, Ester and Nitrile ) that is more easily reduced and an derivative of aluminium
hydride ( DIBAL-H ) that is less reactive than lithium aluminium hydride.

Diisobutylaluminium hydride ( DIBAL-H ) is less reactive than LiAlH4, because


it is much more sterically hindered and ,therefore, have difficulty in
Al
H transferring hydride ions.
Al H (i-Bu) 2 i.e. DIBAL-H
R C N R CHO
H2O
Nitrile
O
Al H (i-Bu) 2 i.e. DIBAL-H
R C OR 1 R CHO
H2O
Ester
O
Al H (i-Bu) 2 i.e. DIBAL-H
R C Cl R CHO
Acid chloride H2O

[3] Preparation of Ketones only


(a) From Acid chlorides
Treatment of acyl chlorides with dialkylcadmium, prepared by the reaction of cadmium
chloride with Grignard reagent, gives ketones.

2 R Mg X + CdCl 2 R 2Cd + 2 Mg(X)Cl


O O
2 R1 C Cl + R 2Cd 2 R1 C R + CdCl 2
(b) From Nitriles
Treatment of nitrile with Grignard reagent followed by hydrolysis yields a ketone.
NMgBr O
+
R1 C N + R Mg X
R1 C R
H3O
R1 C R

AROMATIC PORTION
(a) From alcohols
OH +
K 2Cr 2O 7 / H CHO
CH 2

OH O
+
K 2Cr 2O 7 / H C CH3
CH CH3

OH O
CH C
+
K 2Cr 2O 7 / H

jksahu74@gmail.com Page | 231


(b) From acyl chloride ( Rosenmund reduction )
O O

C C
H2 H
Cl
Pd - BaSO 4

Benzoyl chloride Benzaldehyde

(c) From nitriles ( stephen reaction )


O
C N CH NH
SnCl 2
H2O C
HCl H
Imine
Benzonitrile

(d) From Methyl benzene ( Etard reaction )

O
CH3 CH ( OCrOHCl 2 ) 2
CS 2 + C
H3O
+ CrO 2Cl 2 H

Chromyl chloride
Toluene Chromium complex
Benzaldehyde

Not Etard reaction


CH3 CH ( OCOCH 3 ) 2 O
273 K - 283 K
+ CrO 3 + ( CH 3CO ) 2O C
H
Chromic oxide +
Acetic anhydride H3O
Toluene Benzylidene diacetate

(e) From benzene ( Gatterman - Koch reaction )

An aldehyde group is directly introduced by treating benzene with carbon monoxide and
hydrogen chloride in the presence of aluminium chloride as a catalyst.
AlCl 3 CHO
+ CO + HCl
+ HCl

This is an electrophilic substitution reaction ( modified Friedel-crafts reaction ), where


the most likely electrophile is acylium ion. H

CO + HCl AlCl + 3 +C O + AlCl 4

(f) From Toluene by side chain chlorination

CH3
CHCl 2 CHO
Cl 2 / hν H2O

373 K
Benzalchloride

(g) From Grignard reagent


NMgBr O
MgBr C
Ether CH3 + C
H3O CH3
+ R C N

jksahu74@gmail.com Page | 232


Properties of Aldehydes and Ketones
Nucleophilic Addition Reaction
Nu Nu
R1 +
Slow H

C O + Nu C
O
− Fast
C
OH
R2 R1 R1
R2 R2

[1] Addition of HCN in presence of a base


O OH
Base
R1 C R2 + HCN R1 C R2
CN Cyanohydrin

Base is used to generate Cyanide ion : − −


HCN + OH CN + H2O
Strong Nucleophile

[2] Addition of Sodium hydrogensulphite


OSO 2Na
O OSO 2H
+
H Transfer R1 C R2
R1 C R2 + NaHSO3 R1 C R2
OH
ONa
Bisulphite addition product

This bisulphite addition product is formed by all aldehydes and only by methyl ketones.
The other ketones due to decrease in electrophilicity of carbonyl carbon and also because
of steric factors due to alkyl groups, do not react with sodium bisulphite.

This reaction is used to separate and purify aldehydes and


ketones, because aldehydes and ketones may be recovered from their suphite adducts by
warming them up with sodium carbonate solution.

OSO 2Na O
Na2CO3
R1 C R2 R1 C R2 + Na2SO3 + NaHCO3

OH

[3] Addition of Grignard Reagent

O
− +
O Mg Br HO
− + H2O
R Mg Br + C C R C OH + Mg Br
R
Carbonyl compound
Alcohol
o
Formaldehyde results in the formation of 1 alcohol
Aldehyde ( Other than Formaldehyde ) results in the formation of 2o alcohol
Ketones result in the formation of 3o alcohol

[4] Addition of Alcohols


Addition of alcohols ( Weak nucleophile ) to aldehydes in acidic medium first forms
hemiacetals which on addition of second molecule of alcohol results in the formation of
acetals.
Addition of alcohols to ketones to form hemiketals and ketals does not occur readily
because of the steric hindrance.
OH OR1
O + R1 OH
H R C OR1 R C OR1
R C H + R1 OH
H Hemiacetal H Acetal
Aldehyde
jksahu74@gmail.com Page | 233
O + OH OR1
H R1 OH
R C R + R1 OH R C OR1 R C OR1
Ketone Alcohol R
R Ketal
Hemiketal
R HO CH2 R
O CH2
HCl gas
C O + HO CH2
C
dil. HCl O CH2
R R Ethyleneglycol ketal
Acetals and ketals are hydrolysed with aqueous mineral acids to yield corresponding
aldehydes and ketones.
Nucleophilic Addition - Elimination Reaction
Addition of ammonia and its derivatives
R R
+
H
C O + H2N Z C N Z + H2O

R R

O OH xx OH
+ R H +
H H2N Z −H
+
C C C N Z
R R +
R R
R H
OH
R H
R
− H2O C N Z
C N Z

R R

R R R R
C O + H2 NH C N H C O + H2 NOH C N OH

R Ammonia R Imine R Oxime


R Hydroxyl amine

R R R R

C O + H2 NR C N R C O + H2 N NH 2 C N NH2
Substituted imine Hydrazone
R Amine R Hydrazine R
R
( Schiff's base )

R R
H
C O + H
N NH C N NH
R R Phenylhydrazone
Phenylhydrazine

O 2N O 2N
R R
H
C O + N NH NO2
C N NH NO2
H
R
2, 4 - dinitrophenylhydrazine R
2, 4 - dinitrophenylhydrazone
( Yellow, Orange or Red solid )

R O R O
H
C O
+ H
N NH C NH2 C N NH C NH2
R Semicarbazide R Semicarbazone
jksahu74@gmail.com Page | 234
Reduction Reaction
O
[1] Catalytic Reduction
O
H2 / Pt
R C H R CH2 OH ( Primary alcohol ) H2 / Pt
Aldehyde LiAlH 4 / NaBH 4
OH
O
H2 / Pt
R C R R HC OH ( Secondary alcohol )
LiAlH 4 / NaBH 4
Ketone R ( Secondary alcohol )

[2] Wolff - Kishner Reduction


R R
KOH / ethylene glycol
C O + H2 N NH 2 C N NH2
Heat
R CH2 R + N2
Hydrazone Alkane
R Hydrazine R

[3] Clemmensen Reduction O


Zn - Hg
R C R
Carbonyl compounds Conc. HCl
R CH2 R + H2O
Alkane

Oxidation Reaction
O
Mild oxidising agent Strong oxidising agent
No product R C CH3 RCOOH + HCOOH
Ketone
O
Mild oxidising agent Strong oxidising agent
RCOOH R C H RCOOH
Aldehyde

The oxidation of ketone required cleavage of carbon - carbon bond, which can not break by
mild oxidising agents such as Tollen's reagent and Fehling's solution.

O + COOH
Tollen's reagent K2Cr2O 7 / H
No product ( CH2 ) 4
Fehling's solution OR +
KMnO 4 / H COOH
+
Cyclohexanone OR CrO3 / H Hexanedioic acid ( Adipic acid )

Tollen's Test
To aqueous silver nitrate,a drop of dilute sodium hydroxide is added, forming a brown
precipitate of silver oxide. 2 AgNO 3 + 2 NaOH Ag2O + 2 NaNO 3 + H2O

Concentrated ammonia is then added until the precipitate just dissolves


Ag2O
+ 2 NH 3 + NaNO 3 + H2O [ Ag ( NH 3 ) 2 ] NO 3
+ 2 NaOH

So Tollen's reagent is ammonical silver nitrate i.e. [ Ag ( NH 3 )2 ] NO 3

Silver mirror
Aldehyde+ Tollen's reagent
Ketone + Tollen's reagent No silver mirror

− −
RCHO + 3 OH RCOO + 2 H 2O + 2 e−

+
[ Ag ( NH 3 ) 2 ]
+ e− X 2
Ag + 2 NH3

+ −
RCHO + 3 OH − + 2 [ Ag ( NH 3 ) 2 ] RCOO + 2 H 2O + 4 NH3 + 2 Ag
Silver mirror
jksahu74@gmail.com Page | 235
Fehling's Test
Fehling solution is a mixture of alkaline copper(II) sulphate solution and sodium poatssium
tartarate (Rochelle salt ) Aldehyde + Fehling's reagent Reddish Brown precipitate
Ketone + Fehling's reagent No Reddish Brown precipitate
Aromatic aldehyde + Fehling's reagent No Reddish Brown precipitate

− −
+
RCHO 3 OH RCOO + 2 H 2O + 2 e−
2+ − −
2 Cu + 2 OH + 2e Cu2O
+ H2O
2+ −
RCHO + 2 Cu −
+ 5 OH RCOO + 3 H 2O + Cu2O Reddish Brown precipitate

α−Hydroxyketones give positive Tollen's and Fehling test, because in α−hydroxyketones,the


secondary alcoholic group gets oxidised to ketonic group.
OH O 2 H 2O + 4 NH3 + 2 Ag
H3C CH2 CH C CH3 Tollen's reagent Silver mirror
Fehling's reagent
H3C CH2 C C CH3 +
3-Hydoxypentan-2-one O O 3 H 2O + Cu2O
( α−hydroxyketone ) Pentane- 2, 3 - dione Reddish Brown precipitate

O OH
Idoform Test
The reaction is used to detect the H3C C and H3C CH groups in an organic
compound. The organic compounds containing the above groups when treated with
halogen in presence of an alkali results in the formation of Chloroform,bromoform or
idoform. Idoform is a yellow solid.
O O
i.e. I 2 & NaOH
R C CH3 + NaOI
sodium hypoiodite
R C ONa + CHI3 Idoform
( Yellow ppt. )
OH O O
− 2 HI I2 + NaOH
R CH CH3 + I2 R C CH3 R C ONa + CHI 3 Idoform
( Yellow ppt. )
This oxidation does not affect a carbon- carbon double bond, if present in the molecule.
O O
H3C CH C C CH3 + NaOI H3C CH C C ONa + CHI 3
CH3 sodium hypoiodite CH3 Idoform( Yellow ppt. )

The iodoform reaction is given by:

(1) Ethanol [ The only 1o alcohol ]


OH

(2) All secondary alcohols with H3C CH group.

(3) Acetaldehyde [ the only aldehyde ]


O
(4) All methyl ketones [ aliphatic and aromatic ] with H3C C group.

O O

H3C C group is necessary for idoform reaction, because the C group is an


electron withdrawing group, which makes the α− H of methyl group acidic in nature.
The conjugate base, after removal of α−H, become resonance stabilised.
O −
O O
C CH
− C CH C CH
H B −
jksahu74@gmail.com Page | 236
Aldol condensation [ ALDOL = ALD ehyde + alcoh OL ]

Aldehydes and ketones having at least one α−H undergo a reaction in presence of dilute
alkali as catalyst to form β−hydroxy aldehydes or β−hydroxy ketones

OH
dil NaOH ∆ H3 C CH CH CHO
2 CH 3 CHO H 3C CH CH2 CHO − H2 O
Ethanal 3 -Hydroxybutanal [ Aldol ] But-2-enal

O O OH H
dil NaOH ∆
H CH2 C H
H3 C C + H3 C CH CH CHO − H2 O H3C CH CH CHO

O OH O O
Ba(OH) 2 ∆
2 H3 C C CH3 H3 C CH CH 2 C CH3 H3C C CH C CH3
− H 2O 4-Methylpent- 3-en- 2-one
Propanone H 3C
CH3 [ Ketol ]
4-Hydroxy- 4-methylpentan- 2-one

O O OH H O O
dil NaOH ∆
H3 C C + H CH2 C CH 3 H 3C C CH C CH3
− H 2O
H 3C C CH C CH 3

CH 3 CH3 CH3

When aldol condensation is carried out between two different aldehydes or ketones, it is
called cross aldol condensation.
Self
CH3 CHO H3C CH CH
But- 2-enal
CHO + H3C CH2 CH C CHO condensation
1. NaOH 2-Methylpent - 2-enal product
CH3
+ 2. ∆ Mixed
CH3 CH 2CHO H3 C CH C CHO
+ H3C CH2 CH CH CHO condensation
2-Methylbut- 2-enal CH3 Pent- 2-enal product

O O OH H O
dil NaOH
C + H CH2 C CH CH C

H − H2 O
Benzaldehyde Acetophenone O
No α− hydrogen Having α− hydrogen CH CH C

1,3-diphenylprop-2-en-1-one

Cannizzaro Reaction
Aldehydes having no α−hydrogen,when treated with conc. alkali, undergo self oxidation
and reduction [ disproportionation / dismutation / auto oxidation-reduction ] reaction.
O
Conc. NaOH
2 H C H CH 3OH + HCOONa
Methanal Methanol Sodium methanoate

O O
Conc. NaOH
2 C H CH 2 OH + C ONa

Benzaldehyde Benzyl alcohol Sodium benzoate


jksahu74@gmail.com Page | 237
Electrophilic substitution reaction.

Carbonyl groups act as deactivating and meta - directing group.

Cl 2 / AlCl 3 C
R O R
C Cl

O
HNO 3 / H 2 SO 4
C
R
R = H : aldehyde NO2
R = alkyl : Ketone
O
H 2 SO 4 / SO 3
C
R
SO 3 H

ALIPHATIC PORTION

Preparation of Carboxylic Acids


[a] From alcohol and aldehyde

1. alkaline KMnO 4
RCH 2 OH + RCOOH
2. H 3 O
1o alcohol
1. alkaline KMnO 4
CH 3 (CH 2 ) 8 CH 2 OH CH 3 (CH 2 ) 8 COOH
+
1-Decanol Aldehyde OR 2. H 3 O Decanoic acid

O
Strong oxidising agent
R C H RCOOH
Mild oxidising agent

[b] From Grignard reagents


O
O +
H 3O
- + Dry ether C - + RCOOH
R MgBr + C O R
O MgBr
Dry ice

[c] FromAcid derivatives O


H 2O
R C Cl
Acid chloride
RCOOH + HCl

O O
H2O
R C
O
C R1 RCOOH + R 1 COOH
Acid anhydride

O +
+ H 3O
H 3O NaOH
RCOOH RCOONa + R' OH R C OR' RCOOH + R' OH
( Saponification) Ester

+
+ O H 3O
R C N
H 3O
R C NH 2 ∆
RCOOH + NH3

Nitrile Amide

jksahu74@gmail.com Page | 238


AROMATIC PORTION
CH 2 R
COOK + COOH
KMnO 4 - KOH H 3O
∆ Benzoic acid
Alkyl benzene
[ The entire side chain is oxidised to the carboxylic acid irrespective of length of side chain ]
CONH 2 COOH
+
H3O


+ NH3

Benzamide Benzoic acid


O O
C C H 2O COOH
O CH3
+ CH 3 COOH
Ethanoic acid
Benzoic ethanoic anhydride Benzoic acid
O
C C2 H 5 + COOH
O H3O
Ethyl benzoate + C 2H 5 OH

Properties of Carboxylic Acid


[1] Acidity
Carboxylic acids dissociate in water to give resonance stabilised
− carboxylate anion.
O O O
− +
R C OH + H2O R C O R C O + H3O

2 RCOOH + 2 Na 2 RCOONa + H2

RCOOH + NaOH RCOONa + H 2O

RCOOH + NaHCO 3 RCOONa + H2O + CO 2[ Test for carboxylic group ]


[ Brisk effervescence ]
2 3
SP SP
Acidity of H 2C CH COOH > H3C CH 2 COOH
O O
C OH > CH 2 C OH
2 3
SP SP
2 3
SP carbon is more electronegative than SP carbon,

[2] Esterification
The water should remove from the mixture, because ester may undergo hydrolysis.

O O
+
H
H 3C C OH + H O CH3 H 3C C O CH3 + H 2O

Acid Alcohol Ester


O O
+
H
H 3C C OH + H O H 3C C O + H 2O

Ethanoic acid Phenol Phenyl ethanoate


jksahu74@gmail.com Page | 239
[3] Formation of Acid anhydride
O O O
O H2SO 4 / ∆
H3C C + or P 2O 5 / ∆ C C + H2O
OH H C H3C O CH3
O CH3 Ethanoic anhydride
Ethanoic acid

[4] Formation of Acid chloride


O O

C
C
+ PCl 5
H3C
+ POCl 3 + HCl
H3C O OH O Cl

C C
3 HC
3 OH + PCl 3 3 HC
3 O Cl
+ H3PO 3
O

C
H3C
C
OH + SOCl 2 H3C Cl + SO 2 + HCl

[5] Formation of Acid amide


O O O

C C − ∆ C
H3C OH + NH3
H3C O NH4
+
− H2O H3C NH2
Ammonium ethanoate
Ethanamide
− +
COO NH 4 CONH 2
COOH
+ NH3 ∆
− H2O
Amm.benzoate Benzamide
O

COOH - +NH 4
COO CONH 2 C

+ NH3
-+ − 2 H2O
Strong heating
C
NH

COOH COO NH 4 CONH 2 − NH3


O
Phthalic acid Ammonium phthalate Phthalamide Phthalimide

[6] Formation of Alcohol [ Reduction ]


O
LiAlH 4 / Ether CH 2 [ The reaction involve hydride transfer
C
R OH + R OH and causes reuction of carbonyl group ]
H3 O

O
B 2H 6 CH2
[ Diborane is better for this process, because
C
R OH + it does not easily reduce ester,nitro and halo
H3 O R OH groups]

Sodium borohydride does not reduce the carboxyl group.


[7] Formation of Alkane [ Decarboxylation ]
O

C CaO [ NaOH & CaO in the ratio


R ONa + NaO H

RH + Na2CO 3
of 3:1 is known as sodalime ]
Alkane
Sodium salt of carboxylic acid

Kolbe electrolysis
Electrolysis
RCOONa R R + CO 2 + NaOH + H2

At anode At cathode
jksahu74@gmail.com Page | 240
[8] Hell - Volhard - Zelinsky Reaction ( HVZ reaction )
R CH2 COOH (i) X2 / Red phosphorous
R HC COOH X : Cl , Br
Having α - hydrogen (ii) H 2O
X
If two α − hydrogens are present, both of them may be substituted by halogen.

[9] Ring substitution reaction


Carboxylic group act as deactivating and meta directing group.
HOOC
Cl 2 / AlCl 3

HO O
C Cl
HOOC

HNO 3 / H 2SO 4

NO 2

HOOC

H2SO 4 / SO 3

SO 3H
Explanations
Due to polarity of carbonyl group to ward oxygen,the intermolecular association of carbonyl
compounds occurs which makes the boiling point of carbonyl compounds higher than the
corresponding alkane of comparable mass.
However carbonyl compounds can not form hydrogen bonding with
each other and their boiling points are lower compared to alcohol (of comparable mass)

The lower members of aldehydes and ketones are soluble in water in all proportions,
because they associate with water molecules through hydrogen bonding.

The reaction of Grignard reagent with nitriles is considered to be the best method for the
preparation of carbonyl compounds, because, in the first stage of addition an imine is
formed as an intermediate product. The imine is then isolated and hydrolysed to give a
carbonyl compound as the final product.

Reaction of Grignard reagent with acid chloride and esters produce carbonyl
compounds,but the reaction does not stop at this stage and proceeds further with another
molecule of Grignard reagent to give corresponding alcohol as the final product.

Grignard reagent and dialkyl cadmium both are organometallic compounds, but reaction of
acid halides with

[a] Grignard reagent is not satisfactory method for the preparation of ketones.

[b] Dialkylcadmium is a satisfactory method for the preparation of ketones.

Because, in case of dialkylcadmium, due to less reactivity of cadmium, reaction with acid
chloride occurs only upto the formation of ketone. The reaction of Grignard reagent does
not stop after formation of ketone, since magnesium is more reactive metal,and proceeds
further to form tertiary alcohol.
Aryl ketones do not undergo addition of alcohol to form hemiacetals and ketals because of
O
steric factor.
C +
H
R + R' OH No reaction

R = alkyl / aryl
jksahu74@gmail.com Page | 241
Aryl ketones do not give addition product with sodium bisulfite due to steric factor.
O
C
R
+ NaHSO 3 No reaction

R = alkyl / aryl

The order of reactivity of aldehydes and ketones O O O


toward nucleophilic addition reaction: C C C
H H R H R R

Sterically the presence of two relatively large substituents in ketones hinder the approach
of nucleophile to carbonyl carbon than in aldehydes.
Electronically two alkyl groups in ketone reduce the electrophilicity of the carbonyl
more effectively than in aldehyde.
O
xx The right side -NH2 group is in conjugation with electron
xx
H2N NH C NH2
withdrawing carbonyl group and acquires positive charge and not
[Semicarbazide]
in a position to act as the nucleophile, so does not involve
Involved in the formation in the formation of semicarbazone.
of semicarbazone
O OH
HCN
CN

Cyclohexanone
cyanohydrin
O
H3C CH3
HCN
No product

CH3
2, 4, 6-trimethylcyclohexanone

In 2,4,6-trimethylcyclohexane, the three methyl groups will increase the electron density
on the carbonyl carbon atom and the nucleophile attack does not seem to be feasible.
Moreover,the two methyl substituents at the ortho position will also hinder the attack of
nucleophile CN− ion on the carbonyl group.

Upon heating , formic acid loses a molecule of water and gets dehydrated to give carbon
monoxide. Therefore it does not form anhydride upon heating.

HCOOH Heat
CO
+ H 2O

Phenoxide ion has more number of resonating structures than carboxylate ion, but
carboxylic acid is a stronger acid than phenol. Because, in carboxylate ion the negative
charge is dispersed on two electronegative oxygen atoms while incase of phenoxide ion,
there is only one oxygen atom to disperse the negative charge.
Benzaldehyde and other aromatic aldehydes form silver mirror with Tollen's reagent but
fail to react with fehling's solution and Benedict's solution. Due to resonance, the electron
density on the carbonyl carbon atom inbcreases, as a result the C−H bond becomes
stronger. It can be oxidised to C − OH with strong oxidising agents like Tollen's reagent
( EoAg2+|Ag = +0.8 V ) but not with weaker Fehling's or Bendedict's reagent
( Eo Cu2+ |Cu = +0.34 V )

In Fehling solution , copper sulphate forms a soluble complex of Cu2+ ions with tartarate
ion of the Rochelle's salt. The complex provides the Cu2+ ions and also checks the
precipitation of copper hydroxide in the reaction.

jksahu74@gmail.com Page | 242


Preparation
[1] Reduction Method The primary amines may be prepared by the reduction of
nitro compounds,nitriles,amides and oximes.
H 2 / Pt or H 2/Pd or H 2 / Ni or LiAlH 4
R NO 2 R NH 2
Zn / HCl or Sn/ HCl or Fe / HCl
NO 2 NH 2
Sn/ HCl or Fe / HCl

[Fe/HCl is preferred ,because FeCl2 formed gets


Nitro compounds hydrolysed to release hydrochloric acid,thus a small
amount of HCl is required to initiate the reaction.]

LiAlH 4 or H 2 / Ni or Na(Hg) / C 2H 5 OH
R C N R CH 2 NH2
Nitrile
O
LiAlH 4 / H 2O
R C NH2 R CH 2 NH2
Amide Amines

[2]Hoffmann Bromamide Degradation Reaction


O

R C NH2 + Br 2 + 4 NaOH R NH2 + Na 2CO 3 + 2 NaBr + 2 H 2O

In this reaction ,the amine formed contain one carbon less than those of amide.

[3] Gabriel phthalimide synthesis


O O
O
C KOH C
R X C
- +
N H N K
N R NaOH
C C
C
Phthalimide O N-Alkylphthalimide
O O
- +
COO Na
+ R NH2
- +
COO Na

Aromatic primary amines can not be prepared by this method because aryl halides do not
undergo nucleophilic substitution with the anion formed by phthalimide.
[4] Ammonolysis of alkyl halides
The process of cleavage of the C-X bond by ammonia molecule is known as ammonolysis.

NH3 RX RX RX + -
RX R NH2 R 2 NH R 3N R 4N X

The primary amine thus obtained behaves as a nucleophile and can further react with
alkyl halide to form secondary and tertiary amines, and finally quaternary ammonium
salt.
xx
+ - NaOH
NH3 + R X R NH 3 X R NH2 + H 2O + NaX

Primary amine is obtained as a major product by taking large excess of ammonia.


The order of reactivity of halides with amines is RI > RBr > RCl

jksahu74@gmail.com Page | 243


Properties
[1] Salt formation
+ - NaOH
R NH2 + HX R NH3 X R NH2 + H2O + NaX
Substituted ammonium salt

- NaOH
NH2 + HCl +
NH3 Cl NH2 + H2O + NaCl

Anilinium chloride
[2] Alkylation
RX RX RX + -
R NH2 R2NH R3N R4N X

[3] Acylation
The replacement of hydrogen atom of -NH 2 group by acyl group is known as acylation.
O H O H
Base
H3C C Cl + H N C2H5 H3C C N C2H 5
N-Ethylethanamide
The base stronger than amine, like pyridine is used to removes HCl so formed.
O C2H5 O C2H5
Base
H3C C Cl + H N C2H5 H3C C N C2H5
Ethanoyl chloride N,N-Diethylethanamide
O O H O H

H3C C O C CH3 + H N H3C C N


Ethanoic anhydride N-Phenylethanamide
Aniline
[ acetanilide ]
O
H O H
C Cl + H N CH3 C N CH3
Methanamine N- methylbenzamide
Benzoyl chloride
[4] Carbylamine reaction
Heat
NH2 + CHCl3
+ 3 KOH NC + 3 H2O
+ 3 KCl

Heat
R NH2 + CHCl3 + 3 KOH R NC + 3 H2O + 3 KCl

Primary amine Isocyanide


/ Carbylamine
[ Foul smell ]
[5] Reaction with Nitrous acid

[ Nitrous acid is prepared in situ from mineral acid and sodium nitrite.]

NaNO2 + HCl + - H2O


R NH2 + HNO2 R N2 Cl R OH + N2 + HCl
Alcohol
Aliphatic diazonium salt

NaNO2 + HCl + -
NH2
+ HNO2 N2 Cl + NaCl + H2O

Benzenediazonium chloride
Benzenediazonium chloride is stable due to resonance.

jksahu74@gmail.com Page | 244


[6] Reaction with Hinsberg's reagent
Benzenesulphonyl chloride is known as Hinsberg's reagent.
O O + O
KOH K
R N H + Cl S R N S R N
-
S

H O O O
H
Water soluble
1o amine
Acidic hydrogen potassium salt
N-alkylbenzenesulphonamide of sulfonamide

O O
KOH Water insoluble
R N H + Cl S R N S
precipitate
O R O
R
o N,N-Dialkylbenzenesulphonamide
2 amine
O
R N R + Cl S No reaction
R O
3o amine

O
+ O O
K H2O
R N S
HCl
R N S
R N H + Cl S
- KOH H O
O H O
1o amine
O O
H2O
R N S
KOH
R N H + Cl S

O R O
R
2o amine

These days benzenesulphonyl chloride is replaced by p-toluenesulphonyl chloride.

NH2
[7] Electrophilic substitution
Br Br
Br2 / H 2O

Br

NH2 NH2 NH2 NH2

NO2
HNO3 , H 2SO4 . 288 K
+ +
NO2
Aniline NO2 ( 47 % ) (2%)
( 51 % )

+ - +
NH3 HSO4 NH2 NH3

H2SO4 453 - 473 K

Aniliniumhydrogensulphate -
SO3H SO 3
Sulphanilic acid Zwitter ion
jksahu74@gmail.com Page | 245
O O
Controlled Bromination
NH2 H xx
C H C
N CH3 NH2
N CH3
( CH3CO ) 2O Br 2 - +
OH / H
Pyridine CH3COOH

N-Phenylethanamide
( Acetanilide ) Br Br

O
Controlled Nitration O
xx H C NH2
NH2 H C N CH3
N CH3
( CH3CO ) 2O - +
HNO3 , H2SO 4 , 288 K OH / H
Pyridine

NO 2
NO 2
The lone pair of electrons on nitrogen is less available for donation to benzene ring by
resonance. Therefore, activating effect of -NHCOCH3 group is less than that of amino
group.
Explanations
Aniline and other aryl amines are colourless but get coloured on storage due to
atmospheric oxidation.
Alcohols are more soluble than amine in water because, alcohols are more polar than
amines and form strong intermolecular hydrogen bonds than amine.
The intermolecular association is more in primary amine than in secondary amines as there
are two hydrogen atoms avilable for hydrogen bond formation in primary. Tertiary amines
do not have intermolecular association due to absence of hydrogen atom available for
hydrogen bond formation. Therefore, the order of boiling points of isomeric amines :
Primary > Secondary > Tertiary
The basicity of amines in gaseous phase : 3o amine > 2o amine >1o amine > NH3
H
xx +
+
N + H N

[ Base ] [ Conjugate acid ]


The stability of substituted ammonium ion: R
+ + +
R HN R NH2 +
The positive charge get neutralised by R NH3 NH4
the +I effect of alkyl groups. R R

In aqueous phase the stability of substituted ammonium cation depend upon both the +I
effect of alkyl group and solvation by water molecules. The greater the size of the ion
lesser will be the solvation and less stabilised is the ion.
So the basicity of methyl substituted amines in aqueous phase:
CH3
H3C NH H3C NH2 H3C N NH3
CH3
CH3
2o 1o 3o

C2 H5
H5C2 NH H5 C 2 N
H5C2 NH2 NH3
C2 H5 C2 H5
2o 1o
3o
When alkyl group is bigger methyl group, there will be steric hindrance to H-bonding with
water, so the stability of substituted ammonium ion decreases.
So the basicity of ethyl substituted amines in aqueous phase:
jksahu74@gmail.com Page | 246
Aniline does not undergo Friedel-Crafts reaction (alkylation / acetylation) due to salt
formation with aluminium chloride, the Lewis acid, which is used as a catalyst.Due to this
, nitrogen of aniline acquires positive charge and hence acts as a strong deactivating
group for further reaction.
Besides the ortho and para derivatives, significant amount of meta derivative is also formed
during nitration of aniline.Because, in the presence of mineral acids,the aniline readily
forms anilinium ions. The anilinium ion is a strong electron withdrawing group, which
directs the electrophile towards meta position.
+ -
NH2 NH3 NO 3 NH2
Ring deactivator
HNO3

NO2

Aniline is less basic than ammonia, because the availability of electron on


xx
nitrogen atom of aniline decreases due to delocalisation of electron by NH2
resonance
In case of aliphatic amine due to +I effect of alkyl group, the xx Aniline
electron density on nitrogen atom increases, as a result basic R > NH2
strength increases. Aliphatic amine

Amines are less acidic than alcohols, because N-H bond is less polar than O-H bond. Therefore, amine
releases H+ ion with more difficulty.
Aromatic amines are insoluble in water because, the phenyl group is bulky in size and has
-I effect. As a result, its hydrogen bonding with water is negligible.

Methylamine forms a soluble hydroxide on reacting with water. The OH- ions released by
the hydroxide combine with Fe3+ ions of ferric chloride to give ferric hydroxide or
hydrated ferric oxide which is brown in colour.
xx
+ -
H3C NH2
+ H OH H3C NH3
+ OH
3+ -
2 Fe + 6 OH 2 Fe(OH) 3 or Fe 2O 3. 3H2O ( Brown ppt )

NH2 NH2 NH2


NO2
Basicity of
NO2
NO2 o-Nitroaniline
m-Nitroaniline
p- Nitroaniline

Incase of aromatic amines, the basicity depends upon:


[i] Inductive effect o- > m- > p-
[ii] Resonance effect, observed at o- & p- positions.

The electron withdrawing substituents on aromatic ring reduce the basicity of substituted
anilines. The -R effect of nitro group is observed maximum at ortho and para positions.Thus,
m-nitroaniline is relatively more basic compared to ortho and para isomers as it withdraws
electrons through inductive effect(-I effect) only. Out of o-nitroaniline and p-nitroaniline, -I
effect is found to be stronger in ortho isomers as compared to para. Thus,o-nitroaniline
experiences a strong -R and -I effect and is least basic.

So PKb of o-nitroaniline > p-nitroaniline > m-nitroaniline

jksahu74@gmail.com Page | 247


Benzenediazonium Chloride
+ - NaNO 2 + HCl
Diazotisation
NH2
N 2 Cl + NaCl + HNO 2
+ H2O

Benzenediazonium chloride
Benzenediazonium chloride is stable due to resonance.

Coupling do not H 3PO 2


Diazonium ion being a weak electrophile,can couple

occur in strong H2O


H + N2 + H 3PO 3 + HCl

acidic medium
with a strongly activated aromatic system.

+
NH 2 NH 3 CH 3CH 2OH
+
H + N2 + CH 3CHO + HCl
H
-
OH

CuCl / HCl
Less Cl + N2
activated
aromatic
system CuBr / HBr N2
OH O
- Br +
-
OH
+
H CuCN / KCN CN + N2

+ -
N N Cl Cu / HCl
Cl + N2 + CuCl

-+
O Na
In strongly alkaline medium, the concentration of

N
KI
diazonium ion ( act as electrophile ) decreases

N
I + N2 + KCl
and coupling reaction does not occur.

HBF 4 + -

NaOH N2 BF 4 F + N2 + BF 3

OH

N H2O
N
OH + N2
+ HCl

HBF 4 + - NaNO 2
NaOH
N2 BF 4
Cu ∆
NO 2 + N2 + NaBF 4
N
+
N
H OH

-
N N OH ( Orange dye )
HO Coupling reaction

H NH 2

N N NH 2 ( Yellow dye )
-
HO Coupling reaction
p-Aminoazobenzene

jksahu74@gmail.com Page | 248


Carbohydrates are optically active polyhydroxy aldehydes or ketones or the compounds
which produce such units on hydrolysis.

Aldohexose Ketohexose
D-(+)-Glucose D-(-)-Fructose
H O
CH2OH
C

H *C OH C O

HO *C H HO *C H

H *C OH H *C OH

H *C OH H *C OH

CH2OH CH2OH

Configuration = D ( Last chiral Configuration = D ( Last chiral


carbon contain - OH at right side ) carbon contain - OH at right side )
Specific rotation = + 52.50 Specific rotation = - 92.40
No. of chiral centre = 4 No. of chiral centre = 3
No. of primary alcoholic group = 1 No. of primary alcoholic group = 2
No. of secondary alcoholic group = 4 No. of secondary alcoholic group = 3

Anomers A pair of optical isomers which differ in the configuration only around first carbon
are called anomers. e.g. α- and β- form of glucose (anomers) exist in equilibrium with open
chain structure.
HO H H OH
H O
C C
C
H C OH H C OH
H C OH

HO C H O HO C H O
HO C H

H C OH H C OH
H C OH

H C H C
H C OH

CH2OH CH2OH
CH2OH

β−D-(+)-Glucose α−D-(+)-Glucose

jksahu74@gmail.com Page | 249


Fischer projection Haworth Projection
OH
H OH
C H2C
6 H
5 O
H C OH H

O H H 1
4
− (+)-Glucopyranose HO
α −D- C H OH
H C OH OH 3 2 OH
O H C H OH
Pyran H 2C
OH
OH
HO H
C H2C
6 OH
5 O
H C OH H
HO C H O H H 1
4
β −D-
− (+)-Glucopyranose OH
H C OH 3 2
OH H
H C
H OH
H 2C
OH

HOH2C OH OH
C O CH2 OH
H 2C
6 5 2 1
O H OH
HO C H
OH
α -D- (-)-Fructofuranose H C OH H 4 3

H C OH H
H2C
O OH
Furan
HO CH2OH
C OH OH
H 2C O
6 5 2
HO C H O
β -D- (-)-Fructofuranose H HO
H C OH CH 2 OH
4 3
H 1
H C
OH H
H2C
OH

Open chain structure of glucose could not explain the following :-

(a) Glucose does not give 2,4-DNP test, Schiff's test and it does not form the
hydrogen sulphite addition product with NaHSO3
(b) The pentaacetate of glucose does not react with hydroxylamine indicating the
absence of free -CHO group.
(c) Glucose is found to exist in two different crystalline forms which are named as
α - and β -form.

jksahu74@gmail.com Page | 250


Monosaccharide: A carbohydrate that cannot be hydrolyzed.
Examples : glucose, fructose, ribose, etc.

Oligosaccharides: Carbohydrates that yield two to ten monosaccharide unitson hydrolysis.


Examples : Lactose , Maltose, Sucrose

Polysaccharides :Carbohydrates which yield a large number of monosaccharide units on


hydrolysis. Examples: Starch , Cellulose

Sugars are crystalline substances which are sweet and water soluble.
e.g. monosaccharides and oligosaccharides i.e. glucose, fructose and cane sugar

Non- sugars are amorphous substances which are tasteless and water insoluble. e.g.
Polysaccharides i.e. starch,cellulose, etc.

Reducing sugars are Carbohydrates which reduce Fehling's solution and Tollens' reagent .
e.g., All Monosaccharides

Maltose and Lactose ( Although these are disaccharides )

Non-reducing sugars are Carbohydrates in which aldehydic or ketonic groups are bonded,
can't reduce Fehling's solution and Tollens' reagent.

e.g. sucrose..( disaccharides) and all poly saccharides.

Invert sugar Sucrose is dextrorotatory but after hydrolysis gives dextrorotatory glucose and
laevorotatory fructose. Since the laevorotation of fructose (-92.4°) is more than dextrorotation of
glucose (+ 52.5°), the mixture is laevorotatory. Thus, hydrolysis of sucrose brings about a change
in the sign of rotation, from dextro (+) to laevo (-) and the product is named as invert sugar.

+
H
C12H22O 11
+ H2O C6H12O 6 + C6H12O 6
Sucrose Glucose Fructose

Preparation of Glucose

From sucrose (Cane sugar):


+
If sucrose is boiled with dilute H
HCl or H2SO4 in alcoholic
C12H22O 11
+ H2O C6H12O 6 + C6H12O 6
Sucrose Glucose Fructose
solution,glucose and fructose
are obtained in equal amounts.

From starch: Commercially


+
glucose is obtained by H
hydrolysis of starch by boiling
(C6H10O 5)n
+ n H2 O
393 K , 2 - 3 atm.
n C6H12O 6
it with dilute H2SO4 at 393 K Starch / Cellulose Glucose
under pressure.

jksahu74@gmail.com Page | 251


Evidences regarding structure of Glucose
S.N Experiment Observation Inference
All the six carbon atoms
1 Heating with HI HI are linked in a straight
HOH2C (CHOH)4 CHO H3C CH2 CH2 CH2 CH2 CH3
∆ n-Hexane chain
Glucose

2 Reaction with hydroxylamine NH2OH


HOH2C (CHOH) 4 C O HOH2C (CHOH)4 CH N OH
Glucose H Oxime
Presence of a carbonyl
group
3 Reaction with HCN HO
O
HCN HOH2C (CHOH)4 CH CN
HOH2C (CHOH)4 C H
Glucose Cyanohydrin
Br2 Water Carbonyl groupis
HOH2C (CHOH)4 CHO HOH2C (CHOH)4 COOH
4 Reaction with Bromine water present as an aldehydic
Glucose Gluconic acid
group.

Reaction with acetic


anhydride O
O C
O The presence of five -OH
Acetic anhydride C O O CH3
5 H3C C Glucose groups
H3C
O CH2 CH CHO
H3C C 4
O Glucose pentaacetate

HNO3
HOH2C (CHOH)4 CHO
Reaction with The presence of a
6 nitric acid Glucose HOOC (CHOH)4 COOH primary alcoholic (-OH)
HNO3 Saccharic acid group .
HOH2C (CHOH)4 COOH
Gluconic acid

jksahu74@gmail.com Page | 252


Glycosidic linka ge
A linkage between two monosaccharide units through oxygen atom is called glycosidic linkage.

Disaccharide Monosaccharide - 1 Monosaccharide - 2 Glycosidic linkage

Sucrose α -D-(+)-Glucose β-D-(-)-Fructose C1 - C2

Maltose α -D-(+)-Glucose α -D-(+)-Glucose C1 - C4

Lactose β-D-(+)-Galactose β-D-(+)-Glucose C1 - C4

Since the reducing groups of glucose and fructose are involved in glycosidic bond formation,
sucrose is a non reducing sugar.
OH OH H
H 2C H2C O
6 H 1 2 5
5 O
H HO
H
H H 1 CH2 OH
4
OH 3 4 6 [ Sucrose ]
OH 3 2 H
O OH
H OH
Glycosidic linkage
α-D-(+)-Glucose β-D-(−)-Fructose

The free aldehydegroup can be produced at C1 of second glucose in solution and Maltose shows
reducingproperties so it is a reducing sugar.

OH OH
H2 C H2 C
6 H 6
5
H
5 O O
H H
H H 1 4
H H 1
4
OH OH [ Maltose ]
3 2 3 2
OH O OH
H OH Glycosidic linkage H OH

α-D-(+)-Glucose α-D-(+)-Glucose
Lactose is
also a reducing sugar.
OH
OH
H2 C
H2 C 6 OH
6 5 O
5 O H
OH
H
4
H H 1 O 4
OH
H 1 [ Lactose ]
OH
3 2 H
H 3 2 H
Glycosidic linkage H OH
H OH
β-D-(+)-Galactose β-D-(+)-Glucose

jksahu74@gmail.com Page | 253


Starch Polymer of α-D-(+)-glucose

Amylose Amylopectin
Water soluble Water insoluble
Contribution 15 - 20 % Contribution 80 - 85 %
Unbranched Branched
Chain by C1- C4 glycosidic linkage Branching occurs by C1-C6 glycosidic linkage

Cellulose: Straight chain polymer of β−D-glucose with C1 - C4 glycosidic linkage

Glycogen

a) The carbohydrates are stored in animal body as glycogen.

b) It is also known as animal starch because its structure is similar to Amylopectin.

c) It is present in liver, muscles and brain.

d) When the body needs glucose, enzymes break the glycogen down to glucose.

Amino Acids
COOH
α− amino acid since amino group attached to α - carbon atom.
H2N H
L-configuration since amino group at left side of chiral carbon atom.
R
In aqueous solution amino acid exist as zwitter ion.
O O

R CH C O H R CH C O
+
NH2 NH3
[ Zwitter ion ]

Non-essential amino acids, which can be synthesised in the body.

Amino acid -R Three letter One letter


symbol code
Glycine −H Gly G
Alanine −CH3 Ala A

Aspartic acid − CH2COOH Asp D

There are 20 amino acids. Except glycine, all other are optically active.

jksahu74@gmail.com Page | 254


Essential amino acids , which cannot be synthesised in the body and must be obtained
through diet .

Amino acid -R Three letter One letter


symbol code
H3C CH
Valine Val V
CH3

Leucine H3C CH CH2


Leu L
CH3
Lysine H2N - (CH2)4 − Lys K

Phenylalanine C6H5 - CH 2 − Phe F

Isoelectric point: The PH of a solution in which an α - amino acid exist as a zwitter ion and hence
neither migrates towards anode nor towards cathode is called isoelectric point.
Peptides are the compounds formed when two or more molecules of α -amino acids combine with
the loss of molecules of water.
While writing the name or structure of a peptide or polypeptide , the N-terminal end is written on
the left while C-terminal end on the right.

Peptide linkage O CH3


O CH3
H2N CH2 C OH + H NH CH COOH H2N CH2 C NH CH COOH

Glycine Alanine Glycylalanine [ Gly-Ala ]


[a] Oligopeptide: contain 2 to 9 amino acids

[b] Polypeptide : Contain 10 to 100 amino acids.

[c] Proteins : Contain more than 100 amino acids.

(But insulin, which is a protein, contains 51 amino acids)

Structure of protein
Primary structure Secondary structure
It is the sequence in which various amino It is the manner in which protein chains are
acids are arranged in the protein molecule. arranged in space with respect to each other.
It determines function of a protein Function of protein does not depend on its
secondary structure.
It involves only covalent bonds It involves internal and external hydrogen bonding.
No possibleconformation Two possible conformations areα-Helix and β-
Pleated sheet structure

jksahu74@gmail.com Page | 255


α-Helix structure β- Pleated sheet structure

A polypeptide chain forms all possible hydrogen All peptide chains are stretched out to nearly
bonds by twisting into a right handed screw maximum extension and then laid side by side
(helix) with the -NH group of each amino acid which are held together by intermolecular
residue hydrogen bonded to the C= O of an hydrogen bonds. The structure resembles the
adjacent turn of the helix pleated folds of drapery and therefore is
known as β-pleated sheet
Examples are: α – keratin, skin, wool , claws Examples are: fibroin present in silk etc
and myosin in muscles, etc

Tertiary structure :Theoverall folding of the polypeptide chains i.e., further folding of the
secondary structure.

The 2° and 3° structures of proteins are stabilized by


- Hydrogen bonds
- Disulphide linkages
- Van der Waals
- Electrostatic forces of attraction.

Quaternary structure :Some of the proteins are composed of two or more polypeptide chains
referred to as sub-units. The spatial arrangement of these subunits with respect to each other is
known as quaternary structure.

Types of protein
Fibrous proteins Globular proteins

When the polypeptide chains run parallel and are When the chains of polypeptides coil around
held together by hydrogen bonds and disulphide to give a spherical shape.
bonds, then fiber- like structure are formed
They are insoluble in water These are usually soluble in water

Examples are keratin (present in hair, wool, and Insulin, egg albumin, casein (of milk ),
silk) and myosin (present in muscles), collagen, myoglobin (of skeletal muscle ), haemoglobin
fibroin etc.
(of cells) , thyroglobulin , etc

Denaturation of Proteins
When a protein in its native form, is subjected to physical change like change in temperature or pH,
the hydrogen bonds are disturbed. Due to this, globules unfold and helix gets uncoiled and protein
loses its biological activity. This is called denaturation of protein. During denaturation 2° and 3°
structures are destroyed but 1º structure remains intact. Example: Coagulation of egg white on
boiling, Curdling of milk.

jksahu74@gmail.com Page | 256


Nucleic acids
Nucleic acids are long chain polymers of nucleotides.

Nucleotides= Nucleoside + Phosphate ( Where Nucleoside = Base + Pentose sugar )


= Base + Pentose sugar + Phosphate
Base is attached to 1' position of sugar and Phosphate group is attached to 5' position of sugar
moiety
Bases are Purine (Guanine -G, Adenine-A ) or Pyrimidine ( Cytosine -C, Thymine - T, Uracil - U)
Sugar moiety is β-D-2-deoxyribose or β-D-ribose. Nucleotides are joined together by
phosphodiester linkage between 5' and 3' carbon atoms of the pentose sugar
OH OH
OH OH O
H2C O H2C
5 4 1
5 4 1
H H H H
H 3 2
H
H 3 2 H
OH OH OH H

β-D-ribose β-D-2-deoxyribose

Simplified version of nucleic acid chain is:

Base Base Base


1 1 1
Sugar 5 Phosphate Sugar Phosphate Sugar Phosphate
3 3 5 3 5

Nucleotides Phosphodiester linkage

Primary structure of nucleic acid:The sequence of nucleotides in the chain of a nucleic acid is
called its primary structure

Secondary structure of nucleic acid:


DNA RNA
It has a double stranded α-helix structure in which It has a single stranded α-helix structure
two strands are coiled spirally in opposite
directions.
Sugar present is β-D-2-deoxyribose Sugar present is β - D - ribose
Bases: Bases:
i) Purine bases( Adenine-A and Guanine-G ) i) Purine bases( Adenine-A and Guanine-G )
ii) Pyrimidine bases:(Thymine -T and Cytosine - C ) ii) Pyrimidine bases:(Uracil - Uand Cytosine -C )
It occurs mainly in the nucleus of the cell It occurs mainly in the cytoplasm of the cell
It is responsible for transmission for heredity It helps in protein synthesis.
character

jksahu74@gmail.com Page | 257


Double helix structure of DNA
a) It is composed of two right handed helical polynucleotide chains coiled spirally in opposite
directions around the same central axis.
b)Two strands are held together by H- bonds ( A = T , G ≡ C )
c) Total amount of purine ( A + G ) = Total amount of Pyrimidine ( C + T )

DNA Fingerprinting
A sequence of bases on DNA is also unique for a person and information regarding this is called
DNA fingerprinting. It is same for every cell and cannot be altered by any known treatment. DNA
fingerprinting is now used

(i) In forensic laboratories for identification of criminals.


(ii) To determine paternity of an individual.
(iii)To identify the dead bodies in any accident by comparing the DNA's of parents or children.
(iv) To identify racial groups to rewrite biological evolution.

Biological Functions of Nucleic Acids

[1] DNA is the chemical basis of heredity and regarded as the reserve of genetic information.
[2] DNA is responsible for maintaining the identity of different species over millions of years.
[3] A DNA molecule is capable of self duplication during cell division and identical DNA strands
are transferred to daughter cells.
[4] The proteins are synthesised by various RNA molecules in the cell but the message for the
synthesis of a particular protein is present in DNA.

Vitamins[ vital + amine ]


Organic compounds required in the diet in small amounts to perform specific biological functions
for normal maintenance of optimum growth and health of the organism are called vitamins.
Vitamins are designated by alphabets A, B, C, D, etc. Some of them are further named as sub-
groups e.g. B1, B2, B6, B12, etc. Excess of vitamins is harmful

Classification of Vitamins:

(i) Fat soluble vitamins: Vitamins which are soluble in fat and oils but insoluble in water . These
are vitamins A, D, E and K. They are stored in liver and adipose (fat storing) tissues.

(ii) Water soluble vitamins: 'B' group vitamins and vitamin 'C' are soluble in water so they are
grouped together. Water soluble vitamins must be supplied regularly in diet because
they are readily excreted in urine and cannot be stored (except vitamin B12) in our body.

jksahu74@gmail.com Page | 258


Vitamins Sources Deficiency diseases
Vitamin - A Fish liver oil, carrots, butter Xerophthalmia (hardening of cornea of eye) ,
and milk Night blindness
Vitamin - B1 Yeast,milk,greenvegetables Beriberi (loss of appetite, retarded growth)
(Thiamine)
and cereals
Vitamin B2 Milk, egg white, liver, Cheilosis (fissuring at corners of mouth and lips),
(Riboflavin) kidney digestive disorders and burning sensation of the
skin
Vitamin B6 Yeast, milk, egg yolk, Convulsions
(Pyridoxine) cereals and grams
Vitamin B12 Meat, fish, egg and curd Pernicious anaemia (RBC deficient in
haemoglobin)
Vitamin - C Citrus fruits, amla and Scurvy (bleeding gums)
(Ascorbic acid) green leafy vegetables
Vitamin-D Exposure to sunlight, fish Rickets (bone deformities in children) and
and egg yolk osteomalacia (soft bones and joint pain in adults)
Vitamin- E Vegetable oils like wheat Increased fragility of RBCs and muscular
germ oil, sunflower oil, etc Weakness
Vitamin- K Green leafy vegetable Increased blood clotting time

jksahu74@gmail.com Page | 259


Polymer - Polymers are large molecules having high molecular mass, formed by joining of
repeating
structural units on a large scale.
Monomer - The simple and reactive molecules which combine to give polymer is known as
monomer.
Polymerisation - The process of formation of polymers from respective monoers is called
polymerisation
Linear polymer - Thes polymers consists of long and straight chain. e.g.- Polythene, polyvinyl
chloride.
Branched chain polymer - These polymers contain linear chains having some branches e.g. Low
density polythene.
Cross linked / Net work polymers - These are formed from bi-functional and tri-functional
monomers contain strong covalent bonds between various
linear
polymer chains. e.g Bakelite and melamine.
Addition polymer - These are formed by reaction between monomer molecules possessing
multiple
bonds
The addition polymers formed by the polymerisation of a single monomeric species are known as
homopolymers
Free radical mechanism of addition polymerization(Chain growth
polymerization)

[1] Chain initiation


O O O
x x
Ph C O O C Ph 2 Ph C O 2 Ph
Benzoyl peroxide Phenyl radical

x x
Ph + H2C CH2 Ph CH2 CH2

[2] Chain propagation


x
Ph CH2 CH2 +n H2C CH2 Ph CH2 CH2 CH
n 2
x
CH2

[3] Chain termination


x
Ph CH2 CH2 CH2 CH2
Ph CH2 CH2 CH2 CH2
n n
+ x
Ph CH2 CH2 CH2 CH2 Ph CH2 CH2 CH2 CH2
n n

Condensation Polymer (Step growth polymer)


These are formed by condensation between monomeric units with elimination of small molecules
such as water , ammonia, alcohol etc

jksahu74@gmail.com Page | 260


Addition polymers
Sl.No Polymer Structure Uses
. Monomer
As insulator, anticorrosive,packing
1 Polythene CH2 CH2 H2C CH2
material,house hold and laboratory works.
n
Ethene
Cl CH Cl Rain coats, Hand bags, Vinyl flooring
Polyvinyl chloride H2C
2
( PVC ) CH2 CH Vinyl chloride and water pipe.
n
CH3
H 2C CH CH 3
3 Polypropene CH 2 CH
Ropes, toys,pipes and fibres
n Propene
C 6H5
4 Polystyrene H2C CH C 6H 5 As insulator, Wrapping material,toys,
CH 2 CH Styrene Radio and television cabinets.
n
Teflon As lubricant, insulator and making
5 CF 2 CF 2 F 2C CF 2
( Polytetrafluoro non-stick cooking ware.
ethene ) n Tetrafluoro ethene
CN
Polyacrylonitrile For making synthetic fibres and synthetic
6 (Orlon / Acrilan ) CH2 CH
H 2C CH CN
wool.
n Acrylonitrile
CH3
7 Natural rubber CH3
Cis-1,4-polyisoprene Used for tyres after vulcanisation
. CH 2 C CH CH 2 . H2C C CH CH 2
n Isoprene
Cl
8 Polychloroprene Cl
As insulator, making conveyor belts and
( Neoprene ) . CH 2 C CH CH 2 . H2C C CH CH 2 printing rollers
n Chloroprene
COOCH 3 CH3
9 Polymethyl methacrylate
. CH 2 C . H2C C COOCH3 As a sustitute of glass
( Plexiglass, Lucite )
n Methyl methacrylate
CH3

COOC 2H 5 In making films, house pipes and finishing


10
Polyethyl acrylate H2C CH COOC2H5 fabrics
. HC CH 2 .
n Ethyl prop-2-enoate
The addition polymers formed by the polymerisation of two or more types of monomeric species are known as
Co-polymers
Butadiene-styrene copolymer C6 H5 H 2C CH CH CH2
Buta-1,3-diene In making automobile tyres
11 (Buna-S) . CH2 CH CH CH2 CH CH2 . and foot wears
H2C CH C 6H 5
Synthetic rubber n Styrene
CN H 2C CH CH CH2
Nitrile rubber Oil seals, tank lining
12 . CH2 CH CH CH2 CH CH2 .
Buta-1,3-diene
CN
(Buna-N) H2C CH
n Acrylonitrile

Natural polymer
Polymer Monomer Uses
Cellulose β − Glucose Occurs in cotton
Starch α − Glucose Food material stroage in plants
Protein Amino acid Essential for growth
Natural rubber Isoprene ( 2-Methylbuta-1,3-diene ) Used for tyres after vulcanisation

Inter moecular forces Fibres > Thermo plastic > Elastomers

jksahu74@gmail.com Page | 261


Condensation Polymer (Step growth polymer)
Sl.No Polymer Monomer Structure Uses
Terylene O O O O
1 /Dacron
/Polyester HO C C OH . O CH2 CH2 O C C .
Terephthalic acid n
HO CH 2 CH 2 OH Used for making fibres, safety belts, tents
Ethylene glycol
O O
Glyptal COOH Used as binding material
2 ( Alkyl resin ) Phthalic acid . O CH 2 CH 2 O C C . in preparation of mixed
COOH plastics and paints
n
HO CH 2 CH 2 OH Ethylene glycol

H Used for the manufacture


O H
Nylon-6 O Cyclic amide of tyres cords,fabrics and
3 N
. C (CH2)5 N . ropes
( Caprolactum )
n

H O O
HOOC (CH2)4 COOH H
Nylon-6,6 Adipic acid
4 H2N (CH2)6 NH2 . N (CH 2)6 N C (CH 2)4 C .
n
Hexamethylenediamine For making brushes,paratutes and ropes
NH2CONH2 O For making unbreakable cups
Urea HCHO
5 formaldehyde Urea Formaldehyde . NH C NH CH2 . and laminated sheet
n
H2N N NH2 Melamine . NH N For making
NH CH2 .
Melamine
6 unbreakable crockery
formaldehyde N N HCHO n
N N
resin Formaldehyde
NH2 NH2
OH OH Fo making
HCHO Formaldehyde . CH 2 CH 2 . combs,electrical
7 Bakelite switches,handles
OH Phenol n of utensiles and
computer discs
. .

High density Polythene Low density Polythene


350-370 K 1000-2000 atm
Ethene Ziegler-Natta Catalyst Polythene Ethene Polythene
333-343 K & 6-7 atm Traces of oxygen / Peroxide

Linear molecules Highly branched structure


High density due to close packing of monomer chains. Low density due to loose packing of monomer chains.
Chemically inert ,more tougher and harder Chemically inert , tough and flexible
For the manufacture of Buckets, dustbin, bottle, pipes. For the manufacture of squeeze bottle,flexible pipes.

Ziegler-Natta Catalyst - Triethylaluminium and titanium tetrachloride

Vulcanisation of rubber
Heating of raw rubber with sulphur and an appropriate additive at a temperature range
between 373-415K is called vulcanisation of rubber.
Sulphur forms cross links at the reactive sites double bonds and make the rubber stiff.
Natural rubber is a linear polymer of isoprene ( 2-methylbuta-1,3-diene ) i.e cis-1,4-polyisoprene
Synthetic rubber is either homopolymers of Buta-1,3-diene derivative / copolymers of
Buta-1,3-diene or its derivatives with another unsaturated monomer.

jksahu74@gmail.com Page | 262


Classification of polymer
Natural Found in plants and animals - Protein, starch,resins and rubber
Source
Semi-synthetic Derivative of cellulose i.e Cellulose acetate ( Rayon )
& Cellulose nitrate
Synthetic Plastic ( Polythene ), Synthetic fibres ( Nylon 6,6 ),
synthetic rubbers ( Buna -s )

Linear High density polythene, Poly vinyl chloride

Structure
Branched chain Low density polythene

Formed from bi-functional and tri-functional


Cross linked monomers and contain strong covalent bonds
- Bakelite, Melamine

Mode of
Addition and Condensation
Polymerisation
POLYMER
Polymer chains are held together by the weakest
Elastomers intermolecular forces.-
Buna-S ,Buna-N and neoprene
Polymer chains are held together by the strong
Fibres intermolecular forces like H-bonding
Molecular forces Polyamides ( nylon 6,6 ) and Polyesters ( terylene )
Possess inter molecular forces of attraction
Thermo plastic intermediate between elastomer and fibre.
( Softening on heating and hardening on cooling )
- Polythene, Polystyrene, and polyvinyls

Thermosetting On heating undergo extensive cross linking and


can not be reused. -Bakelite,Urea-formaldehyde
resins

Chain growth Free radical addition


Growth

Step growth Condensation polymerisation

Biodegradable polymer
The polymers which are decomposed by enzymatic hydrolysis and by oxidation.
[1] Nylon-2-nylon-6
O O O O

H NH CH2 C OH +H NH ( CH 2 ) 5 C OH . NH CH2 C NH ( CH2 ) 5 C


n
.

Glycine Amino caproic acid Nylon-2-nylon-6

[2] Poly β − hydroxybutyrate-co-β


β −hydroxy
− valerate ( PHBV )
O O O O
H O CH CH2 C OH + H O CH CH2 C OH . O CH CH2 C O CH CH2 C .

CH3 H2 C CH3 CH3 H2 C CH3 n


3-Hydroxybutanoic acid 3-Hydroxypentanoic acid PHBV

jksahu74@gmail.com Page | 263


Drugs : - Drugs are chemicals of low molecular masses, interact with macromolecular targets and
produce a biological response.

Medicines : - Drugs which have therapeutic biological response and used in diagnosis ,
prevention and treatment of diseases; are called medicines.

i.e. All medicines are drug, but all drugs are not medicine

Chemotherapy :- The treatment of disease by means of chemicals that have specific toxic effect
upon the disease producing micro-organisms or that selectively destroy neoplastic tissues is called
chemotherapy.
O
Classification of drugs : -
Drugs are classified on the basis of H2N S NH R
(1) Pharmacological effect
(2) Drug action O
(3) Molecular targets Sulphonamide
(4) Chemical structure
Interaction of drugs with targets
Targets- Macromolecules of biologcal origin such as Proteins, carbohydrates, nucleic acids and
lipids
Enzymes - Proteins which perform the role of biolgical catalysts in the body
Receptors- Proteins which are responsible for the communication system in the body
Communication system in the body
Following steps are involved for the communication system in the body :-
(1) Receptor at its binding site receive chemical messenger
(2) Shape of the receptor changed after attachment of messenger
(3) After transfer of message to the cells,the messenger leaves the site of receptor, and receptor
regains its structure.
Receptors as drug targets
(a) Drugs ( as antagonists) bind to the active site of receptor and inhibit its natural functioning.
These are useful when blocking of the message is required.
(b) Drugs ( as agonists) mimic the natural messenger by switching on on the receptor.
These are useful when there is lack of natural chemical messenger.

jksahu74@gmail.com Page | 264


Enzymes as drug targets
Drugs hinder the formation of enzyme-substrate complex i.e. inhibit the catalytic activity of the
enzyme.. It can be done in two ways :-
(a) Drugs [ as competitive inhibitor] block the active site of enzyme for the sustrate
(b) Drugs [ as binding inhibitor] bind in the allosteric site of the enzyme and change
the shape of the active site in such a way that substrate can not recognise it.
You May Know
Penicillin was the first antibiotic discovered by Alexander Fleming from the mould penicillium
notatum
Phenol act as antiseptic ( 0.2% phenol) and as disinfectant ( 1% phenol)
Bithionol/Bithional is added to soap to impart antiseptic properties.
Dettol is a mixture of chloroxylenol and terpeneol
Pathogon is an organism that causes disease.
Bactericidal antibiotics:- These drugs kill the organisms in the body ( Penicillin,
Ofloxacin,Aminoglycosiders)
Bacteriostatic antibiotics:- These drugs inhibit the growth of organism(Tetracycline,
Chloramphenicol, Erythromycin)
Spectrum of an antibiotics:- The full range of micro-organism attacked by an antibiotic is called
its spectrum.
The antibiotics which are effective aganist several different types of micro-organism are called
broad spectrum antibiotics ( Tetracycline, Chloramphenicol,Ofloxacin, Vancomycin)
The antibiotics which are effective only aganist certain specific group of micro-organisms are
called narrow spectrum antibiotics ( Penicillin)
Artificial sweetening agents: -
Saccharin - ( ortho-sulphobenzamide) 550 times as sweet as cane sugar.It is excreted from the
body in urine unchanged. It appears to be entirely inert and harmless when taken.
Aspartame- ( methyl ester of dipeptide formed from aspartic acid and phenylalanine) 100 times as
sweet as cane sugar. It is limited to cold foods and soft drinks because it is unstable at cooking
temperature.
Alitame - 2000 times as sweet as cane sugar. It is stable but it is difficult to control its sweetness.
Sucrolose - ( trichloro derivative of sucrose) 600 times as sweet as cane sugar. It is stable at
cooking temperature and does not provide calories.

jksahu74@gmail.com Page | 265


Medicines Examples Role
Analgesics Aspirin (2-acetoxybenoic acid ),
Drugs used for relieving pain
( Non-narcotic ) Analgin, Seridon, Anacine, Aspro.
Analgesics Opium, Heroin , Pethidine , They reduce tension and pain. They produce
(Narcotic ) Codeine, Morphine sleep and unconsciouness.

Antimicrobials Chlorine, Formaldehyde, The substance which kills or destroys


Phenol, Ozone micro-organism

Antibiotics Penicillin, Streptomycin, The chemical substances produced by micro -


chloramphenicol organism (bacteria,fungi,molds ) that can inhibit the
growth or even destroy other micro-organism.
Antiseptics Dettol, Bithional,Tincture iodine, The chemical sustances which prevent the growth
Methyl blue, Salicylic acid, 0.2% of micro-organism or kill them but are not harmful
phenol, picric acid, Resorcinol to the living human tissues.

Disinfectants 1% phenol, chlorine (Cl2) , A substance that kills micro-organisms, but is


Sulphurdioxide ( SO2) not safe for contact with living tissues. It is
used for toilets, instruments.

Antacids
NaHCO3 , Al(OH) 3 gel , MgCO3 , The chemical substances which can reduce or
Mg(OH)2 , AlPO 4 neutralise the acidity.

Antihistamines Brompheniramine ( Dimetapp) It is used to treat allergy


Terfenadine ( Seldane)

Tranquilizers Valium, Serotonin and It reduce the mental anxiety by acting on higher
Barbiturates i.e derivative of centres in the central nervous system.
barbituric acid such as Veronal,
Equanil,Amytal,nembutal.luminal
and seconal
Antipyretics Aspirin, Paracetamol, Analgin, It is used to bring down the temperature of
Phenacetin. human body.
Anaesthetics Cyclopropane,nitrous oxide, These drugs produce loss of sensation.
xylocain
Hypnotics Luminal, seconal, soaryl It produce sleep and are habit forming

Sedatives Valium, Barbiturates They act as depressant and suppress the


activities of central nervous system.
Sulpha drug Sulphanilamide, Sulphadiazine, It can be used inplace of antibiotics. They inhibit
Sulphaguanidine the growth of micro-organisms.
Antifertility Norethindrone, These are the steroids used to control the pregnancy
drugs Ethynylestradiol ( novestrol )

jksahu74@gmail.com Page | 266


Food preservatives
They prevent spoilage of food due to microbial growth. e.g.- Sodiumbenzoate, salts of propanic acid
and sorbic acid.
Sodium benzoate ( C6H5COONa ) is metabolised by conversion to hippuric acid ( C6H5CONHCH2COOH )
which is ultimately excreted in the urine.
Saponification Base hydrolysis of ester is known as saponification
H 2C O COC17H35 H 2C OH
HC O COC17H35 ∆
+ 3 NaOH 3 C17H35COONa + HC OH
H 2C O COC17H35
[ Sodium stearate ] H 2C OH
[ Glyceryl ester of stearicacid ( fat) ] [ soap ] Glycerol ( Glycerine )
Soaps are sodium or potassium salts of stearic, Oleic or palmitic acid
The soap obtained by saponification remains in colloidal form and is precipitated from the solution by
adding sodium chloride.
Potassium soaps are soft to the skin than sodium soap.
Toilet soaps are prepared by using better grade fats and oils and care is taken to remove excess alkali.
Soaps that float in water are made by beating tiny air bubbles before their hardening
Transparent soaps are made by dissolving the soap in ethanol and then evaporating the excess solvent.
Shaving soaps contain glycerol to prevent rapid drying and rosin ( forms sodium rosinate ) to produce lathers
Laundry soaps contain fillers like sodium rosinate, sodium silicate, borax and sodium carbonate.
Synthetic detergents
These are salts of long chain sulphonates and sulphates.
They are two types:-
[1] Sodium salts of long chain benzenesulphuric acid

H3C ( CH2) 11
H2SO 4
H3C ( CH2) 11 SO3H NaOH (aq) H3C ( CH2 ) 11

SO3 Na
+

Dodecylbenzene Dodecylbenzenesulphonic acid Sodium dodecylbenzenesulphonate


[2] Sodium salts of long chain alkyl hydrogen sulphate
H2SO 4 − +
CH3(CH2)10CH2OH NaOH (aq)
CH3(CH2)10CH2OSO 3H CH3(CH2)10CH2OSO 3 Na
Lauryl alcohol Lauryl hydrogensulphate Sodium laurylsulphate

jksahu74@gmail.com Page | 267


Finkelstein CH3Br + NaI CH3-I + NaBr
1.
Swarts CH3Br + AgF CH3F + AgBr
2.
CH 3

Friedel-Crafts Anhydrous AlCl3


Alkylation + H 3C Cl

3.
COCH 3

Friedel-Crafts CH3COCl
Acylation
Anhydrous AlCl3
4.
2Na
Wurtz H3C Cl + Cl CH3 H3C CH3 + Na Cl
5.
Cl Cl

Fittig 2Na
+ + Na Cl
6. Dry ether
Cl

Wurtz-Fittig 2Na
+ Cl CH3 CH3 + Na Cl
Dry ether
7.
OH OH
ONa
Na OH i) CO2 COOH
Kolbe
ii) H+
8.
OH
ONa OH
CH3Cl + Na OH CHO CHO
Reimer-Tiemann H+

9.

Williamson CH3-Br + CH3-ONa CH3-O- CH3 + NaBr


10.
H3O+
Stephen H3C CN + SnCl2 + HCl H3C CH NH H3C CHO
11.
CH3 CHO

CrO2Cl2
Etard
H3O+
12.
CHO

Gatterman – CO / HCl
Koch
Anhydrous AlCl3
13.

jksahu74@gmail.com Page | 268


O H2 O
Rosenmund
C C
reduction H3C Cl H3C H
Pd / BaSO4
14.
O
Clemmensen Zn - Hg
C H3C CH2 CH3
reduction Conc. HCl
15. H3C CH3
O
Wolff-Kishner i) NH2-NH2
C H3C CH2 CH3
reduction
16. H3C CH3 ii) KOH / Ethylene glycol / ∆

Tollens’ test R-CHO + 2 [Ag(NH3)2]+ + 3 OH- R-COO- + 2Ag + 2H2O + 4 NH3


17.
Fehling’s test R-CHO + 2 Cu2+ + 5 OH- R-COO- + Cu2O + 3H2O
18.
O
I2 / NaOH
Iodoform C CHI3 + CH3COONa
19. H3C CH3 OR, NaOI
OH
Aldol dil NaOH ∆
2 H3C H3C CH CH2 CHO H3C CH CH CHO
20. condensation
CHO

Conc. NaOH
21.
Cannizzaro HCHO
+ HCHO HCOON a + H3C OH

i) Cl2 / Red Phosphorus


Hell-Volhard- H3C COOH H2C COOH
Zelinsky (HVZ) ii) H2O
22. Cl
O
Hoffmann Br2
bromamide H3C C NH2 H3C NH2
23. degradation NaOH

Carbylamine R-NH2 + CHCl3 + 3 KOH R-NC + 3 KCl + 3 H2O
24.
NH2 + -
N 2 Cl

NaNO2 + dil HCl


Diazo
273 - 278 K
25.
+ -
N 2 Cl Cl

Sandmeyer. CuCl / HCl


+ N 2

26.
+ -
N 2 Cl Cl

Gatterman Cu / HCl
+ N2

27.

+ - OH-
Coupling N2 Cl + H OH N N OH
28.

jksahu74@gmail.com Page | 269


ORGANIC CHEMISTRY MECHANISM
1
1. SN mechanism
OH
Br
Aq. KOH C
C ( Racemic mixture )
H5 C2 H
H5 C2 H
CH3
CH3
Br
H
C slow +
Step -I H H5 C2 C
H5 C2
CH3 CH3
Carbocation

OH
H
-
+ HO C
Step -II H5 C2 C
Fast H5 C2 H
CH3 CH3

2. SN2 mechanism
OH
Br
Aq. KOH C
C
H H
H H
H H

H H
OH
Slow Fast
- C Br HO C Br C
HO H
H H H H
H
H T.S
3. Elimination reaction in haloalkane

In general elimination reaction occurs:

(i) in the presence of a strong and/or bulkier base,

(ii) at high temperature and

(iii) with bulkier haloalkanes


Alc. KOH
H3C CH2 Br H2C CH2

The alcoholic KOH results in the formation of a strong base, ethoxide ion.
-+
H C CH
3 OH 2 + CH CH O K
KOH 3 2 + H2O

The much stronger ethoxide ion abstracts a proton to form alkene


H
-
CH3CH2 O
H 2C CH2
β − elimination H2C CH2 + H3C CH2 OH

Ethene
Br
jksahu74@gmail.com Page | 270
4. Effect of substituent on nucleophilic substitution on aryl halides

5. Hydration of Alkenes
HO
+
H
R CH CH2 + H2O R CH CH3

[1] Protonation of alkene


+
+
H2O + H H3O

H
+ +
R CH CH2 + H O H R CH
Carbocation
CH3

[2] Nucleophilic attack of water on carbocation H


+
H O
+ xx
R CH CH3 + H2O
xx
R CH CH3

[3] Deprotonation to form an alcohol

+
H
H O H O
xx +
H 3O
R CH CH3 + H2O
xx
R CH CH3 +

6. Addition of Grignard Reagent to Aldehydes & Ketones

- + HO
O O MgBr
-+ H2O
R MgBr + C
C C + Mg(OH)Br

R R

jksahu74@gmail.com Page | 271


7. Dehydration of alcohol for the formation of Alkene

Conc. H2SO 4
R CH2 CH2 OH R CH CH2
Alcohol 443 K
Alkene
[1] Protonation of alcohol H
xx + Fast +
R CH2 CH2 O
xx
H + H R CH2 CH2 O H

[2] Formation of carbocation H


Slow +
+ R CH2 CH2
R CH2 CH2 O H
Carbocation

[3] Deprotonation to form an alkene H


+
+ −H
R HC CH2 R CH CH2

8. Dehydration of alcohol for the formation of Ether

Conc. H2SO4
R CH2 OH R CH2 O CH2 R
413 K
H
[1] Protonation of alcohol
xx + Fast +
R CH2 O
xx
H + H R CH2 O H
Oxonium ion
[2] Attack of alcohol on oxonium carbon ( addition )
H H
xx Slow
R CH2 O
xx
H + R CH2 O
+
H
2
R CH2 O
+
CH2 R
SN
[3] Deprotonation to form an ether
H +
+
−H
R CH2 O CH2 R R CH2 O CH2 R

9. Reaction of HI with ether

[1] Protonation of ether


H
xx + -
H3C O
xx
CH2 CH3 + HI H3C O CH2 CH3 + I

[2] I− acts as nucleophile and attack the least substituted carbon of the oxonium ion by SN2
mechanism H
+
I− + H3C O CH2 CH3 H3C I + H3C CH2 OH

[3] If excess of HI is there and temperature is very high:

xx +
H3C CH2 OH
xx + H I H3C CH2 OH 2 + I−

+
I− H3C CH2 OH 2 H3C CH2 I + H2O

jksahu74@gmail.com Page | 272


If there is tertiary alkyl group, then the reaction will proceed by SN1 mechanism.
CH3 CH3
xx +
H3C C O
xx
CH3 + H I H3C C O CH3 + I−
CH3 CH3 H

CH3 CH3
+ slow +
H3C C O CH3 H3C C + H3C OH

CH3 H CH3

CH3 CH3
fast
H3C C+ + I− H3C C I

CH3 CH3

10. Nucleophilic addition of aldehydes & ketones


Nu
R1 - +
C O
Nu - E
C OE
Nu - E Nu + E

R1 R2
R2

R1 Nu
- Nu
Nu
Nu +
Step-1 C O C O
- Step-2 - E
Slow C O
R2 R1 C OE
R2 R1 Fast
Planar R2 R1 R2
Tetrahedral

11. Willamson synthesis

This is used to prepare both simple and mixed ether

The reaction involves SN2 attack of an alkoxide ion on primary alkyl halide.

If alkyl halide used is tertiary then alkene will be the main product.

-+
R O Na + R1 X R O R1 + NaX

CH3 CH3
- +
H3C C O Na + H3C Br H3C C O CH3

CH3 Ether
CH3

H CH3 CH3
- + CH2 C Br H2C C Alkene
H3C O Na +
CH3 CH3

- + H3C Br
NaOH
OH O Na O CH 3
Anisole

jksahu74@gmail.com Page | 273


12. Acylation of amines

CH3 H CH3
xx Base +
H 5C 2 N H + C Cl H 5C 2 N C Cl H 5C 2 N C Cl
H -
O H O H O

13. Esterification
O
O Bond cleavage +
H
R C OR 1
R C OH + H O R1
Ester
Carboxylic acid Alcohol
[1] Protonation of carboxylic acid +
O O H
+
H
R C OH R C OH

[2] Attack of alcohol on carbonyl carbon ( addition ) O H


+
O H H R C OH
+
R C OH + xO
xx x R1
H
O
R1
[3] Elimination of leaving group
xx +
x O H
O H x O H
+
R C OH R C O H R C
+
O H O
O
H R1 R1 R1

[4] Deprotonation + O
O H +
−H
R C R C OR 1

O
R1
14. Reduction by LiAlH4
O OH
LiAlH 4
R C R or NaBH 4 R CH R

O OAlH 3
LiAlH 4 +
AlH 4−

Li + AlH 3 H + R C R R CH R

− O
OAlH 3 R
− OH
3 R C R
CH O Al 3 H2O
R CH R
R
3 R CH R + Al(OH) 3
4

jksahu74@gmail.com Page | 274


DISTINGUISH TESTS
Group Test Observation Description
Br Br
CCl 4
R CH CH
+ ( Alkene )
R R CH HC R
Br2 Br Br
Bromine water Disappearance of orange- CCl 4
red colour of bromine water ( Orange - red colour )
+ R C C R R C C R
Unsaturation

( Alkyne ) Br Br
(Colourless )
When alkaline KMnO4 is R CH CH R R CH CH R
added to an unsaturated ( Alkene ) OH
+
HO
hydrocarbon, Cold dilute KMnO4
R C C R1 MnO 2
Baeyer’s test disappearance of pink
colour indicates ( Pink ) ( Alkyne )
RCOOH + R1 COOH +
Brown Ppt.
unsaturation. In this R C C H
process MnO2 is formed ( Terminal Alkyne )
RCOOH + CO2
+ H2O +
( brown colour).
Terminal alkynes’

Tollen’s reagent: White precipitate will obtain Ag(NH3)2OH


R C CH R C C Ag
Ammonical silver nitrate
White ppt
Ammonical cuprous
chloride: Red precipitate will obtain Cu(NH3) 2Cl
R C CH R C C Cu
Amm. Cuprous chloride Red ppt.
( single bond )

The halo compound boiled aqueous KOH HNO3


with aq. KOH and then R X
Haloalkane Boil
R OH + KX
AgNO3
Ag X
C–X

AgNO3 Test treated with acidified Precipitate


AgNO3. ( For Cl white ppt
and for Br yellow ppt.will
obtain )
jksahu74@gmail.com Page | 275
Group Test Observation Description
R R
Alcohol treated with Lucas reagent. R C OH
ZnCl 2 + HCl R C Cl
Primary, Secondary and Tertiary

Room temperature
R R
Lucas Test (HCl & ZnCl2)
If turbidity appears immediately ---
Tertiary alcohol. Tertiary alcohol Cloudiness appears immediately

If turbidity appears after some time --- R R


Secondary alcohol R CH OH
ZnCl 2
R
+ HCl
CH Cl
Room temperature
Secondary alcohol
If turbidity appears after heating--- Cloudiness appears within five minutes
Primary alcohol
Alcohol

R CH2 OH Lucas reagent


R CH2 Cl
Heat
Primary alcohol
Cloudiness appears upon heating
Iodoform test
Ethyl alcohol

When ethyl alcohol or 2-hydroxy alcohol OH


2-hydroxy

treated with I2 and NaOH ( NaOI ) a H3C CH R + I2 NaOH


CHI3 + RCOONa
alcohol
and all

yellow precipitate of iodoform will form. Alcohol Iodoform


( Yellow precipitate )

Neutral ferric Phenol when reacts with neutral ferric FeCl 3 3−


Phenol chloride, chloride, it produces violet colour. 6 OH
[ Fe (OC 6H5)6 ]
+ 3 HCl
Violet colouration
hydrazine test:

O 2N O 2N
Carbonyl compound react with 2,4-
dinitrophenyl

H
Carbonyl dinitrophenylhydrazine to form yellow or
group orange crystals.
C O + H
N NH NO 2 C N NH NO 2

2,4-DNP Yellow / orange crystal


Carbonyl compound
2,4-

jksahu74@gmail.com Page | 276


Group Test Observation Description
R OH
Aldehydes and Sodium NaHSO3
+-
only methyl bisulphate Aldehydes and methyl ketones both form
C O
H Aldehyde
+ R C SO 3 Na
H
ketone test: crystalline addition product with NaHSO3 OH
R +-
C O + NaHSO3 R C SO 3 Na
H3C Methylketone CH3

Tollen Test On warming with ammonical silver nitrate O +


2 [Ag(NH3)2]
solution(Tollen reagent) in alkaline R C H RCOO- + 2 Ag + 2 H 2O + 4 NH3
medium, aldehyde gives silver mirror on 3 OH-
Aldehydes the side of test tube.

Fehling Test Fehling solution is alkaline solution of O


2 Cu 2+
copper sulphate containing sodium R C H Cu2O
+ 3 H 2O + RCOO-
potassium tartarate. Aldehyde gives 5 OH
-
reddish brown precipitate of
Cu2O with Fehling solution.

O
H3C C Iodoform When aldehyde or ketone containing O
Iodoform test
group Test such type of groups are treated with I2 H3C C R + I2
+ NaOH CHI3 + RCOONa
and NaOH ( NaOI ) a yellow precipitate of Iodoform
( Yellow precipitate )
iodoform will form
Aromatic
aldehyde Tollen’s test With Fehling solution give negative test
Aromatic aldehyde + Tollen's reagent Silver mirror

and with Tollen’s reagent give positive Aromatic aldehyde + Fehling solution Negative test
test.
Carboxylic Sodium
acid bicarbonate Brisk effervescence ( production of CO2 )
test will produce when carboxylic acid treated
RCOOH + NaHCO3 RCOONa
+ H2O + CO2

with sodium bicarbonate.

jksahu74@gmail.com Page | 277


Group Test Observation Description

On warming with ammonical silver Tollen's reagent 2−


Tollen Test nitrate solution(Tollen reagent) in
HCOOH Ag + CO3 + H2O
alkaline medium, Formic acid gives Silver mirror
silver mirror on the side of test tube.
Formic acid

Fehling solution is alkaline solution of 2−


Fehling solution Cu O
Fehling Test copper sulphate containing sodium HCOOH 2 + CO3 + H2O
potassium tartarate. Formic acid gives Reddish brown ppt
reddish brown precipitate of
Cu2O with Fehling solution.

NaNO2 + HCl + -
Aromatic Azo-dye test NH2
Diazotisation
N2 Cl + NaCl + H2O
primary Aromatic primary amine produce
amine diazonium salt when treated with Benzenediazonium chloride
NaNO2 and HCl. If the diazonium salt
treated with phenol an orange dye will
H OH
produce , with aniline it will produce
N N OH
yellow dye. HO
-
Coupling reaction
+
- ( Orange dye )
N2 Cl
H NH 2 N N NH2
-
HO Coupling reaction p-Aminoazobenzene

( Yellow dye )
When a primary amine is heated with
Aliphatic and Carbylamine alcoholic caustic potash and
aromatic test chloroform, an offensive smelling
Primary amine
+ KOH + CHCl 3 Isonitrile
( Offensive smell )
primary compound
amine called carbylamine ( alkyl or
arylisocyanide) is formed.

jksahu74@gmail.com Page | 278


Group Test Observation Description
O O

R N H + Cl S R N S

H O H O
o
1 amine
Acidic hydrogen
It gives sulphonamide with N-alkylbenzenesulphonamide
hinsberg reagent, this
Primary amine KOH
sulphonamide is soluble in
Hinsberg Test
NaOH or KOH. + O
K
R N S
-
O
(The reagent used Water soluble
potassium salt
in this test is
of sulfonamide
benzene sulphonyl O O

chloride)
With hinsberg reagent,it
R N H + Cl S R N S

O R O
R
forms sulphonamide, which N,N-Dialkylbenzenesulphonamide
2o amine
Secondary amine is insoluble in NaOH or
KOH
KOH
Water insoluble
precipitate
Tertiary amine do not react O

Tertiary amine
with hinsberg reagent R N R + Cl S No reaction
R O
,because it is not having 3o amine
replaceable hydrogen.

jksahu74@gmail.com Page | 279


[1] Propene and propane Br Br
CCl 4
H 3C CH CH2 Br 2
+ H3 C CH CH 2
propene ( Orange - red colour )
(Colourless )
CCl 4
Br 2
H 3C CH 2 CH3 + Orange - red colour
( Orange - red colour )
propane
[2] Propyne and propane

H 3C C CH + Cold dilute KMnO4 MnO 2 + CH 3COOH + CO 2 + H2 O


Propyne ( pink ) Brown Ppt. (Colourless )

H 3C CH 2 CH3 + Cold dilute KMnO4 Pink colour


propane ( pink )
[3] Propyne and propene
Cu(NH 3 ) 2 Cl Ag(NH 3 ) 2 OH
H 3C C C Cu H3 C C CH H3C C CAg
Ammonical Cuprous chloride Propyne Ammonical silver nitrate
Red ppt. White ppt
Cu(NH 3 ) 2 Cl Ag(NH 3 ) 2 OH
No red ppt. H3C CH CH2 No white ppt.
propene

[4] Chloroethane and chlorobenzene


H 5 C 2 Cl Boil HNO 3
+ aqueous KOH H 5C 2 OH + KCl
AgNO 3
AgCl
Chloroethane
White ppt.
Boil
Cl
+ aqueous KOH No action
Chlorobenzene
[5] Chlorocyclohexane and chlorobenzene
Boil HNO 3
Cl
+
aqueous KOH OH
+ KCl
AgNO 3
AgCl
Chlorocyclohexane White ppt.
Boil
Cl + aqueous KOH No action
Chlorobenzene
[6] Chloroethane and Bromoethane
H 5 C 2 Cl Boil HNO 3
+ aqueous KOH H 5C 2 OH + KCl
AgNO 3
AgCl
Chloroethane
White ppt.
H 5 C 2 Br Boil HNO 3
+ aqueous KOH H 5C 2 OH + KBr
AgNO 3
AgBr
Chloroethane
Yellow ppt.
[7] Benzyl chloride and chlorobenzene
Cl OH HNO 3
Boil
CH 2
+ aqueous KOH CH 2 + KCl
AgNO 3
AgCl
Benzyl chloride White ppt.

Boil
Cl
+ aqueous KOH No action
Chlorobenzene
[8] Ethyl chloride and Vinyl chloride
Boil HNO 3
H 5C 2 Cl + aqueous KOH H 5 C 2 OH + KCl
AgNO 3
AgCl
Ethyl chloride White ppt.
Boil
H2 C CH Cl + aqueous KOH No action
Vinyl chloride

jksahu74@gmail.com Page | 280


[9] n-Propylalcohol and isopropyl alcohol
OH Cl
ZnCl 2
H 3C CH CH3 + HCl H3C CH CH3
Room Temp.
isopropyl alcohol + Lucas reagent Cloudiness appear
ZnCl 2
CH 3CH 2CH 2 OH + HCl CH 3 CH 2 CH 2Cl
Room Temp.
n-propyl alcohol + Lucas reagent No cloudiness
[10] Methyl alcohol and ethyl alcohol
CH 3 CH 2 OH + 4 I2 + 6 NaOH CHI 3
Yellow ppt.
+ HCOONa

CH 3 OH
+ 4 I2
+ 6 NaOH No reaction
[11] Ethyl alcohol and Acetone
O 2N O 2N
H 3C
H3C H

C O + H
N NH NO 2 C N NH NO 2

H 3C H 3C
Precipitate
2,4-Dinitrophenylhydrazine
Acetone

CH 3 CH 2OH + 2,4-Dinitrophenylhydrazine No reaction


[12] Ethyl alcohol and Phenol

Phenol + Neutral ferric chloride Violet colouration


3−
6 OH
+ FeCl 3 [ Fe (OC 6 H 5) 6 ]
+ 3 HCl
Violet colouration

CH 3 CH 2 OH + Neutral ferric chloride No Violet colouration


[13] Benzoic acid and Phenol
Benzoic acid + Sodiumbicarbonate Brisk effervescence
C 6H 5 COOH + NaHCO 3 C 6 H5 COONa + CO 2
+ H2O

Phenol + Sodiumbicarbonate No effervescence


( Phenol is less acidic than benzoic acid )
[14] Propanone and Propanol
Propanone + 2,4-Dinitrophenylhydrazine Yellow / orange crystal
O 2N O 2N
H 3C H
H 3C

C O + N NH NO 2 C N NH NO 2

H3C H H 3C

Propanol + 2,4-Dinitrophenylhydrazine No crystal


[15] Propanal and Propanone
Propanal + Tollen's reagent Silver mirror
O
3 OH- -
+
H 3C CH 2 C H + + 2 [Ag(NH 3) 2] CH 3CH 2COO + 2 H 2O + 4 NH 3 + 2 Ag
O Propanal + Fehling solution Reddish brown precipitate
- -
H 3C CH 2 C H +
2 Cu 2+ 5 OH + CH 3CH 2 COO + Cu O
3 H 2O + 2

Negative test Fehling solution Tollen's reagent


Propanone Negative test

jksahu74@gmail.com Page | 281


[16] Propanal and Ethanal O ( Yellow precipitate )
Iodoform test
H3C C H + I2
+ NaOH CHI3 + HCOONa
Ethanal Iodoform
O
Iodoform test
H3C CH2 C H + I2
+ NaOH No yellow ppt.
Propanal
[17] Pentan-2-one and Pentan-3-one
O ( Yellow precipitate )
Iodoform test
H3C CH 2 CH 2 C CH3 + I2
+ NaOH CHI3 + CH3CH2CH2COONa

Pentan-2-one Iodoform
O
Iodoform test
H3C CH 2 C CH 2 CH3
+ I2
+ NaOH No yellow ppt.
Pentan-3-one
[18] Propanal and Benzaldehyde
O Propanal + Fehling solution Reddish brown precipitate
5 OH - -
H C 3CH C H 22 Cu 2+ + + CH3CH2COO + Cu O
3 H2O + 2
Fehling solution
Benzaldehyde + Fehling solution No precipitate
-
CHO
+ 2 Cu 2+ + 5 OH No reaction
[19] Ethanoic acid and Methanoic acid
Fehling solution
Reddish brown ppt Methanoic acid Tollen's reagent Silver mirror

2− Fehling solution HCOOH Tollen's reagent 2−


H2O + CO3 + Cu2O Ag + CO3 + H2O
Fehling solution
No brown ppt Ethanoic acid Tollen's reagent No Silver mirror

[20] Ethanal and Methanal


Iodoform test
CH3CHO + I2
+ NaOH CHI3 + HCOONa
Ethanal Iodoform ( Yellow precipitate )
Iodoform test
HCHO
Methanal
+ I2 + NaOH No yellow ppt.

[21] Acetophenone and Benzophenone


O
Iodoform test
C CH3
+ I2
+ NaOH CHI3 + COONa

Acetophenone ( Yellow precipitate )


O
Iodoform test
C
+ I2
+ NaOH No ppt.

Benzophenone
[22] Benzoic acid and Ethyl benzoate
Benzoic acid + Sodiumbicarbonate Brisk effervescence
C6H5COOH + NaHCO3 C6H5COONa + CO2
+ H 2O
Ethyl benzoate + Sodiumbicarbonate No effervescence
[23] Acetophenone and Benzaldehyde
Benzaldehyde + Tollen's reagent Silver mirror
- + -
CHO 3 OH+ 2 [Ag(NH ) ] + COO 3 2 + 2 H2O + 4 NH3 + 2 Ag
Tollen's reagent

Acetophenone + Tollen's reagent No Silver mirror

jksahu74@gmail.com Page | 282


[24] Methylamine and dimethylamine
CH3NH2
+ CHCl 3
+ 3 KOH (alc ) CH3NC +
Methylisonitrile
3 KCl + 3 H 2O
Methylamine
CH3 ( Offensive smell )

H3C NH + CHCl 3 + 3 KOH (alc ) No offensive smell


Dimethylamine
[25] Ethylamine and Aniline
OH
NH2
NaNO 2
+ HCl +−
N2 Cl N N OH
Diazotisation
Aniline Orange dye
CH 3CH 2 NH2
NaNO 2
+ HCl
CH 3CH2 OH
OH

No Orange dye
Ethylamine
[26] Aniline and N-methylaniline

NH2
+ CHCl3 + 3 KOH (alc ) NC
+
Phenylisonitrile
3 KCl + 3 H 2O
Aniline
( Offensive smell )
NH
N-Methylaniline
CH3
+ CHCl 3 + 3 KOH (alc ) No offensive smell

[27] Aniline and Benzylamine


OH
NH2
NaNO 2
+ HCl +−
N 2 Cl N N OH
Diazotisation
Aniline Orange dye
NaNO 2 HCl

OH
NaNO 2 + HCl
CH 2 NH2 CH 2 OH No Orange dye
Benzylamine

[28] Secondary and tertiary amine


Secondary amine + Benzenesulphonyl chloride Precipitate KOH
Water insoluble
Tertiary amine + Benzenesulphonyl chloride No reaction

[29] Glucose and Fructose


Glucose
+ Br 2 + H 2O Gluconic acid
+
2 HBr
[ Red colour ] [ Colour less ]

Fructose + Br 2 + H2O Red colour


[ Red colour ]
[30] Glucose and Sucrose
Glucose + Tollen's reagent Silver mirror

Sucrose
+ Tollen's reagent No Silver mirror

[31] Glucose and starch


Glucose + Fehling solution Red ppt.

Starch + Fehling solution No Red ppt.

OR
Glucose + I 2 Solution No Blue colour

Starch + I 2 Solution Blue colour

jksahu74@gmail.com Page | 283


Broadly you can classify conversions into two types – aliphatic and aromatic.

Aliphatic Conversions
a). For stepping up the series:

Stepping up the series means that the product has one carbon more than that of reactant. For
such kind of a conversion, convert the given compound to an alkyl halide and then to a cyanide
and then to the required organic compound as asked in the question. This way, the product has
one carbon more than the reactant.

HX alc. KCN LiAlH 4


R OH R X R CN R CH 2 NH2
Anhydrous ZnCl2 Reduction
Alcohol Alkyl halide Nitrile Primary amine

HNO 2
X 2 / UV Hydrolysis
( NaNO 2 & HCl )
+
Hydrocarbon H3O
LiAlH 4
R CH 2 OH RCOOH R CH 2 OH
+ Oxidation
Primary alcohol H3O Primary alcohol
Carboxylic acid

b). For stepping down the series:

Stepping down the series means that the product has one carbon less than that of its reactant. For
such kind of a conversion, convert the given compound to an amide and then let it undergo
Hoffmann bromamide degradation reaction. Then you convert it to the required product as asked
in the question.
NH 3 Br 2 HNO 2
RCOOH RCONH 2 R NH 2
R OH
HX
R X
Carboxylic acid ∆ Acid amide KOH Amine ( NaNO 2 & HCl )
Primary alcohol
Anhydrous ZnCl2
Alkyl halide

Alkaline KMnO4 Aqueous KOH


R CH2 OH R CH2 X
Primary alcohol Alkyl halide

c). Reaction with Grignard’s reagent:

Grignard’s reagent is a versatile reagent and can be used for many conversions. Some of its
reactions are given below:

Dryether
R X + Mg RMgX

RMgX + CO2 RCOOH

jksahu74@gmail.com Page | 284


RMgX + HCHO Primary Alcohol

RMgX + RCHO Secondary Alcohol


O
RMgX + R C R Tertiary Alcohol
O
RMgX + R C N R C R

Aromatic Conversions
There are two types of conversions possible in this case:

a. When the functional group contains carbon atom:


Cl OH CH3

NaOH , 623 K, 300 atm Zn dust CH3Cl


+ Anhydrous AlCl3
H

H2 / Pd , BaSO 4 , S SOCl 2 Alkaline KMnO 4


OHC ClOC HOOC
Boiling Xylene

b. When the functional group does not contain carbon atom:

OH
Warm H2O

I
H3PO 2
Zn dust

H2O
KI
+
N2 BF4- +
N2 Cl- Cl

HBF4
CuCl
HCl
Conc. HNO 3


H2SO4 , 333 K

CuBr
2
NO

F Br
HBr
NaNO2
,∆
Na

HCl
Cu

NO 2 NH2
Sn / HCl

jksahu74@gmail.com Page | 285


Ethyne into benzene
Red-Hot-Cu Tube
3 CH CH
Polymerisation

Ethene into benzene Br Br


Br2 Alc. KOH Red-Hot-Cu Tube
H2C CH2 H2 C CH2 HC CH
Polymerisation

Hexane into benzene CH3


H2C CH3 Cr2O7 / V 2O5 / Mo2O3 Aromatisation
H2C CH 2 773 K / 10-20 atm. ( - H 2 ) − 3 H2
Hexane CH2
Cyclohexane Benzene

Benzene into p-nitrobromobenzene


Br 2 / FeBr 3 HNO 3 / H 2SO 4
Br O 2N Br

NO 2 NO 2
Benzene into m-nitrochlorobenzene
HNO 3 / H 2SO 4 Cl 2 / FeCl 3

Cl

Benzene into p-nitrotoluene


CH3Cl / anhy. AlCl3 HNO 3 / H 2SO 4
O 2N CH3
CH3

Benzene into Acetophenone O


O C
Anhy. AlCl 3 CH3
+ H3C C Cl

Ethanol into but-1-yne


SOCl 2 HC CNa
CH3CH2OH CH3CH2Cl HC CCH2CH3
( Pyridine ) NaCl

Ethane to Bromoethene
Br Br
Br 2 Alc. KOH Br2 Alc. KOH
H3C CH3 CH3CH2Br H2C CH2 H2 C CH2 H2C CH Br

Propene into 1-nitropropane


HBr AgNO2
H3C CH CH2 H3C CH2 CH2 Br H3C CH2 CH2 NO2
( Peroxide ) − AgBr

Ethanoic acid into Benzene

aq NaOH Sodalime Cl 2 / hν Na / Dryether


H3C COOH H3C COONa CH4 H3C Cl H3C CH3

Red hot iron tube Cl 2 / hν


873 K
Br Br Cl
NaNH2 Alc. KOH Br2 Alc. KOH CH2 CH3
HC CH H2C CH Br H2 C CH2 H2C CH2

jksahu74@gmail.com Page | 286


Toluene into Benzyl alcohol
Cl OH
Cl 2 / Boil Aq. KOH
CH3 CH 2 CH 2
− HCl
Propene into Propyne Br Br
Br 2 alc. KOH
H3C CH CH2 H3 C HC CH 2 H3C C CH
CCl 4 NaNH 2

Ethanol into Fluoroethane


SOCl 2 Hg 2F 2
H3C CH 2 OH H3C CH 2 Cl H3C CH 2 F
( Pyridine ) − Hg2 Cl 2
Bromomethane into Propanone CH3 CH3
+
Alc. KCN CH 3MgBr H2O / H
CH 3 Br CH 3CN H3C C NMgBr H3 C C O
− KBr ( ether )

But-1-ene into But-2-ene Br


HBr alc. KOH
H3C CH 2 CH CH2 H3C CH 2 HC CH3 H3C CH CH CH3

1- Chlorobutane into n- Octane Dry ether


H3C CH 2 CH 2 CH 2 Cl + 2 Na
( Wurtz reaction )
H3C ( CH 2 ) 6 CH3

Benzene into Biphenyl Br 2 / FeBr 3 Na / Dryether


2 Br

Propene into Propan-1-ol H3C HBr Aq. KOH


CH CH2 H3C CH 2 CH 2 Br H3C CH 2 CH 2 OH
Peroxide
Br
1- Bromopropane into 2-Bromopropane
alc. KOH HBr H3C CH CH3
H3C CH 2 CH 2 Br H3C CH CH2

Benzyl alcohol into 2-Phenylethanoic acid


+
CH 2
OH SOCl 2 Cl KCN CN H2 O / H
CH 2 CH 2 CH 2COOH

Ethanol into Propanenitrile SOCl 2 KCN


H3C CH 2 OH H3C CH 2 Cl H3C CH 2 CN
( Pyridine )

Aniline into Chlorobenzene NaNO2 / HCl + - CuCl / HCl


NH2 N 2 Cl Cl

2-Chlorobutane into 3,4-Dimethylhexane CH3


CH3
Na / Dryether
2 H 3C CH 2 CH Cl H3 C CH 2 CH CH CH 2 CH3
CH3

2-Methylpropene into 2-Chloro- 2-methylpropane Cl


HCl
H3C C CH2 H3C C CH3
CH3 CH3

Ethylchloride into Propanoic acid H2O / H


+
KCN H3C CH 2 COOH
H3C CH 2 Cl H3C CH 2 CN

But-1-ene into n-butyliodide


HBr NaI
H3C CH 2 CH CH2 H3C CH 2 CH 2 CH 2 Br H3C CH 2 CH 2 CH 2 I
Peroxide Acetone
2-Chloropropane into Propan-1-ol Cl
alc. KOH
B2H6
H3C CH 2 CH 2 OH
H 2 O 2 / OH−
CH CH3 H 3C CH CH2
H 3C

Isopropyl alcohol into Idoform OH I 2 / Na 2CO 3


CHI 3
H 3C CH CH 3
− CH 3COONa
Chlorobenzene into p-nitrophenol NaOH
HNO 3 / H 2SO 4
O 2N OH
Cl OH
623 K / 300 atm.

jksahu74@gmail.com Page | 287


2- Bromopropane into 1-Bromopropane alc. KOH H Br
H 3C CH CH3 H3C CH CH2 H3C CH 2 CH 2 Br
Peroxide
Br
tert.butyl bromide into Isobutyl bromide
CH3 CH3
alc. KOH H Br
H 3C C CH3 H 3C C CH2 H3C CH CH 2 Br
Peroxide
Br CH3

Aniline into Phenylisocyanide ( warm )


NH2
+ CHCl 3
+ 3 KOH NC + 3 KCl + 3 H 2O

Propene into Propan-2-ol Br HO


HBr aq. KOH
H 3C CH CH2 H3 C CH CH3 H3 C CH CH3

Benzyl chloride into Benzyl alcohol aq. KOH


CH 2 Cl CH 2 OH

Ethylmagnesium chloride into Propan-1-ol


O +
H2 O / H
H 3C CH 2 MgCl + H C H H 3C CH 2 CH 2 OMgCl H 3C CH 2 CH 2 OH

Methylmagnesium bromide into 2-Methylpropan-2-ol


O OMgBr + OH
H2O / H
CH 3 MgBr + H3C C CH3 H3C C CH3 H3C C CH3
H3C H3C

Butan-1-ol into Butanoic acid CH 3CH 2CH 2 CH 2OH Jones reagent ( CrO 3 - H 2SO 4 ) CH 3CH 2CH 2 COOH

Benzyl alcohol into phenylethanoic acid


+
HBr KCN H 3O
CH 2 OH CH 2 Br CH 2 CN CH 2 COOH

3-Nitrobromobenzene into 3-Nitrobenzoic acid
+
Mg CO 2 H 3O
Br Mg Br COOMgBr COOH
Ether ( Dry ice )

O 2N O 2N O 2N O 2N

4-Methylacetophenone into Benzene-1,4-dicarboxylic acid


KMnO 4 / KOH dil. H 2 SO 4
H 3C COCH 3 KOOC COOK HOOC COOH

Cyclohexene into Hexane-1,6-dioic acid KMnO 4 - H 2SO 4 COOH


Heat COOH Adipic acid

Butanal into Butanoic acid H C CH 2 CH 2 CHO Tollen's reagent


H 3C CH 2 CH 2 COOH
3

+
Ethylbenzene into Benzoic acid CH 2CH 3
KMnO 4 / KOH
COOK
H 3O
COOH

+
Acetophenone into Benzoic acid KMnO 4 / KOH H 3O
COCH 3 COOK COOH

Bromobenzene into Benzoic acid O


O
Br
Mg C O H2O
Mg Br C COOH
OMg Br

Phenylethene ( Styrene ) CH 2
H 3O
+
KMnO 4 / KOH COOK
CH COOH
into Benzoic acid
Ethanal into Butane-1,3-diol OH OH OH
dil. NaOH LiAlH 4
H 3C CHO H3C HC CH2 CHO H 3C CH CH 2 CH 2

jksahu74@gmail.com Page | 288


Ethanal into But-2-enal OH Heat
dil. NaOH H3C HC CH2 CHO H3C CH CH CHO
H3C CHO − H2 O

Ethanal into But-2-enoic acid


dil. NaOH
OH
Heat Tollen's reagent
H3C CH CH CHO H3C CH CH COOH
H3C CHO H3C CH CH 2 CHO
−H2O
CH3 COOH COOCH 3
CH 3Cl −
Benzene into methylbenzoate KMnO 4 / OH CH3OH
Anhy. AlCl3 ( H 2 SO 4 )

CH3 COOH COOH


CH 3Cl
Benzene into m-Nitrobenzoic acid KMnO 4 / OH − HNO 3
Anhy. AlCl3 H2 SO 4
NO2

Benzene into CH 3Cl


CH3
HNO 3
KMnO 4 / OH−
H 3C NO 2 HOOC NO 2
p-Nitrobenzoic acid Anhy. AlCl3 H2 SO 4

Benzene into p-Nitrobenzaldehyde


CH 3Cl HNO 3 CrO 2Cl 2 / CS 2
CH3 H 3C NO 2 OHC NO 2
Anhy. AlCl3 H2 SO 4 +
H 3O

Benzene into Phenylacetic acid


CH 3 CH 2Cl CH 2 CN CH 2COOH
Cl2 / Boil KCN +
CH 3Cl H2O / H
Anhy. AlCl3

Propanone into propene O OH


LiAlH4 H2SO4 (conc.)
H3C C CH3 H3C HC CH3 H3C CH CH2
Heat
Propanal into butanone OH O
CH3MgBr K2Cr 2 O7 / H2 SO4
CH3CH2CHO + H3C CH2 CH CH3 H3C CH2 C CH3
H2O / H

Ethanol into 3-hydroxybutanal OH


Anhydrous CrO3 Dil. NaOH
CH3CH2OH CH3CHO H3C HC CH2 CHO
OR Cu / 573 K Aldolcondensation
O
Benzaldehyde into benzophenone CHO COOH
K2 Cr 2O7 / H 2SO4 Ca(OH)2 C
Heat

Benzaldehyde into 3-phenylpropan-1-ol


CHO CH CH CHO CH 2 CH 2 CH 2 OH
dil.NaOH H2 / Ni
+ CH3CHO
( Cross aldolcondensation )

Benzaldehyde into 3-phenylprop-2-en-1-ol


CHO CH CH CHO CH CH CH 2 OH
dil.NaOH NaBH 4 / CH 3OH
+ CH3CHO
( Cross aldolcondensation )

HO HO
Benzaldehyde into α -Hydroxyphenylacetic acid
O CN + COOH
CH CH
H2O / H
C
H
+ HCN

Benzoic acid into m-Nitrobenzyl alcohol COOH COOH CH 2 OH


+
HNO 3 / H 2 SO 4 B 2H 6 / H 3O

NO 2 NO 2

Benzoic acid into Benzaldehyde COOH COCl


Pd / BaSO4
CHO
SOCl 2
Rosenmund reduction

jksahu74@gmail.com Page | 289


Benzene into m-Nitroacetophenone O
O 2N
O
CH 3COCl / AlCl 3 HNO3 / H 2SO 4
C CH 3 C CH 3

Bromobenzene into 1-Phenylethanol O -


O MgBr
+
HO
+
Mg H3C C H H3O
Br Mg Br CH CH3 CH CH3

Chloroethane into propan-1-amine Ethanolic NaCN LiAlH 4


H 3C CH 2 Cl H 3C CH 2 C N H3C CH 2 CH 2 NH2
Reduction

Chlorophenylmethane into 2-Phenylethanamine H 2 / Ni


Ethanolic NaCN CH 2 C N CH 2 CH 2 NH2
CH 2 Cl
Reduction

Benzene into Aniline HNO 3 / H 2SO 4 Sn / HCl NH 2


NO 2
Reduction

Benzene into N,N-dimethylaniline CH3


CH 3 Br
HNO 3 / H 2 SO 4 Sn / HCl N
NO 2 NH2
Reduction CH 3

1,6-Dichlorobutane into hexane-1,6-diamine


LiAlH 4
Cl ( CH2 ) 4 Cl KCN NC ( CH 2 ) 4 CN H 2NH 2C ( CH 2 ) 4 CH 2NH 2

4-Nitrotoluene into 2- bromobenzoic acid


CH 3 CH 3 CH 3 CH 3 CH 3 COOH
Br Br Br Br Br
Br 2 Sn / HCl NaNO 2 / HCl H 3PO 2 KMnO 4
273-278 K H2O OH−

NO 2 NO 2 NH 2 N N Cl

+

+ - NO 2
3-Methylaniline into 3-nitrotoluene NH 2 + -
N 2 Cl
N 2BF 4

NaNO 2
NaNO 2 + HCl HBF4
Cu ∆
CH3
CH3
CH 3 CH3
NH 2 + -
N 2 Cl
Aniline into 1,3,5-tribromobenzene Br Br
Br Br Br Br H 3PO 2
NH 2
aq. Br 2 NaNO 2 + HCl
Br
Br Br
Hexanenitrile into 1-aminopentane
-
H3C ( CH 2)4
H2O / H O
C N H3C ( CH 2)4 CONH 2
Br 2 + 4 KOH
H3C ( CH2)3 CH2 NH2
H 2O 2
H2O / H +
Methanol into ethanoic acid H3C OH
SOCl 2
H3C Cl
KCN
H3C CN CH 3COOH

Etahnamine into methanamine


CH3 CH2 NH 2
NaNO 2 + HCl KMnO 4
CH3 COOH
NH3
CH 3CONH 2
Br 2 + 4 KOH
CH 3NH 2
CH3 CH2 OH
( HNO 2 ) OH− ∆
Methanamine into ethanamine LiAlH 4
NaNO 2 + HCl SOCl 2 KCN CH3 CH 2NH 2
CH 3NH 2 H3C OH H3C Cl H3C CN
( HNO 2 )

Ethanoic acid into Propanoic acid


+
B 2H 6 / H 3O SOCl 2 KCN H2O / H +
CH3 COOH CH 3 CH2 OH CH3 CH2 Cl CH3 CH2 CN CH 3CH 2 COOH

Propanoic acid into ethanoic acid


CH3 CH2 COOH
NH3
CH3 CH2 CONH2
Br 2 + 4 KOH
CH3 CH2 NH2
NaNO 2 + HCl
CH3 CH2 OH
KMnO 4
CH3 COOH
∆ ( HNO 2 ) OH−

jksahu74@gmail.com Page | 290


Nitromethane into dimethylamine CH 3 NO 2
Sn / HCl
CH3 NH 2
CH 3 Cl
(CH 3 )2 NH
6 [H]

Ethanoic acid into methanamine


CH3COOH
NH3
CH3CONH2
Br 2 + 4 KOH
CH3NH2 + K2CO 3 + 2 KBr + 2 H 2O

Nitrobenzene into benzoic acid
Sn / HCl NaNO 2 / HCl + − CuCN H2O / H+
NO 2 NH2 N 2 Cl C N COOH
273-278 K

Benzoic acid into aniline


COOH
NH3
CONH 2
Br 2 + 4 KOH
NH2

Benzamide into toluene
Br 2 / KOH NaNO 2 / HCl + − H 3PO 2 / H 2 O CH 3 Cl
CONH 2 NH2 N 2 Cl CH3
273-278 K Anhyd. AlCl 3

Aniline into p-Bromoaniline O O


- +
Br 2
NH2
( CH 3 CO ) 2 O
NH C CH3 Br NH C CH3
HO / H Br NH2
Pyridine CH 3 COOH

+ − + −
Aniline into 2,4,6-tribromofluorobenzene N 2 Cl N2 BF 4 F
NH2 NH2 Br Br Br Br Br Br
Br Br NaNO 2 / HCl HBF4 NaNO 2
aq. Br 2
273-278 K Cu ∆
Br Br Br
Br

Aniline into benzyl alcohol


NaNO 2 / HCl + − CuCN LiAlH 4 NaNO 2 / HCl
NH2 N 2 Cl CN CH2NH2 CH 2OH
273-278 K

Chlorobenzene into p-Chloroaniline Cl


HNO 3
Cl NO 2
Sn/ HCl Cl NH2
H 2SO 4

Benzene into m-bromophenol + −


N 2 Cl OH
NO 2 NO 2 NH2

Br 2 Boil H 2 O
HNO 3 Sn/ HCl NaNO 2 / HCl
H 2SO 4 FeBr 3 273-278 K
Br Br
Br Br

Benzyl chloride into 2- Phenylethanamine KCN LiAlH 4


CH 2 Cl CH 2 CN CH 2CH 2NH 2

m-Nitroaniline into m-Chloroiodobenzene + −


NH2 N 2 Cl I
NO 2 NO2 NO 2

NaNO 2 / HCl CuCl Sn/ HCl NaNO 2 / HCl KI / Heat


273-278 K 273-278 K
+ − Cl Cl
NH2 N2 Cl Cl Cl

+
Aniline into Benzoic acid NaNO 2 / HCl + − CuCN H2 O / H
COOH
NH2 N 2 Cl CN
273-278 K

p-Chloroaniline into p-Chlorobenzylamine


NaNO 2 / HCl + − CuCN LiAlH 4
Cl NH2 Cl N 2 Cl Cl CN Cl CH 2 NH 2
273-278 K

Toluene into m-Bromotoluene CH3


CH3
CH3
CH3 CH3 CH3
CH3
Br 2 + (i) Diazotisation
( CH 3CO ) 2 O H2 O / H
HNO 3 LiAlH 4 (ii) H 3 PO 2 / H 2O
CH 3 COOH
H 2SO 4 Pyridine Br Br Br
NO 2 NHCOCH 3 NH2
NH2 NHCOCH3
NO 2 NH 2 + −
N Cl Cl
m- Dinitrobenzene into m-Dichlorobenzene 2

NaNO 2 / HCl CuCl


Sn/ HCl
273-278 K
NH2 + − Cl
NO 2 N 2 Cl

jksahu74@gmail.com Page | 291

Potrebbero piacerti anche